UW Internal Medicine Flashcards

1
Q

Lady has history of chest pain worse with emotional stress. No change with deep inspiration or exercise, no syncope, no SOB. Vitals, heart/ lung sounds, and EKG are all normal. Order an echo or stop testing for CAD?

A

Stop testing for CAD

  • She’s a low-risk patient (no cardiac risk factors), so even if we did a stress test and it came back positive, it would likely be a false positive
  • A transthoracic echo (TTE) would test for wall motion abnormalities during chest pain/ stress test or valvular abnormalities (like aortic stenosis) in a patient with exertional chest pain and a murmur
How well did you know this?
1
Not at all
2
3
4
5
Perfectly
2
Q

Old guy has problems with his central vision. Peripheral vision is good. History of cataracts, but otherwise healthy. Diagnosis?

A

Age-related macular degeneration (AMD)

*This is seen in patients >50 y.o.
Results from degeneration and atrophy of the central retina (macula) and surrounding structures.

**Macula= center of retina (the light-sensitive tissue that lines the inside of the eye and allows us to see in color)

How well did you know this?
1
Not at all
2
3
4
5
Perfectly
3
Q

Lady with stabbing pain on cheek when lightly touched, radiates from ear to jaw line. Treatment?

A

Carbamazepine (if this causes adverse effects: Oxcarbazepine, Baclofen, or surgical decompression of the trigeminal nerve)

This is trigeminal neuralgia (neuropathic pain along the V2/ maxillary and V3/ mandibular branches of the trigeminal nerve)

How well did you know this?
1
Not at all
2
3
4
5
Perfectly
4
Q

Monitor what in a patient on Carbamazepine?

A

Blood count

Carbamazepine can cause leukopenia (low WBC count) and aplastic anemia

How well did you know this?
1
Not at all
2
3
4
5
Perfectly
5
Q

What does chemosis mean?

A

Swelling/ edema of the conjunctiva

How well did you know this?
1
Not at all
2
3
4
5
Perfectly
6
Q

Normal platelet count?

A

150,000- 400,000

How well did you know this?
1
Not at all
2
3
4
5
Perfectly
7
Q

Guy traveled to Mexico. Came back with abdominal pain, vomiting, and diarrhea. These symptoms went away, but then the guy got a fever, swelling around the eyes, double vision, and splinter hemorrhages. Has eosinophilia and elevated creatine kinase. Diagnosis?

A

Trichinellosis (aka Trichinosis) caused by the roundworm Trichinella (“porky trickster”)

Eat undercooked pork with cysts-> gastric acid releases larvae from cysts (intestinal stage)-> female worms release new larvae a few weeks later that migrate into striated muscle (muscle stage)

  • most common presentation: eosinophilia + periorbital edema + myositis (inflamed, weak muscles)
  • *since the larvae migrate, patients may get systemic symptoms like fever and splinter hemorrhages
How well did you know this?
1
Not at all
2
3
4
5
Perfectly
8
Q

Dengue fever usually presents with pain where?

A

Behind the eye (retro-orbital pain)

How well did you know this?
1
Not at all
2
3
4
5
Perfectly
9
Q

Why might shistocytes/ helmet cells show up on blood smear in a patient with a replaced heart valve?

A

Prosthetic heart valves can destroy RBCs that cross the valve-> schistocytes/ helmet cells (fragmented RBCs)

How well did you know this?
1
Not at all
2
3
4
5
Perfectly
10
Q

Is haptoglobin increased or decreased in hemolytic anemia?

A

Decreased

Haptoglobin binds up free hemoglobin (Hb). In hemolytic anemia, there’s a lot of broken up RBCs, so there’s a lot of free Hb for haptoglobin to pick up. Since haptoglobin is getting used up, it is decreased.

How well did you know this?
1
Not at all
2
3
4
5
Perfectly
11
Q

Meaning of conjunctival pallor?

A

Pale conjunctiva

How well did you know this?
1
Not at all
2
3
4
5
Perfectly
12
Q

Patient has macrocytic anemia, shiny tongue, SOB on exertion, and depigmented areas over the arm suggestive of vitiligo. Cause of the anemia?

A

Pernicious anemia (antibodies against intrinsic factor)-> vitamin B12 deficiency

*Macrocytic= B12 or folate deficiency. Glossitis is seen in both these, since these deficiencies impair DNA synthesis (and therefore impair epithelial replication). Folate deficiency is usually seen in alcoholics or malnourished. Having vitiligo makes pernicious anemia more likely- if you have 1 autoimmune dz you’re at greater risk for another since your immune system sucks

How well did you know this?
1
Not at all
2
3
4
5
Perfectly
13
Q

Patient went to the Caribbean and came back with fever, malaise, rash, lymphadenopathy, and polyarthralgias (pain in joints). Diagnosis?

A

Chikungunya fever

  • caused by the Aedes mosquito in the Caribbean
  • supportive treatment
How well did you know this?
1
Not at all
2
3
4
5
Perfectly
14
Q

Patient has aortic dissection (tearing chest pain) + orthopnea (SOB when lying down) to the point that he refuses to lay down. Most likely cause of the SOB?

A

Aortic regurg

-Aortic dissection can lead to aortic regurg if blood from the intimal tear extends to the aortic valve
-This can cause 4 symptoms:
1. Hypotension (blood is regurging backwards, so less blood is being pumped forwards)
2. Sudden onset worsening chest pain
3. Pulmonary edema
4. Orthopnea (SOB lying down)
(2-4 are due to: blood from aorta-> LV and backing up into lungs)

**aortic dissection can also cause cardiac tamponade (if blood from the intimal tear enters into the pericardium and restricts filling of the heart). This would NOT cause pulmonary edema.

How well did you know this?
1
Not at all
2
3
4
5
Perfectly
15
Q

Patient just took Bactrim for a UTI. She is on Phenytoin for seizures. Recently, she has had an unsteady gait and nystagmus. What’s going on?

A

Phenytoin toxicity (ataxia and nystagmus are side effects of Phenytoin)

-Bactrim (TMP-SMX)= P450 inhibitor, so it blocks P450 metabolism in the liver—> drug stays in body longer. Phenytoin working in the body longer—> toxicity.

How well did you know this?
1
Not at all
2
3
4
5
Perfectly
16
Q

40 y.o. Lady from India has episodes of upper abdominal pain and burning that waxes and wanes. Feels bloated after meals. Positive stool guaiac (occult blood in stool). Most likely diagnosis?

A

H. Pylori

*dyspepsia (indigestion/ heartburn) can be—> from NSAIDs, GERD, H Pylori, etc.
GERD wouldn’t cause blood in stool. Since she has blood in stool and from India (low-income), H. Pylori is most likely.

**diagnose with urea breath test or stool antigen test. Endoscopy is reserved for patients >55 y.o. Or those with alarm symptoms (weight loss, bleeding, anemia, dysphasia, persistent vomiting).

How well did you know this?
1
Not at all
2
3
4
5
Perfectly
17
Q

What is dyspepsia?

A

Indigestion/ heartburn

How well did you know this?
1
Not at all
2
3
4
5
Perfectly
18
Q

What is bradyarrhythimia?

A

Bradycardia <60 bpm

How well did you know this?
1
Not at all
2
3
4
5
Perfectly
19
Q

TIA vs. stroke?

A

TIA (transient ischemic attack or “mini stroke”)- blockage is temporary and blood flow returns on its own. Stroke symptoms go away on their own

Stroke- “brain attack” where blockage is permanent

How well did you know this?
1
Not at all
2
3
4
5
Perfectly
20
Q

Lady with a hx of MS and hyperlipidemia has speech arrest and right arm weakness for <30 minutes. What are 2 meds we should give her?

A

Aspirin (ASA) and a statin

Why? She had a TIA (classic stroke symptoms for <24 hrs that resolved on their own). The treatment for TIA is to address the risk factors- so give aspirin (to prevent platelet aggregation and stroke) and give a statin (to lower her cholesterol).

**This presentation is NOT an MS exacerbation (MS attacks last for days or weeks). If it were, we could treat the flare up with glucocorticoids, immunoglobulins, or plasma exchange therapy (since MS is an autoimmune condition and we want to decrease the immune system’s damage).

How well did you know this?
1
Not at all
2
3
4
5
Perfectly
21
Q

Women has burning upper abdominal pain, constipation, and blood in stool. Calcium is high, phosphorus is low. What syndrome does she likely have?

A
MEN 1 (multiple endocrine neoplasia type 1)= “pans of pitted pears” 
-pancreatic, pituitary, and parathyroid tumors
  • Has a pancreatic tumor (ZE syndrome)—> burning upper abdominal pain and GI bleed (tumor can invade into duodenum).
  • Has a parathyroid tumor—> release PTH (primary hyperparathyroidism), which increases calcium absorption and phosphorus wasting.
  • That’s 2/3, so MEN 1 is likely and we would want to check for a pituitary tumor also.
How well did you know this?
1
Not at all
2
3
4
5
Perfectly
22
Q

Sarcoidosis effect on calcium levels?

A

Can cause hypERcalcemia

Due to increased conversion to 1,25 active vitamin D—> more calcium and phosphorous reabsorption

How well did you know this?
1
Not at all
2
3
4
5
Perfectly
23
Q

What is Milk-alkali syndrome?

A

Hypercalcemia + metabolic alkalosis + AKI due to taking too much calcium (can be from taking too many Tums pills…since Tums= calcium carbonate)

**too much calcium has multiple effects on the kidneys and ultimately causes diuresis and stimulates bicarb reabsorption-> met alkalosis

How well did you know this?
1
Not at all
2
3
4
5
Perfectly
24
Q

What is a plantar reflex?

A

Babinski

How well did you know this?
1
Not at all
2
3
4
5
Perfectly
25
Q

Old guy with memory loss and muscle spasms when startled. He also has nystagmus, hypokinesia, and positive Babinski (plantar reflex). EEG shows periodic sharp-wave complexes. Diagnosis?

A

Creutzfeldt-Jakob disease (CJD)

  • prion disease (usually sporadic, but can be due to contaminated corneal transplants).
  • key findings: startle response and sharp waves on EEG
  • *as it progresses, patients may lose ability to move and speak (txt is supportive)
How well did you know this?
1
Not at all
2
3
4
5
Perfectly
26
Q

Lady has temporal (giant cell) arteritis and gets treated. 6 months later her muscles are weak and she has trouble going up stairs or standing from a chair. Most likely cause?

A

Drug-induced myopathy

-glucocorticoids (used to treat giant cell arteritis) can cause myopathy (think of Cushing’s secondary to too much glucocorticoid use…skin thinning, fat redistribution, and muscle thinning!)

How well did you know this?
1
Not at all
2
3
4
5
Perfectly
27
Q

What are the pneumonia vaccine guidelines for seniors 65+?

A

One 13-valent pneumococcal conjugate vaccine (PCV13) +

One 23-valent pneumococcal polysaccharide vaccine (PPSV23) at least 6-12 months later

*give add’l PCV13 and PPSV23 vaccines to high-risk patients (CSF leaks, sickle cell, cochlear implants, asplenia/ immunocompromised)

**give one PPSV23 before age 65 too for some patients (smokers or chronic medical problems like heart/ lung/ liver dz or DM)

How well did you know this?
1
Not at all
2
3
4
5
Perfectly
28
Q

What’s a normal pupil size?

A

2-4 mm in light

How well did you know this?
1
Not at all
2
3
4
5
Perfectly
29
Q

How would Phenytoin toxicity present?

A

Nystagmus, ataxia, confusion

How well did you know this?
1
Not at all
2
3
4
5
Perfectly
30
Q

Decreased sensation over 4th and 5th digits and weak grip. What nerve is probably damaged and where?

A

Ulnar nerve
At the elbow (medial epicondylar groove)

*ulnar nerve syndrome can be caused by prolonged compression on the ulnar nerve from leaning on the elbows while working at your desk

How well did you know this?
1
Not at all
2
3
4
5
Perfectly
31
Q

Incidence vs. prevalence?

A

Incidence= # new cases

Prevalence= total cases

How well did you know this?
1
Not at all
2
3
4
5
Perfectly
32
Q

If a man abuses steroids (testosterone), will his levels of GnRH, LH, and FSH be high or low?

A

All low

-the exogenous testosterone will feedback and inhibit GnRH, LH, and FSH production (-> low endogenous testosterone, small testes, possible infertility)

How well did you know this?
1
Not at all
2
3
4
5
Perfectly
33
Q

After treatment with Dapsone, a patient developed fatigue and dark urine. Why?

A

G6PD deficiency

  • in this condition, RBCs can’t make enough NADPH to protect against oxidative injury. Some meds (Dapsone, TMP/SMX, Primaquine), foods (Fava beans), and infections cause oxidative damage and bring on anemia attacks in patients with this
  • *hemolytic anemia-> blood in urine (dark urine)
How well did you know this?
1
Not at all
2
3
4
5
Perfectly
34
Q

What type of drug is Enoxaparin?

A

Low-molecular weight heparin (LMWH)

How well did you know this?
1
Not at all
2
3
4
5
Perfectly
35
Q

Patient has a DVT. Is treated with Enoxaparin (LMWH). Platelet count goes down. Why?

A

This is heparin-induced thrombocytopenia (HIT)

  • bad reaction to heparin where heparin binds to platelet factor 4 (PF4) and antibodies form against the hep-PF4 complex. These antibodies attack platelets—> thrombocytopenia (low platelets) and stick them together—> clotting (opposite of what you’d expect when giving an anti-coagulant).
  • discontinue the hep immediately and pick another anticoagulant to treat with!
How well did you know this?
1
Not at all
2
3
4
5
Perfectly
36
Q

Reg chest X-rays, what does “bilateral basilar lucency” mean?

A

Lucency= shows up dark on x-ray
-means it’s less dense (like air-filled lungs in emphysema)—> more X-rays pass through

Basilar= bottom of lungs (opposite of apex, which is the pointy top of the lungs)

So…”bilateral basilar lucency” means the bottom of both lungs appears darker (air-filled) on X-ray

How well did you know this?
1
Not at all
2
3
4
5
Perfectly
37
Q

38 y.o. man with SOB and cough, worse on exertion. Smoked 2 years, otherwise no medical hx. Breath sounds are decreased on the bottom of both lungs. Diagnosis?

A

Alpha-1 antitrypsin deficiency—> panacinar emphysema (destruction of lower lobes *vs. emphysema due to smoking alone would be centriacinar and cause destruction of upper lobes “smoke rises”)

How well did you know this?
1
Not at all
2
3
4
5
Perfectly
38
Q

How do you diagnose Alpha-1 antitrypsin (AAT) deficiency (besides going off the history and physical)?

A

Measure serum AAT levels (should get pulmonary function tests too)

How well did you know this?
1
Not at all
2
3
4
5
Perfectly
39
Q

What is nephrolithiasis?

A

Kidney stones

How well did you know this?
1
Not at all
2
3
4
5
Perfectly
40
Q

Alcoholic is having seizures. Cr is 2.4. RBCs are found in his urine. Diagnosis?

A

Rhabdomyolysis

*seizures-> skeletal muscles overexertion-> release of myoglobin into the blood-> clogs up renal tubules and causes renal injury

How well did you know this?
1
Not at all
2
3
4
5
Perfectly
41
Q

Normal Cr? Normal BUN? Normal BUN:Cr ratio?

A

Cr- 0.6-1.2
BUN- 7-25

BUN/CR- around 15

How well did you know this?
1
Not at all
2
3
4
5
Perfectly
42
Q

Normal calcium levels?

A

8.5-10ish

How well did you know this?
1
Not at all
2
3
4
5
Perfectly
43
Q

Normal potassium (K+) level?

A

3.5-5.0

How well did you know this?
1
Not at all
2
3
4
5
Perfectly
44
Q

Normal Na+ level?

A

135-145

How well did you know this?
1
Not at all
2
3
4
5
Perfectly
45
Q

Paraneoplastic syndromes in small cell lung CA (4) vs. squamous cell lung CA (1)?

A

Small cell:

  1. SIADH
  2. Lambert-Eaton
  3. ACTH (Cushing’s)
  4. Anti-Hu antibodies

Squamous cell:
1. PTHrP (-> hypercalcemia)

How well did you know this?
1
Not at all
2
3
4
5
Perfectly
46
Q

Former smoker has a lung mass and calcium level of 14.5. Why is calcium at this level?

A

Calcium is high (normal would be 8.5-10ish…14 is very high)

This is due to squamous cell carcinoma of the lung-> PTHrP (paraneoplastic syndrome)-> excess calcium reabsorption (and phosphorous wasting)
-This is also known as “hypercalcemia of malignancy”

How well did you know this?
1
Not at all
2
3
4
5
Perfectly
47
Q

How do we treat hypercalcemia in the short-term? Long-term?

A

Short term: Give normal saline (and calcitonin)

  • why? Hypercalcemia-induced DI (high calcium interferes with renal channels that allow ADH to work to retain water)-> hypercalcemia patients pee a lot and are volume depleted, so you have to restore volume/ hydrate
  • may also give Calcitonin (directly blocks osteoC’s from breaking down bone and releasing calcium-> so, they lower calcium levels in the blood)
  • *don’t give loop diuretics (even though they can lower calcium levels) unless they have HF (not worth the side effect of worsening volume depletion)

Long term: Give bisphosphonates

*why? Bisphosphonates stimulate apoptosis of osteoClasts (they stop osteoC’s from breaking down bone and releasing calcium-> so, they lower calcium levels in the blood)

How well did you know this?
1
Not at all
2
3
4
5
Perfectly
48
Q

Normal platelet count?

A

150,000- 400,000

How well did you know this?
1
Not at all
2
3
4
5
Perfectly
49
Q

Normal Hb in male? Female?

A

Normal Hb in males= 13.5- 17.5

Normal Hb in females= 12-16
a little lower due to bleeding once a month/ menstrual cycles

How well did you know this?
1
Not at all
2
3
4
5
Perfectly
50
Q

Treatment for botulism?

A

Horse-derived (equine) antitoxin therapy

How well did you know this?
1
Not at all
2
3
4
5
Perfectly
51
Q

Treatment for Guillain-Barre?

A

Plasmaphoresis (therapeutic plasma exchange)

*GB occurs when the immune system attacks nerves (can be after Campylobacter or a flu shot…)

How well did you know this?
1
Not at all
2
3
4
5
Perfectly
52
Q

What is antiphospholipid antibody syndrome?

A

The presence of antiphospholipid antibodies + you have blood clots (DVT, PE, stroke, or MI) or pregnancy problems (miscarriages or premature birth from placental insufficiency or preeclampsia)

**Antiphospholipid antibodies are seen in SLE (lupus) (though not specific to it). They can cause 3 problems: (1) antiphospholipid antibody syndrome, (2) increased PTT, (3) false positive syphilis test (RPR/ VDRL)

How well did you know this?
1
Not at all
2
3
4
5
Perfectly
53
Q

What is spontaneous bacterial peritonitis (SBP)?

A

A bacterial infection of the ascitic fluid of the peritoneum
(So, it’s when a patient has ascites, usually secondary to cirrhosis, and that fluid gets infected)

How well did you know this?
1
Not at all
2
3
4
5
Perfectly
54
Q

Man with hx of cirrhosis secondary to chronic hep C presented with confusion. He was found to have hepatic encephalopathy and spontaneous bacterial peritonitis (infection of ascitic fluid). He was given Lactulose and Cefotaximine. Since admission and all of this, his Creatinine has gone up. Most likely cause?

A

Hepatorenal syndrome

How well did you know this?
1
Not at all
2
3
4
5
Perfectly
55
Q

Most likely diagnosis in a Mississippi patient who was initially thought to have sarcoidosis (cough, hilar lymphadenopathy, erythema nodosum, and non-caseating granulomas)?

A

Histoplasmosis

*Blasto is also seen in the Mississippi region, but usually does not involve hilar lymphadenopathy

How well did you know this?
1
Not at all
2
3
4
5
Perfectly
56
Q

Where are these fungi found?

  1. Histoplasmosis
  2. Blastomycoses
  3. Coccidioides
  4. Paracoccidioides
A
  1. Histoplasmosis- Mississippi/ Ohio river valley (Midwest)
  2. Blastomycoses- Great Lakes/ Ohio river valley (Midwest)
  3. Coccidioides- Southern US (Cali, Arizona, northern Mexico)
  4. Paracoccidioides- South America, Brazil
How well did you know this?
1
Not at all
2
3
4
5
Perfectly
57
Q

Guy has a hx of IV drug abuse, endocarditis, and stroke (made him wheelchair-bound). Presents with right calf pain and swelling. On physical exam, there is hepatosplenomegaly. Abdomen is distended with shifting dullness suggestive of ascites. Most likely cause of the ascites?

A

Chronic liver disease

  • IV drug use= high-risk for hep C w/ chronic liver dz (cirrhosis)—> ascites (the portal HTN causes fluid to accumulate in the peritoneum)
  • IV drug use also—> endocarditis with embolic stroke. Stasis (wheelchair-bound) following the stroke—> DVT. But DON’T be distracted by everything going on w/ this patient! The question is just asking why does he have ascites? 80% of ascites is due to liver failure and you can trace this to his IV drug abuse.
How well did you know this?
1
Not at all
2
3
4
5
Perfectly
58
Q

Man with hx of IV drug abuse has dark urine and jaundice. Total bilirubin, direct (conjugated) bilirubin, alk phos, and AST are all high. Abdomen is not tender. Next best step to diagnose?

A

Ultrasound the abdomen

(High serum alk phos suggests cholestasis/ reduced bile flow, possibly due to an obstruction/ stone. U/S will help you look for intrahepatic or extrahepatic causes of biliary obstruction.)

How well did you know this?
1
Not at all
2
3
4
5
Perfectly
59
Q

50 year old lady with hx of GERD. Has episodes of hands turning bluish in the cold. Her hands are puffy and lungs have crackles. Diagnosis? Associated with what 2 antibodies?

A

Systemic sclerosis (diffuse type)

  1. anti-Scl-70 antibody (aka anti-DNA topoisomerase-1 antibody)
  2. anti-RNA polymerase III

*Systemic sclerosis= autoimmunity + non inflammatory vasculopathy + collagen deposition w/ fibrosis.
2 sub-types: (1) diffuse (involves the skin all over and early visceral involvement- interstitial lung dz), (2) limited aka CREST syndrome (face and fingers)

**CREST= Calcinosis (calcium deposits) and anti-Centromere antibody
Raynaud’ s phenomenon
Esophageal dysmotility (GERD0
Sclerodactyly (thickened skin on hands and feet)
Telangiectasia (dilated blood vessels)

How well did you know this?
1
Not at all
2
3
4
5
Perfectly
60
Q

What does CREST syndrome stand for?

A

Calcinosis (calcium deposits) and anti-Centromere antibody
Raynaud’ s phenomenon
Esophageal dysmotility (GERD0
Sclerodactyly (thickened skin on hands and feet)
Telangiectasia (dilated blood vessels)

How well did you know this?
1
Not at all
2
3
4
5
Perfectly
61
Q

What is the drug Memantine used for?

A

Severe Alzheimer’s disease

  • it blocks NMDA glutamate receptors
  • may improve cognitive symptoms
How well did you know this?
1
Not at all
2
3
4
5
Perfectly
62
Q

Lady just flew home from Central Asia. Has SOB, chest pain, and hemoptysis. HR is 106, RR is 28. Most likely diagnosis?

A

Pulmonary embolism (PE)

  • She was on a long flight, has SOB, and is coughing up blood. Red flags!
  • typical PE presentation= tachypnea (rapid breathing) and tachycardia in addition to SOB. This is because she has the pulm artery occluded-> dec oxygenation-> heart tries to compensate.
How well did you know this?
1
Not at all
2
3
4
5
Perfectly
63
Q

Patient is anxious, confused, has conjunctival injection. What substance?

A

Marijuana

(Conjunctival injection= bloodshot eyes)
*increases appetite, HR, breathing, causes dry mouth, and bloodshot eyes. Also slows reaction time-> car accidents.

How well did you know this?
1
Not at all
2
3
4
5
Perfectly
64
Q

What acid-base disturbance can AKI cause?

A

Non-nation gap metabolic acidosis

-from retention of uremia and impaired acid excretion (if you can’t pee out the acid, it accumulates in the blood)

How well did you know this?
1
Not at all
2
3
4
5
Perfectly
65
Q

Why might hypoventilation cause fatigue?

A

Breathing less= less oxygen being delivered to organs in the body to provide energy

How well did you know this?
1
Not at all
2
3
4
5
Perfectly
66
Q

In nephrogenic DI (like due to lithium), will sodium levels go up or down? Will blood or urine have a higher osmolarity?

A

Nephrogenic DI (not responding to ADH)-> not retaining water-> pee more-> [Na+] will go up-> HYPERNATREMIA (high Na+)

Blood will be more concentrated (HIGHER OSMOLARITY) than the urine (urine is more dilute since you’re peeing more)

How well did you know this?
1
Not at all
2
3
4
5
Perfectly
67
Q

How can sodium levels differentiate nephrogenic DI vs. psychogenic polydipsia?

A

Nephrogenic DI (not responding to ADH)-> not retaining water-> pee more-> [Na+] will go up-> HYPERNATREMIA (high Na+)

Psychogenic polydipsia (drinking water like a crazy person)-> you’re putting so much water into the body that it dilutes [Na+]-> HYPONATREMIA (low Na+)

How well did you know this?
1
Not at all
2
3
4
5
Perfectly
68
Q

What is the DASH diet?

A

DASH= Dietary Approaches to Stop Hypertension
-this is the diet you put HTN patients on
(Involves eating fruits, veggies, whole grains, low-fat, low salt, high potassium foods)

How well did you know this?
1
Not at all
2
3
4
5
Perfectly
69
Q

What is UP/Cr?

A

Urine protein over creatinine ratio

A way to estimate how much protein is being spilled in the urine per day

How well did you know this?
1
Not at all
2
3
4
5
Perfectly
70
Q

What is ankle-brachial index?

A

Ratio of BP at the ankle to BP in the upper arm (brachial)

  • If BP in the ankle is lower than in the arm, it suggests blocked arteries in the ankle due to peripheral vascular disease (PAD)
  • used to check when patient has claudication, dermal atrophy, and absent distal pulses
How well did you know this?
1
Not at all
2
3
4
5
Perfectly
71
Q

A patient had a previous MI complicated by LV systolic dysfunction (the heart isn’t pumping so well anymore-> ejection fraction <30%). What deadly condition is this patient at risk of?

A

Sudden cardiac death (SCD) due to ventricular arrhythmia

How well did you know this?
1
Not at all
2
3
4
5
Perfectly
72
Q

Guy is brought in by his GF for confusion. He moved into a mobile home yesterday and had headache, nausea, and dizziness. This morning she went over to check on him and found him confused in bed in his urine. What test will most likely diagnose him?

A

Measuring carboxyhemoglobin level

*this is likely carbon monoxide (CO) poisoning from an old heater. Hb binds CO instead of oxygen (CO has a greater affinity)—> carboxyhemoglobin. Less oxygen delivered to tissues (left shift).

How well did you know this?
1
Not at all
2
3
4
5
Perfectly
73
Q

What is Well’s criteria?

A

A calculator to assess the probability a patient has a PE (pulmonary embolism)

*if total score (based on clinical signs of DVT, previous DVT/ PE, tachycardia, recent surgery, hemoptysis/ cancer) is >4, a PE is likely

How well did you know this?
1
Not at all
2
3
4
5
Perfectly
74
Q

If PE is on your differential but is unlikely (based on Wells criteria), what test can you do to rule it out completely?

A

Check the D-dimer (make sure it’s less than 500)

  • D-dimer is sensitive, but not specific for PE. So, you get this to make sure you can rule out PE.
  • *If PE is likely (based on Wells criteria), don’t get a D-dimer…go straight to CT pulmonary angiogram (aka spiral CT) (or V/Q scan if you can’t do the pulmonary angiogram bc the patient has bad kidneys and can’t handle the contrast) to diagnose.
How well did you know this?
1
Not at all
2
3
4
5
Perfectly
75
Q

If PE is on your differential and is LIKELY (based on Wells criteria), what test do you do?

A

Get a CT pulmonary angiogram (aka spiral CT) to diagnose

Unless your patient has kidney problems (in which case they can’t handle the IV contrast needed to do the pulmonary angiogram)—> then get a V/W scan to diagnose

How well did you know this?
1
Not at all
2
3
4
5
Perfectly
76
Q

What number is an elevated D-dimer?

A

> 500

How well did you know this?
1
Not at all
2
3
4
5
Perfectly
77
Q

What antibiotic can you give as empiric treatment of traveler’s diarrhea (ETEC)?

A

Ciprofloxacin (short-course)

*if symptoms fail to improve, test for something else (ex: may be parasitic like Giardia and require Metronidazole)

How well did you know this?
1
Not at all
2
3
4
5
Perfectly
78
Q
35 year old guy with PMH of Bipolar (on Risperidone) comes in for fatigue, headache, and decreased libido. 
Testosterone- low
TSH- low
T4- low
Prolactin- high 
Most likely diagnosis?
A

Pituitary adenoma

  • anterior pituitary—> FLAT PiG (FSH, LH, ACTH, TSH, Prolactin, GnRH)
  • mass effect—> headache and visual disturbance. Production of most hormones will be decreased due to compression of normal pituitary cells. But, prolactin is high due to anatomic disruption of the dopamine brake onto prolactin.
  • *NOT Hashimotos/ hypothyroidism bc T4 would be low, but TSH would be high due to negative feedback.
  • *NOT Risperidone side effect bc that’s an anti-dopamine drug-> takes brake off prolactin-> high prolactin, but does not affect these other hormones.
How well did you know this?
1
Not at all
2
3
4
5
Perfectly
79
Q

What valvular problem is common in ankylosing spondylitis?

A

Aortic regurg

*Ankylosing spondylitis= a seronegative spondyloarthritis (FA pg 461- lack of Rheumatoid factor, axial skeleton involvement, HLA-B27) with spinal fusion—> bamboo spine. Patients can also get uveitis and aortic regurg (aortic inflammation-> aortic aneurysm-> pulls on valve-> regurg).

**These patients are at risk for vertebral fractures with minimal trauma! (Increased osteoclast activity in the setting of chronic inflammation)

How well did you know this?
1
Not at all
2
3
4
5
Perfectly
80
Q

A patient has a MI of the LAD. 4 days later, she is hypotensive with tachycardia. There are crackles over her lungs and her extremities are cold. EKG shows deep T wave inversion in leads V1-V5. Most likely cause of her deterioration?

A

Interventricular septum rupture (complication 3-5 days post-MI)

-> cardiogenic shock (hypotension, pulm edema, confusion due to poor pumping and poor organ perfusion). Cold extremities due to shunting of blood to core vital organs.

How well did you know this?
1
Not at all
2
3
4
5
Perfectly
81
Q

Male patient undergoing chemo for Seminoma comes to the ED for fever and chills, nothing else going on. He has mucosal pallor (pale skin), low Hb and Hct, and low leukocytes and neutrophils. Diagnosis and treatment?

A

Febrile neutropenia (fever + low neutrophils, specifically absolute neutrophil count <1500)

Treat with an anti-pseudomonas beta-lactam agent (these bugs are often responsible)—
Cefepime (4th gen cephalosporin), Meropenem (carbapenem), or Piperacillin-Tazobactam (extended-spectrum penicillin)

*Chemo patients are at higher risk for this and you need to give emergent empiric antibiotics to prevent progression to sepsis!

How well did you know this?
1
Not at all
2
3
4
5
Perfectly
82
Q

Patient with PMH of Genital herpes comes in with scaley rashes looking like psoriasis. What screening test should you offer him?

A

HIV test

*HIV is associated with flares of psoriasis. Other things that can worsen psoriasis— skin trauma, withdrawal from glucocorticoids, and certain meds (anti-malaria, indomethacin, propranolol)

How well did you know this?
1
Not at all
2
3
4
5
Perfectly
83
Q

AIDS patients are at greatest risk for a CMV (cytomegalovirus) infection with a CD4 count less than what?

A

CD4 < 50

“Charity drive 50 cents”

How well did you know this?
1
Not at all
2
3
4
5
Perfectly
84
Q

Treatment for CMV esophagitis (AIDS patient with CD4 <50 with linear ulcerations)?

A

Ganciclovir

*Herpes Simplex esophagitis (well-circumscribed, round ulcers as opposed to linear ulcers in CMV) is treated with Acyclovir

How well did you know this?
1
Not at all
2
3
4
5
Perfectly
85
Q

HIV patient has a CD4 count of 300. She has a lobular pneumonia, diagnosed by symptoms and chest X-ray. What is the most likely organism responsible for the pneumonia?

A

Strep pneumo

*Don’t assume it’s PCP pneumonia just bc she has HIV! That is seen in HIV patients with CD4 <200. Still, step pneumo is the no. 1 cause of lobular CAP!

How well did you know this?
1
Not at all
2
3
4
5
Perfectly
86
Q

AIDS patient with CD4 <50 and bloody diarrhea. Most likely cause?

A

Cytomegalovirus (CMV colitis)

  • common causes of diarrhea in HIV patients:
    1) Cryptosporidium—CD4 <180 (severe watery diarrhea, low-grade fever)
    2) Microsporidium—CD4 <100 (crampy watery diarrhea, usually no fever)
    3) MAC—CD4 <100 (watery diarrhea and high fever)
    4) CMV—CD4 <50 (bloody diarrhea, abdominal pain, low-grade fever)
How well did you know this?
1
Not at all
2
3
4
5
Perfectly
87
Q

HIV patient has had a cough for 2 months, no hemoptysis. Was hospitalized 6 months ago for seizures. CXR shows cavitation in the right upper lobe. Most likely diagnosis?

A

Reactivation TB (affects upper lobes)

  • not likely to be aspiration pneumonia bc that is found in lower lobes (due to gravity)
  • you do not HAVE TO HAVE hemoptysis for it to be TB!!! Also, note this guy’s HIV status puts him at increased risk for TB
How well did you know this?
1
Not at all
2
3
4
5
Perfectly
88
Q

What the heck is Strep Sanguinus?

A

It’s in the family of Step Viridans

*think of it when there is a patient with a heart problem who gets a dental procedure done!

How well did you know this?
1
Not at all
2
3
4
5
Perfectly
89
Q

Lady has painful swelling of the left face. There is a raised red rash with sharply-demarcated borders including the left ear. She also have fever + chills. What bug is responsible?

A

Group A strep (Strep Pyogenes)

This is Erysipelas (superficial skin infection of the upper dermis *like cellulitis but more superficial)

How well did you know this?
1
Not at all
2
3
4
5
Perfectly
90
Q

When is Toxoplasma prophylaxis indicated?

A

In HIV patients with CD4 <100

How well did you know this?
1
Not at all
2
3
4
5
Perfectly
91
Q

When is CMV prophylaxis indicated?

A

In some organ transplant recipients

How well did you know this?
1
Not at all
2
3
4
5
Perfectly
92
Q

HIV patient not taking his meds. Has had weight loss, fever, night sweats, cough, and SOB for 3 weeks. There are small ulcers on his hard palate, enlarged lymph nodes, and lung crackles. What test should you order?

A

Urine Histoplasma antigen
(This sounds like disseminated histoplasmosis)

*treat with Amp B

How well did you know this?
1
Not at all
2
3
4
5
Perfectly
93
Q

What is the most appropriate empiric antibiotic to start an endocarditis patient on with a hx of IV drug abuse?

A

Vancomycin

-it is broad spectrum and has MRSA coverage

How well did you know this?
1
Not at all
2
3
4
5
Perfectly
94
Q

Wisconsin guy has fever, night sweats, productive cough, and weight loss. Also has crusty skin lesions, lytic lesions on ribs, and consolidation on his left upper lung. Diagnosis?

A

Blastomycosis

*seen in the Mississippi/ Ohio river valley (Wisconsin has the highest infection rate)

How well did you know this?
1
Not at all
2
3
4
5
Perfectly
95
Q

Teenager has episodes of “chest fluttering.” BP is normal, but pulse is 210. EKG shows narrow complex tachycardia. What diagnosis should be on the differential?

A

Wolff-Parkinson-White syndrome

-going through an accessory “secret pathway” (bundle of Kent) that bypasses the rate-slowing AV node-> ventricles get depolarized before they should (shows up on EKG as delta waves).

How well did you know this?
1
Not at all
2
3
4
5
Perfectly
96
Q

What is long QT syndrome?

A

When you have a long QT interval (QT= ventricular depolarization + depolarization, so the entire systole)

  • > 450 mace in males or >470 mace in females
  • can be congenital or acquired
  • risk for sudden cardiac death (due to Torsades or polymorphic ventricular tachycardia)
How well did you know this?
1
Not at all
2
3
4
5
Perfectly
97
Q

What is micturition?

A

Urination

How well did you know this?
1
Not at all
2
3
4
5
Perfectly
98
Q

What is an electrocardiogram?

A

ECG! (Same thing as EKG)

How well did you know this?
1
Not at all
2
3
4
5
Perfectly
99
Q

Old guy felt like he was going to pass out while running. He had a similar episode before. You did a physical exam, significant for a systolic ejection murmur at the right 2nd intercostal space. You order an EKG, which showed LV hypertrophy. Next step?

A

Order an echocardiogram

This guy most likely has aortic stenosis (exertional syncope, systolic murmur, LV hypertrophy). Get an echo any time you suspect a structural/ valvular heart disease (aortic stenosis, HOCM, LV dysfunction, cardiac tamponade, etc.)
*do NOT stress test symptomatic severe aortic stenosis

How well did you know this?
1
Not at all
2
3
4
5
Perfectly
100
Q

20 year old lady with episodes of sharp chest pain for 3 weeks. Has systolic murmur at the apex that shortens with squatting. Diagnosis?

A

Mitral valve prolapse

*Apex= mitral region
Systolic murmur= MVP or mitral regurg
Squatting= increased preload (kink veins in legs-> more ‘milking’ of blood to heart)
Murmur is softer with squatting= MVP (more preload helps the valve to be more crisp as opposed to floppy where it doesn’t align well)

**Do an echo to confirm the diagnosis. Symptoms are benign.

How well did you know this?
1
Not at all
2
3
4
5
Perfectly
101
Q

Guy comes in with sudden onset palpitations. He is found to be in a-fib. BP is 112/70 and O2 sat is 92. He is full code.
He suddenly becomes unresponsive. Cardiac monitor still shows a-fib, but there’s no palpable pulse and he has agonal breathing (gasping). What do you do next?

A

Chest compressions

  • The heart has an abnormal rhythm going on, but it is not effectively pumping blood out to the extremities, so you can’t feel a pulse (this is called pulseless electrical activity).
  • ACLS guidelines say do CPR and give epi (beta>alpha agonist, so helps improve contractility and is a vasopressor-> vasoconstricts to improve cerebral and coronary perfusion).

*Although we should cardiovert new a-fib patients (had it <48 hrs) or a-fib patients who are unstable, if they are so unstable they go into a non-perfusing rhythm, you follow ACLS guidelines!

How well did you know this?
1
Not at all
2
3
4
5
Perfectly
102
Q

Asystole vs. Pulseless electrical activity (PEA)?

A

Asystole- Cardiac arrest
No electrical activity
Monitor is flat-lined

Pulseless electrical activity (PEA)- A non-perfusing rhythm
There is an abnormal heart rhythm (ex: a-fib), but the heart isn’t effectively pumping blood out, so there’s no palpable pulse (or measurable BP)
Monitor shows the abnormal rhythm

**NOTE: 3 rhythms cannot be PEA: (1) V-fib, (2) V-tach, (3) Asystole. Why? Bc we don’t expect a pulse with these rhythms. PEA is an organized electrical activity where we expect to see a pulse but we don’t have one.

How well did you know this?
1
Not at all
2
3
4
5
Perfectly
103
Q

What does ACLS stand for? What does CPR stand for?

A

ACLS= Advanced Cardiac Life Support

(guidelines on what to do if a patient is in cardiac arrest)

CPR= Cardiopulmonary Resuscitation

How well did you know this?
1
Not at all
2
3
4
5
Perfectly
104
Q

What are the reversible causes of asystole/ pulseless electrical activity? “5 H’s and 5 T’s”

(You want to consider these when doing chest compressions and giving epi…if you figure out the source of the problem, you may be able to correct the heart’s rhythm and save the patient’s life!)

A
Hypovolemia
Hypoxia
Hydrogen ions (acidosis) 
HypOkalemia/ HyPERkalemia
Hypothermia
Tension pneumothorax
Tamponade, cardiac
Toxins (narcotics, benzos) 
Thrombosis (pulmonary or coronary) 
Trauma
How well did you know this?
1
Not at all
2
3
4
5
Perfectly
105
Q

Can you shock a patient in Asystole?

A

NO

Asystole= they are flat-lined. There is ZERO electrical activity in the heart, so shocking them will not work (shocks work by feeding off of electrical activity). Do chest compressions, give epi (every 3-5 min), and try to figure out why the patient went into asystole bc there may be a reversible cause.

How well did you know this?
1
Not at all
2
3
4
5
Perfectly
106
Q

2 shockable rhythms? 2 non-shockable rhythms?

A

SHOCK:

  1. V-fib (ventricular fibrillation)
  2. V-tach (ventricular tachycardia)
    * *note: Torsades is a subcategory of ventricular tachycardia- shock it.

NON-SHOCKABLE:

  1. Asystole
  2. Pulseless electrical activity (PEA)

Rap song: “Defib for V-fib and pulseless V-tach. Don’t defib asystole, you won’t get them back!”

How well did you know this?
1
Not at all
2
3
4
5
Perfectly
107
Q

Define orthostatic hypotension.

A

Decrease in BP by 20 systolic or 10 diastolic when standing

*may be accompanied by an increase in HR

How well did you know this?
1
Not at all
2
3
4
5
Perfectly
108
Q

Why are old people more likely to get orthostatic hypotension (big drop in BP when they stand, making them momentarily lightheaded)?

A

Aging-> some baroreceptor activity is lost

*normally: you stand-> gravity is working against you so vessels have to constrict in the lower extremities to ‘milk’ blood up to the heart so it can pump enough to maintain BP. For the moment that BP is dropped right when you stand, baroreceptors (on carotid sinus) respond to decreased stretch-> decrease firing-> this increases sympathetic stimulation to raise the BP to baseline.

How well did you know this?
1
Not at all
2
3
4
5
Perfectly
109
Q

Young man faints when exercising. Has a crescendo-decrescendo systolic murmur in the left sternal border. Diagnosis?

A

HOCM

Basically a functional aortic stenosis, since the intraventricular septum is in the way of aortic outflow

How well did you know this?
1
Not at all
2
3
4
5
Perfectly
110
Q

IV drug user had an episode of syncope. Has a diastolic murmur at the left sternal border. Platelet count is elevated and EKG shows 2nd degree AV block. Diagnosis?

A

Perivalvular abscess

*Be suspicious of perivalvular abscess anytime a patient has endocarditis + conduction abnormalities on EKG (the abscess can extend into the conduction pathway, messing it up). Endocarditis alone wouldn’t cause AV block or syncope.

How well did you know this?
1
Not at all
2
3
4
5
Perfectly
111
Q

Man comes in for palpitations and SOB on exertion. He recently binge drank. HR is 130. EKG shows no clear P waves. Echo shows EF=35%, mitral regurg, and dilated LA and LV with global hypokinesis. What’s going on?

A

Alcohol-> A-fib w/ RVR (rapid ventricular response)-> tachycardia-mediated cardiomyopathy (the prolonged arrhythmia and rapid ventricular rate is causing HF)

*treat with aggressive rate and rhythm control

**note: the mitral regurg in this patient is just due to the fact that the ventricles are overloaded

How well did you know this?
1
Not at all
2
3
4
5
Perfectly
112
Q

What is tachycardia-mediated cardiomyopathy? How do you treat it?

A

When you have a tachyarrhythmia (heart is beating super fast, out of rhythm for a while) that leads to HF
(*can be due to a-fib, a-flutter, v-tach, etc.)

Treat with aggressive rate or rhythm control
(*AV nodal blocking agents, antiarrhtymic drugs, ablation of arrhythmia, etc.)

How well did you know this?
1
Not at all
2
3
4
5
Perfectly
113
Q

37 year old lady who immigrated from Cambodia presents with stroke symptoms. Over the past several months, she has had palpitations, SOB on exertion, and hemoptysis. Most likely explanation?

A

Rheumatic heart disease-> mitral stenosis-> a-fib-> stroke

*remember that mitral stenosis (MS) over time-> increased LA pressure. This predisposes to a-fib. This also leads to pulmonary vascular congestion, explaining the hemoptysis.

How well did you know this?
1
Not at all
2
3
4
5
Perfectly
114
Q

What is “flash pulmonary edema?”

A

Acute pulmonary edema secondary to increased cardiac filling pressures (such as after an MI)

How well did you know this?
1
Not at all
2
3
4
5
Perfectly
115
Q

What is micturition?

A

Urination

How well did you know this?
1
Not at all
2
3
4
5
Perfectly
116
Q

What is an electrocardiogram?

A

ECG! (Same thing as EKG)

How well did you know this?
1
Not at all
2
3
4
5
Perfectly
117
Q

Old guy felt like he was going to pass out while running. He had a similar episode before. You did a physical exam, significant for a systolic ejection murmur at the right 2nd intercostal space. You order an EKG, which showed LV hypertrophy. Next step?

A

Order an echocardiogram

This guy most likely has aortic stenosis (exertional syncope, systolic murmur, LV hypertrophy). Get an echo any time you suspect a structural/ valvular heart disease (aortic stenosis, HOCM, LV dysfunction, cardiac tamponade, etc.)
*do NOT stress test symptomatic severe aortic stenosis

How well did you know this?
1
Not at all
2
3
4
5
Perfectly
118
Q

20 year old lady with episodes of sharp chest pain for 3 weeks. Has systolic murmur at the apex that shortens with squatting. Diagnosis?

A

Mitral valve prolapse

*Apex= mitral region
Systolic murmur= MVP or mitral regurg
Squatting= increased preload (kink veins in legs-> more ‘milking’ of blood to heart)
Murmur is softer with squatting= MVP (more preload helps the valve to be more crisp as opposed to floppy where it doesn’t align well)

**Do an echo to confirm the diagnosis. Symptoms are benign.

How well did you know this?
1
Not at all
2
3
4
5
Perfectly
119
Q

Guy comes in with sudden onset palpitations. He is found to be in a-fib. BP is 112/70 and O2 sat is 92. He is full code.
He suddenly becomes unresponsive. Cardiac monitor still shows a-fib, but there’s no palpable pulse and he has agonal breathing (gasping). What do you do next?

A

Chest compressions

  • The heart has an abnormal rhythm going on, but it is not effectively pumping blood out to the extremities, so you can’t feel a pulse (this is called pulseless electrical activity).
  • ACLS guidelines say do CPR and give epi (beta>alpha agonist, so helps improve contractility and is a vasopressor-> vasoconstricts to improve cerebral and coronary perfusion).

*Although we should cardiovert new a-fib patients (had it <48 hrs) or a-fib patients who are unstable, if they are so unstable they go into a non-perfusing rhythm, you follow ACLS guidelines!

How well did you know this?
1
Not at all
2
3
4
5
Perfectly
120
Q

Asystole vs. Pulseless electrical activity (PEA)?

A

Asystole- Cardiac arrest
No electrical activity
Monitor is flat-lined

Pulseless electrical activity (PEA)- A non-perfusing rhythm
There is an abnormal heart rhythm (ex: a-fib), but the heart isn’t effectively pumping blood out, so there’s no palpable pulse (or measurable BP)
Monitor shows the abnormal rhythm

**NOTE: 3 rhythms cannot be PEA: (1) V-fib, (2) V-tach, (3) Asystole. Why? Bc we don’t expect a pulse with these rhythms. PEA is an organized electrical activity where we expect to see a pulse but we don’t have one.

How well did you know this?
1
Not at all
2
3
4
5
Perfectly
121
Q

What does ACLS stand for? What does CPR stand for?

A

ACLS= Advanced Cardiac Life Support

(guidelines on what to do if a patient is in cardiac arrest)

CPR= Cardiopulmonary Resuscitation

How well did you know this?
1
Not at all
2
3
4
5
Perfectly
122
Q

What are the reversible causes of asystole/ pulseless electrical activity? “5 H’s and 5 T’s”

(You want to consider these when doing chest compressions and giving epi…if you figure out the source of the problem, you may be able to correct the heart’s rhythm and save the patient’s life!)

A
Hypovolemia
Hypoxia
Hydrogen ions (acidosis) 
HypOkalemia/ HyPERkalemia
Hypothermia
Tension pneumothorax
Tamponade, cardiac
Toxins (narcotics, benzos) 
Thrombosis (pulmonary or coronary) 
Trauma
How well did you know this?
1
Not at all
2
3
4
5
Perfectly
123
Q

Can you shock a patient in Asystole?

A

NO

Asystole= they are flat-lined. There is ZERO electrical activity in the heart, so shocking them will not work (shocks work by feeding off of electrical activity). Do chest compressions, give epi (every 3-5 min), and try to figure out why the patient went into asystole bc there may be a reversible cause.

How well did you know this?
1
Not at all
2
3
4
5
Perfectly
124
Q

2 shockable rhythms? 2 non-shockable rhythms?

A

SHOCK:

  1. V-fib (ventricular fibrillation)
  2. V-tach (ventricular tachycardia)
    * *note: Torsades is a subcategory of ventricular tachycardia- shock it.

NON-SHOCKABLE:

  1. Asystole
  2. Pulseless electrical activity (PEA)

Rap song: “Defib for V-fib and pulseless V-tach. Don’t defib asystole, you won’t get them back!”

How well did you know this?
1
Not at all
2
3
4
5
Perfectly
125
Q

Define orthostatic hypotension.

A

Decrease in BP by 20 systolic or 10 diastolic when standing

*may be accompanied by an increase in HR

How well did you know this?
1
Not at all
2
3
4
5
Perfectly
126
Q

Why are old people more likely to get orthostatic hypotension (big drop in BP when they stand, making them momentarily lightheaded)?

A

Aging-> some baroreceptor activity is lost

*normally: you stand-> gravity is working against you so vessels have to constrict in the lower extremities to ‘milk’ blood up to the heart so it can pump enough to maintain BP. For the moment that BP is dropped right when you stand, baroreceptors (on carotid sinus) respond to decreased stretch-> decrease firing-> this increases sympathetic stimulation to raise the BP to baseline.

How well did you know this?
1
Not at all
2
3
4
5
Perfectly
127
Q

Young man faints when exercising. Has a crescendo-decrescendo systolic murmur in the left sternal border. Diagnosis?

A

HOCM

Basically a functional aortic stenosis, since the intraventricular septum is in the way of aortic outflow

How well did you know this?
1
Not at all
2
3
4
5
Perfectly
128
Q

IV drug user had an episode of syncope. Has a diastolic murmur at the left sternal border. Platelet count is elevated and EKG shows 2nd degree AV block. Diagnosis?

A

Perivalvular abscess

*Be suspicious of perivalvular abscess anytime a patient has endocarditis + conduction abnormalities on EKG (the abscess can extend into the conduction pathway, messing it up). Endocarditis alone wouldn’t cause AV block or syncope.

How well did you know this?
1
Not at all
2
3
4
5
Perfectly
129
Q

Man comes in for palpitations and SOB on exertion. He recently binge drank. HR is 130. EKG shows no clear P waves. Echo shows EF=35%, mitral regurg, and dilated LA and LV with global hypokinesis. What’s going on?

A

Alcohol-> A-fib w/ RVR (rapid ventricular response)-> tachycardia-mediated cardiomyopathy (the prolonged arrhythmia and rapid ventricular rate is causing HF)

*treat with aggressive rate and rhythm control

**note: the mitral regurg in this patient is just due to the fact that the ventricles are overloaded

How well did you know this?
1
Not at all
2
3
4
5
Perfectly
130
Q

What is tachycardia-mediated cardiomyopathy? How do you treat it?

A

When you have a tachyarrhythmia (heart is beating super fast, out of rhythm for a while) that leads to HF
(*can be due to a-fib, a-flutter, v-tach, etc.)

Treat with aggressive rate or rhythm control
(*AV nodal blocking agents, antiarrhtymic drugs, ablation of arrhythmia, etc.)

How well did you know this?
1
Not at all
2
3
4
5
Perfectly
131
Q

Lady presents with stroke symptoms. Over the past several months, she has had palpitations, SOB on exertion, and hemoptysis. Most likely explanation?

A

Rheumatic heart disease-> mitral stenosis-> a-fib-> stroke

*remember that mitral stenosis (MS) over time-> increased LA pressure. This predisposes to a-fib. This also leads to pulmonary vascular congestion, explaining the hemoptysis.

How well did you know this?
1
Not at all
2
3
4
5
Perfectly
132
Q

What is “flash pulmonary edema?”

A

Acute pulmonary edema secondary to increased cardiac filling pressures (such as after an MI)

How well did you know this?
1
Not at all
2
3
4
5
Perfectly
133
Q

Patient has episodes of squeezing chest pain at rest. EKG and exercise stress test are normal. Most likely diagnosis?

A

GERD

How well did you know this?
1
Not at all
2
3
4
5
Perfectly
134
Q

Gold standard for diagnosing type 2 DM in PCOS patients?

A

Oral glucose tolerance test

It is more sensitive in PCOS patients than fasting glucose and A1c standard screenings tests

How well did you know this?
1
Not at all
2
3
4
5
Perfectly
135
Q

Pathophys of PCOS?

A

Inc LH (related to insulin resistance)—> theca cells in ovaries use this to make androgens, then granulosa cells of ovaries take the androgens and make estrogen (estradiol).

With all the excess LH, there’s too many androgens for the granulosa cells to convert to estradiol, so excess androgens circulate in the blood and go into the peripheral fat—> fat cells make it into estrone and this feeds back onto the GnRH axis, inhibiting it.

Excess androgens—> hirsutism
Inhibition of GnRH—> anovulatory, infertile

How well did you know this?
1
Not at all
2
3
4
5
Perfectly
136
Q

55 year old guy has urinary frequency, urgency, hesitancy, and low back pain + perineal pain during ejaculation. The prostate is smooth but slightly enlarged. Urine culture is negative. Most likely diagnosis?

A

Chronic prostatitis
(Noninfectious chronic prostate inflammation)

  • this is a diagnosis of exclusion based on urinary symptoms + perineal or genital pain + pain or blood with ejaculation in the setting of a negative urine culture
  • antibiotics (fluoroquinolones) can help (even though a bacteria is not identified)
  • alpha-adrenergic inhibitors (Tamsulosin) and 5-alpha-reductase inhibitors (Finasteride) can also help
How well did you know this?
1
Not at all
2
3
4
5
Perfectly
137
Q

How does epididymitis (inflammation of the epididymis of the testicles) typically present?

A

Urinary/ voiding symptoms + scrotum pain/ swelling + purulent (pus) urethral discharge

How well did you know this?
1
Not at all
2
3
4
5
Perfectly
138
Q

Why would you test a patient with Raynaud phenomenon for autoantibodies and inflammatory markers (ex: ANA)?

A

It can occur in the setting of an autoimmune or systemic disease (ex: CREST syndrome)

How well did you know this?
1
Not at all
2
3
4
5
Perfectly
139
Q

What drugs may be helpful for Raynaud phenomenon?

A

Dihydropyridine CCBs

Vasodilate, so help improve perfusion to the fingertips

How well did you know this?
1
Not at all
2
3
4
5
Perfectly
140
Q

Lichen planus (pruritic, purple, papules, plaques) is associated with what disease?

A

Hepatitis C

How well did you know this?
1
Not at all
2
3
4
5
Perfectly
141
Q

If FENa (fraction of excreted sodium) is >2%, what does that tell you?

A

It’s either post-renal (later stage) or intrinsic AKI

Normal FeNa= <1%. If FeNa is higher, you know the kidneys are not working to reabsorb Na+

How well did you know this?
1
Not at all
2
3
4
5
Perfectly
142
Q

Lady has pyelonephritis. Later develops intrinsic AKI. What agent was she likely treated with?

A

An aminoglycoside

-these drugs treat serious gram-negative infections (pyelonephritis is often due to E-coli—a bladder infection that ascends) and are nephrotoxic

How well did you know this?
1
Not at all
2
3
4
5
Perfectly
143
Q

Minimal bright red blood per rectum (not mixed in stool) is most often due to benign disorders such as __________ and __________.

A
Hemorrhoids (varicose veins in the rectum that can cause bleeding) 
OR
Anal fissures (aka anal ulcer) (small tear in the lining of the anal wall)
How well did you know this?
1
Not at all
2
3
4
5
Perfectly
144
Q

What is a barium enema?

A

They give you a contrast solution and then take an x-ray of the colon

How well did you know this?
1
Not at all
2
3
4
5
Perfectly
145
Q

Patient under 40 years old has minimal bright red blood per rectum. What test should you do first?

A

Anoscopy (anal speculum)

Why? This is minimally invasive and allows you to look in there for hemorrhoids (varicose veins) or rectal fissures (ulcer/ tear in anal wall). If you don’t find the cause with the anoscopy, then consider a sigmoidoscopy or colonoscopy.
*If the patient were 40-49, you may consider a sigmoidoscopy. If the patient were 50+ and it’s been 2-3 years since the last colonoscopy, you’d want to do a colonoscopy.

How well did you know this?
1
Not at all
2
3
4
5
Perfectly
146
Q

How long does a seizure have to go on for there to be risk of permanent brain injury?

A

> 5 minutes

*Status epilepticus used to be defined as a seizure <30 min, but recent studies show all it takes is 5+ minutes of seizing for there to be risk of permanent brain injury (from the excitatory cytotoxicity)

How well did you know this?
1
Not at all
2
3
4
5
Perfectly
147
Q

How do we get rid of Basal Cell Carcinoma (pearly nodules) on the face and other cosmetically sensitive areas?

A

Mohs micrographic surgery

(Sequential removal of thin layers of skin with microscopic inspection to confirm that the margins have been cleared of malignant tissue)

How well did you know this?
1
Not at all
2
3
4
5
Perfectly
148
Q

Name for brown plaques/ nodules with “stuck-on” appearance?

A

Seborrheic keratosis

How well did you know this?
1
Not at all
2
3
4
5
Perfectly
149
Q

Drug-induced acne is a common side effect of what drugs?

A

Corticosteroids

How well did you know this?
1
Not at all
2
3
4
5
Perfectly
150
Q

If a patient cannot run on a treadmill, how do we do a stress test?

A

Give them Adenosine (vasodilator)

  • to normal coronary arteries—> this causes a big increase in blood flow (dilates them a lot)
  • to stenotic coronary arteries—> this does not cause an increase in blood flow (the body’s mechanisms have already dilated these stenotic vessels to the max, so adding in a vasodilator doesn’t do a thing)
  • the difference in blood flow allows us to diagnose CAD
How well did you know this?
1
Not at all
2
3
4
5
Perfectly
151
Q

Guy has an MI. You also notice an S3 heart sound and crackles in the lungs on examination. He’s already been given ASA, Clopidogrel, and Atorvostatin. What should you give him next?

A

Furosemide

-Diuretics are recommended for acute pulmonary edema (“flash pulmonary edema”) secondary to an MI.

*Although beta-blockers are part of the therapy for MI (“MONA BASH”), do NOT give a beta-blocker to a patient with decompensated CHF (dilated cardiomyopathy) or bradycardia
(This guy has this based on S3 and crackles—fluid overloaded). Why? Beta-blockers cause decreased contractility and HR…if you already got a pumping problem, this will worsen it!

How well did you know this?
1
Not at all
2
3
4
5
Perfectly
152
Q

What lung conditions are associated with CREST syndrome?

A

Interstitial fibrosis and pulmonary HTN

*if pulm HTN, would expect a RV heave (impulse palpated immediately left of the sternum that suggests RV enlargement) and/or a loud pulmonary component of the 2nd heart sound

How well did you know this?
1
Not at all
2
3
4
5
Perfectly
153
Q

Girl with hx of depression and chronic back pain overdosed on meds in a suicide attempt. Has a seizure on her way to the ED. She’s tachy, hypotensive, skin is warm and flushed, pupils dilated, bowel sounds decreased, and QRS prolonged.
What did she overdose on and what are you going to do about it?

A

TCA (tricyclic antidepressants) overdose
Give sodium bicarbonate

*Can cause QRS prolongation (or QT prolongation-> Torsades). Also causes anti-cholinergic effects.

How well did you know this?
1
Not at all
2
3
4
5
Perfectly
154
Q

Calcium gluconate is given in what arrhythmia?

A

Hyperkalemia (peaked T waves)

How well did you know this?
1
Not at all
2
3
4
5
Perfectly
155
Q

Homeless guy comes in smelling like alcohol. BP is 90/60, HR 95, RR 5. Has normal pupil size. Extremities are cool to touch. Decreased bowel sounds and muscular tone. Diagnosis? What is your priority in managing this patient?

A

Opioid overdose
Priority= airway protection—intubate and give Naloxone (mu-opioid antagonist to reverse acute opioid toxicity)

  • What gives it away is the respiratory depression (RR=5)!
  • Other findings also support this diagnosis (extremities cool to touch is due to hypothermia from impaired thermogenesis)

*don’t rule out opioid use just cuz the pupils are normal sized (not pinpoint)! They can be normal (could even be dilated if the patient took amphetamines too, for example)

How well did you know this?
1
Not at all
2
3
4
5
Perfectly
156
Q

What do you give for beta-blocker overdose/ intoxication?

A

Glucagon!

How well did you know this?
1
Not at all
2
3
4
5
Perfectly
157
Q

What’s a “scaphoid stomach?”

A

A sunken in stomach

How well did you know this?
1
Not at all
2
3
4
5
Perfectly
158
Q

Teenage girl has constipation and says her stomach is “puffed up.” She has cold intolerance. BP and HR are low. BMI is 18. On exam, abdomen is non-distended and normal bowel sounds. Diagnosis?

A

Anorexia nervosa

BMI <18.5 + distorted body image (thinks stomach is “puffed up” when it is not)

  • Constipation and cold intolerance are also consistent with hypOthyroidism. BUT, in hypOthyroidism you’d expect weight GAIN (everything is slowed down) and HTN (not low BP).
  • In Anorexia, electrolyte abnormalities—> slowed bowels and constipation. Malnutrition—> loss of muscle mass (which includes the heart)—> heart shrinks and beats slower to conserve energy (think of it as the heart is weakened bc it is starving)
How well did you know this?
1
Not at all
2
3
4
5
Perfectly
159
Q

Treatment options for psoriasis?

A

Topical glucocorticoids (betamethasone, fluocinonide) or Vitamin D derivatives

How well did you know this?
1
Not at all
2
3
4
5
Perfectly
160
Q

Another name for esophagogastroduodenoscopy (EGD)?

A

Endoscopy

Tube that goes down the mouth-> esophagus-> stomach-> duodenum

How well did you know this?
1
Not at all
2
3
4
5
Perfectly
161
Q

Guy with HTN (taking a Thiazide diuretic) is peeing all the time, thirsty all the time. Glucose is 90, Na+ is 150, serum osmolality is 300, urine osmolality is 125. Most likely diagnosis?

A

Diabetes insipidus (DI)

  • glucose is normal, so rule out DM
  • Na+ is high, so rule out side effect of the Thiazide diuretic alone (this causes hypOnatremia)
  • the blood is more concentrated than the urine…in DI, you are not making/ responding to ADH, so you cannot retain water and just pee it all out, makes sense
How well did you know this?
1
Not at all
2
3
4
5
Perfectly
162
Q

Lady complains of daily headaches. She used to get migraines a lot and uses over-the-counter analgesics (pain relievers for headache, like Tylenol) each day. What advice should you give her?

A

Stop taking the over-the-counter headache meds!

Using these too often—> medication overuse headaches (like a caffeine headache, these occur do to getting addicted to the meds)

How well did you know this?
1
Not at all
2
3
4
5
Perfectly
163
Q

Why do you get chills with fever?

A

Fever-> cytokines cause the hypothalamus to change the body’s setpoint to a higher level-> to reach this new setpoint, peripheral blood vessels vasoconstrict (to shunt blood to the core) and this can make a person feel cold

*also, muscles may contract (shivering) during this process

How well did you know this?
1
Not at all
2
3
4
5
Perfectly
164
Q

Patient comes in for recurrent sinusitis. He also has hematuria, found on urinalysis. Past medical history is significant for chronic joint and back pain. Likely diagnosis and next best step?

A

Granulomatosis with polyantiitis (Wagner’s)

  • small-vessel vasculitis involving the nasopharynx, lungs (but not in this patient), and kidneys
  • systemic symptoms are common (anemia, fatigue, fever, joint pains)

Next step= check for c-ANCA (autoantibodies against neutrophils)
*tissue biopsy is the confirmatory test

How well did you know this?
1
Not at all
2
3
4
5
Perfectly
165
Q

What test can you do to confirm lactose intolerance and how does it work?

A

Hydrogen breath test

  • Lactose intolerance= you lack lactase, a brush border enzyme that breaks down lactose in dairy products (decreases with age, esp in Asians). Since lactose doesn’t get broken down into simple carbs, its draws in water-> osmotic diarrhea.
  • Since it’s not broken down, more stomach acid will try to compensate to digest it—> positive H+ breath test.
How well did you know this?
1
Not at all
2
3
4
5
Perfectly
166
Q

High, low, or normal osmotic gap in lactose intolerance?

A

High

How well did you know this?
1
Not at all
2
3
4
5
Perfectly
167
Q

Old lady just diagnosed with colon cancer comes in complaining of intermittent pressure-like chest pain since the morning. EKG shows T wave inversion in leas V2-V4. Angiogram shows no blockage. Diagnosis?

A

Stress-induced (takotsubo) cardiomyopathy
Aka “broken heart syndrome”

  • believed to be brought on by a catecholamine surge from a stressor-> microvascular spasm and impaired contraction of the heart (hypokinesis).
  • EKG usually shows evidence of ischemia (ST elevation, T wave inversion, etc.) in the anterior precordial leads (V1-V4) and troponin may be a little high. But, angiogram shows no CAD.
  • self-limiting in weeks
How well did you know this?
1
Not at all
2
3
4
5
Perfectly
168
Q

EKG changes in:

  1. Panic attack
  2. Stress-induced cardiomyopathy (“broken heart syndrome”)?
A
  1. Panic attack—> no EKG abnormalities

2. Broken heart syndrome—> ischemic EKG symptoms (ST elevation, T wave inversion, moderately elevated troponin, etc.)

How well did you know this?
1
Not at all
2
3
4
5
Perfectly
169
Q

Are the following levels high, low, or normal in Paget’s dz of the bone?

  1. Serum calcium
  2. Serum phosphorus
  3. Alkaline phosphatase
  4. Urgent hydroxyproline
A
  1. Serum calcium- NORMAL
  2. Serum phosphorus- NORMAL
  3. Alkaline phosphatase- HIGH
  4. Urgent hydroxyproline- HIGH (an elevated bone turnover marker)
  • Paget disease of the bone- imbalance between osteoclast and osteoblast function (osteoC’s go crazy breaking down bone w/o the regulation of osteoB’s-> osteoB’s rush to compensate and lay down bone, but do a poor job of it-> thick bone that fractures easily)
  • Symptoms: bone pain, increasing hat size, hearing loss, lion face, isolated elevated alk phos (due to activation of osteoB’s)
How well did you know this?
1
Not at all
2
3
4
5
Perfectly
170
Q

Lady in the ED has an S3 and bilateral crackles. O2 sat is 78% with 40% oxygen, so she is intubated for respiratory failure and given nitrates and diuretics. Breath sounds on the left continue to be decreased. How can you restore them?

A

Reposition the endotracheal tube

(When you intubate, the goal is to get the tube just before the carina, where the main bronchus bifurcates into the right and left bronchus. If only one lung is expanding after intubation, you advanced the tube too far into either the right bronchus or the left bronchus.)

How well did you know this?
1
Not at all
2
3
4
5
Perfectly
171
Q

Patient had a URI 2 weeks ago that resolved. Now he has tingling in his foot, bilateral muscle weakness, and absent knee and ankle reflexes. No headache or photophobia. Diagnosis?

A

Guillain-Barré syndrome

-ascending paralysis from a URI or diarrheal illness (Campylobacter jejuni)

How well did you know this?
1
Not at all
2
3
4
5
Perfectly
172
Q
Will the following be high, low, or normal on examination of CSF in a patient with Guillain-Barré syndrome? 
Protein
WBC’s
RBC’s 
Glucose
A

Protein- HIGH
WBC’s- NORMAL
RBC’s- NORMAL
Glucose- NORMAL

  • CSF protein may be high due to increased permeability of the blood-nerve-barrier
  • treat with IV Ig or plasmapheresis
  • takes patients months to recover (can be chronic too- I saw a patient like this in the hospital)
How well did you know this?
1
Not at all
2
3
4
5
Perfectly
173
Q

When is it ok to accept a gift from a medical conference?

A

When the gift directly benefits patient care and it is of low monetary value

How well did you know this?
1
Not at all
2
3
4
5
Perfectly
174
Q

What are the indications for long-term home oxygen therapy? What about in patients with cor pulmonale, right HF, or hematocrit >55%?

A
  1. PaO2 < 55 or oxygen sat SaO2 < 88%
  2. In patients with cor pulmonale, right HF, or hematocrit >55%, PaO2 < 59 or oxygen sat SaO2 < 89%
  • *PaO2= Oxygen in blood, not bound to Hb
  • *SaO2= Percent of oxygen that is bound to Hb (saturated)
How well did you know this?
1
Not at all
2
3
4
5
Perfectly
175
Q

What do you do with a patient who has an epidural hematoma?

A

Emergent neurosurgery to get rid of the hematoma

How well did you know this?
1
Not at all
2
3
4
5
Perfectly
176
Q

A patient is having an MI. What’s the max time you can take to place that stent (PCI)?

A

90 min

How well did you know this?
1
Not at all
2
3
4
5
Perfectly
177
Q

Middle age man has nerve palsy, low-grade fevers, enlarged parotid glands, enlarged cervical lymph nodes. Labs are normal, except calcium is high. He is sexually active with multiple women. Diagnosis?

A

Sarcoidosis

-systemic granulomatous disease that can involve the nervous system (Bell’s palsy), lymphadenopathy, parotid gland swelling, hypercalcemia (1-alpha hydroxylase activity)

How well did you know this?
1
Not at all
2
3
4
5
Perfectly
178
Q

What EKG chance does TCA (tricyclic antidepressant) overdose cause?

A

TCAs block fast sodium channels—> slower conduction speed= QRS prolongation
(*can also lead to Torsades)

*Treat with sodium bicarb bc it increases serum pH (more alkaline) and extracellular sodium. Both the increased pH and extra extracellular Na+ decrease the drug’s binding to cardiac Na+ channels.

How well did you know this?
1
Not at all
2
3
4
5
Perfectly
179
Q

Normal calcium level?

A

8.4-10.2

For simplicity, remember about 8.5-10

How well did you know this?
1
Not at all
2
3
4
5
Perfectly
180
Q

Patient has breast cancer that metastasized to bone. She had a mastectomy and is undergoing chemo.
She is having vague bone pain. Has a palpable supraclavicular lymph node and high calcium.
What is this and how do you treat it?

A

Hypercalcemia of malignancy

Bisphosphonates

She has metastatic breast cancer to the bone—> more bone breakdown—> bone pain and higher levels of calcium in the blood (you can also get hypercalcemia of malignancy if you have a tumor secreting PTHrP).
Bisphosphonates cause osteoClast apoptosis so they can’t keep breaking down bone

How well did you know this?
1
Not at all
2
3
4
5
Perfectly
181
Q

Patient has an adrenal mass, high BP, headache, and hypokalemia. Diagnosis?

A

Primary hyperaldosteronism due to adrenal adenoma

*aldosterone-> Na+ reabsorption, K+ and H+ wasting, so hypokalemia makes sense

**treat with an aldosterone antagonist (Spironolactone, Eplerenone) or surgery

How well did you know this?
1
Not at all
2
3
4
5
Perfectly
182
Q

Old man has weakness and pain in his shoulders since shoveling snow 3 weeks ago. There is weakness on shoulder aBduction and decreased sensation on left forearm. Next step?

A

Get an MRI of the cervical spine

-He most likely has C5-6 nerve root impingement (cervical radiculopathy) from underlying cervical spondylitis (wear and tear of spinal disks in neck/ cervical spine degeneration)

How well did you know this?
1
Not at all
2
3
4
5
Perfectly
183
Q

Man has 1 year history of diarrhea, cramps, flushing, systolic murmur over left sternal border that increases with inspiration, and high LFTs. Diagnosis?

A

Carcinoid syndrome

  • Serotonin-secreting tumor
  • Usually in the GI tract, but if Mets to the liver, it can’t be broken down and serotonin travels through the bloodstream causing systemic effects. Can affect right heart only (tricuspid regurg in this patient) bc lungs (like the liver) have the enzyme to break down serotonin.
  • Symptoms= diarrhea, flushing, tricuspid regurg, bronchospasm
How well did you know this?
1
Not at all
2
3
4
5
Perfectly
184
Q

How can Carcinoid tumors lead to Niacin (vit B3) deficiency?

A
Carcinoid= serotonin-secreting tumor
Tryptophan gets used up to make serotonin 
Since tryptophan (and vit B6) are required to make niacin (vit B3), you’ll get a niacin deficiency as a result
How well did you know this?
1
Not at all
2
3
4
5
Perfectly
185
Q

Lady with hx of DM presents with sudden-onset double vision and ptosis of her right eye. The right eye is down and out. Pupils are responsive and there are no other neuro defects. Most likely diagnosis?

A

Diabetic ophthalmoplegia- poorly controlled DM—> ischemic neuropathy (tiny blood vessels that supply CN 3 are damaged)—> CN 3 palsy

How well did you know this?
1
Not at all
2
3
4
5
Perfectly
186
Q

Tendon vs. ligament?

A

Tendon- muscle to bone

Ligament (“Like”)- bone to bone

How well did you know this?
1
Not at all
2
3
4
5
Perfectly
187
Q

What is enthesitis?

A

Inflammation and pain at sites where tendons (muscle to bone) and ligaments (bone to bone) attach to bone

  • most often at the Achilles’ tendon (but also can be at the costosternal junction, shoulders, elbows, hips, iliac crests, tibial tuberosities, etc.)
  • can be part of an isolated disorder (like plantar fasciitis) or in spondyloarthropathies (like ankylosing spondylitis)
How well did you know this?
1
Not at all
2
3
4
5
Perfectly
188
Q

30 year old guy has low back pain that improves with use. He has limited spine flexion and sacroiliac joint inflammation. Has also had diarrhea going on. Diagnosis?

A

IBD (Crohn’s or Ulcerative Colitis)

*remember they are associated with arthritis, including spondylarthritis or sacroilitis

How well did you know this?
1
Not at all
2
3
4
5
Perfectly
189
Q

What does a positive pronator drift test tell you?

A

There is upper motor neuron (UMN) or pyramidal/ CST (corticospinal tract) disease

*UMN lesions cause more weakness in supinator muscles compared to pronator muscles of the upper limb—> affected arm drifts down and the palm turns (pronates) toward the floor

How well did you know this?
1
Not at all
2
3
4
5
Perfectly
190
Q

Middle aged woman is having strong urges to urinate and sometimes leaks urine on her way to the bathroom. What med can you give her?

A

Oxybutynin (anti-muscarinic for treatment of urinary incontinence)

How well did you know this?
1
Not at all
2
3
4
5
Perfectly
191
Q

What is presbycusis?

A

Age-related hearing loss

*due to cochlear hair cell loss (and cochlear neuron degeneration)

How well did you know this?
1
Not at all
2
3
4
5
Perfectly
192
Q

What can you advise patients to do when they get vasovagal syncope episodes- when they feel fainting coming on (prodromal phase)?

A

Cross legs, tense calf muscles, handgrip, and tense arm muscles with clenched fists…)
-These maneuvers increase venous return to the heart (more ‘milking’ of blood to heart)—> more CO, which can stop the syncope from happening

How well did you know this?
1
Not at all
2
3
4
5
Perfectly
193
Q

Fever and chills 1-6 hrs after blood transfusion. What reaction is this?

A

Febrile nonhemolytic transfusion reaction
(Due to cytokines in stored blood products)

*prevention of this is by leukoreduction of blood products (filter out most the WBCs after blood collection, before storage to prevent cytokine production within the packed RBC product)

How well did you know this?
1
Not at all
2
3
4
5
Perfectly
194
Q

What 2 medication classes can lead to severe hypotension if combined with a PDE-5 inhibitor like Sildenafil (Viagra) for ED?

A
  1. Nitrates

2. Alpha blockers (ex: Doxazosin)

How well did you know this?
1
Not at all
2
3
4
5
Perfectly
195
Q

Patient has fever and new holosystolic murmur at the apex. He was treated empirically with vancomycin. Blood culture results came back sensitive to penicillin. What should you change his antibiotics to (2 options)?

A

IV Penicillin G or IV Ceftriaxone

  • Fever + murmur suggests endocarditis
  • Initial emperic treatment with Vanco is appropriate to cover MRSA (just in case)
  • Now we got cultures back and know it is not MRSA (it is sensitive to penicillin), so switch to IV penicillin or Ceftriaxone (easier to administer due to once daily dosing vs. every 4-6 hrs)
  • Do not use oral Penicillin- should treat IV for endocarditis
How well did you know this?
1
Not at all
2
3
4
5
Perfectly
196
Q

Do you have a high A-a gradient in PE (pulmonary embolism)?

A

YES

You have a high A-a in any V/Q mismatch where there is impaired gas exchange!

  • in the case of PE, the problem is with Q (perfusion)
  • oxygen is getting from the alveoli fine, but the blood isn’t there to pick it up bc it is stuck (there’s a clot in major pulmonary vessels)
How well did you know this?
1
Not at all
2
3
4
5
Perfectly
197
Q

Do patients with PE typically have a high or low PaCO2?

A

LOW

PE—> hyperventilation (in effort to compensate)—> blowing off more CO2, so less CO2 stays behind in the blood

How well did you know this?
1
Not at all
2
3
4
5
Perfectly
198
Q

1st line treatment for female pattern hair loss?

A

Topical Minoxidil

A vasodilator that increases blood flow to the scalp
*side effects: irritation and itching

How well did you know this?
1
Not at all
2
3
4
5
Perfectly
199
Q

Old lady with PMH of DM and HTN has began having difficulty planning meetings for 9 months. She can no longer cook for herself. Gait is unsteady. She is sad. Diagnosis?

A

Vascular dementia
(Stepwise/ gradual decline, often presents with motor, gait, urinary, and psych symptoms)

*not likely to be Alzheimer’s bc that starts with memory problems and other stuff goes later

How well did you know this?
1
Not at all
2
3
4
5
Perfectly
200
Q

Patient flexes right hip and knee and slaps foot to ground w/ each step. He is complaining of recurrent falls. Cause of gait abnormality?

A

Common peroneal neuropathy
(L5 radiculopathy-> back pain radiating to the foot and weakness on foot inversion and plantar flexion)

*remember the common peroneal (fibular) nerve:
PED= Peroneal Everts and Dorsiflexes
If injured-> foot dropPED

How well did you know this?
1
Not at all
2
3
4
5
Perfectly
201
Q

Patients with SOB due to CHF have high BNP levels, typically above what number?

A

BNP >400

(Remember BNP is released by the heart in response to stretch, which there is more of when you have CHF and are fluid overloaded)

*BNP is a highly sensitive test for CHF. Clinical signs (JVD, crackles, lower extremity edema) are highly specific, but not sensitive.

How well did you know this?
1
Not at all
2
3
4
5
Perfectly
202
Q

Why might a male with a prolactinoma have decreased testosterone levels?

A

HIGH prolactin will feedback on GnRH, making it LOW (negative feedback)—> LOW testosterone production

How well did you know this?
1
Not at all
2
3
4
5
Perfectly
203
Q

35 year old guy has recurrent headaches. BP is 190/100 and you hear a S4 and continuous murmur. Diagnosis?

A

Coarctation of the aorta

  • narrowing of descending aorta-> pressure overload in proximal part of aorta-> HTN in arms (where BP is normally measured), headaches (pressure goes back to carotids-> head)
  • murmur for coarctation is systolic (blood through constricted aorta like aortic stenosis) or continuous (if collateral vessels are present)
  • S4 may be heard due to LV hypertrophy (induced by HTN)
How well did you know this?
1
Not at all
2
3
4
5
Perfectly
204
Q

What may be seen on CXR of a patient with coarctation of the aorta?

A

Notching of the ribs (erosions of inferior costal surfaces)

-this finding is due to collateral vessel formation

How well did you know this?
1
Not at all
2
3
4
5
Perfectly
205
Q

Patient has watery diarrhea, flushing, and low K+ causing muscle cramps. CT shows a mass in the pancreatic tail. Diagnosis?

A

VIPoma

-rare tumor of pancreatic cells—> releases vasoactive intestinal peptide (VIP)

How well did you know this?
1
Not at all
2
3
4
5
Perfectly
206
Q

Patient has fever, acute-onset respiratory distress, hypoxemia, and bilateral opacities on CXR. Diagnosis?

A

ARDS (Acute Respiratory Distress Syndrome)

*acute inflammatory response w/ cytokines in the lungs-> epithelial damage/ necrosis of alveolar cells + endothelial damage/ inc capillary permeability leaking proteins-> hyaline membranes

How well did you know this?
1
Not at all
2
3
4
5
Perfectly
207
Q

What happens in ARDS?

A

Acute inflammatory response w/ cytokines in the lungs-> epithelial damage/ necrosis of alveolar cells + endothelial damage/ inc capillary permeability leaking proteins-> hyaline membranes
(The release of cytokines/ overreactive inflammatory response to a stressor like sepsis, PNA, whatever is the problem…causes damage to lung tissue)

**rarely can lead to irreversible pulmonary fibrosis (from too much collagen deposition as endothelial cells, pneumocytes, and fibroblasts proliferate to repair damaged lung)

How well did you know this?
1
Not at all
2
3
4
5
Perfectly
208
Q

What are the 2 ways to improve oxygenation in a ventilated patient?

A
  1. Increase FiO2 (fraction of inspired oxygen)
  2. Increase PEEP (positive end-expiratory pressure)
    * This is the best way to improve hypoxemia in a patient with ARDS bc it additionally prevents alveolar collapse
How well did you know this?
1
Not at all
2
3
4
5
Perfectly
209
Q

If high levels (>60%) of FiO2 are required to maintain oxygenation in a patient on a ventilator, what setting should you increase?

A

PEEP (positive end-expiratory pressure)

  • There are 2 ways to improve oxygenation of a ventilated patient:
    1. Increase FiO2 (fraction of inspired oxygen)
    2. Increase PEEP (positive end-expiratory pressure)
How well did you know this?
1
Not at all
2
3
4
5
Perfectly
210
Q

Patient has nasal breathing, stuffy nose, dry cough for more than a year. No allergies. Diagnosis and treatment (2 options)?

A

Nonallergic rhinitis (aka vasomotor rhinitis)

Treat with an intranasal antihistamine and/or intranasal glucocorticoid

How well did you know this?
1
Not at all
2
3
4
5
Perfectly
211
Q

Man comes in after long plane flight with LUQ pain. Has high indirect bilirubin (UCB), high reticulocyte count, and splenomegaly. Most likely cause of his symptoms?

A

Splenic infarction
(Acute occlusion of splenic artery, causing LUQ pain)

  • High retic count and UCB= Intravascular hemolysis
  • Could be due to sickle cell trait (usually asymptomatic, but can get Intravascular hemolysis and splenic infarction in stressors like a long flight- do Hb electrophoresis to diagnose
How well did you know this?
1
Not at all
2
3
4
5
Perfectly
212
Q

What is “salvage therapy?”

A

Treating a disease a different way when the standard treatment fails

(Ex: guy gets radical prostatectomy for prostate adenocarcinoma and PSA is undetectable. Months later, PSA is high and he gets radiation= a salvage therapy)

How well did you know this?
1
Not at all
2
3
4
5
Perfectly
213
Q

Old guy has progressive loss of vision in his right eye. Vertical lines look bent and wavy to him. Diagnosis?

A

Macular degeneration

  • often age-related, the macula (center of retina in back of eye responsible for visual acuity) wears down
  • early finding= straight lines look wavy
  • eventually they loose central vision

*most common cause of blindness in industrialized nations

How well did you know this?
1
Not at all
2
3
4
5
Perfectly
214
Q

How do we confirm a diagnosis of Scabies and how do we treat it (2 drug options)?

A

Diagnosis: light microscopy of skin scrapings

Treatment: topical permethrin or oral ivermectin

How well did you know this?
1
Not at all
2
3
4
5
Perfectly
215
Q

30 y.o. African American woman
Painful, stiff hands/ wrists, esp in morning
Muscle aches
Ulcer on buccal mucosa
Lymphadenopathy
Low Hb (anemia), low platelets (thrombocytopenia)

Diagnosis?

A

Lupus (SLE)

Diagnosis if 4+ of these criteria:

  1. Butterfly rash
  2. Photosensitivity
  3. Oral/ nasopharyngeal ulcers
  4. Discoid rash (coin-shaped, worse with sun exposure)
  5. Arthritis
  6. Pericarditis
  7. Hematologic dz (anemia, leukopenia, thrombocytopenia)
  8. Renal disease (proteinuria, casts)
  9. CNS- seizures, psychosis
  10. Immuno markers (false positive Syphilis, anti-dsDNA, anti-Smith Ab)
  11. ANA

*She has: (1) arthritis, (2) oral ulcer, (3) lymphadenopathy, (4) anemia and thrombocytopenia

How well did you know this?
1
Not at all
2
3
4
5
Perfectly
216
Q

What is Felty syndrome?

A

Triad of:
RA + splenomegaly + neutropenia (low neutrophils-> high-risk for bacterial infections)

*“SANTA’s suit is felty” Splenomegaly, Anemia, Neutropenia, Thrombocytopenia, Arthritis (Rheumatoid)

How well did you know this?
1
Not at all
2
3
4
5
Perfectly
217
Q

Patient who got a liver transplant 5 months ago (taking immunosuppressants, no antibiotics) comes in with SOB, nonproductive cough, fever, chills. O2 sat is 82%, has bilateral crackles, diffuse interstitial infiltrates, elevated lactate dehydrogenase (LDH).
What test should you order?

A

Bronchoalveolar lavage

This is likely PCP (Pneumocystis) pneumonia
*immunosuppressed/ organ transplant patients are at high risk for PCP and CMV pneumonia. Therefore, they should be on prophylactic Bactrim (for PCP) and Ganciclovir-Valganciclovir (for CMV in seropositive patients)

How well did you know this?
1
Not at all
2
3
4
5
Perfectly
218
Q

What does recumbent mean?

A

Lying down

How well did you know this?
1
Not at all
2
3
4
5
Perfectly
219
Q

Guy smoker has blood in urine and fever for 4 weeks. Family hx of blood disorder. Has a left-sided varicocele even when lying down.
Next test?

A

Abdominal CT

  • Patient most likely has RCC (renal cell carcinoma) (*classic triad: flank pain, hematuria, palpable abdominal mass is in few patients)
  • smoking is a risk factor
  • varicocele is due to tumor pressing against left renal vein and left testicular vein, impairing venous drainage from testicle
  • fever is a nonspecific cancer finding (also weight loss, anorexia, fatigue, night sweats)
  • ectopic production of EPO by kidney tumor-> polycythemia
How well did you know this?
1
Not at all
2
3
4
5
Perfectly
220
Q

Increased QRS voltage (really high QRS peaks) tells you what?

A

LV hypertrophy

How well did you know this?
1
Not at all
2
3
4
5
Perfectly
221
Q

Guy has narrowed retinal vessels and blurred vision and increased QRS voltage on EKG and T wave inversion. BP is 130/80.
What’s going on?

A

Isolate ambulatory HTN (aka masked HTN)

  • even though BP is okay rn, it is probably high on average (avg over day of >135/85= HTN)
  • he has evidence of end-organ failure from HTN (retinal AV nicking aka hypertensive retinopathy and LV hypertrophy shown by peaked QRS’s)
How well did you know this?
1
Not at all
2
3
4
5
Perfectly
222
Q

Woman had 2 miscarriages. Positive VDRL. Low platelets (thrombocytopenia). High PTT.
Diagnosis?
Treatment?

A

Anti-phospholipid antibody syndrome
Treat/ prophylaxis with aspirin and LMWH (low molecular weight heparin) to avoid clotting problems and pregnancy loss

  • the presence of antiphospholipid antibodies + you have blood clots (DVT, PE, stroke, MI) or pregnancy problems (miscarriages or premature birth from placental insufficiency or preeclampsia)
  • antiphospholipid antibodies are seen in SLE (but not specific to it). They can cause 3 problems: (1) antiphospholipid antibody syndrome, (2) increased PTT, (3) false positive syphilis test (RPR, VDRL)
How well did you know this?
1
Not at all
2
3
4
5
Perfectly
223
Q

Guy has cough, fatigue, unintentional weight loss, enlarged mediastinum, enlarged hilar and mediastinal lymph nodes.
PET (positron emission tomography) scan with 18-fluorodeoxyglucose shows uptake in the brain, kidneys, bladder, supraclavicular and mediastinal lymph nodes.
Diagnosis?

A

Hodgkin lymphoma

  • B symptoms
  • painless lymphadenopathy
  • mediastinal mass
  • PET scan radiotracer is taken up by cancer cells w/ a high metabolic rate (supraclavicular and mediastinal LN’s) and healthy organs with high metabolic rates (brain, kidneys, bladder)
How well did you know this?
1
Not at all
2
3
4
5
Perfectly
224
Q

Guy has diverticulitis and gets treated. A couple days later, he develops acute pain and redness in his right ankle (had a similar episode before). Diagnosis?

A

Gout

  • usually occurs in big toe, but can occur in ankle or knee too
  • triggers: alcohol, surgery/ trauma, dehydration, some meds (diuretics)
How well did you know this?
1
Not at all
2
3
4
5
Perfectly
225
Q

Explain what goes down when the RAA System is activated.

*triggers that activate RAAS: low BP, low NaCl delivery to macula densa, sympathetic NS kicks in

A
  1. Liver releases AT
  2. Kidney JGA cells release Renin, which converts AT-> AT I
  3. Lungs release ACE (angiotensin converting enzyme), which converts AT I-> AT II
    * ACE also breaks down bradykinin
  4. AT II has multiple effects on the body to raise BP:
    - vasoconstriction
    - constricts efferent arteriole (raise GFR)
    - aldosterone release from adrenal gland-> reabsorption of Na+ (wasting of K+, H+)
    - ADH (retains water) and thirst
How well did you know this?
1
Not at all
2
3
4
5
Perfectly
226
Q

Explain amaurosis fugax.

A

Temporary loss of vision (in 1 or both eyes)

-For example, due to emboli from severe carotid stenosis can cause ischemia to the optic nerve-> transient (on/off) vision loss

How well did you know this?
1
Not at all
2
3
4
5
Perfectly
227
Q

Young woman comes in with really high BP of 165/100 and transient (on/off) vision loss in an eye. She has a right carotid bruit. Renin and aldosterone levels are high. Next step?

A

CT Angiogram of the abdomen (look at the renal arteries)

Why? She has such a high BP that it is causing transient vision loss (amaurosis fugax- emboli from severe ipsilateral carotid stenosis can cause ischemia to the optic nerve), so you need to work it up. High Renin and Aldosterone means it’s secondary hyperaldosteronism. This is most likely due to FIBROMUSCULAR DYSPLASIA (developmental defect in blood vessel wall-> irregular thickening of renal, carotid, vertebral arteries)-> kidneys read low BP-> activate RAAS-> increased renin, aldosterone, and BP.

How well did you know this?
1
Not at all
2
3
4
5
Perfectly
228
Q

Atrioventricular (AV) nodal reentrant tachycardia (AVNRT) is a sub-type of what tachycardia?

A

Paroxysmal supraventricular tachycardia (PSVT)

How well did you know this?
1
Not at all
2
3
4
5
Perfectly
229
Q

What is the diving reflex? (Our body’s response to immersion in cold water)

A

Reflex on immersion in cold water that slows HR and causes peripheral vasoconstriction to divert blood flow to central, vital organs (brain, heart, lungs) to conserve oxygen until breathing resumes and to delay potential brain damage

(*the body knows it cannot breathe in oxygen while underwater, so it slows the HR down so that the body can survive longer with the blood/ oxygen it already has to pump)

colder water (drops core temp super fast)= stronger diving reflex response (and therefore better chance of survival if you drown in ice cold water vs. a pool)
(
supraventicular means it is coming from the atria)

How well did you know this?
1
Not at all
2
3
4
5
Perfectly
230
Q

How would immersion in cold water relieve heart palpations? (state mechanism involving baroreceptors)

A

Vagal maneuver (cold water immersion/ diving reflex, carotid sinus massage, Valsalva, eyeball pressure)—> increased stretching and firing of baroreceptors—> more parasympathetic response—> lowered HR

*diving reflex (immersion in cold water slows the heart to conserve energy/ so you could survive longer + vasoconstricts to shunt blood to central, vital organs)

How well did you know this?
1
Not at all
2
3
4
5
Perfectly
231
Q

In hypovolemic shock, are the following values high or low?

  1. pulmonary capillary wedge pressure (PCWP)
  2. cardiac output (CO)
  3. blood pressure (BP)
  4. systemic vascular resistance (SVR)
A
  1. PCWP (=LA pressure)- LOW (due to decreased preload)
  2. CO- LOW (due to decreased preload)
  3. BP- LOW (due to deceased CO)
  4. SVR- HIGH (vasoconstriction in attempt to raise BP and maintain CO)
How well did you know this?
1
Not at all
2
3
4
5
Perfectly
232
Q

Why might an inferior MI (ST elevations in II, III, aVF) cause AV block (prolonged PR intervals w/ dropped QRS’s)?

A

Inferior MI—> occluded RCA, which supplies the AV node—> since the AV node has cut off blood supply, it won’t be able to do a good job of delaying the signal from the atria to ventricles—> AV block
(prolonged PR’s= impulse from atria to ventricles gets too delayed, dropped QRS’s= part of the AV node is blocked so failing to send the impulse from the atria to ventricles)

How well did you know this?
1
Not at all
2
3
4
5
Perfectly
233
Q

60 y.o. smoker has sudden-onset chest pain, diaphoresis, and N/V. BP is 85/55 and HR is 50 bpm. EKG shows ST-elevations in leads II, III, aVF. Diagnosis?

A

Inferior MI- occlusion of the RCA

*His vitals tells you he’s in cardiogenic shock (really low HR and BP). This makes sense bc MI-> heart not getting fed enough blood to pump well

How well did you know this?
1
Not at all
2
3
4
5
Perfectly
234
Q

In pericarditis, what do you see on EKG?

A

Diffuse ST-segment elevations

How well did you know this?
1
Not at all
2
3
4
5
Perfectly
235
Q

Guy had an MI 3 days ago. Now he’s coming in with sharp pain on his left chest that improves with leaning forward. Diagnosis?

A

Acute pericarditis (aka fibrinous pericarditis aka peri-infarction pericarditis)

*EKG would show diffuse ST-segment elevation

How well did you know this?
1
Not at all
2
3
4
5
Perfectly
236
Q

Woman with history of rheumatic heart dz has acute onset SOB. She has a mid-diastolic rumble at the apex, crackles over the lungs, and EKG shows irregularly, irregular rhythm and no P waves. What’s going on?

A

She has mitral stenosis (from rheumatic heart disease which causes mitral regurg, later mitral stenosis)—> increased pressure in LA causes the LA to dilate—> predisposes to a-fib—> worsening flow through the stenotic mitral valve—> more backing up of blood/ congestion to lungs, explaining the SOB

*up to 70% of patients with mitral stenosis will develop a-fib from the significant LA dilation

How well did you know this?
1
Not at all
2
3
4
5
Perfectly
237
Q

Most reliable finding to differentiate between an epileptic seizure and syncope?

A

Tongue biting

(*95% specificity for epileptic seizure, esp if on the lateral tongue- bc rarely syncope can cause frontal tongue biting)

How well did you know this?
1
Not at all
2
3
4
5
Perfectly
238
Q

What are these EKG findings suggestive of?

High-voltage QRS complexes, lateral ST segment depression, and lateral T wave inversion

A

LV hypertrophy

How well did you know this?
1
Not at all
2
3
4
5
Perfectly
239
Q

30 year old guy has a nosebleed requiring nasal packing. His BP is 180/120. EKG shows high-voltage QRS complexes, ST-segment depression, and T-wave inversion. Most likely diagnosis?

A

Coarctation of the aorta

  • high-voltage QRS’s, ST-segment depression, and T-wave inversion= LV hypertrophy from long-standing HTN
  • prob not essential HTN bc that rarely causes end-organ damage in young patients <40
  • coarctation of the aorta (narrowing at distal aorta) usually causes asymptomatic HTN, but can cause headache, nosebleeds, lower extremity claudication

*check bilateral arm and leg BP measurements

How well did you know this?
1
Not at all
2
3
4
5
Perfectly
240
Q

Old lady has acute-onset epigastric pain associated with N/V. What test should you do 1st?

A

ECG

*You need to rule out acute coronary syndrome (STEMI, NSTEMI, unstable angina), which could have an atypical presentation like this! Once ruled out, then you can consider GI things like pancreatitis

**women, elderly, and diabetes patients are more likely to have atypical heart symptoms

How well did you know this?
1
Not at all
2
3
4
5
Perfectly
241
Q

How is vasospastic (Prinzmetal) angina similar to Raynaud phenomenon?

A

Vasospastic (Prinzmetal) angina- due to episodic vasospasm of pulmonary vessels

Raynaud phenomenon- due to episodic vasospasm in fingers and toes

How well did you know this?
1
Not at all
2
3
4
5
Perfectly
242
Q

What is the screening protocol for AAA?

A

One-time abdominal ultrasound screen for men 65+ who have ever smoked

How well did you know this?
1
Not at all
2
3
4
5
Perfectly
243
Q

How do you treat Wolff-Parkinson White syndrome patients with A-fib with RVR:

  • if unstable?
  • if stable?
A

Unstable—> immediate cardioversion

Stable—> IV procainamide or Ibutilide (rhythm control)

*do NOT use AV node blockers like adenosine, beta blockers, CCBs, digoxin—they would promote conduction across the accessory ‘secret’ pathway, causing a-fib—> v-fib

How well did you know this?
1
Not at all
2
3
4
5
Perfectly
244
Q

Meniere disease is a disorder of the inner ear (too much endolymph/ fluid) characterized by what triad?

A
  1. Episodes of vertigo (20 min- 24 hrs, associated with nausea/ vomiting)
  2. Sensorineural hearing loss (fluctuates)
  3. Tinnitus (ringing of ear)
How well did you know this?
1
Not at all
2
3
4
5
Perfectly
245
Q

Woman comes in for episodes of dizziness (room spinning) associated with ringing in the ears, improved with lying down and eyes closed. When you do Rinne test, air conduction > bone conduction on both ears. When you do Webber test, the sound is heard better on the left. Diagnosis?

A

Meniere Disease (disorder of inner ear where you have too much volume/ pressure of endolymph fluid)

Triad:

  1. Episodes of vertigo
  2. Sensorineural hearing loss
  3. Tinnitus
How well did you know this?
1
Not at all
2
3
4
5
Perfectly
246
Q

You need 2/3 criteria to diagnose acute pancreatitis. What are they?

A
  1. Acute-onset epigastric pain radiating to the back
  2. Increased amylase or lipase >3x the upper limit of normal (lipase more specific)
  3. Abdominal imaging suggestive of pancreatitis- CT of abdomen (pancreatic enlargement)
    * NOTE: you do NOT need CT to make the diagnosis…epigastric pain radiating to back + lipase >3x upper limit of normal is sufficient!

**U/S is not used (gas prevents visualization) and X-ray is not used (poor test for pancreatitis)

How well did you know this?
1
Not at all
2
3
4
5
Perfectly
247
Q

Guy has an NSTEMI (substernal chest pain w/ diaphoresis, ST depressions, up-trending troponin) so he gets a stent placed. Besides ASA, a beta-blocker, statin, ACE inhibitor, what do you need to give him?

A

A P2y12 receptor blocker
(like Clopidogrel aka Plavix)

*normally, ADP binds this receptor on platelets to promote platelet aggregation (we block it to stop platelet aggregation)

Dual Anti-Platelet Therapy (DAPT) (ASA + P2y12 receptor blocker) is given to patients with stents for 12 mo to reduce the risk of stent thrombosis. Also, it reduces risk of recurrent MI in NSTEMI compared to ASA alone!

How well did you know this?
1
Not at all
2
3
4
5
Perfectly
248
Q

If a patient gets a stent, how long do they need to be on DAPT (Dual Anti-Platelet Therapy= Aspirin + P2y12 receptor blocker like Clopidogrel/ Plavix)?

A

At least 12 months

Significantly reduces risk of stent thrombosis and recurrent MI

How well did you know this?
1
Not at all
2
3
4
5
Perfectly
249
Q

Younger guy who has a sore throat for 4 days is coming in due to hematuria. U/A shows protein and RBCs in his urine. Diagnosis?

A

IgA nephropathy

  • protein in urine= nephrotic syndrome
  • most common nephrotic syndrome in adults= IgA nephropathy, esp after a URI (“sore throat”)
How well did you know this?
1
Not at all
2
3
4
5
Perfectly
250
Q

Lady has right ear pain, red vesicles in the ear canal, and right facial droop. Diagnosis?

A

Herpes zoster oticus
aka Ramsay Hunt syndrome

Varicella zoster (VZV) reactivates—> Herpes zoster. If the herpes occurs in the dermatome of the geniculate ganglion (collection of sensory nerves from the facial nerve in the facial canal of the head), you get this. It disrupts CN 7/ facial nerve motor fibers and spreads to CN 8/ vestibulocochlear nerve—> facial paralysis and ear pain there.

How well did you know this?
1
Not at all
2
3
4
5
Perfectly
251
Q

Old man presents with worsening SOB and nonproductive cough. Has had an ongoing fever, headache, sore throat, runny nose, anorexia, and body aches since Christmas shopping at a mall 5 days ago. He has crackles. Diagnosis?

A

Influenza virus
(Muscle aches + nonspecific flu symptoms after shopping in a germy mall)

*pneumonia is the most common complication of the flu

How well did you know this?
1
Not at all
2
3
4
5
Perfectly
252
Q

Most common complication of influenza?

A

Pneumonia!

  • This can be from secondary bacterial infection (ex: Strep pneumo, Staph a.) or from direct viral attack (influenza pneumonia)
  • give oxygen support and oseltamivir (Tamiflu) + treat the pneumonia with Abx
How well did you know this?
1
Not at all
2
3
4
5
Perfectly
253
Q

50 y.o. Guy has nausea, fatigue, bilateral flank pain over the last couple yrs. BP is 160/100. Liver is large, mass is felt on palpation of flank, prostate is enlarged. BUN and Cr are high. Diagnosis?

A

Autosomal Dominant Polycystic Kidney Disease (ADPKD)—> chronic kidney dz (CKD)

  • evidence of ADPKD: flank pain, HTN (would be due to excess renin release), large liver (cysts)
  • evidence of CKD: nausea, fatigue, high Cr (due to structural degeneration of the kidneys)
  • NOTE: BPH is a separate issue here
How well did you know this?
1
Not at all
2
3
4
5
Perfectly
254
Q

Car crash victim is having severe pain from a leg fracture. He is a former heroin addict. You give him ketorolac (NSAID) for pain management, but it doesn’t cut it. What do you give now?

A

IV Morphine

*INITIAL management for acute pain (including opioids) is about the same for all patients—regardless of addiction history (you will just need to monitor this guy closely to avoid relapse)

How well did you know this?
1
Not at all
2
3
4
5
Perfectly
255
Q

Why do you get the following in aortic stenosis?

  1. Diminished and delayed carotid pulse (“pulsus parvus et tarsus”)
  2. Systolic murmur (late-peaking, crescendo-decrescendo)
  3. Soft and single S2
A
  1. Diminished and delayed carotid pulse (“pulsus parvus et tarsus”)—> since you have a stenotic aortic valve, there’s less blood flow making it through
  2. Systolic murmur (crescendo-decrescendo)—> aortic valve opens during systole so makes sense that aortic stenosis (problem opening) is heard during systole
  3. Soft and single S2—> normally, you have physiologic split of S2 (inspiration-> more blood to right heart-> pulmonic valve takes slightly longer to close than aortic valve in S2)…with aortic stenosis, the stenotic aortic valve takes longer to close too so they close at the same time= single S2
How well did you know this?
1
Not at all
2
3
4
5
Perfectly
256
Q

40 y.o. guy comes in for a physical. He has fatigue, muscle aches, weight gain. His total cholesterol and triglycerides are high. What is the next best step to manage his dyslipidemia?

A

Order TSH

Fatigue, muscles aches, weight gain-> suspicious for hypothyroidism, which can cause lipid abnormalities!

You need to confirm this diagnosis—if it is in fact hypothyroidism, you will need to treat the underlying cause with Levothyroxine (will improve lipid levels, but does take a few months). Don’t jump straight to a statin (can worsen myopathy) or another anti-lipid drug until you do the work-up.

How well did you know this?
1
Not at all
2
3
4
5
Perfectly
257
Q

Mechanism responsible for angina pain relief with nitrates?

A

Decreased LV wall stress (afterload)

Nitrates-> venodilate-> decrease preload/ blood in heart-> decrease demand on the heart (heart has to pump less blood) to match decreased supply (heart fed less blood) from angina

*they also vasodilate, resulting in less pressure (afterload) the heart has to pump against

**nitrates-> decreased LV end diastolic volume and wall stress-> decreased myocardial oxygen demand

How well did you know this?
1
Not at all
2
3
4
5
Perfectly
258
Q

What is a proctocolectomy?

A

Surgical removal of the rectum and colon

Done in FAP and UC, for example

How well did you know this?
1
Not at all
2
3
4
5
Perfectly
259
Q

Kid has FAP (familial adenomatous polyposis). How do you manage it?

A

Frequent colonoscopic surveillance

  • screening sigmoidoscopies for kids starting at 10-12 yrs
  • once you detect adenoma (or if >50 yrs) do annual colonoscopies
  • do proctocolectomy (surgical removal of rectum + colon) if they initially present bad with high-grade dysplasia or adenomas or once they get into their 20’s (prophylactic since nearly 100% FAP patients will get colon cancer)
How well did you know this?
1
Not at all
2
3
4
5
Perfectly
260
Q

Lady has sudden onset left-sided weakness. Has positive Babinski. Has fatigue and a mitral regurg murmur (holosystolic at apex). ANA (antinuclear antibody) and RPR (rapid plasma regain) are positive. Diagnosis?

A

Antiphospholipid antibody syndrome
(These antibodies are present in SLE/ lupus and we call it a syndrome if you also have blood clots or miscarriages/ placental problems)

-sudden-onset left-sided weakness= stroke
(Babinski= UMN signs)
-fatigue and mitral regurg are lupus symptoms (antibodies can damage valves-> exposure of subendothelial collagen-> vegetations of platelets/ clotting factors form)
-ANA is non-specific to autoimmune dz’s
-false positive Syphilis (RPR/ VDRL) is a thing with antiphospholipid antibody syndrome (*as well as high PTT)

How well did you know this?
1
Not at all
2
3
4
5
Perfectly
261
Q

What 2 types of pneumonia should you be suspicious of in an immunocomprimised pneumonia patient?

A
  1. CMV (cytomegalovirus)

2. PCP (pneumocystis)

How well did you know this?
1
Not at all
2
3
4
5
Perfectly
262
Q

Tachycardia…
Narrow QRS, what is it?
Wide QRS, what is it?

A

Narrow QRS—> SVT (supraventricular tachycardia) or A-fib
*In SVT, the fast rhythm is coming from the atria. QRS is narrow (shorter ventricular contraction) bc it the atria send a rapid signal to the ventricles-> ventricles contract faster, keeping up with incoming signals.

Wide QRS—> V-tach (ventricular tachycardia) or Torsades
*In V-tach, the fast rhythm is coming from a rapid-firing pacemaker in the ventricles. QRS is wide (takes longer for ventricular contraction) bc it takes longer for a signal to go from ventricular myocyte to myocyte over the normal conduction pathway. That said, since it’s not paying attention to atrial signals and the ventricle is just going crazy on its own, the overall pace is fast/ tachy.

How well did you know this?
1
Not at all
2
3
4
5
Perfectly
263
Q

How do you treat SVT (supraventricular tachycardia) (narrow QRS) if the patient is stable? Unstable?

A

Stable SVT—> Adenosine (x3, rate control)

Unstable SVT—> shock

  • Sketchy: Adenosine (swing dancing) is the agent of choice of SVT
  • *Valsalva, carotid sinus massage, breath holding, and immersion in cold water may help temporarily by lowering HR
How well did you know this?
1
Not at all
2
3
4
5
Perfectly
264
Q

How do you treat V-tach (wide QRS) if the patient is stable? Unstable?

A

Stable V-tach—> Amiodarone

Unstable V-tach—> shock

How well did you know this?
1
Not at all
2
3
4
5
Perfectly
265
Q

Panic attacks usually cause what type of tachycardia?

A

Sinus tachycardia

Normal P waves followed by QRS’s
*do NOT cause narrow QRS= supraventricular tachy (PSVT) or wide QRS= V-tach

How well did you know this?
1
Not at all
2
3
4
5
Perfectly
266
Q

In paroxysmal supraventricular tachycardia (PSVT) will you see P-waves?

A

No, probably not

You get narrow QRS complexes and can have “hidden P-waves”

How well did you know this?
1
Not at all
2
3
4
5
Perfectly
267
Q

Lady had an MI of the LAD. Got a stent. On day 4 of hospitalization, she develops SOB, confusion, cold extremities, JVD, crackles. Has a loud systolic murmur at the left sternal border with a palpable thrill. What happened?

A

Rupture of the interventricular septum

  • can happen 3-5 post-MI of the LAD or RCA
  • when the interventrciular septum ruptures you basically get a VSD with flow from the LV-> RV, which explains the holosystolic murmur and cardiogenic shock presentation (too much backward flow of blood, not enough forward flow)
How well did you know this?
1
Not at all
2
3
4
5
Perfectly
268
Q

Where is aortic regurg heard best?

A

The left sternal border

(Not the usual right 2nd intercostal space where you hear many aortic murmurs echo best…aortic regurg is actually heard best directly over the aortic valve)

How well did you know this?
1
Not at all
2
3
4
5
Perfectly
269
Q

What should you think of when you hear flashes, floaters, or curtain across vision?

A

Retinal detachment

How well did you know this?
1
Not at all
2
3
4
5
Perfectly
270
Q

Does the retina share a blood supply with the macula?

A

No! They have their own blood supply

How well did you know this?
1
Not at all
2
3
4
5
Perfectly
271
Q

Patient has both a metabolic acidosis from AKI (kidneys not working to reabsorb bicarb-> low bicarb in blood) and respiratory acidosis from hypoventilation (breathe less-> blow off less CO2-> high CO2 in blood). The patient feels lethargic. What is the lethargy due to?

A

The respiratory acidosis/ CO2 retention (hypercapnia)

How well did you know this?
1
Not at all
2
3
4
5
Perfectly
272
Q

Why can AKI cause metabolic acidosis?

A

The kidneys aren’t working to reabsorb bicarb-> less bicarb in the blood

How well did you know this?
1
Not at all
2
3
4
5
Perfectly
273
Q

What is the CURB-65 scoring?

A

MD Calc tool to determine whether a pneumonia patient should be admitted to the hospital

C- confusion (+1)
U- urea >20 (+1)
R- respirations >30/ min (+1) 
B- BP, hypotensive <90/60 (+1)
Age >65 (+1) 

If CURB-65 score is 3 or greater—> admit to hospital (*if 1-2 you might still admit)

**basically you’re looking at their vitals (is this a sepsis-like picture, really sick patient?) to determine if they need inpatient vs outpatient care

How well did you know this?
1
Not at all
2
3
4
5
Perfectly
274
Q

What should you give a patient with CAP admitted to the hospital floor (bc they are really sick)?

A

Beta-lactam + macrolide (ex: Ceftriaxone + Azithromycin)

OR

Fluoroquinolone (ex: Moxifloxacin)

How well did you know this?
1
Not at all
2
3
4
5
Perfectly
275
Q

Reg respiratory settings on a ventilator, what 2 things should you think of changing when oxygenation is a problem? Ventilation?

A

Oxygenation is a problem—> consider changing FiO2 or PEEP (the percent oxygen in the air you are giving to the person or the positive end expiratory pressure to keep the alveoli from collapsing)

Ventilation is a problem—> consider changing Tv (tidal volume) (mL of air the patient is getting in/out) or the RR (respiratory rate) (how fast they are breathing/ blowing off CO2)

How well did you know this?
1
Not at all
2
3
4
5
Perfectly
276
Q

Patient has been taking Naproxen (Aleve) for years to manage chronic back pain. Labs show her kidney function is getting worse. What is the cause of her renal failure?

A
Analgesic nephropathy (too many NSAIDs damaged the kidneys)
-most often—> Tubulointerstitial nephritis or Papillary necrosis 

*Naproxen (Aleve)= NSAID
Remember that NSAIDs block PG’s (PG’s dilate the afferent)—> constriction of afferent—> decreased GFR and RPF (decrease blood flow to kidneys/ put more pressure on the kidneys)

**chronic NSAID use also can cause premature aging, atherosclerotic valvular disease, and urinary tract cancer

How well did you know this?
1
Not at all
2
3
4
5
Perfectly
277
Q

If you suspect a chest pain patient may have aortic dissection, should you give them aspirin?

A

NO! Aspirin is an antiplatelet agent-> stops clotting, and therefore causes bleeding, which would worsen aortic dissection. In fact, aspirin can kill a patient with aortic dissection.

How well did you know this?
1
Not at all
2
3
4
5
Perfectly
278
Q

Mechanism of action of Aspirin (ASA)?

A

Blocks COX-1 (and COX-2)—> decreased production of TXA2 (thromboxane A2) (*and PGs) within platelets—> decreased platelet aggregation
(It is an NSAID/ antiplatelet agent that stops platelets from sticking)

How well did you know this?
1
Not at all
2
3
4
5
Perfectly
279
Q

What should you give chest pain patients with possible ACS (acute coronary syndrome) (and low risk for aortic dissection) ASAP?

A

Aspirin

Blocks COX-1-> TXA2-> platelet aggregation

How well did you know this?
1
Not at all
2
3
4
5
Perfectly
280
Q

Guy has fatigue, lower extremity edema, dark urine. U/A shows protein and blood in the urine. Kidney biopsy shows dense deposits in the glomerular BM. Immunofluorescene is positive for C3, not Ig’s. Diagnosis?

A

Membranoproliferative glomerulonephritis (nephrotic syndrome)

*complement gets activated/ used up—> decreased C3 protein

How well did you know this?
1
Not at all
2
3
4
5
Perfectly
281
Q

FEV1/FVC ratio in restrictive vs obstructive lung dz?

A

Restrictive= problem getting air IN
FEV1 low/ FVC low= NORMAL or HIGH (less air in means less air out)

Obstructive= problem getting air OUT
FEV1 lower/ FVC low= LOW

*remember FEV1= max air you can blow out in 1 second; FVC= max air you can blow out after max inspiration

How well did you know this?
1
Not at all
2
3
4
5
Perfectly
282
Q

Middle aged guy has SOB on exertion. CXR shows mediastinal fullness and scattered reticular opacities in the upper lungs. Labs show calcium is high. Diagnosis?

A

Sarcoidosis-associated lung disease (restrictive pattern, so problem breathing in)

*Clues: he’s young, “mediastinal fullness and scattered reticular opacities”= hilar lymphadenopathy, hypercalcemia (vit D conversion by lung macrophages)

How well did you know this?
1
Not at all
2
3
4
5
Perfectly
283
Q

Are the following values high, low, or normal in sarcoidosis-associated lung disease?
FEV1
TLC (total lung capacity)
DLCO (diffusion capacity for carbon monoxide)

A

Restrictive= problem breathing in (less air in= less air out)

FEV1 (max air you can breathe out in 1 sec)= normal to low

TLC= low (problem breathing in, so lungs hold less air)

DLCO= low (inflammation and scarring interfere with gas exchange)

How well did you know this?
1
Not at all
2
3
4
5
Perfectly
284
Q

In ARDS, would you want to put the patient on high or low tidal volume (Vt) ventilator support?

A

Low tidal volume ventilation (LTVV)

-to prevent overdistention of alveoli

How well did you know this?
1
Not at all
2
3
4
5
Perfectly
285
Q

Lady who recently underwent chemo presents with right-sided severe abdominal pain that is constantly burning. No other symptoms, but when you touch the area it hurts a lot and no NSAIDs or antacids have helped. Likely diagnosis?

A

Herpes Zoster (Shingles)

*The pain can come before the vesicular rash by several days!

How well did you know this?
1
Not at all
2
3
4
5
Perfectly
286
Q

Patient with surgical history of gastrectomy presents with fatigue and SOB on exertion. He has a shiny tongue, low Hb, high LDH. Diagnosis?

A

Vitamin B12 deficiency

  • gastrectomy (surgical removal of part or all of the stomach)—> loss of intrinsic factor, which is needed to absorb vit B12–> vit B12 deficiency, which is a macrocytic anemia
  • why hemolysis (inc LDH)? Vit B12 is needed for DNA synthesis. If it is deficient, you get ineffective erythropoiesis (making of RBCs in the bone marrow)-> intramedullary hemolysis
How well did you know this?
1
Not at all
2
3
4
5
Perfectly
287
Q

Lady is having difficulty swallowing solid foods (not liquids). Lost weight, has oral thrush, dental caries, and firm submandibular nodules. Only 1 sexual partner. Next step to diagnose?

A

Get antibodies to Ro/SSA and La/SSB

  • this sounds like Sjogren syndrome (autoimmune disorder of exocrine glands)—> dry mouth (leading to difficulty swallowing, thrush, and dental carries)
  • to diagnose you need (1) evidence of dry mouth/ eyes (positive Schirmer test for decreased lacrimation) and (2) histologic evidence of lymphocytic infiltration of the salivary glands OR serum antibodies against SSA (Ro) and/or SSB (La)
How well did you know this?
1
Not at all
2
3
4
5
Perfectly
288
Q

Old guy with hx of metastatic squamous cell cancer has had low back pain for 2 months. In the last 2 weeks, he’s had progressive leg weakness and dribbling of urine. He has decreased strength and sensation in the lower extremities. Diagnosis?

A

Cauda equina syndrome

  • compression of 2+ spinal nerve roots in the lumbar cistern (in his case, likely due to a tumor *other cases: large lumbar disc herniation or abscess in the region)
  • the urinary incontinence is due to damage to S3-S5 (can also get sexual dysfunction)
  • get an urgent MRI of the lumbosacral spine
  • emergency surgical decompression within 1-2 days is required to prevent irreversible neuro damage
How well did you know this?
1
Not at all
2
3
4
5
Perfectly
289
Q

Most common cause of primary adrenal insufficiency in developed countries (US)?

A

Autoimmune adrenalitis (autoantibodies against the adrenal gland)

How well did you know this?
1
Not at all
2
3
4
5
Perfectly
290
Q

If you suspect lupus, what antibody should you check first?

A

ANA (antinuclear antibody)

-this is sensitive, so it is the best screening test. Anti-dsDNA and anti-Sm (smith) antibodies are specific, so they are confirmatory tests.

How well did you know this?
1
Not at all
2
3
4
5
Perfectly
291
Q

What disease do you associate each of the following antibodies with?

  1. Anti-cyclic citrullinated peptide
  2. Anti-histone
  3. Anti-neutrophil cytoplasmic (ANCA)
  4. Anti-Ro/SSA
  5. Anti-Scl-70 (anti-topoisomerase)
A
  1. Anti-cyclic citrullinated peptide—> Rheumatoid arthritis
  2. Anti-histone—> drug-induced lupus (from hydralazine, procainamide, etc.)
  3. Anti-neutrophil cytoplasmic (ANCA)—> vasculitis
  4. Anti-Ro/SSA—> Sjogren syndrome
  5. Anti-Scl-70 (anti-topoisomerase)—> scleroderma (systemic sclerosis)
How well did you know this?
1
Not at all
2
3
4
5
Perfectly
292
Q

What is serositis?

A

Inflammation of the serous membranes in the body (for example, the pleural lining of the lungs or the pericardium lining of the heart)

  • can cause positional chest pain and SOB
  • if this is part of the presentation along with fatigue, polyarthritis, etc. you would suspect lupus (SLE)
How well did you know this?
1
Not at all
2
3
4
5
Perfectly
293
Q

When/ how often should you do colonoscopy screening for patients with Ulcerative Colitis?

A

8 years after initial diagnosis and every 1-2 years thereafter

How well did you know this?
1
Not at all
2
3
4
5
Perfectly
294
Q

Initial treatment of frostbite?

A

Rapid rewarding with warm water (37-39C/ 98.6-102.2F)
*also give pain meds (rewarming tissue is painful)

*frostbite= freezing of tissue-> disruption of cell membranes, ischemia, vascular thrombosis, inflammatory changes (usually involves face, ears, distal limbs)

**do NOT do hot air rewarming (not good enough control over the temp) or rewarm by placing the limb near your abdomen in the field if there’s the possibility of refreezing before getting definitive care (freezing-> warm-> freezing can cause more damage than just freezing-> warm)

How well did you know this?
1
Not at all
2
3
4
5
Perfectly
295
Q

Lady twisted her ankle a month ago. The swelling improved, then got worse over the last couple weeks. Her foot, ankle, and calf are really painful now all the time. There is erythema, warmth, edema up to the calf and imaging shows patchy area of osteopenia. Diagnosis?

A

Complex regional pain syndrome

  • can happen 4-6 wks post-trauma due to inflammatory cytokine effects
  • causes burning/ tingling pain out of proportion to the injury
  • associated with erythema, edema, skin changes, X-ray may show patchy demineralization
How well did you know this?
1
Not at all
2
3
4
5
Perfectly
296
Q

Lady on OCPs explains she has been having severe migraines with aura. What should you tell her to do?

A

Give off the OCPs!

Women with migraines, esp with aura, are at increased risk for ischemic stroke. OCPs (and other estrogen-containing contraceptives) increase risk for stroke too. So you don’t want to have both these stroke risk factors at once! OCPs are contraindicated in migraine sufferers for this reason.

How well did you know this?
1
Not at all
2
3
4
5
Perfectly
297
Q

How do you treat symptomatic prolactinoma?

A

Dopamine agonist (Cabergoline, Bromocriptine)

*remember that dopamine puts the break on prolactin, so by giving a dopamine agonist, you are stopping prolactin (helps normalize prolactin levels and reduce tumor size)

**only do surgery (transsphenoidal resection) if the tumor is really large (>3 cm) or not responding to meds

How well did you know this?
1
Not at all
2
3
4
5
Perfectly
298
Q

Patient was diagnosed with adrenal insufficiency and got put on hydrocortisone. She continues to have light-headedness and salt cravings. Next step in her management?

A

Add fludrocortisone (a synthetic mineralcorticoid/ aldosterone)

-she is already taking hydrocortisone (a synthetic glucocorticoid/ cortisol)…since this doesn’t do much for the mineralcorticoid/ aldosterone deficiency, she gets those symptoms (hypOnatremia/ salt cravings/ light-headedness, hyperkalmeia)

How well did you know this?
1
Not at all
2
3
4
5
Perfectly
299
Q

Guy was wearing contact lenses for a week straight. Is having difficulty removing the contact lenses now. He woke up with eye pain and discharge. The cornea is hazy and there is sclera injection (red/ bloodshot). Diagnosis?

A

Contact lens associated keratitis
-usually due to Pseudomonas and serratia (gram-neg) and require removal of contacts and antibiotics emergently! (Otherwise can-> corneal perforation, scarring, permanent vision loss)

How well did you know this?
1
Not at all
2
3
4
5
Perfectly
300
Q

You do a CXR on a girl with a shoulder sprain and find an incidental coin-sized lesions on the upper lobe of her lung. 1st step to work it up?

A

Compare to a previous CXR

Most likely it’s a solitary pulmonary nodule (SPN)= benign nodule

  • no further testing if it’s stable in size >2 yrs
  • if it’s changed, move on to a CT and continue to rule out or rule in cancer
How well did you know this?
1
Not at all
2
3
4
5
Perfectly
301
Q

Patient has rash, joint pain, fatigue. Has palpable purpura, hepatosplenomegaly. Bun and Cr are elevated, complement is low, anti-HCV (hep C) and Rheumatoid factor are both positive. Diagnosis?

A

Mixed cryoglobulinemia

Vasculitis (from immune complexes)

  • palpable purpura on legs, joint pain, liver involvement, kidney disease/ glomerulonephritis
  • associated with hep C and hypocomplementemia (depleted complement levels)
How well did you know this?
1
Not at all
2
3
4
5
Perfectly
302
Q

What is an AV fistula (used in dialysis) and how can it cause high-output cardiac failure?

A

AV fistulas connect the arteries to the veins directly

Rather than the normal path: arteries-> arterioles-> capillaries-> venous system, AV fistulas directly connect arteries-> veins (blood can dump right into the venous system at the top of the arm where the fistula is placed for dialysis access). Now the lower arm isn’t getting good perfusion-> heart makes up for it by pumping harder. This constant increase in contractility (heart having to work harder)-> high-output cardiac failure.

How well did you know this?
1
Not at all
2
3
4
5
Perfectly
303
Q

Upper abdominal systolic-diastolic bruit indicates what?

A

Renal artery stenosis

**A bruit does not usually indicate AAA (abdominal aortic aneurysm)- this would be a pulsatile mass (in <50% of patients) or picked up on screening Doppler U/S.

How well did you know this?
1
Not at all
2
3
4
5
Perfectly
304
Q

Reduced systemic vascular resistance (SVR) means what?

A

Vasodilation

How well did you know this?
1
Not at all
2
3
4
5
Perfectly
305
Q

Most patients with AAA (abdominal aortic aneurysm) present with what symptoms?

A

Most are asymptomatic! (Trick Q)

How well did you know this?
1
Not at all
2
3
4
5
Perfectly
306
Q

Guy has CABG (coronary artery bypass grafting) and aortic valve replacement done. 2 days later gets sudden-onset weakness, chest tightness, SOB. He is hypotensive and tachy. EKG shows irregularly irregular rhythm. Diagnosis and treatment?

A

A-fib

Cardiovert (it is <48 hrs plus he is hemodynamically unstable)

How well did you know this?
1
Not at all
2
3
4
5
Perfectly
307
Q

Defibrillation vs. cardioversion?

A

Defibrillation= unsynchronized shock

  • “drops a bomb”/ shocks at a random point in the cardiac cycle
  • use for V-fib and V-tach (Rap: “defib for V-fib and pulseless V-tach. Don’t defib asystole you won’t bring em back”)

Cardioversion= SYNCHRONIZED shock (press that sync button)

  • shock is given at the QRS complex (purpose is to minimize shock occurring during depolarization, which could throw the patient into V-fib)
  • example: use for a-fib if unstable, onset <48 hrs, or in a young person with a structurally normal heart after anti-coagulating with Warfarin for 3 weeks and do TEE right before to make sure there’s no clots (don’t want to throw a clot with the CV-> stroke)
How well did you know this?
1
Not at all
2
3
4
5
Perfectly
308
Q

Lady with HTN (on Lisinopril) and OSA (on CPAP during sleep) presents with leg swelling, worse in evenings. She has pitting edema in the bilateral ankles, dilated and tortuous superficial veins, and a small ulcer noted on the left medial ankle. All pulses are normal. Diagnosis and solution?

A

Chronic venous insufficiency (CVI)
(Incompetent venous valves-> backflow of blood and pooling in legs)
*ankles often appear hyperpigmented, may have ulcers

Leg elevation, compression stockings, exercise
**patients who don’t respond to these conservative treatments should get venous duplex U/S to confirm the diagnosis (by retrograde venous blood flow)

How well did you know this?
1
Not at all
2
3
4
5
Perfectly
309
Q

What 4 drugs improve survival in patients with LV systolic HF?

A
  1. ACE inhibitors
  2. ARBs
  3. Beta-blockers
  4. Mineralcorticoid receptors antagonists (Spironolactone/ Eplerenone)

*also hydralazine + nitrates in African American patients

How well did you know this?
1
Not at all
2
3
4
5
Perfectly
310
Q

What are electrical alternans?

A

Varying amplitude (height) of QRS complexes

Suggests cardiac tamponade

How well did you know this?
1
Not at all
2
3
4
5
Perfectly
311
Q

Holosystolic murmur beast heard over the cardiac apex with radiation to the axilla

A

Mitral regurg

*clinical features: SOB on exertion (dec CO due to the regurg and inc LA pressure) and fatigue…can lead to a-fib (stretched LA) and HF

How well did you know this?
1
Not at all
2
3
4
5
Perfectly
312
Q

Man has MI of his LAD and gets a stent placed. 10 days later presents with another MI (mid sternal chest pain, diaphoresis, SOB) with ST-elevation in leads V1-V4. Cause of this?

A

Medication non adherence

He’s having stent thrombosis. To prevent this, patients should be on ASA + Clopidogrel (Plavix)= dual anti-platelet therapy (DAPT). Most common cause of stent thrombosis= not taking these meds as instructed.

How do you know this is stent thrombosis, not a recurrent MI? LAD= leads V1-V4 where the stent was placed! So this is far more likely.

How well did you know this?
1
Not at all
2
3
4
5
Perfectly
313
Q

Young guy with HTN and alcohol abuse comes into the ED with SOB for 2 days, worse last night. BP is 220/110. He has crackles, S4, and cotton-wool spots on eyes. Cr is 2.1. He is given IV furosemide + Nitroprusside with improvement of symptoms but the next day he has a seizure. Diagnosis?

A

Hypertensive emergency—> next day Cyanide (CN) toxicity

Really high BP + end-organ damage (CHF, renal failure, and eye signs/ symptoms)
*cotton-wool spots= white patches on retina (high pressure damages blood supply to the nerve fibers)
IV Nitroprusside is given in HTN emergency, but gives off nitric oxide and cyanide byproducts. CN toxicity= neurologic change (seizure, altered mental status, lactic acidosis, coma)

How well did you know this?
1
Not at all
2
3
4
5
Perfectly
314
Q

Man with CAD and HTN is brought to the ED after a suicide attempt. BP is 75/40, HR is 40 bpm. Extremities are cold and clammy. EKG shows sinus Brady with 1st degree AV block. He is given IV fluids + Atropine w/o improvement. Next thing to give him?

A

GLUCAGON
-Bradycardia, AV block, hypotension and wheezing suggests beta-blocker overdose

*Remember that beta-blockers dec HR (block beta-1 on heart), can cause AV block (AV node is slowing the signal from atria to ventricles too much), can cause hypotension (block beta-1, which causes renin release), and can cause wheezing (block beta-2, which bronchodilates-> bronchoconstriction)

**extremities are cold due to peripheral vasoconstriction (inc BP, dec blood flow) to shunt blood to centrally located vital organs

***also, the fact he has CAD and HTN tells you he prob was prescribed a beta-blocker and has it available to overdose on in a suicide attempt

How well did you know this?
1
Not at all
2
3
4
5
Perfectly
315
Q

Guy with CAD, hx of MI comes in for retrosternal chest pain and burning, worse at night (not w/ exercise). He has cough and occasional hoarseness. Resting EKG is normal. Exercise EKG shows ST depression in the inferior leads, but myocardial perfusion study shows no evidence of stress-induced ischemia. Treatment?

A

PPI (like Omeprazole)
-this is GERD

  • GERD symptoms mimic angina, worse at night bc acid can reflux easier when laying down
  • cough and hoarseness from laryngeal irritation (from the acid)

*Although the exercise EKG shows ST depression (subendocardial ischemia), evidence of ischemia when exercising at 90% of maximal HR is NOT unusual in patients with known CAD

How well did you know this?
1
Not at all
2
3
4
5
Perfectly
316
Q

In terms of the EKG, how is uremic pericarditis different from other etiologies of pericarditis?

A

Does not typically cause diffuse ST elevation (or PR depression)

  • this is bc the inflammation doesn’t affect the myocardium
  • *uremic pericarditis presents in patients with advanced renal failure requiring dialysis (they come in with pleuritic chest pain and pericardial friction rub)
How well did you know this?
1
Not at all
2
3
4
5
Perfectly
317
Q

What is the CHADS-VASC score? What do each of the letters stand for and how do you interpret the results?

A

CHADS-VASC score is used to determine whether an a-fib patient should be started on lifelong anticoagulation

C- CHF
H- HTN
A- Age >75 (*2 points) 
D- DM
S- Stroke/ TIA (*2 points)
V- Vascular dz (prior MI, PAD, aortic plaque) 
A- Age 65-74
Sc- Sex (female) 
*everything positive counts as 1 point except age >75 and stroke count as 2 points 

0–> no anticoagulation
1–> none, ASA, or oral anticoagulation
2+-> oral anticoagulation

How well did you know this?
1
Not at all
2
3
4
5
Perfectly
318
Q

44 year old guy with no medical hx presented with new-onset a-fib w/ RVR. Echo showed mildly dilated LA, but no valvular abnormalities. Should we have him on long-term anticoagulation, like ASA and/or Warfarin?

A

NO
His CHADSVASc score is 0 so it is not indicated.

C- CHF
H- HTN
A- Age >75 (*2 points) 
D- DM
S- Stroke/ TIA (*2 points)
V- Vascular dz (prior MI, PAD, aortic plaque) 
A- Age 65-74
Sc- Sex (female) 
*everything positive counts as 1 point except age >75 and stroke count as 2 points 

0–> no anticoagulation
1–> none, ASA, or oral anticoagulation
2+-> oral anticoagulation

How well did you know this?
1
Not at all
2
3
4
5
Perfectly
319
Q

Pregnant lady present with SOB, S3, and holosystolic murmur at the apex. U/A shows trace protein and she has a little ankle edema. EKG shows sinus tachy. Next best step?

A

Echocardiogram

You need to check for peripartum cardiomyopathy (dilated cardiomyopathy that can occur during or up to 5 mo. after pregnancy). It is often associated with mitral regurg (holosystolic murmur at the apex).

How well did you know this?
1
Not at all
2
3
4
5
Perfectly
320
Q

Patient comes in after a car crash. Vitals are stable and physical exam is unremarkable. 30 min after arrival, his BP is 80/50 and HR is 120. Now he is having trouble breathing and has a erythematous rash and wheals over his chest and abdomen. Most likely cause?

A

Latex allergy (anaphylactic shock)

How well did you know this?
1
Not at all
2
3
4
5
Perfectly
321
Q

Fat embolism presents with what type of rash? How long does is occur after a long-bone fracture?

A

Petechial rash (also SOB and confusion)

12-24 hrs later! Not immediate

How well did you know this?
1
Not at all
2
3
4
5
Perfectly
322
Q

An aortic regurg patient is asymptomatic. How does his heart adapt to explain this?

A

Increased LV compliance

(The LV undergoes eccentric hypertrophy of myocardial fibers in series, meaning it stretches out/ dilates in response to the backflow of blood)

How well did you know this?
1
Not at all
2
3
4
5
Perfectly
323
Q

We start a patient on a statin if their ASCVD (atherosclerotic cardiovascular disease) score is what?

A

> 7.5-10%

Depends on the resource you’re looking at

How well did you know this?
1
Not at all
2
3
4
5
Perfectly
324
Q

Most common cardiac manifestation of lupus (SLE)?

A

Pericarditis

(*typically presents as pleuritic chest pain, pericardial friction rub, and diffuse EKG changes w/ other SLE symptoms such as joint pain)

How well did you know this?
1
Not at all
2
3
4
5
Perfectly
325
Q

Systolic murmur at the sternal border

A

Tricuspid regurg

How well did you know this?
1
Not at all
2
3
4
5
Perfectly
326
Q

Normal range for total bilirubin?

A

0.1-1.0

How well did you know this?
1
Not at all
2
3
4
5
Perfectly
327
Q

Alcoholic has obvious signs of cirrhosis (ascites, spider angiomas, hemorrhoids, AST:ALT >2:1). Do you need to do a liver biopsy to confirm the diagnosis of cirrhosis?

A

No
-though liver biopsy is the gold standard for diagnosis of cirrhosis, if the history and physical is obvious for cirrhosis, do not put the patient through this

*you also don’t need to do unnecessary work-up for rare causes of cirrhosis like alpha-1 antitrypsin deficiency or hemochromatosis. This guy is an alcoholic so you can say with confidence that the cirrhosis resulted from alcohol intake.

How well did you know this?
1
Not at all
2
3
4
5
Perfectly
328
Q

What screening test should you do in a patient with liver cirrhosis?

A

Endoscopy (EGD= EsophagoGastroDuodenoscopy)

-you need to check for esophageal varices, determine their risk for variceal hemorrhage, and come up with strategies for prevention of variceal hemorrhage (these patients bleed so much it’s life threatening!)

How well did you know this?
1
Not at all
2
3
4
5
Perfectly
329
Q

How do you manage acute diverticulitis (presents as LLQ pain and confirmed by CT)? Be specific about the antibiotics used. What do you need to follow up with?

A

Antibiotics- Ciprofloxacin + Metroniadazole (Flagyl)
And bowel rest

Follow up with colonoscopy (4-8 wks later) since colon cancer can mimic the presentation and CT findings of diverticulitis!
*also advice patients to increase fiber intake and exercise and lose weight and stop eating red meats, smoking, and straining on the toilet

**Cipro is a fluoroquinolone that is broad spectrum and good for gram neg GI bugs + Metronidazole is good for anaerobic coverage

How well did you know this?
1
Not at all
2
3
4
5
Perfectly
330
Q

50 y.o. heavy alcoholic presents with SOB, even at rest. He has an S3 and crackles. CXR shows pulmonary venous congestion. Echo shows dilated LV and EF of 25%. He has no history of any heart problems. What is the most likely cause of his symptoms?

A

Alcoholic dilated cardiomyopathy

  • he has evidence of systolic dysfunction in the setting of alcohol abuse. Know that dilated CM from alcohol is a dx of exclusion (he had no prior CAD or valvular heart dz and heavy alcoholism is evident)
  • abstinence from alcohol should improve or normalize LV function over time
How well did you know this?
1
Not at all
2
3
4
5
Perfectly
331
Q

Peripheral vascular dz vs. acute arterial occlusion? What are they

A

Peripheral vascular dz- blood flow is reduced to a limb (same concept as angina)

Acute arterial occlusion- blood flow is cut off to a limb (same concept as MI, emergency!)

How well did you know this?
1
Not at all
2
3
4
5
Perfectly
332
Q

Old man ex-smoker has worsening cramping in his left leg. He has decreased pulses in the left leg and a reduced ankle-brachial index. Diagnosis and treatment?

A

Peripheral artery disease

Treat with smoking cessation, graduated exercise program (walk until claudicaiton/ pain from ischemia, then rest, then do 1 more cycle), atherosclerotic risk factor reduction, avoid temperature extremes, aspirin + a stain, can also give Clopidogrel, cilostazol (surgery- angioplasty or bypass grafting- if severe)

  • this usually occurs with CAD/ is a manifestation of atherosclerotic cardiovascular dz (ASCVD)
  • *do not confuse with acute arterial occlusion- this is an emergency where blood flow through an artery to a limb is fully cut off! Presents with 6 P’s: pain, pallor, polar (cold), paralysis, paresthesias, and pulselessness (not just reduced pulse/ blood flow…none at all!)
How well did you know this?
1
Not at all
2
3
4
5
Perfectly
333
Q

Patient complains of on/off epigastric abdominal pain for 1 year, worse with eating. Hb is low, MCV is >100. Diagnosis?

A

Autoimmune gastritis—> pernicious anemia

(Antibodies against parietal cells—> cannot absorb vit B12–> macrocytic anemia)

*look for hypochlorhydria (decreased stomach acid- since parietal cells produce acid) and elevated gastrin (gastrin stimulates parietal cells to release acid, so it will get up-regulated in response to the low acid as feedback)

How well did you know this?
1
Not at all
2
3
4
5
Perfectly
334
Q

Name some functions of the liver.

A
  1. Synthetic (makes stuff)
    - clotting factors
    - cholesterol
    - proteins (ex: protein c and s, albumin)
  2. Metabolic
    - metabolism of drugs and corticosteroids (P450 system)
    - detox
    - breaks down estrogen
  3. Excretory
    - bile excretion
How well did you know this?
1
Not at all
2
3
4
5
Perfectly
335
Q

40 y.o guy has chronic diarrhea with fat in it (no pathogens or WBCs). He’s given a oral D-xylose solution. Urinary excretion of the D-xylose is low. He is treated with Rifaximin and D-xylose test is repeated but results didn’t change. Diagnosis?

A

He is peeing out a LOW amount of D-xylose (monosaccharide), meaning he has a mucosal reabsorption problem like celiac dz or SIBO…but it’s not SIBO since that’s treated with Rifaximin and that did not help…so it’s celiac
(problem with reabsorption due to flattened villi-> less of the monosaccharide pushed into the blood-> less filtered through the kidneys and peed out)

*patients with malabsorption due to enzyme deficiencies would have normal absorption and urinary output of D-xylose (bc their problem is with enzymes breaking polysaccharides into monosaccharides and D-xylose is already a monosaccharide)

How well did you know this?
1
Not at all
2
3
4
5
Perfectly
336
Q

Explain how the D-xylose test can be used to distinguish pancreatic insufficiency from mucosal causes of malabsorption (celiac dz, bacterial overgrowth, etc.).

A

D-xylose is a monosaccharide and gets fully absorbed into the blood.

Pancreatic insufficiency (not getting enough pancreatic enzymes into the duodenum to break down polysaccharides-> monosaccharides)—since D-xylose is already a monosaccharide it doesn’t need the help of pancreatic enzymes, so all the D-xylose gets appropriately absorbed into the blood and a normal amount is peed out

Celiac dz (flattened villi/ problem at small intestine level with pushing nutrients into the blood/ reabsorption)—so some D-xylose does NOT get reabsorbed into the blood as it should and a low amount is peed out

How well did you know this?
1
Not at all
2
3
4
5
Perfectly
337
Q

Mechanism of spider angiomas and palmar erythema in cirrhosis?

A

Both due to hyperestrinism

Liver not working to break down estrogen—> too much estrogen

How well did you know this?
1
Not at all
2
3
4
5
Perfectly
338
Q

Mechanism of caput medusae in cirrhosis?

A

Caput medusae= dilation of superficial veins on the abdominal wall DUE TO PORTAL HTN
(Backs up portal vein-> periumbilical vein)

How well did you know this?
1
Not at all
2
3
4
5
Perfectly
339
Q

Truck driver comes in with SOB and sharp chest pain for 10 hrs, relieved by taking shallow breaths. BP is normal, HR is tachy, RR is 30. EKG is normal. Most likely diagnosis?

A

PE (pulmonary embolism)

*presents with SOB, pleuritic chest pain, tachypnea, tachycardia (CXR may appear normal)

How well did you know this?
1
Not at all
2
3
4
5
Perfectly
340
Q

Guy with long extremities, flexible joints, and pectus carinatum has sudden-onset chest and neck pain. Diagnosis?

A

Aortic dissection

-this guy has Marfan syndrome (mutation of fibrillin-1-> connective tissue integrity problems throughout the body, including aortic root dilation)

How well did you know this?
1
Not at all
2
3
4
5
Perfectly
341
Q

20 y.o guy is having depression and involuntary movements. Total bili, direct bili, AST, and ALT are elevated. Diagnosis?

A

Wilson disease

  • rare AR disease in which copper transport is impaired—> copper accumulation in tissues
  • consider this when there is liver dysfunction + neuro psych symptoms!
How well did you know this?
1
Not at all
2
3
4
5
Perfectly
342
Q

Which has a greater impact on BP reduction: following the DASH diet or reducing sodium in your diet?

A

DASH diet

*the following can decrease BP by this much: 
Weight loss—5-20 per 10kg loss
DASH diet—8-14
Exercise—4-9
Dietary sodium reduction—2-8
Limiting alcohol—2-4

**tell patients to do all of the above, but losing weight and eating a DASH diet (all-around healthy diet rich in fruits and veggies, low in saturated and total fats) is key to lowering BP, more so than limiting salt

How well did you know this?
1
Not at all
2
3
4
5
Perfectly
343
Q

Patient has S3 and SOB. Best initial therapy?

A

Loop diuretics

He has LV failure (backing up to lungs). Taking off fluid is the first step in providing symptomatic relief.

*yes, drugs like beta-blockers are helpful in the long-term and reduce mortality, but this wouldn’t be the best initial therapy—wouldn’t take the excess fluid off to make the patient feel better

How well did you know this?
1
Not at all
2
3
4
5
Perfectly
344
Q

What heart problem can amyloidosis cause?

A

Restrictive cardiomyopathy

Amyloidosis= misfolded protein that deposits around the body

How well did you know this?
1
Not at all
2
3
4
5
Perfectly
345
Q

Diarrhea + leukocytosis. Think of what?

A

C diff

How well did you know this?
1
Not at all
2
3
4
5
Perfectly
346
Q

What is SBP?

A

Spontaneous bacterial peritonitis

-infection of the ascitic fluid

  • presents as abdominal discomfort in a cirrhosis patient, possibly mental status change
  • diagnose by PMN (neutrophil) count >250 + positive peritoneal fluid culture
How well did you know this?
1
Not at all
2
3
4
5
Perfectly
347
Q

Woman with SLE on Prednisone comes in for fever, chills, fatigue, cough, and SOB. Pulse ox is 86% and she’s using accessory muscles to breathe. LDH is elevated. CXR shows bilateral interstitial infiltrates. Diagnosis?

A

PCP pneumonia

*it’s associated with AIDS, but also immunosuppressed people (she’s taking glucocorticoids/ steroids, which block the immune system)

How well did you know this?
1
Not at all
2
3
4
5
Perfectly
348
Q

Patient has burning chest pain for 2 hrs. Lungs sound clear. EKG shows ST elevation in leads II, III, and aVF. You give him nitroglycerin. A few minutes later, he’s lightheaded and his BP dropped from normal to 75/50. What happened? Next step?

A

ST elevation in II, III, aVF= inferior STEMI (occlusion of the RCA). Inferior wall MI’s have a 50% chance of involving the RV. Since he has clear lungs (meaning left heart is good), it probably did involve the right heart.

In a right sided MI, you don’t give nitrates bc the right heart is preload dependent and this will drop BP too much!

Since you already gave nitros and his BP did drop too much, give a normal saline bolus (IV fluids) to get the preload back up (which will improve CO and BP).

How well did you know this?
1
Not at all
2
3
4
5
Perfectly
349
Q

Normal CD4+ count?

A

> 500 (*10^6/ L)

How well did you know this?
1
Not at all
2
3
4
5
Perfectly
350
Q

Does it help if alcoholic cirrhosis patients stop drinking?

A

Yes! Alcoholic cirrhosis-> liver inflammation after drinking involving fibrosis and nodule formation. This fibrogenesis improves with abstinence from alcohol.
Although advanced cirrhosis is irreversible, avoiding alcohol still carries a huge survival benefit.

How well did you know this?
1
Not at all
2
3
4
5
Perfectly
351
Q

How does disseminated MAC (mycobacterium avium complex) infection present?

A

Constitutional (fever, night sweats, fatigue, malaise, weight loss) and GI (diarrhea, abdominal pain) symptoms

How well did you know this?
1
Not at all
2
3
4
5
Perfectly
352
Q

Normal AST? Normal ALT?

A

8-40 is the normal range for both AST and ALT

How well did you know this?
1
Not at all
2
3
4
5
Perfectly
353
Q

Obese patient has high AST of 80 and high ALT of 90 + mild hepatomegaly. Not an alcoholic. No other signs/ symptoms. Diagnosis?

A

Nonalcoholic fatty liver disease (NAFLD)

  • we see a mild elevation in liver enzymes in these patients (AST/ALT ratio <1) and mild hepatomegaly. Alk phos may also be on the high side. Albumin and bili are typically normal.
  • diagnose by labs and U/S showing hyperechoic texture of the liver (liver biopsy would confirm but we don’t do this for fatty liver)
  • treat with weight loss/ manage metabolic risk factors
How well did you know this?
1
Not at all
2
3
4
5
Perfectly
354
Q

Regarding class I antiarrhythmic drugs, what is “use dependence?”

A

Class I anti-arrhythmic drugs block Na+ channels in depolarization (phase 0)

In patients with faster heart rates, the drug has less time to dissociate from the Na+ channels—> longer effect of the drug (more widening of the QRS interval/ ventricular contraction)

**this is why class I anti arrhythmic drugs (esp class IC) are good at treating supraventricular tachyarrhythmias

How well did you know this?
1
Not at all
2
3
4
5
Perfectly
355
Q

What happens to SVR (systemic vascular resistance) and afterload in HF?

A

SVR goes up (vasoconstriction)
Afterload goes up

*Here’s the cycle of hemodynamics in HF:
Dec contractility-> dec CO-> poor perfusion to kidneys activates RAAS-> vasoconstriction (inc SVR)-> inc afterload (the vasoconstriction/ inc BP increases the pressure the heart now has to pump against)-> decreases the contractility even more since the extra afterload makes it harder to pump…so its a nasty cycle

How well did you know this?
1
Not at all
2
3
4
5
Perfectly
356
Q

Why might a HF patient have mitral regurg (holosystolic murmur at the cardiac apex)?

A

LV dilation (due to decreased CO)/ remodeling—> functional mitral regurg

How well did you know this?
1
Not at all
2
3
4
5
Perfectly
357
Q

What is cardiac index?

A

Basically the same as cardiac output, but takes into account the person’s body weight

*Cardiac Index (CI)= CO/ body surface area

How well did you know this?
1
Not at all
2
3
4
5
Perfectly
358
Q

Septic shock and anaphylactic shock both fall under what category of shock?

A

Distributive shock

“Warm shock” from vasodilation

How well did you know this?
1
Not at all
2
3
4
5
Perfectly
359
Q

Most common cause of mitral regurg?

A

Mitral valve prolapse (MVP)

Myxomatous degeneration of the mitral valve

How well did you know this?
1
Not at all
2
3
4
5
Perfectly
360
Q

Patient with known HFrEF on ASA, Digoxin, Lasix, Metoprolol, Lisinopril, and Atorvostatin presents with worsening SOB and lower extremity edema. You give her IV Lasix and edema improves but a few days later she has a few beats of wide complex V-tach. Next step?

A

Check serum electrolytes!

If a patient is stable in V-tach, you need to find out why there are having this arrhythmia (if unstable defibrillate)! She likely has electrolyte imbalance due to the IV Lasix/ Fuorosemide, which is a loop diuretic and causes hypokalemia and hypomagnesemia. Low K+ and Mg can lad to V-tach. If this is the case, you need to correct her electrolytes.

How well did you know this?
1
Not at all
2
3
4
5
Perfectly
361
Q

What is another name for vasospastic angina?

A

Variant angina or Prinzmetal angina

How well did you know this?
1
Not at all
2
3
4
5
Perfectly
362
Q

Mechanism and treatment for Vasospastic/ Variant/ Prinzmetal angina?

A

Mechanism= the intima layer of the artery smooth muscle is hyperactive—> intermittent coronary artery vasospasm (ST elevation during chest pain episodes lasting <15 min)

Treatment= CCB’s (Diltiazem, Amlodipine)

  • cause coronary artery vasodilation to prevent vasospasm
  • Nitrates given to stop chest pain during an episode
How well did you know this?
1
Not at all
2
3
4
5
Perfectly
363
Q

Guy comes in with sudden-onset severe left-sided chest pain. Has HR of 125, BP of 160/90, ST depression and T-wave inversion in leads V4-V6. He is agitated, pupils are dilated, and nasal mucosa is atrophic. What should you give him?

A

IV Benzo (like Diazepam)

This guy is on cocaine (tachy, HTN, signs of MI specifically NSTEMI, dilated pupils, nose jacked up from snorting cocaine) having cocaine-induced MI.
Benzos bind to GABA and enhance its inhibitory CNS activity-> less sympathetic activity so reduced agitation, improved HR and BP, improvement in cardiac symptoms.

*also give ASA (antiplatelet to stop thrombus formation enhanced by cocaine), nitrates, and CCBs to vasodilate. Do NOT give beta-blockers due to unopposed alpha-1 constriction-> worsening vasoconstriction!!

  • *Remember cocaine blocks NET and DAT (NE and dopamine reuptake)-> inc NE and dopamine in the cleft. NE= alpha-1> beta-1 agonist, so causes vasoconstriction (including coronary vasoconstriction).
  • *Use alpha-blockers in cocaine toxicity where BP is super high
How well did you know this?
1
Not at all
2
3
4
5
Perfectly
364
Q

Patient is found to have premature atrial complexes (PACs) on EKG. Is not symptomatic. Drinks 1-2 beers/ day. Best way to manage this patient?

A

Advise quitting alcohol

PACs are early beats from the atria firing on its own, you see early P waves (QRS is normal bc conduction below the atria is normal). Usually asymptomatic and does not require treatment, however advice to quit tobacco, alcohol, caffeine, stress bc these things can be what’s causing it!
*if symptomatic (palpitations) a beta-blocker can be given

How well did you know this?
1
Not at all
2
3
4
5
Perfectly
365
Q

Old guy comes into the ED with the worst pain in his life in his chest and back. He has a diastolic decrescendo murmur at the sternal border and is tachy. Has nonspecific ST-segment elevation and T-wave changes. CXR shows widening of the mediastinum. Diagnosis?

A

Acute Aortic Dissection

*Diastolic murmur at sternal border= aortic regurg (dissection can pull apart at aortic valve)

How well did you know this?
1
Not at all
2
3
4
5
Perfectly
366
Q

What fasting blood glucose and hemoglobin A1c do you need to meet criteria for a diagnosis of diabetes?

A

Fasting blood glucose >125

Hb A1c of 6.5% or more

How well did you know this?
1
Not at all
2
3
4
5
Perfectly
367
Q

For patients <40 yo with ASCVD or >40 yo with DM, what med do you give?

A

A statin! (Long-term)

How well did you know this?
1
Not at all
2
3
4
5
Perfectly
368
Q

Asthma patient in respiratory distress is given nebulized albuterol, IV methylprednisolone, and supplemental O2. Repeat labs show leukocytosis (high WBCs). Why?

A

Glucocorticoids (methylprednisolone) cause mobilization of marinated neutrophils-> inc WBCs
(Medication effect)

How well did you know this?
1
Not at all
2
3
4
5
Perfectly
369
Q

What are the 4 severity levels of asthma? State the frequency of symptoms (times they need to use SABA albuterol rescue inhaler) for each.

A
  1. Intermittent- symptoms 2 or less days/ week (treatment= SABA prn)
  2. Mild persistent- symptoms >2 days/ week but not daily (SABA prn + low-dose ICS)
  3. Moderate persistent- symptoms daily (SABA prn + low-dose ICS + LABA (or medium-dose ICS))
  4. Severe persistent- symptoms throughout the day (SABA prn + medium-dose ICS + LABA)
    * if worse: SABA prn + high-dose ICS + LABA + oral corticosteroid and/or Omalizumab (if allergies too)
How well did you know this?
1
Not at all
2
3
4
5
Perfectly
370
Q

Smoker guy is planning to get a knee replacement surgery in a month. He has COPD and HTN. What will decrease his risk of post-operative pulmonary complications the most?

A

Quitting smoking right now

There is no evidence that drugs (such as Moxifloxacin) or anything else decreases PNA risk in the setting of surgery. There is, however, evidence that quitting smoking 1-2 months prior to surgery decreases post-op pulmonary risk by a lot!

How well did you know this?
1
Not at all
2
3
4
5
Perfectly
371
Q

What valvular issue can cause hemoptysis?

A

Mitral stenosis

(Mitral valve has a hard time opening-> blood backs up to LA and lungs-> pulmonary edema and that extra pressure in the lungs can burst capillaries-> cough up blood)

How well did you know this?
1
Not at all
2
3
4
5
Perfectly
372
Q

Another name for bronchogenic carcinoma?

A

Lung cancer!

How well did you know this?
1
Not at all
2
3
4
5
Perfectly
373
Q

Most common cause of hemoptysis?

A

Pulmonary airway disease
(Chronic bronchitis, bronchiectasis, lung CA)

-Chronic bronchitis (the constant coughing and irritation from mucus plugs can cause bursting of blood vessels in the bronchus), Bronchiectasis (same reason- you also have chronic coughing and mucus trapping from the abnormal dilation of the bronchus), and Bronchogenic carcinoma/ Lung cancer (likely due to coughing and maybe related to the rich vascular supply of the tumor)

How well did you know this?
1
Not at all
2
3
4
5
Perfectly
374
Q

Asthma patient comes in with worsened SOB and cough (asthma attack). Pulse ox= 88%. Patient is wheezing and using accessory muscles to breathe. ABG shows: pH of 7.43, PaO2 of 65, PaCO2 of 40. CXR shows hyperinflated lungs. What is most concerning and indicates the patient is getting worse?

A

Most concerning= the fact that PaCO2 is normal-high

  • Why? The normal response to an asthma exacerbation is hyperventilation (compensation for not breathing well)—> blowing off more CO2–> low PaCO2 (<40). If you are not hyperventilating in an asthma attack, you are not blowing off as much CO2 and PaCO2 will be high >40–this suggests respiratory muscle fatigue or severe air trapping suggesting RESPIRATORY COLLAPSE.
  • CXR showing hyperinflation is expected in an asthma exacerbation (obstructive= hard to breathe out= air trapping)
How well did you know this?
1
Not at all
2
3
4
5
Perfectly
375
Q

Patient with PMH of leukemia who got stem cell transplantation and developed neutropenia presents with fever, pleuritic chest pain, hemoptysis. CXR shows right upper lobe infiltrate. CT of chest shows nodular lesions surrounded by ground-glass opacities in the right upper lobe. Sputum gram stain shows inflammatory cells but no organisms. Diagnosis and treatment?

A

Invasive aspergillosis
(Fever, pleuritic chest pain, and hemoptysis in immunocompromised)

Voriconazole + an echinocandin (Caspofungin)

How well did you know this?
1
Not at all
2
3
4
5
Perfectly
376
Q

Most common cause of Cor-pulmonale in the US?

A

COPD (accounts for 25% of cases)

(Widespread vasoconstriction in pulmonary vasculature to shunt blood to areas where there’s better oxygen…but problem is all areas are diseased, so this just creates a greater pressure that the right heart has to pump against-> right-sided HF)

How well did you know this?
1
Not at all
2
3
4
5
Perfectly
377
Q

HIV patient off his meds presents with fever, cough, SOB. Has bilateral lung crackles. CXR shows bilateral interstitial infiltrates. Patient is given antibiotics and IV NS (normal saline) at 150mL/hr. 2 days later his SOB improved but he is confused and Na+ levels are lower than on admission. Cause of his hypOnatremia?

A

SIADH

  • He has PCP pneumonia (HIV, classic fever + cough + SOB symptoms, crackles, CXR infiltrates)
  • SIADH can happen from PNA= too much retention of water-> low sodium concentration
  • Giving normal saline will worsen this kind of hypOnatremia (although it’s salty and you’re giving back some Na+, it is FLUID and that fluid dilutes the [Na+] more)
How well did you know this?
1
Not at all
2
3
4
5
Perfectly
378
Q

When you suspect lung CA what should you do?

A

CT of the chest

How well did you know this?
1
Not at all
2
3
4
5
Perfectly
379
Q

How can cirrhosis cause lower extremity edema?

A

Bad liver—> not making albumin like it should—> low oncotic pressure holding fluid in vessels—> peripheral edema

How well did you know this?
1
Not at all
2
3
4
5
Perfectly
380
Q

What is Spontaneous bacterial peritonitis (SBP)?

A

An infection of the ascitic fluid

*usually occurs in patients with underlying chronic liver disease and ascites. Spontaneous bacterial peritonitis (SBP) is often associated with fever and abdominal pain (can also present with hepatic encephalopathy alone)

How well did you know this?
1
Not at all
2
3
4
5
Perfectly
381
Q

What does “serial” mean in medicine?

A

Repeat

For example, “serial troponin” means you are measuring troponin several time (usually q8 hrs). “Serial CT scan” for lung cancer screening means you are doing a CT annually.

How well did you know this?
1
Not at all
2
3
4
5
Perfectly
382
Q

What is empyema?

A

Collection/ pocket of pus in the pleural space

  • empyema often results from untreated exudative pleural effusion (usually secondary to bacterial PNA)
  • give antibiotics and do aggressive draining of the pleura with thoracentesis. If that doesn’t do it, do throacotomy (surgical opening up of the chest to gain access to the pleural space/ open drainage)
How well did you know this?
1
Not at all
2
3
4
5
Perfectly
383
Q

Renal failure= GFR of less than what?

A

GFR <30 mL/min

How well did you know this?
1
Not at all
2
3
4
5
Perfectly
384
Q
You have a patient that needs to be started on anticoagulation (for PE, for example). The patient has a GFR of 20mL/min. Which initial anticoagulation med is best and why?
A) Enoxaparin 
B) Fondaparinux
C) RivaroXaBAN 
D) Unfractionated heparin 
E) Warfarin
A

D) Unfractionated heparin

A) Enoxaparin= low-molecular-weight heparin (LMWH)
B) Fondaparinux= IV factor Xa inhibitor
C) RivaroXaBAN= oral factor Xa inhibitor
E) Warfarin
Cannot use A, B, C, E^ because the patient has renal failure (GFR <30) and these anticoagulants are not recommended for patients with renal insufficiency (takes too long to clear them-> increased factor X activity and bleeding risk)
*Remember that LMWH (IV) is the best choice for renal insufficient patients and bridge to Warfarin (PO)!

How well did you know this?
1
Not at all
2
3
4
5
Perfectly
385
Q

Small cell lung cancer patient has SIADH (paraneoplastic syndrome) causing hypOnatremia. What’s the best initial treatment for the patient’s hypOnatremia?

A

Fluid restriction

SIADH-> too much ADH-> too much retention of fluid diluting [Na+] so that it is low
Less fluid will help this concentration come back up
*SIADH drugs (Vaptans and Demeclocycline) can be used to block V2 ADH receptors, but start with fluid restriction and see if that is enough

How well did you know this?
1
Not at all
2
3
4
5
Perfectly
386
Q

Most common side effect of Beclomethasone?

A

Most common side effect of inhaled corticosteroids (ICS) like Beclomethasone (in asthma, COPD)= OROPHARYNGEAL THRUSH (oral candidiasis)

How well did you know this?
1
Not at all
2
3
4
5
Perfectly
387
Q

Patient takes the following meds at home: Aspirin, Diltiazem, Atorvostatin, and Albuterol as needed. Was recently diagnosed with stable angina and recently had a nasal polyp removed. Present with SOB and wheezing. Diagnosis?

A

Aspirin-exacerbated respiratory distress (AERD)

*ASA and all NSAIDs block COX—> more shunting toward LOX in AA pathway—> more leukotriene synthesis, which causes bronchoconstriction and can bring on an asthma attack

How well did you know this?
1
Not at all
2
3
4
5
Perfectly
388
Q

What is a parapneumonic effusion?

A

A pleural effusion that arises as a result of a pneumonia

*subtypes: uncomplicated, complicated, and empyema

How well did you know this?
1
Not at all
2
3
4
5
Perfectly
389
Q

Light’s criteria?

A

P:S (pleural: serum ratios)—

Protein >0.5
LDH >0.6
Pleural LDH > 2/3rds the upper limit of normal for serum LDH

If any of these are true, it is exudative (“extra shit in it”) vs. transudative (fluid only)

How well did you know this?
1
Not at all
2
3
4
5
Perfectly
390
Q

What are the 3 types of Aspergillus infections and their presentations?

A
  1. Allergic Bronchopulmonary Aspergillosus (ABPA)
    - Pulm infiltrates, wheezing, IgE in an asthma or CF patient
  2. Aspergilloma (fungal ball)
    - Gravity-dependent lung cavitation, fever, hemoptysis in a patient with prior TB
  3. Invasive pulmonary Aspergliosis
    - Fever, hemoptysis, chest pain, SOB-> kidney failure, endocarditis, ring-enhancing lesions in brain, and sinus tissue necrosis in an immunocomprimised/ neutropenic patient
How well did you know this?
1
Not at all
2
3
4
5
Perfectly
391
Q

How do you diagnose Aspergilloma?

A

Will present as a lung cavitation in a patient with prior TB (seen on chest CT) and you get Aspergillus IgG serology to confirm

How well did you know this?
1
Not at all
2
3
4
5
Perfectly
392
Q

Does lung compliance increase or decrease in ARDS?

A

Decreased lung compliance

-ARDS= inflammatory condition in the setting of infection (sepsis, PNA, etc.), trauma, or other conditions (pancreatitis, massive transfusion). Lung injury causes the release of proteins, inflammatory cytokines, and neutrophils into the alveolar space (hyaline membranes). This leads to leakage of bloody and proteinaceous fluid into the alveoli, alveolar collapse due to loss of surfactant, and DIFFUSE ALVEOLAR DAMAGE-> inability to expand/ stiff lungs/ dec compliance

How well did you know this?
1
Not at all
2
3
4
5
Perfectly
393
Q

What happens to the ratio of arterial oxygen tension to fraction of inspired oxygen in ARDS?

A

It decreases

Arterial oxygen (PaO2) to fraction of inspired oxygen (FiO2) ratio: 
PaO2/ FiO2 (also called P/F ratio)
Decreases (<300) bc the lung injury/ hyaline membranes impair gas exchange-> decreases PaO2 and raises the FiO2 requirement (the oxygen that you have to give them through the ventilator)
How well did you know this?
1
Not at all
2
3
4
5
Perfectly
394
Q

70 y.o. man has a history of attacks of dyspnea on exertion. He has an ejection-type systolic murmur along the left sternal border that decreases with squatting. What does he have?

A

HOCM

  • remember that HOCM is a SYSTOLIC murmur at the left sternal border (systolic bc the problem is when the blood is being pumped out through the aorta)
  • remember that squatting increases preload (compress leg veins= more ‘milking’ of blood to heart), which makes the problem better (more blood to push septal defect out of the way)
How well did you know this?
1
Not at all
2
3
4
5
Perfectly
395
Q

Inheritance pattern of HOCM?

A

Autosomal dominant

*caused by mutations in sarcomere genes (ex: cardiac myosin binding protein C and cardiac beta-myosin heavy chain gene) for the myocardial contractile proteins of the heart

How well did you know this?
1
Not at all
2
3
4
5
Perfectly
396
Q

If you suspect aortic dissection, what imaging do you do?

A

TEE (trans esophageal echocardiogram) is preferred

But you can also do a chest CT with contrast (if patient’s kidneys are ok/ can tolerate contrast) or an MRI (if non-emergency and patient can lie still)
*do whichever of these is available at the time

How well did you know this?
1
Not at all
2
3
4
5
Perfectly
397
Q

What is the recommendation for AAA screening?

A

Abdominal ultrasound once on men 65+ who EVER smoked

How well did you know this?
1
Not at all
2
3
4
5
Perfectly
398
Q

What is the recommendation for breast cancer screening?

A

Mammogram every 2 years on women 50-74

How well did you know this?
1
Not at all
2
3
4
5
Perfectly
399
Q

What is the recommendation for cervical cancer screening?

A

Pap smear every 3 years on women 21-65

*women 30-65 may substitute w/ HPV testing every 5 yrs

How well did you know this?
1
Not at all
2
3
4
5
Perfectly
400
Q

What is the recommendation for colon cancer screening?

A

Fecal occult blood test (FOBT) or fecal immunochemical test (FIT) every year
OR
Colonoscopy every 10 years

How well did you know this?
1
Not at all
2
3
4
5
Perfectly
401
Q

What is the recommendation for HIV screening?

A

Do HIV antibody screen once in adults 15-65

How well did you know this?
1
Not at all
2
3
4
5
Perfectly
402
Q

What is the recommendation for lung cancer screening?

A

Low-dose CT scan every year in adults 55-80, 30+ pack-years of smoking, current smoker, or quit in last 15 years

How well did you know this?
1
Not at all
2
3
4
5
Perfectly
403
Q

What is the recommendation for osteoporosis screening?

A

DEXA scan in women 65+

How well did you know this?
1
Not at all
2
3
4
5
Perfectly
404
Q

Patient presents with worsening SOB, pitting edema in legs, hepatojugular reflux. Has a pleural effusion—what is it most likely due to?

A

CHF

-CHF—> increased hydrostatic pressure pushing fluid out of vessels—> transudative pleural effusion

*you’d expect:
Pleural protein: serum ratios to be...
Protein <0.5
LDH <0.6
LDH < upper 2/3rds limit of normal serum
How well did you know this?
1
Not at all
2
3
4
5
Perfectly
405
Q

A glucose of <60 (or pleural: serum glucose ratio of <0.5) represents what type of pleural effusion?

A

Complicated parapneumonic effusion (exudative pleural effusion in the setting of pneumonia)

How well did you know this?
1
Not at all
2
3
4
5
Perfectly
406
Q

Criteria for extubation (of a person on a ventilator)?

A
  1. pH > 7.25
  2. Adequate oxygen on minimal support (FiO2 less than or equal to 40%, PEEP less then or equal to 5)
  3. Intact inspiratory effort and sufficient mental alertness to protect the airway
    * if a patient is ready to be exubated based on the above criteria, do a spontaneous breathing trial (keep them intubated but turn off ventilator support to check that they breathe on their own and ABG remains normal throughout this effort)!
How well did you know this?
1
Not at all
2
3
4
5
Perfectly
407
Q

Symptoms of CHF + peeing out protein. Think about what?

A
Cardiac amyloidosis 
(Amyloid, a misfolded protein, deposits in various tissues, including the heart-> restrictive CM and you pee out some excess protein)
How well did you know this?
1
Not at all
2
3
4
5
Perfectly
408
Q

Patient has sharp chest pain worse with deep breathing, better with leaning forward. BUN and Cr are elevated a lot. Diagnosis and treatment?

A

Uremic pericarditis
(Toxins accumulate and cause inflammation of the heart sac)

Hemodialysis

How well did you know this?
1
Not at all
2
3
4
5
Perfectly
409
Q

How does acute HIV infection present?

A

Mono-like symptoms (fever, lymphadenopathy, sore throat, athralgias) 2-4 weeks post-exposure

*Viral load is really high (>10,000 copies/mL) (but HIV antibody testing may be negative adn CD4 still normal)

How well did you know this?
1
Not at all
2
3
4
5
Perfectly
410
Q

Patient has fatigue, SOB on exertion, lower extremity edema, proteinuria, easy bruising. Echo shows concentric thickening of ventricular walls w/ diastolic dysfunction. Diagnosis?

A

Amyloidosis

  • Causes a restrictive CM + peeing out protein
  • Remember, it’s a misfolded protein that deposits in various places (systemic dz)
How well did you know this?
1
Not at all
2
3
4
5
Perfectly
411
Q

What is dumping syndrome?

A

Seen in gastric bypass patients

The stomach is now so small that food, esp simple carbs, gets dumped into the intestines really fast-> diarrhea (can also cause abdominal pain, nausea, hypotension, tachycardia, lightheadedness)

How well did you know this?
1
Not at all
2
3
4
5
Perfectly
412
Q

Mechanism of vasovagal syncope?

A

Severe dehydration, etc.-> dec preload to heart-> dec CO-> cardiopulmonary receptors recognize there’s less volume in the heart-> initially this causes inc sympathetic activity, so inc HR and contractility to raise CO to normal-> contortion/ weird twisting of the LV (bc it’s pumping normal but there’s not enough blood to pump out)-> messes up cardiopulmonary receptors so that they have the opposite effect (they think the heart is over full)-> dec sympathetic activity, so dec HR and contractility-> lack of blood to brain-> body faints to inc preload back again (gravity isn’t against you when you’re laying on the floor)

How well did you know this?
1
Not at all
2
3
4
5
Perfectly
413
Q

What is a pericardial knock and when do you hear it?

A

Mid-diastolic sound associated with constrictive pericarditis

Remember, constrictive pericarditis= fibrotic, scarred pericarditis (from repeated inflammation/ recurrent pericarditis). This restricts diastolic filling of the heart-> “pericardial knock” during diastole.

How well did you know this?
1
Not at all
2
3
4
5
Perfectly
414
Q

What screening do you need to do for cirrhosis patients?

A

EGD (endoscopy) to screen for esophageal varices

How well did you know this?
1
Not at all
2
3
4
5
Perfectly
415
Q

What med can you give for esophageal varices?

A

Non-selective beta blocker (Propranolol or Nadolol)

-Block beta-2 (vasodilates)-> vasoconstricts-> reduces portal blood flow

How well did you know this?
1
Not at all
2
3
4
5
Perfectly
416
Q

Would you expect PT and albumin to be high or low in a cirrhosis patient?

A

PT- High (liver makes clotting factors. If the liver is bad, it’s not making as many clotting factors and PT goes up bc takes longer to clot)

Albumin- Low (liver makes albumin. If the liver is bad, makes less of it)

How well did you know this?
1
Not at all
2
3
4
5
Perfectly
417
Q

Besides in B12 and folate deficient individuals, we see macrocytic anemia in what population of patients?

A

Alcoholics

How well did you know this?
1
Not at all
2
3
4
5
Perfectly
418
Q

Explain high-output cardiac failure in a hemodialysis patient with an AV fistula.

A

AV fistula= direct connection between artery and vein (blood goes in arteries away from heart and meets up right away with vein back to heart rather than having to journey through tiny vessels first)

Way more blood in venous system to heart—> heart has to deal with more blood, so a preload overload—> high-output cardiac failure

*these patients get a wide pulse pressure (ex: 160/90) bc in systole the heart pumps out a huge amount of blood (due to having more preload) and during diastole blood is shunting across the AV fistula to VEINS so the pressure drops a lot in the ARTERIES

How well did you know this?
1
Not at all
2
3
4
5
Perfectly
419
Q

Patient had penetrating injury to right eye which resulted in blindness in the right eye. 2 weeks later presents with “floating spots” and blurry vision in his left eye. Most likely cause?

A

“Spared eye injury”

Damage to eye-> exposes antigens your body has never seen (‘hidden antigens’)-> immune system responds to them and attacks other eye

How well did you know this?
1
Not at all
2
3
4
5
Perfectly
420
Q

Guy presents with SOB, cough, hemoptysis for 2 days. He has had weight loss and rhinosinusitis. Labs show low Hb, high BUN and Cr, high WBCs. CXR shows lung nodules. Diagnosis?

A

Granulomatosis with polyangiitis (Wegner’s)

-systemic vasculitis that involves nasopharynx + lungs + kidneys

How well did you know this?
1
Not at all
2
3
4
5
Perfectly
421
Q

Patient flexes his right hip and knee and slaps his foot to the floor with each step. What gait is this?

A

“Steppage gait”

L5 radiculopahty adn compression peroneal neuropathy are common causes

How well did you know this?
1
Not at all
2
3
4
5
Perfectly
422
Q

Lady had a liver transplant 2 weeks ago (is on immunosuppressant meds). Has fever, chills, weakness, vomiting. Fever, low BP, tachy. Most likely cause of her presentation?

A

Bacterial infection

  • most infections within the first month are bacterial causes, during months 1-6 opportunistic (in setting of immunosuppression)
  • hemodynamic instability (look at the vital signs) makes bacterial infection more likely than transplant rejection
How well did you know this?
1
Not at all
2
3
4
5
Perfectly
423
Q

What med can treat an actively bleeding esophageal varices?

A

Octreotide

How well did you know this?
1
Not at all
2
3
4
5
Perfectly
424
Q

Lady comes in with periorbital edema and proteinuria. She is started on diuretics and the edema improves. Then she gets severe right-sided abdominal pain, fever, and gross hematuria. Why?

A

Renal vein thrombosis

Remember that nephrotic syndrome (esp membranous GN)-> peeing out protein, including antithrombin III-> hypercoagulable state

How well did you know this?
1
Not at all
2
3
4
5
Perfectly
425
Q

60 year old man has diarrhea w/ blood and mucus. Has not been on antibiotics. Colonoscopy 4 years ago was normal. Has elevated WBC count and ESR, low Hb. Sigmoidoscopy shows erythematous friable mucosa from the rectum to sigmoid colon. Diagnosis?

A

Ulcerative colitis (UC)

*has a bimodal distribution- more common in young (15-40) and old (50-80)

**NOT colon cancer…although low Hb in elderly is colon cancer until proven otherwise, the colonoscopy 4 years ago did prove otherwise (only need 1 every 10 years). Plus, the sigmoidoscopy showing involvement of just the distal colon is constituent with UC (bleeding-> low Hb from that)

How well did you know this?
1
Not at all
2
3
4
5
Perfectly
426
Q

Patient has coffee ground emesis + black, tarry stools. What are you thinking?

A

Upper GI bleed

*look for elevated BUN (due to increased GI digestion/ breakdown of Hb)

How well did you know this?
1
Not at all
2
3
4
5
Perfectly
427
Q

Lady with osteoarthritis (takes NSAIDs) has conjunctival pallor. Most likely cause of her anemia?

A

Iron deficiency anemia from NSAIDs (block GI protective prostaglandins-> gastritis and/or gastric ulcers leading to chronic GI blood loss and depletion of iron stores)

*osteoarthritis does NOT cause anemia of chronic dz!

How well did you know this?
1
Not at all
2
3
4
5
Perfectly
428
Q

Patient with BMI of 55 and extensive smoking history has SOB, clear lungs, distant heart sounds, low-voltage QRS complexes on EKG, Hb 16, pH 7.3, PaCO2 60. Diagnosis?

A

Obesity hypoventilation syndrome

  • BMI >30 (obese)
  • Lungs are clear
  • Distant heart sounds and low-voltage QRS= heart is further away due to excess fat in between (*think of this and pericardial effusion)
  • Reactive polycythemia (poor oxygenation due to shallow breaths-> kidneys inc EPO-> inc Hb to try to compensate by having more O2 binding sites)
  • Respiratory acidosis (retained CO2 from shallow breathing)
How well did you know this?
1
Not at all
2
3
4
5
Perfectly
429
Q

In a patient with poor oxygenation, why might you see inc Hb?

A

Reactive polycythemia

Poor oxygenation-> kidneys pump out more EPO-> more Hb to try to compensate by providing more oxygen binding sites

How well did you know this?
1
Not at all
2
3
4
5
Perfectly
430
Q

3 diagnostic criteria for obesity hypoventilation syndrome?

A
  1. Obesity with BMI >30
  2. Daytime hypercapnia (PaCO2>45)
  3. No alternate explanation of their hypoventilation
  • work it up by getting ABG (hypercapnia, normal A-a gradient), r/o pulm disease w/ CXR, restrictive pattern on PFTs (all the fat restricts them from expanding their lungs), normal TSH, polysomnography
  • treat with weight loss/ bariatric surgery, nocturnal positive-pressure ventilation
  • *most of these patients have coexisting OSA
How well did you know this?
1
Not at all
2
3
4
5
Perfectly
431
Q

Patient with cardiomyopathy is on Furosemide (Lasix), Carvedilol, Lisinopril, and Digoxin. She presents with a-fib w/ RVR and you treat her with RivaroXaBAN and Amiodarone. 2 weeks later, she comes back with anorexia, nausea, weakness. Why?

A

Digoxin toxicity (causes GI symptoms)

She was already taking Digoxin. You added Amiodarone (anti-arrhythmic) to her regime, which blocks the P450 system (broken chrome bumper in uno, does, tres sketchy)-> inc Digoxin in blood.

How well did you know this?
1
Not at all
2
3
4
5
Perfectly
432
Q

Normal leukocyte (WBC) count?

A

4,500-11,000

*just remember it as roughly <10,000

How well did you know this?
1
Not at all
2
3
4
5
Perfectly
433
Q

What is Myasthenia Gravis and how do meds like Pyridostigmine work for it?

A

MG= antibodies against the nicotinic ACh receptors on the post-synaptic at the motor end plate (skeletal muscle NMJ)-> proximal muscle weakness worse w/ use, ptosis, diplopia

Pyridostigmine= ACh-esterase inhibitor-> blocks the breakdown of ACh-> more ACh in the cleft to outcompete MG antibodies

How well did you know this?
1
Not at all
2
3
4
5
Perfectly
434
Q

What is a Myasthenic crisis?

A

Exacerbation of Myasthenia Gravis (antibodies going crazy against post-synaptic nicotinic ACh receptors-> muscle weakness)

  • can be brought on by infection, surgery, pregnancy/ childbirth, tapering off immunosuppressant drugs, some meds (aminoglycosides, beta blockers)
  • presents with oropharyngeal and general weakness, respiratory insufficiency
  • intubate, plasmapheresis or IVIG + corticosteroids
How well did you know this?
1
Not at all
2
3
4
5
Perfectly
435
Q

Lady with Myasthenia Gravis and Hypothyroidism presents with SOB, productive cough, and choking sensation. T 101, BP 130/80, HR 110, RR 22. She takes shallow breaths and has crackles at lung bases. High WBCs, resp acidosis (low pH, high CO2). Diagnosis and next step?

A

Myasthenia crisis
Endotracheal intubation

  • This patient has PNA (SOB, productive cough, shallow breathing, crackles, WBC count). Resp acidosis (low pH, high CO2) explained by not breathing deep-> less air getting down into lungs for gas exchange. This was a trigger for MG crisis/ exacerbation.
  • Do elective intubation bc of evidence of impending respiratory failure (ill-appearing, tachypnea/ breathing rapid and shallow, muscle weakness, difficulty clearing secretions/ choking sensation, and respiratory acidosis).
How well did you know this?
1
Not at all
2
3
4
5
Perfectly
436
Q

We should do elective intubation in patients with evidence of impending respiratory failure. What kind of subjective and objective signs serve as evidence that they’re going to lose ability to support their airway and you should intubate?

A

Subjective:

  • they look really sick
  • tachypnea (breathing fast and shallow)
  • muscle weakness
  • can’t clear secretions/ choking

Objective:

  • low vital capacity (after max inspiration, the max air you can exhale/ breathe out) *low means they have a weak ability to exhale
  • respiratory acidosis (retaining CO2 from poor gas exchange)
How well did you know this?
1
Not at all
2
3
4
5
Perfectly
437
Q

You see low Na+ and high K+ in the labs. What should you automatically think of?

A

A problem with aldosterone!

Remember, aldosterone-> Na+ absorption, K+ and H+ wasting. If aldosterone is low, you get dec Na+ in the blood and inc K+, H+ in the blood-> hypOnatremia, hypERkalemia, and metabolic acidosis.

How well did you know this?
1
Not at all
2
3
4
5
Perfectly
438
Q

Normal fasting glucose?

A

70-110

How well did you know this?
1
Not at all
2
3
4
5
Perfectly
439
Q

Guy from Southeast Asia has lightheadedness, cough, unexpected weight loss. Has fever, low BP. Labs show low Na+, high K+, low glucose, eosinophilia. CXR shows airspace disease in upper lung w/ lymphadenopathy. Diagnosis?

A

Military TB-> infectious adrenalitis/ primary adrenal insufficiency (Addison’s disease)

  • Remember, no.1 cause of Addison’s in the US= autoimmune destruction of adrenal glands. But no.1 cause of Addison’s in developing countries= TB! The inflammation from military TB destroys adrenal glands so that they stop producing hormones.
  • Addison’s= adrenal glands aren’t making aldosterone and cortisol-> low Na+ in blood (low BP, weakness/ lightheadedness, hypOnatremia), high K+ in blood (hypERkalemia).
How well did you know this?
1
Not at all
2
3
4
5
Perfectly
440
Q

Girl is treated for asthma exacerbation with albuterol nebulization + IV methylprednisolone. Her asthma symptoms improve, but now she has severe muscle weakness and hand tremor. Next step?

A

Check serum electrolyte panel

Albuterol= beta-2 agonist-> hypokalemia (stimulates sodium potassium pumps, so brings K+ into cells). HypOkalmeia can cause muscle weakness and tremor.
*Another side effect is palpitations (speed up the heart bc has some agonist effects on beta-1 too)

How well did you know this?
1
Not at all
2
3
4
5
Perfectly
441
Q

Old lady has lethargy and SOB. Hypotension, tachycardia, reduced breath sounds and crackles. Systolic ejection murmur at right upper sternal border. High cardiac index, normal RA pressure, normal pulmonary capillary wedge pressure (LA pressure). Low SVR. What clinical picture does this fit?

A

Septic shock

  • note: you can’t meet all the SIRS criteria based off this question, have to roll with just the info they give you. But the clinical picture is septic shock- possible PNA for infection source (crackles and SOB), hemodynamic instability (low BP, tachy), low SVR= widespread vasodilation in response to inflammatory cytokines.
  • high CI (which is CO taking into account body size) and no problems with the heart, so not a cardiogenic shock picture (inc CO is the heart trying to compensate for low BP).

**do NOT get thrown off by the murmur (systolic at right upper sternal border). This is aortic stenosis- a lot of old people have it due to calcifications!

How well did you know this?
1
Not at all
2
3
4
5
Perfectly
442
Q

What types of shock present with cold extremities?

A
  1. Cardiogenic shock
    (Heart is not pumping blood out well to extremities and vasoconstriction to divert blood to vital organs)
  2. Hypovolemic shock (hemorrhagic shock)
    (Body vasoconstricts to divert blood away from extremities and toward vital organs)
How well did you know this?
1
Not at all
2
3
4
5
Perfectly
443
Q

Do “bounding pulses” always indicate aortic regurg?

A

No

Just means the heart is working overtime (inc CO)

How well did you know this?
1
Not at all
2
3
4
5
Perfectly
444
Q

Usually use BiPAP or CPAP in:

  1. COPD exacerbation
  2. Obstructive Sleep Apnea
A

COPD exacerbation—> BiPAP

Obstructive Sleep Apnea—> CPAP

How well did you know this?
1
Not at all
2
3
4
5
Perfectly
445
Q

What is the goal for O2 sat in a COPD exacerbation patient?

A

88-92%

*if higher, you will inhibit their respiratory drive (based on low oxygen, not high CO2 like normal people since they have chronic CO2 retention and are desensitized to it)-> hypoventilation (slows or stops their breathing)

How well did you know this?
1
Not at all
2
3
4
5
Perfectly
446
Q

Patient is treated for COPD exacerbation. Treatment includes noninvasive ventilation (BiPAP), which is stopped. Patient has improved respiratory symptoms, but now has altered mental status. O2 sat is 96%, vitals normal. Next step?

A

Get an ABG (arterial blood gas analysis) to confirm acute hypercapnic respiratory failure

  • remember our O2 sat goal in COPD’ers is 88-92% due to their respiratory drive being controlled by low O2 (vs. high CO2)…giving too much O2 will stop their respiratory drive to breathe
  • also, too much O2 may encourage hypercapnia (CO2 retention) and worsened VQ mismatch (in COPD you have vasoconstriction in lungs to shunt blood to better oxygenated areas…if you flood in the oxygen, there will be areas that vasodilate thinking the oxygenation is good now, so blood will be sent to this damaged tissue but can’t gas exchange well so the CO2 just builds up more)
  • hypercapnia (CO2 retention)-> delirium, confusion, lethargy, eventually coma and seizures
How well did you know this?
1
Not at all
2
3
4
5
Perfectly
447
Q

Increased or decreased lung compliance in COPD?

A

Increased lung compliance (more stretchy lungs)

*remember COPD is obstructive= hard to get air out. The lungs are filling well, but not pushing air out well.

448
Q

Patient with PMH of HTN, CAD (recent CABG), recent hospitalization for PNA, and 35 pack-year smoking hx presents with SOB. Has decreased breath sounds at both lung bases, crackles, and wheezing. ABG shows high pH, low O2, low CO2. Diagnosis?

A

CHF exacerbation

  • Given his hx of CAD and smoking, be thinking of COPD or CHF exacerbation
  • High pH, low O2, low CO2= respiratory alkalosis (hyperventilating/ blowing off more CO2)
  • NOT COPD exacerbation (that would have CO2 retention)
  • NOT PE (this may have a unilateral, not bilateral pleural effusion and would have more history like recent travel or surgery, tachycardia)
  • CHFe makes sense—bilateral pleural effusions (dec breath sounds), crackles from backing up of fluid from left heart to lungs. Pumping problem, so patient hyperventilates to compensate and get more oxygen in
449
Q

Why might a patient with shallow/ restricted breathing have low O2, high CO2 (respiratory acidosis)?

A

Not expanding lungs well-> air not reaching as many alveoli for gas exchange-> low O2 into blood, high CO2 in blood bc cannot get out

450
Q

Can you hear an S4 in a patient with a prior MI?

A

Yes, sometimes

Think of S4 as “stiff heart”
Can be due to diastolic dysfunction/ filling problem/ hypertrophy/ thick LV wall OR scar tissue making the heart stiff

451
Q

Reg ventilator settings, FiO2 should usually be less than what?

A

FiO2 <60%

  • for comparison, normal air has roughly 20% oxygen content in it
  • don’t go higher than FiO2 of 60% long-term or else you risk oxygen toxicity to the patient (pro inflammatory oxygen free radicals-> atelectasis/ collapsing of alveoli)
452
Q

High or low Tv (tidal volume) and PEEP (positive end-expiratory pressure) in a patient with ARDS?

A

High PEEP (prevent alveolar collapse)

Low Tv (Tv= the air going into the lungs. You don’t want to shove too much air into the lungs at once or you cause alveoli to pop open and slam shut. Goal is to have LOW trauma to alveoli)

453
Q

What is the ‘rule’ for how high to make tidal volume (Tv) on a ventilator?

A

6 mL per kg

Ex: patient weighs 50kg. Make Tv 6*50= 300 mL

**Tv= the air going into the lungs. The reason you don’t want it too high is to avoid trauma to alveoli. If you shove too much air into the lungs at once or you cause alveoli to pop open and slam shut.

454
Q

Do you always see a low RR in obesity hypoventilation syndrome?

A

No, not necessarily (RR can be within normal range)

Obesity-> fat restricts lungs from expanding well-> not as much air reaches alveoli for gas exchange (more dead space)-> CO2 retention= respiratory acidosis due to ‘hypoventilation’

455
Q

What would chloride and bicarb levels look like in a patient with obesity hypoventilation syndrome (OHS) and obstructive sleep apnea (OSA)?

A

Chronic CO2 retainers (impaired gas exchange)-> high CO2 (respiratory acidosis)-> body compensates by increasing HCO3/ bicarb retention…
So, INCREASED bicarb and DECREASED chloride (for every negatively charged bicarb you keep, you get rid of a negatively charged Chloride)

456
Q

Are transudative pleural effusions usually unilateral or bilateral?

A

Bilateral

Transudative= due to high hydrostatic pressure or low oncotic pressure driving fluid out (no other inflammatory cells in the effusion)

  • CHF, nephrotic syndrome, cirrhosis
  • these things don’t just target one side like a pneumonia does, so you’d expect if there’s a pleural effusion from it, it is bilateral
457
Q

What is a hepatic hydrothorax?

A

Right-sided pleural effusion in cirrhosis patient (ruled out other causes)

  • Pathophys is incompletely understood, but is thought to happen from ascites crossing the diaphragm (travels from the peritoneal cavity into the pleural cavity through small diaphragmatic defects)
  • increased permeability of right hemidiaphragm
458
Q

If pleural effusion fluid has glucose <60, consider what cause?

A

Rheumatoid arthritis (RA can rarely lead to inflammation affecting the lungs= pleuritis/ pleurisy, and that inflammation can cause effusion)

  • StepUpToMed page 82, 2nd quick hit box
  • But when glucose is low, still consider other causes of pleural effusion (complicated parapneumonic effusion, empyema, malignant effusion, TB pleurisy, lupus pleuritis, esophageal rupture)! (Just put this in the differential)
459
Q

Patient with CHF and cirrhosis has SOB and dullness to percussion at right lung base. CXR shows right sided pleural effusion. Thoracentesis shows pleural fluid has glucose of 30 and LDH of 250. Cause?

A

Pneumonia (this is a parapneumonic effusion, or pleural effusion 2/2 PNA)
-Specifically, it’s likely an empyema (low glucose due to high metabolic activity of WBCs and bacteria in the pleural fluid)

  • LDH of fluid= 250, which is >2/3rds upper limit of normal serum LDH (45-90), so EXUDATIVE
  • NOT a pleural effusion due to CHF (high hydrostatic pressure) or cirrhosis (low albumin= low oncotic pressure) bc these are transudative (plus would expect them to cause bilateral, not unilateral, pleural effusions—these disease processes don’t isolate to one side) *hepatic hydrothorax (right-sided pleural effusion in cirrhosis patients from ascites crossing diaphragm is also usually transudative)
460
Q

Think of what causes if a pleural effusion has elevated amylase?

A

Pleural effusion due to pancreatitis, esophageal rupture, or malignancy

461
Q

Inflammatory conditions can cause pleural effusions by increasing what?

A

Pleural membrane and capillary permeability

462
Q

Patient has epigastric pain radiating to back associated with nausea/ vomiting. Elevated alk phos, ALT (>150), AST, lipase, WBCs. RUQ U/s shows gallstones w/o gallbladder wall thickening. She is given supportive care (pain control, IV fluids), NPO. Enzymes trend down over a couple days. Next step?

A

Cholecystectomy

  • This patient has acute pancreatitis from a gallstone (*ALT>150 is pretty specific for it—plus patient has high lipase, AST, alk phos)
  • Early cholecystectomy is recommended in stable patients who receiver from acute pancreatitis
  • ERCP would have been the answer (to remove obstructing stone) if we saw common bile duct dilation/ obstruction on U/S (gallstone pancreatitis), or if liver enzymes keep rising
  • *Note: My surgery patient had choledocolithiasis (stone lodge in common bile duct) so we did—
    1. RUQ U/S
    2. MRCP (MRI of the biliary tract to prove it)
    3. ERCP (to remove the stone blocking the common bile duct)
    4. Wait for the pancreas to cool down/ labs to downtrend (bc inflammation complicates surgery)
    5. Cholecystectomy (take out the gallbladder to avoid this happening again)
463
Q

Patient with cirrhosis has worsening renal function during his hospital stay. Cr and urine output are both down-trending. U/A shows minimal hematuria, no casts. Renal U/S shows no hydronephrosis. Renal function is not improving with IV fluids. Diagnosis?

A

Hepatorenal syndrome

  • Kidney function is declining in the setting of cirrhosis
  • You know it’s not just AKI because: no casts (not ATN/ intrinsic), no hydronephrosis/ obstruction (not post-renal), no improvement with fluids (not pre-renal)

*treat with stopping alcohol, liver transplant if possible (if not, can try splanchnic vasoconstrictors like Octreotide, NE, Midodrine or albumin or dialysis, but may not help too much)

464
Q

“Splanchnic arterial dilation” should make you think what?

A

Hepatorenal syndrome

465
Q

Patient has foul smelling stools and weight loss over the last 6 mo. Has iron def anemia. Immunoglobulin A anti-tissue transglutaminase antibody is negative. Small-bowel biopsy shows villous atrophy. Diagnosis?

A

Celiac disease

  • IgA anti-tissue transglutaminase and IgA anti-endomysial antibodies are screening tests, but can be negative in a person with celiac dz + IgA deficiency (if negative but clinical suspicion is high, order total IgA to check for IgA def or do IgG-based serologic testing)
  • Biopsy showing villous atrophy is the confirmatory test
  • associated with iron def anemia (not absorbing enough iron from the food they eat)
466
Q

Low Hb and low ferritin. What type of anemia?

A

Fe deficiency

*Ferritin= Fe in storage site (this will be the first thing depleted in iron deficiency anemia)

467
Q

Guy with cirrhosis comes in with bloating. There is shifting dullness and fluid wave. Diagnostic parade tests shows stay-colored fluid. In addition to abstinence from alcohol, what treatment do you recommend?

A

Low-sodium diet (restricting Na+= less water will follow) and diuretics, specifically Furosemide (Lasix) + Spironolactone (loop and K+ sparing)

*you always do diagnostic paracentesis, but only do regular therapeutic paracentesis if the patient is having symptoms from the ascites (SOB, tense, early satiety)

468
Q

A patient has ascites. You do diagnostic paracentesis. If the ascites is due to portal HTN (from cirrhosis), what would you expect in the ascites fluid?

A
  • yellow or straw-colored
  • low total protein in it (<2.5 g/dL)
  • high SAAG (serum-ascites albumin gradient) (>1.1)
  • if SAAG <1.1 ascites is likely due to another cause (CHF, CKD, massive fluid overload, TB peritonitis, malignancy, hypoalbuminemia, hepatorenal syndrome)
  • get cell count and differential—if neutrophils >250, spontaneous bacterial peritonitis (SBP)
469
Q

How can you determine the cause of ascites based on SAAG (serum ascites albumin gradient) (from doing diagnostic paracentesis)?

A

SAAG >1.1 means portal HTN (due to cirrhosis)

SAAG <1.1 means the ascites is due to another cause (CHF, CKD, massive fluid overload, TB peritonitis, malignancy, hypoalbuminemia, hepatorenal syndrome)

470
Q

Patient is positive for Anti-HBs and Anti-HBc. What does this mean?

A

Recovered from a past Hep B infection

*remember “SpECiES”
HBsAg: 1st marker of infection 
HBeAg: marker of infectivity 
Anti-HBc: core antibody  
Anti-HBe: e antibody 
Anti-HBs: surface antibody (indicates vaccination if alone, recovery if with anti-HBc)
471
Q

If you suspect acute cholecystitis and RUQ U/S is inconclusive, what should you do?

A

HIDA scan

472
Q

Lady with PMH of HTN (recently put on hydrochlorothiazide and amlodipine) has epigastric pain radiating to the back associated with N/V. RUQ U/S shows no gallstones or duct dilation. She doesn’t drink alcohol. TAGs are normal. Next step?

A

Discontinue hydrochlorothiazide

  • Most common causes of pancreatitis=
    1. Gallstones
    2. Alcohol abuse
    3. Triglycerides >1,000
  • Once you rule out these 3 causes, consider less common causes including drug-induced pancreatitis. Loop and Thiazide diuretics can cause pancreatitis as a side effect.
473
Q

Lady has thyroid nodule. Calcitonin is elevated. Ultrasound-guided biopsy shows malignant cells. Next step to work-up this patient?

A

DO A PLASMA METANEPHRINE ASSAY (may also do RET mutation testing)

Thyroid nodule w/ malignant cells and high calcitonin= Medullary Carcinoma of the thyroid. This is associated with MEN II (2a and 2b have Medullary Carcinoma + Pheochromocytoma. 2a also has parathyroid cancer. 2b also has acoustic neuromas/ marfanoid habitus). So you need to screen for Pheochromocytoma!

*this is important bc a pheochromocytoma can be asymptomatic at the time of diagnosis, but cause life-threatening HTN crisis during surgical procedures (thyroidectomy) if not caught and respected!

474
Q

Lady had diarrhea after a picnic that resolved. Now presents with ptosis, diplopia, weakness with upward gaze, loss of deep tendon reflexes, and dysmetria (lack of coordination) with heel-to-shin testing bilaterally. Diagnosis?

A

Guillain-Barre syndrome
Specifically, the sub-type Miller Fisher syndrome (abnormal muscle coordination, absent tendon reflexes, and paralysis of eye muscles)

475
Q

40 year old guy has tinea corporis, with widespread fungal “ringworm” rash. You’ll give him oral fluconazole to treat, but what other screening should you offer?

A

HIV screening

Tinea corporis with extensive skin involvement raises the question of whether the patient is immunocompromised

476
Q

Old guy with recent MI two months ago (on ASA, Clopidogrel (Plavix), Metoprolol, Lisinopril, and Atorvostatin) presents with SOB. Has crackles in lower lung fields and systolic murmur over cardiac apex. Deep Q waves in leads I, aVL, V2-V5. Most likely cause?

A

Ventricular aneurysm

  • Months post-MI there are 3 complications to remember:
    1. Aneurysm (scar tissue outpouches bc it is weaker than the myocardium)
    2. Mural thrombus (clotting within the wall-> blood clot thrown elsewhere)
    3. Dressler syndrome (autoantibodies against the pericardium)
477
Q

Concentric vs. Eccentric LV hypertrophy?

A

Concentric:

  • LV hypertrophy (thickened LV)
  • myocardial contractile fibers are in parallel (stacked up)

Eccentric:

  • LV dilation/ dilated cardiomyopathy (stretched out LV)
  • myocardial contractile fibers are in series (stacked out)
478
Q

What defines typical angina/ chest pain?

A
  1. Substernal
  2. Provoked by exercise or emotional stress
  3. Relieved by rest or nitroglycerin
  • 3/3= typical/ classic angina
  • 2/3= atypical
  • <2= non-anginal
479
Q

Old guy has substernal chest discomfort when walking, better with rest. Echo shows normal EF, mild LA dilation. Aortic valve is calcified with aortic valve area of 1.6 cm^2. What condition is most likely responsible for his symptoms?

A

Coronary artery disease (CAD)

  • he has typical angina symptoms (substernal, with exercise, better with rest)
  • Although he also has aortic stenosis and this too can be a cause of angina, it’s unlikely to be the cause unless it’s severe aortic stenosis (valve area <1 cm^2). Since his aortic valve is 1.3 cm^2 and the angina is mild-moderate, its most likely CAD causing it.
480
Q

20 year old guy is having a resting tremor, muscle rigidity, clumsy gait, slurred speech recently. He also has hepatomegaly, high total bili, alk phos, ALT, and AST. Liver biopsy shows inflammation, portal fibrosis, hepatocyte necrosis, macrovesicular steatosis, vacuolated hepatocellular nuclei, and Mallory bodies. Diagnosis?

A

Wilson disease

AR mutation in copper transporting channel-> accumulation of copper in liver, eyes (Kayser-Flesher corneal deposits), and brain (psychosis, Parkinson’s and behavior symptoms)
*diagnose with decreased serum ceruloplasmin (copper transporter) and increased urinary copper

481
Q

Guy who’s been binge drinking and using cocaine comes in for weakness. His thighs and calves are swollen and tender, strength decreased. What complication is he at risk for over the next couple days in the hospital?

A

AKI secondary to rhabdomyolysis

-binge drinking and cocaine-> acute alcohol myopathy esp in lower extremities= drug-induced rhabdo-> myoglobin gets released into blood-> can block renal tubules-> AKI

482
Q

What does tumor lysis syndrome do to the following electrolytes?

  • uric acid
  • potassium
  • phosphate
  • calcium
A
  • uric acid: INCREASED (nucleic acids are released and metabolized to uric acid)
  • potassium (K+): INCREASED (released from lysed cells)
  • phosphate (P): INCREASED (released from lysed cells)
  • calcium (Ca2+): DECREASED (the extra phosphate binds it up-> decreased free calcium levels in the blood)
483
Q

What is type 2 heparin induced thrombocytopenia (HIT)?

A

A bad reaction to heparin where the heparin binds to PF4 (platelet factor 4) and then antibodies get made against that complex (hep-PF4). The antibodies attack platelets-> thrombocytopenia and cause platelets to stick/ aggregate (leading to paradoxical blood clots/ thrombosis).

*usually happens in 5-10 days post-heparin administration, stop heparin and give another anticoagulant (Argatroban, Fondaparinux)!

**type 1 HIT is milder (nonimmuned-mediated platelet aggregation) and doesn’t require treatment

484
Q

35 year old lady has dry cough. CXR shows bilateral hilar lymphadenopathy. Diagnosis?

A

Sarcoidosis

Chronic non-caseating granulomatous inflammation

485
Q

Man with hyperlipidemia (on Simvastatin) and gout (on Colchicine) presents with muscle pain/ weakness and dark urine. Labs show elevated K+, BUN, Cr. U/A shows blood. What should you order to establish the diagnosis?

A
Creatinine phospholipase (CPK) 
(Marker of skeletal muscle damage) 
  • Sounds like Rhabdo! Statin + Colchicine-> drug-induced rhabdomyolysis (both are myotoxins and muscle breakdown is more likely in combination)
  • Dark Urine, blood on U/A w/o RBCs on urine microscopy (myoglobinuira), high BUN and Cr all point to AKI 2/2 rhabdo
486
Q

How do you treat a mild tinea corporis infection (ringworm)?

A

Topical terbinafine or azoles (like Miconazole)

487
Q

Lady has been having fever and malaise for 2 weeks. Has systolic murmur at cardiac apex. Today she presents with sudden-onset right-sided flank pain. CT abdomen shows wedge infarct. What’s going on?

A

Renal infarction due to embolization of valvular vegetation

(Fever, mitral regurg point to infective endocarditis. A vegetation embolized/ got thrown to infarct the kidney- an ‘MI’ of the kidney causing sudden pain and wedge on CT)

488
Q

ARDS patient is intubated and ventilated with high PEEP, low FiO2. He begins to deteriorate, vital signs because unstable and breath sounds absent on one side. Most likely reason for this?

A

Pneumothorax
-This is a complication of PEEP

*Remember, in ARDS inflammation and edema-> alveolar collapse. PEEP maintains airway pressure at the end of expiration to prevent alveolar collapse. But, since the lung tissue is already weak and predisposed to barotrauma, PEEP can rupture alveoli-> leakage of air to pleural space-> pneumothorax (sudden-onset SOB, hypotension, tachy, tracheal deviation, unilateral absence of breath sounds).

489
Q

What is the FeNa (fraction of excreted sodium) and BUN/Cr ratio in pre-renal AKI?

A

FeNa <1% (normal- the kidneys reabsorb most Na+)

BUN/Cr >20:1 (high- blood flow to the kidneys is slowed, so there is more time for Na+ to get reabsorbed at the PCT)

490
Q

Patient has meningococcal meningitis and wants to leave the hospital AMA. The patient understands the risks and is mentally normal. What do you do?

A

Hospitalize and isolate the patient against his wishes

You cannot let a patient leave AMA when the patient is a risk to public health! This is highly contagious and serious.

491
Q

Patient is treated for severe asthma exacerbation. He has diminished breath sounds and faint wheezing. ABG shows pH 7.3, PaO2 65, PaCO2 50. Next step?

A

Endotracheal intubation

This patient has signs of impending respiratory failure (need to intubate to maintain adequate oxygenation and ventilation)

  • Diminished breath sounds/ wheezing
  • Respiratory acidosis (low pH, high CO2)
  • Note that a normal response to an asthma exacerbation is hyperventilation (blow off more CO2)-> decreased CO2. So, CO2 retention is a bad sign that they’re not maintaining their airway well!
  • Low PaO2/ hypoxia
492
Q

What does lymphocyte pleocytosis mean?

A

Increase in cell count, so more lymphocytes/ WBCs (lymphocytosis)

493
Q

CSF findings in bacterial vs viral vs fungal meningitis?

A

Bacterial- neutrophils + decreased CSF glucose

Viral- lymphocytes + normal CSF glucose (elevated protein)

Fungal- lymphocytes + decreased CSF glucose

494
Q

Patient has herpes simplex viral meningitis and is treated with high-dose IV acyclovir. His neurologic status improves, but 2 days later he develops AKI (rise in BUN, Cr). Most likely cause?

A

IV Acyclovir-> Crystalline Nnephropathy (type of intrinsic AKI where crystals obstruct renal tubules)

*note: this is NOT the same as acute interstitial nephritis—drug (NSAID, Penicillin, diuretic)—> fever, rash, and dec urine output

495
Q

Guy with drinking history (no other med hx) presents with altered mental status. Labs show low bicarb (10), normal Na (140), normal Cl (100), high BUN and Cr. ABG shows low pH (7.2), low PaCO2 (30). U/A shows rectangular, envelope-showed crystals. Diagnosis?

A

Ethylene glycol poisoning

  • low pH, low bicarb, low PaCO2 means metabolic acidosis
  • calculate anion gap: AG= Na-Cl-HCO3= 140-100-10= 30. This is >12, so anion-gap met acidosis
  • keep in mind “MUDPILES”: Methanol, Uremia, DKA, Porpylene glycol, Iron tablets or INH, Lactic acidosis, Ethylene glycol, Salicylates
  • Ethylene glycol blocks alcohol dehydrogenase (competitive inhibitor)-> build up of toxic metabolites-> Ca2+ oxalate kidney stones (envelope-shaped) an AG met acidosis
496
Q

What are your “MUDPILES”—causes of anion gap metabolic acidosis?

A
Methanol
Uremia
DKA
Porpylene glycol
Iron tablets or INH
Lactic acidosis
Ethylene glycol
Salicylates
497
Q

Equation for calculating anion gap?

A

AG= Na-Cl-HCO3

Sodium minus chloride minus bicarb

498
Q

1st line medication for Restless Leg Syndrome?

A

Dopamine agonists (Pramipexole)

  • Also check for iron deficiency anemia and treat that with Fe supplementation
  • Alpha-2-delta calcium channel ligand (gapapentin) may also help
499
Q

Guy with BPH comes in due to more difficulty urinating over the last day. The last 2 days he’s been taking diphenhydramine for cough, otherwise no changes in medical hx. Cause?

A

Diphenhydramine (1st gen anti-histamine)-> anticholinergic side effects, including urinary retention from detrusor hypOcontractility

*BPH wouldn’t cause urination to go downhill more so in just one day

500
Q

Middle aged woman has abdominal pain (esp over RLQ) and non-bloody diarrhea for 4 months. Lost some weight. Has oral ulcers. Hb is low, WBCs and ESR high. Diagnosis?

A

Crohn’s disease

  • Inflammatory markers are suggestive of inflammatory bowel dz rather than celiac or IBS
  • Crohn’s inflames anywhere along GI tract from mouth (mouth ulcers) to anus (*vs UC affects only distal GI tract-> LLQ pain, bloody diarrhea)
  • IBD associated with iron def anemia (malabsorption of iron)
501
Q

30 year old woman had an episode of hand numbness 5 months ago. Now presents with bilateral shooting facial pain triggered by the cold, brushing her teeth, chewing, or touching her cheeks. NSAIDs do not help. What condition does she have?

A

Trigeminal neuralgia most likely due to MS (multiple sclerosis), one of the few conditions that causes BILATERAL trigeminal neuralgia (due to demyelinating of the trigeminal nerve axon/ root)

*hand numbness was probably her first MS symptom

502
Q

Always start with what to rule out brain bleeds?

A

NONCONTRAST CT

No contrast bc contrast appears white and so does blood

503
Q

Guy falls to the ground and has a seizure leaving a bar. He’s confused, but fully responsive. You order a CBC, serum electrolytes, echo, cervical spine imagine, and urine tox screen. Now what?

A

Do NONCONTRAST CT head

CT of brain w/o contrast is the initial imaging choice to r/o acute neuro problems (intracranial or subarachnoid bleeds). Note that MRI is more sensitive than CT is identifying structural causes of epilepsy but takes longer, so this is the choice if non-emergent.
*EEG is a good test for helping to decide course of antiepileptic agents, but ruling out brain bleeds is the 1st imaging step!

504
Q

Most common cause of peripheral neuropathy?

A

Diabetes mellitus!

*don’t jump straight to rare junk like Charcot-Marie-Tooth disease

505
Q

60 year old man with T2DM and PMH of diabetic retinopathy presents for evaluation of recurrent falls. He sways when standing with eyes closed, has bilateral hammer toe deformities, decreased proprioception and vibration sense in his feet, and absent ankle reflexes. Diagnosis (be specific)?

A

Large fiber neuropathy secondary to DM

  • small fiber neuropathy-> positive symptoms such as pain, paresthesias, allodynia (pain out of proportion like light touching of the skin makes you scream out in pain)
  • pain and temp
  • burning and stabbing pain
  • large fiber neuropathy-> negative symptoms such as numbness, loss of proprioception/ vibration, diminished reflexes (this guys’ symptoms fit this picture)
  • pressure/ proprioception/ balance
  • numbness and poor balance
506
Q

Old lady presents with productive cough and low-grade fever, crackles over lung base. She’s been getting a lot of upper respiratory infections lately. ROS is significant for weakness and right-sided chest pain. Labs show low Hb, high WBCs. CXR shows consolidation in lower lung and osteo lytic lesions with fractures in ribs. Diagnosis?

A

Multiple myeloma

malignancy of plasma cells

  • cytokines activate osteoClasts-> lytic lesions
  • inc serum protein (M spike)
  • inc infection risk (monoclonal antibody lacks antigenic diversity)
  • Rouleaux formation of RBCs
  • primary amyloidosis from light chains depositing in tissue
  • proteinuria (Bence Jones proteins)-> kidney failure
  • normocytic anemia (bone marrow replaced by plasma cells + renal failure so dec EPO to drive production of RBCs)
507
Q

Treatment for cluster headaches (severe unilateral periorbital pain sometimes with autonomic symptoms)?

A

Oxygen facemask or Sumatriptan

508
Q

Cancer patient with Mets to vertebrae is having uncontrolled pain. Right now is only on ibuprofen 3-4x/ day for pain. What med should you prescribe at this time for pain management?

A

Short-acting opioid (morphine, hydromorphone)

*If the pain is still bad, you can add a long-acting opioid (morphine, fentanyl) for help with sleep, but he’s never been on opiods before so do not start him right away on long-acting due to risk of respiratory depression! Start with short-acting to determine appropriate dosing for the patient.

509
Q

Old guy comes in due to urinary frequency waking him up throughout the night. Has no burning when peeing. Has diffusely enlarged, smooth prostate on digital rectal exam. Next step?

A

Get a urinalysis (U/A)

  • Seems he has BPH, but all patients with urinary symptoms (frequency, nocturia, hesitancy) should get a U/A to check for hematuria and infection.
  • Also get a PSA as prostate cancer screening unless predicted life expectancy is <10 yrs.
510
Q

What is osteopenia?

A

Bone loss/ weakened bones

511
Q

Guy with PMH of Crohn’s disease s/p small bowel resection (on Mesalamine and Infliximab) presents with generalized bone pain. Radiographs show osteopenia of the spine w/ pseudo fractures in femoral necks. Diagnosis? Effect on serum calcium, phosphate, PTH?

A

Osteomalacia (vit D deficiency)

-Crohn’s disease-> inflammation esp at terminal ileum-> impaired reabsorption of bile acids-> malabsorption of fats and fat-soluble vitamins, including vit D-> DECREASED calcium + P, INCREASED PTH (to try to compensate)

512
Q

Lady who abuses heroin daily presents with arm swelling w/ needle marks. She is treated with clindamycin. The next day she feels awful with congestion, myalgia, nausea/ vomiting, loose stools. WBC count is low. Next step?

A

This is opioid withdrawal (she is off her heroin and has symptoms within ~12 hrs of last use)

-N/V, cramps, diarrhea (opposite of constipation from opioids), restlessness, rhinorrhea, lacrimation, myalgia, arthralgia, HTN, tachy, mydriasis (dilated pupils), hyperactive bowel sounds

Give OPIOID WITHDRAWAL MEDS: Methadone (done timer) or Buprenorphine (blueprint)
*long half-life

513
Q

From Sketchy, what drug is used for acute opioid toxicity? What 2 used for opioid withdrawal?

A

Opioid toxicity—> Naloxone (no lax zone)
*short-acting

Opioid withdrawal—> Methadone (done timer) or Buprenorphine (blueprint)
*long half-life

514
Q

HIV patients with CD4 <100 should be on what med for Toxoplasmsis prophylaxis?

HIV patients with CD4 <200 should be on what med for PCP (pneumocystis pneumonia) prophylaxis?

A

Bactrim (TMP-SMX, trimethoprim-sulfamethoxazole)

*Bactrim reduces the risk of toxo in CD4<100 so that getting toxoplasmosis is only 0-2%! Bactrim also reduces the risk of PCP PNA in CD4 <200.

515
Q

Triple therapy for H Pylori?

A

PPI + Amoxicillin (Metronidazole if allergic) + Clarithromycin

516
Q

Alcoholic has epigastric pain radiating to the back. He is given opioids for pain + IV fluids. On day 2 of hospitalization his BP drops to 80/60, HR tachy at 120. He is satting at 92% on 2L O2 nasal canula and has crackles. BUN, Cr aare high, urine output is low. Diagnosis?

A

Acute severe pancreatitis

  • premature release of pancreatic enzymes-> inflammation and increased vascular permeability around the pancreas-> widespread vasodilation-> septic shock with multisystem organ dysfunction (respiratory and renal failure)
  • old people >75, alcoholics, obese, CRP>150 at 48 hrs, and inc BUN/CR in 48 hrs at higher risk for this (most other patients with acute pancreatitis have mild dz and recover in 3-5 days with fluids/ conservative treatment)
517
Q

Patient presents with urinary frequency, fever of 102, nausea. Costovertebral angle tenderness. WBC count. U/A shows nitrate, WBCs, bacteria in urine. Diagnosis?

A

Pyelonephritis (kidney infection)

  • presents as cystitis (bladder infection/ UTI symptoms) + flank pain, costovertabral angle tenderness, fever, and/or nausea/ vomiting
  • get urine culture and treat empirically with antibiotics that cover gram-neg organisms (ex: Fluoroquinolones)
518
Q

You have a patient with pyelonephritis. When is a CT of the abdomen/ pelvis indicated?

A
  • Patient does not clinically improve within 72 hours (3 days)
  • Has history of nephrolithiasis (kidney stones)
  • Has complicated pyelonephritis (progression to renal abscess, emphysematous pyelonephritis where there’s gas accumulation in the tissues, papillary necrosis, or sepsis)
  • Has unusual urinary findings (gross hematuria, suspicion for urinary obstruction)
519
Q

Patient with Hodgkin lymphoma on chemo (last dose 3 days ago) presents with crampy abdominal pain and vomiting. Labs show mild anemia and thrombocytopenia. What can you do for this paitent?

A

Give Ondansatron (serotonin 5HT receptor antagonist/ anti-emetic)

*mild anemia and thrombocytopenia is normal for a patient with a lymphoproliferative disorder on chemo—she’s just having N/V from chemo

520
Q

Old guy with PMH of HTN, DM, hyperlipidemia, PVD, and recent MI has sudden, painless loss of vision in the right eye. Hours ago, he had similar loss of vision in the same eye but only for 5 min. On exam, he has retinal whitening and a carotid bruit. Diagnosis and next step?

A

Retinal artery occlusion (from embolism)

Treat with high-flow oxygen and ocular massage

  • he has all the risk factors for retinal artery occlusion
  • note: episode of vision loss for 5 min is “amaurosis fugax” (like TIA of retinal artery) and then persisting loss of vision is retinal artery occlusion (like stroke of retinal artery)
521
Q

Middle aged woman with PMH of RA (on Methotrexate) has unilateral knee pain with edema and fever/ chills. Treatment?

A

Antibiotics

This is septic arthritis
Do synovial fluid analysis to confirm diagnosis (and blood cultures)
Gram (+)—> Vanco
Gram (-)—> 3rd gen cephalosporin

522
Q

Alcoholic stumbles into the ED with muscle cramps and perioral numbness. He is found to have hypOcalcemia. Labs also significant for: low Hb, high MCV, low Mg, low Phosphorus. Most likely cause of his hypOcalcemia?

A

Hypomagnesemia

Low Mg (common in alcoholics)—> decreases PTH secretion and decreases responsiveness to it—> Low calcium (bc PTH increases calcium and you don’t have as much of it) 
(**PTH also dumps Phosphorus, so you’d think with less PTH you’d get higher Phosphorus levels, but oddly Phosphorus levels are normal-low, possibly due to intracellular P depletion)
523
Q

Woman with SLE (on Prednisone and Hydroxychloroquine) has pain in her right hip that is worse with weight bearing. She denies trauma to the hip. Range of motion is normal. X-ray of hip is normal. Next step?

A

Get an MRI of the hip

  • This sounds like avascular necrosis (risk is increased with SLE and glucocorticoid use)
  • MRI is more specific than X-ray (can visualize the boundary between normal and ischemic bone)
  • note that in the first few months of having this, range of motion and x-rays may be normal but this will decline later on
524
Q

What effect can Trimethoprim (in TMP-SMX aka Bactrim) have on potassium and creatinine levels?

A

Trimethorpim—> hyperkalemia (by blocking Na+ channel in CD) and artificial increase in Cr (w/o affecting GFR)

525
Q

What is carotid endarterectomy?

A

A vascular surgical procedure done to reduce the risk of stroke by correcting stenosis in the internal carotid artery

-indicated in patients with TIA/ stroke with high-grade carotid stenosis (>70%)

526
Q

Patient has right shoulder pain for 2 months. Cannot raise arms. Has decreased passive and active aBduction, flexion, and rotation of the right shoulder. Palpation does not cause pain. X-ray normal. Diagnosis?

A

Adhesive capsulitis aka “Frozen shoulder”
Thickened, tight shoulder (glenohumeral joint) capsule

-due to chronic inflammation, fibrosis, and contracture

  • pain with aBduction, external rotation
  • decreased passive and active range of motion
527
Q

50 year old guy with T2DM comes in for follow-up lab work. His total cholesterol is normal, LDL normal, TAGs are elevated. What med should you start him on?

A

A statin

Give statins to any patient >40 years old with DM regardless of LDL level!
*Although a fibrate (like gemfibrozil) seems reasonable to reduce the elevated TAGs, statins are 1st line in ASCVD prevention (you would give a fibrate just to reduce TAGs in someone with TAGs>1000 at risk for acute pancreatitis)

528
Q

Can Dermatomyositis be a paraneoplastic syndrome (seen in lung cancer)?

A

YES

529
Q

Heat stroke can be exertional or non-exertional. What does this mean?

A

Exertional- when heat stroke happens to patients who have over exerted themselves

Non-exertional- when heat stroke happens to patients with chronic medical conditions (have underlying impaired thermoregulation)

  • HEAT STROKE: Too hot-> blood is shunted away from central organs and to skin to sweat and cool down body-> distributive shock, where organs lack adequate perfusion-> organs suffer and die
  • SIGNS/ SYMPTOMS: temp >105, altered mental status, hypotension, tachycardia, tachypnea
530
Q

Guy is going to die at your ED unless you give him a blood transfusion. His fiancé tells you at the scene he is Jehovah’s Witness, don’t give the transfusion. He doesn’t have a card on him/ advance directive/ living will that states this. What do you do?

A

Give the blood transfusion

In the absence of an advance directive, a life-saving blood transfusion can be given to a Jehovah’s Witness who lacks decision-making capacity

531
Q

50 year old woman with hypothyroidism presents with itching and fatigue. She has hepatomegaly and bilateral xanthelasma and skin excoriations. Labs are notable for high total cholesterol, total bili, and alk phos. RUQ U/S is normal. Next step?

A

Check anti-mitochondrial antibodies

For Primary Biliary Cholangitis (autoimmune destruction of intrahepatic bile ducts)
*classically in middle-aged women and associated with other autoimmune conditions (hypothyroidism)

532
Q

To treat Wernicke encephalopathy (encephalopathy + ocular dysfunction + gait ataxia) what do you give IN WHAT ORDER?

A

Give thiamine before glucose! Do NOT give glucose, then thiamine.
(Pixorize: don’t eat sugar before you work out your thighs)

533
Q

Lady presents with fatigue, weight loss. Has splenomegaly. Has anemia and thrombocytosis. FISH shows abnormality in chromosome 22. What is the disease and treatment target?

A

Chronic myeloid leukemia (CML) t(9,22), BCR-ABL
*too many granulocytes in myeloid path, esp basophils

Treatment target= tyrosine kinase inhibitor (imatinib)

534
Q

60 year old man presents with scrotal pain. Only sexually active with his wife. Has a tender mass in the left scrotum and erythema. Cremasteric reflex is intact and no discharge. Diagnosis and cause?

A

Acute epididymitis (swelling of the epididymis around the testicle), most likely due to E.coli (from bladder outlet obstruction)

*In age <35 it would be most likely due to chlamydia, gonorrhea from STD

535
Q

Patient has elbow pain radiating to the forearm, worse with activity like after baggage handling work. Pain is reproduced by flexion of the wrist. Diagnosis?

A

Lateral epicondylitis= tennis elbow= swollen extensor tendon (where extensor muscles insert into lateral epicondyle of the humerus)

*remember from anatomy: Medial-> flexors. Lateral-> extensors.

536
Q

40 year old guy with alcoholic history has epigastric pain, especially after meals. He has ascites and paracentesis shows serosanguinous fluid with high total protein, high amylase, low serum-ascites albumin gradient (SAAG <1.1). Most likely cause of his ascites?

A

Pancreatitis

  • he has chronic pancreatitis (recurrent episodes)
  • pancreatic ascites is a rare complication of pancreatitis! (Repeated inflammation damages pancreatic duct-> leakage of pancreatic juice into peritoneal space-> ascites)
  • serosanguinous fluid= straw-colored ascites fluid (yellow w/ blood streaks)—pancreatic ascites is usually this color
  • pancreatic ascites has high amylase (>1000), high total protein (>2.5), and low SAAG (<1.1 means NOT due to portal HTN from cirrhosis)
537
Q

30 year old guy who recently traveled and had unprotected sex presents with RUQ pain and fever. Breath sounds are decreased on the right lung base, liver is palpable. Labs show WBC count, high alk phos, high ALT and ALT (in the 90’s). U/S of abdomen shows a hypoechoic lesion in the right liver lobe. Diagnosis?

A

Entamoeba histolytica
(A parasite, protozoan that causes liver abscess)

  • RUQ pain, fever, hepatomegaly, transaminases (*but note that ALT, AST are not SUPER high in the hundreds or thousands as in viral hepatitis)
  • recent travel and sex
  • hypoechoic lesion= dark on U/S (less dense, fluid-filled, consistent with liver abscess)

*But why decreased right lung breath sounds?? Don’t get hung up in this—possibly due to a small right-sided pleural effusion due to the inflammation in the nearby liver

538
Q

Lady has hyperthyroidism w/ a nodule in the left thyroid gland. Radioactive iodine scan shows uptake only in the nodule. If she gets pregnant, will baby get hyperthyroidism?

A

No

Remember the most common causes of hyperthyroid are:
1. Graves dz (TSI autoantibodies that stimulate TSH receptors)
2. Toxic adenoma
3. Toxic multi-nodular goiter
(Toxic adenoma and goiter caused by activating mutation in TSH receptor)

She has a single “hot nodule” (radioactive iodine uptake) suggestive of toxic adenoma. Baby won’t get this. *If Graves were the reason she had hyperthyroidism, then the autoantibodies would cross the placenta and baby would be affected.

539
Q

If you don’t treat hyperthyroidism, what can happen to the bones?

A

Bone loss

-excess thyroid hormone increases osteoclast activity-> more bone resorption (breakdown)
(-> hypercalcemia-> dec PTH-> more urinating/ wasting of calcium)

540
Q

How can atelectasis lead to pneumonia?

A

Atelectasis= deflated/ collapsed alveoli—> allows for formation of mucus plugs—> inc risk of infection (always a higher risk of infection when any tube is blocked- from Pathoma)

*spirometers help patients breathe deeper to avoid shallow breathing-> atelectasis-> possible PNA as a complication. So, spirometers are used in the hospital for prevention of atelectasis and indirect prevention of PNA.

541
Q

Rib fracture can be deadly in the elderly because it can lead to pneumonia. Explain how a fractured rib can lead to pneumonia.

A

Rib fracture-> pain-> hypoventilation (hurts to take deep breaths, so patient takes shallow breaths)-> atelectasis (alveolar collapse)-> allows for formation of mucus plugs (and any time you block a tube you inc infection risk)-> pneumonia

542
Q

40 year old lady comes in due to abnormal LFTs (AST is 450, ALT is 512). She has no symptoms. Family history is negative for liver disease and she does not drink. Most likely diagnosis?

A

Autoimmune hepatitis

  • more common in women than men (esp consider if they have another autoimmune disorder)
  • initial presentation varies—can involve abdominal pain, pruritus (itching), possibly cirrhosis (jaundice, ascites), but 25% are asymptomatic!
  • positive serology (anti-smooth muscle antibody, anti-liver/ kidney microsomal type 1, antinuclear antibodies)
  • may have high gamma gap (serum protein-serum albumin of >4) prob due to bad liver producing less albumin
543
Q

Old man with past hospitalization for necrotizing pancreatitis due to alcohol use presents with loose, watery stools that are hard to flush and intermittent epigastric pain for 2 months. He also lost weight unintentionally. Most likely diagnosis?

A

Chronic pancreatitis
(Progressive inflammatory disease of the pancreas)

  • epigastric pain (worse w/ meals and intermittent, but may become constant later on)
  • steatorrhea (loose, fatty stools) (pancreas isn’t functioning well to release enzymes that will help break down fat)
  • weight loss
  • history of pancreatitis episode(s)

*treatment involves pancreatic enzyme replacement

544
Q

What can you look for in the stool to aid in diagnosing chronic pancreatitis?

A

Low fecal ELASTASE

  • this is an inactive enzyme (zymogen) that is released by the pancreas and normally activated by trypsin in the duodenum
  • low levels= exocrine insufficiency
545
Q

Explain how vitamin B12 gets digested/ absorbed (what things does it get bound to)?

A
  1. In mouth, vit B12 is bound to amylase (salivary protein)
  2. In stomach, the acid (pepsin) breaks up vit B12—amylase and vit B12 binds to R-binder
  3. In duodenum, pancreatic proteases break up vit B12—R-binder and vit B12 binds to IF (intrinsic factor), which is made by parietal cells in the stomach
  4. vit B12—IF gets absorbed in the ileum
546
Q

25 year old with PMH of asthma presents with difficulty swallowing solids and liquids. He’s had heartburn lately (doesn’t respond to antacids), but this swallowing problem is new. He has a piece of steak stuck in his throat, and when given water to swallow, he vomits it right back up. Diagnosis?

A

Eosinophilic esophagitis-> food impaction (food stuck in the esophagus due to underlying problem)

*history of asthma and refractory heartburn makes eosinophilic esophagitis the most likely thing predisposing to food getting stuck

547
Q

40 year old man has fatigue, abdominal pain, and bloody diarrhea. Two years ago, he has a colonoscopy w/ biopsy that showed pseudopolyps, mucosal ulcerations, and crypt abscesses. 5-ASA helped his symptoms then. But now he has LLQ tenderness. T. Bili and alk phos are elevated. ALT and AST are mildly elevated. Most likely diagnosis and next step?

A

Primary sclerosing cholangitis
(Inflammation and fibrosis of intrahepatic and extrahepatic bile ducts)
*highly associated with UC (ulcerative colitis), which this guy has

Get a MRCP (magnetic resonance cholangiopancreatography, MRI of bile ducts)
-will show you the “bead on a string” appearance of bile ducts from “onion-skinning” fibrosis

548
Q

35 year old lady with PMH of T1DM (not always compliant with insulin) presents with abdominal pain + confusion. The abdominal pain is worse in the RUQ and epigastrium and is associated with vomiting. She has fever, hypotension, tachycardia. Labs show leukocytosis, glucose of 400, low bicarb, high total and direct bili, mildly elevated ALT, and U/A shows glucose (no ketones) in urine. Diagnosis?

A

Acute (ascending) cholangitis

Charcot triad: (1) RUQ pain, (2) fever, (3) jaundice
Reynolds pentad: (4) hypotension/ shock, (5) altered mental status
*this patient has it all except jaundice is not noted (but bili is high and it jaundice may develop as this worsens)
*anion gap metabolic acidosis due to lactic acidosis from severe sepsis
**no ketones so no DKA. Her hyperglycemia is likely due to infection

Life-threatening emergency! Sepsis-> death happens fast. Give broad-spectrum antibiotics, aggressive IV fluids, and relief of common bile duct obstruction with ERCP or percutaneous drainage

549
Q

How would acetaminophen toxicity present?

A

RUQ pain, nausea/ vomiting, elevated bilirubin, maybe anion gap metabolic acidosis, and SUPER high liver enzymes (AST, ALT in the thousands)

550
Q

50 year old man complains of black stools and occasional epigastric abdominal discomfort. Food makes it better, it is worse at night. A right-sided carotid bruit is noted on exam. Fecal occult blood testing is positive. Most likely diagnosis?

A

Peptic ulcer disease (most likely due to H. Pylori)

  • epigastric abd pain + melena (black stool from digested blood)
  • food makes it better and worse at night when not eating, which suggests a duodenal ulcer (acid-neutralizing enzymes are released there)

*peptic ulcer dz is one of the most common causes of GI bleeds! Think of it and diagnose it with upper GI endoscopy

**he has a carotid bruit, suggesting atherosclerosis. This is a throw-off here

551
Q

Crohn’s vs. UC—which is more likely to present with melana vs. hematochezia?

A

Crohn’s (affects anywhere in GI tract, skip lesions)- MELENA
(Upper GI tract bleeding-> digested blood= black stool)

UC (affects distal colon/ rectum)- HEMATOCHEZIA (lower GI tract bleeding-> bright red blood per rectum)

552
Q

Is ischemic colitis more associated with hematochezia or melena?

A

Hematochezia (bright red blood per rectum)

553
Q

“Hot nodule” (hyperthyroid symptoms with low TSH + single palpable nodule). Diagnosis?

A

Toxic adenoma of the thyroid

*not Graves, even though that’s the most common cause of hyperthyroidism—Graves affects the thyroid in a diffuse manner (TSI thyroid-stimulating antibodies stimulate the entire thyroid), so wouldn’t just be a single hot nodule

554
Q

Nitrofurantoin (Macrobid) is a medication used for what?

A

UTI’s

It is a bactericidal antibiotic

555
Q

HIV patient presents with all the signs and symptoms of TB and CXR shows a upper lobe cavitary lesion. But, his PPD test is negative. Can you rule out TB?

A

No

PPD test can be false negative in an immunocompromised patient! (Their immune system sucks, so does not mount the appropriate response to the injected antigen)

556
Q

Lady presents with weight loss, irritability, insomnia, and palpitations. Exam shows lid retraction, dry skin, and hand tremor. Diagnosis?

A

Thyrotoxicosis/ hyperthyroidism

*lid retraction likely is referring to the protruding eyes seen in Graves dz, the most common cause of hyperthyroidism

557
Q

WHY does hyperthyroidism/ thyrotoxicosis cause HTN?

A

Due to increased myocardial contractility

Thyroid hormone (T3) acts on cardiac myocytes-> increased cardiac output (more blood is getting pumped out, so more blood is in circulation= hyperdynamic circulation)-> so inc BP!

558
Q

Old lady presents with constipation and abdominal pain. She also has urinary frequency and thirst. PMH includes a-fib (on Diltiazem and Rivaroxaban) and osteoporosis (refused medical treatment, taking vitamins and minerals instead). Mucous membranes are dry, no tenderness on palpation of abdomen. Normal urine dipstick. Diagnosis?

A

Milk-alkali syndrome
(Excessive intake of calcium/ inflammation absorbable alkali like calcium carbonate)
*she has osteoporosis, so makes sense she’d take calcium as the vitamin

  • symptoms of hypercalcemia: abdominal pain, constipation, polydipsia (urinary frequency) (*too much calcium-> kidneys have to work harder to filter it. Causes renal vasoconstriction and dec GFR)
  • Milk-alkali syndrome often presents as symptomatic hypercalcemia + metabolic alkalosis + AKI
559
Q

You do a thyroid ultrasound to look at a high-risk thyroid nodule. When should you go onto doing a FNA (fine needle aspiration)?

A

If the nodule is >2 cm or >1 cm with high-risk U/S features

microcalcifications, irregular margins, internal vascularity

560
Q

Man has right shoulder and arm pain after playing golf all day. His arm is weak, especially when lifting objects. On exam, he has decreased right biceps reflex with cervical paraspinal muscle spasm. His pain improves when he is asked to lift his right arm above his head and rest his hand on his head. Diagnosis?

A

Cervical radiculopathy (of C6 nerve root)

-caused by disc herniation-> compression of nerve root
(*could also be caused by spondylosis/ osteoarthritis of neck-> compression of nerve root, but this would be chronic not acute)

  • acute right shoulder/ arm pain, biceps weakness, diminished biceps reflex with cervical muscle spasm
  • shoulder aBduction w/ hand on head provides relief bc it takes tension off the impinged nerve root (diagnostic and therapeutic!)
561
Q

Lady gets treated for TB. Two months later on follow-up exam her sputum is clear (no longer has acid-fast bacilli), but she has fatigue, malaise, nausea, tender hepatomegaly. T bili, ALT (350), AST (400) are elevated, viral hep panel is negative. Liver biopsy shows mononuclear infiltrates and hepatic cell necrosis. Diagnosis?

A

Hepatitis secondary to Isoniazid (causes liver injury that presents similarly to viral hep)

562
Q

Patient with PMH of Crohn’s disease s/p intestinal surgery presents with worsening fatigue and painful paresthesia in her feet. Hb is 8 (low) and MCV is 98 (normal). Next step?

A

Serum vitamin B12 level (test for vitamin B12 deficiency)
**Methylmalonic acid and homocysteine if inconclusive

  • Remember that vit B12 is reabsorbed at the terminal ileum (“iron fist bro”- iron at duodenum, folate/ B9 at jejunum, B12 at ileum) and Crohn’s disease affects the terminal ileum (and his surgery was likely removal of the ileum, which means no B12 reabsorption).
  • Vit B12 is needed for DNA synthesis and myelin formation, so B12 deficiency-> macrocytic anemia + neuro demyelination (causing paresthesias).

**Why is his anemia normocytic? He prob has iron deficiency too (due to GI bleeding from Crohn’s). Vit B12 def (macrocytic) + Fe def (microcytic)= normocytic.

563
Q

What neuro problems can vitamin B12 deficiency cause?

A

Subacute combined degeneration- demyelination of dorsal column (DC-ML)-> dec vibration and proprioception AND lateral corticospinal tracts (CST)-> spastic paresis, enhanced reflexes.
*Peripheral nerves often affected first-> lower extremity paresthesias.

564
Q

60 year old man with MI complicated by cardiogenic shock is in the ICU. He now has acute renal failure and lower GI bleeding from anticoagulation therapy. He has no previous history of thyroid disease, but T3 is low (TSH and T4 are normal). Thyroid diagnosis?

A

Euthyroid sick syndrome (“low T3 syndrome”)

  • Any patient with acute, severe illness may have abnormal thyroid function tests w/ low T3 (less conversion of T4-> T3 due to caloric deprivation, inflammation, and FFA’s/ meds that inhibit 5’-deiodinase)
  • It is not recommended to test thyroid function in sick patients for this reason.
565
Q

What lobe are Broca’s and Wernicke’s areas in?

A

Broca’s—> frontal lobe

Wernicke’s—> temporal lobe

*both on dominant side of brain (if right-handed, on right brain)

566
Q

What lobe is Broca’s area in? A lesion here will cause what?

A

Broca’s is in the frontal lobe.

Nonfluent aphasia (can’t talk well, but can understand)

567
Q

What lobe is Wernicke’s area in? A lesion here will cause what?

A

Wernicke’s is in the temporal lobe.

Fluent aphasia (can talk, but can’t understand—“Wernicke word salad,” words you say don’t make sense)

568
Q

70 year old man with PMH of alcohol use disorder is brought in due to falls over the past 6 months. He is also forgetful to the point where his wife thinks he forgets to use the bathroom bc he urinates on himself. Mini-mental state exam score is low. Diagnosis?

A

Normal pressure hydrocephalus

“Wet, wobbly, and wacky”

  • Urinary incontinence (urinating on self)
  • Gait dysfunction (falls)
  • Dementia (forgetful)

*It is a type of communicating hydrocephalus (decreased CSF absorption) in elderly-> more CSF in ventricles-> stretching of corona radiata (nerve fibers running along edge of ventricles)-> motor neurons are disrupted
(Normal pressure means the CSF opening pressure is normal)

569
Q

What would CT of the brain show in a patient with normal pressure hydrocephalus?

A

Large ventricles (ventriculomegaly) with normal sulci

*It is a type of communicating hydrocephalus (decreased CSF absorption) in elderly-> too much CSF in ventricles (elevated central pressure, improvement with LP)-> stretching of corona radiata (nerve fibers running along edge of ventricles)-> motor neurons are disrupted
(Normal pressure means the CSF opening pressure is normal)

570
Q

Management of a myasthenic crisis patient (severe respiratory weakness-> respiratory failure)?

A
  1. Intubate to protect airway
  2. Hold AChE inhibitors (Pyridostigmine) to reduce excess airway secretions and reduce aspiration risk
  3. Give IV plasmapheresis (or IVIG) + corticosteroids
571
Q

What kind of diarrhea does nocturnal diarrhea (diarrhea waking you up from your sleep) suggest?

A

Secretory diarrhea

*causes: chronic infection, microscopic colitis, bile salt diarrhea, hormone secreting tumor (gastrinoma, VIPoma)

572
Q

Patient comes in with IBS symptoms (fluctuating diarrhea and constipation relieved with a bowel movement, worse with stress). Her diarrhea has become more persistent- she has 6 loose stools/ day and they frequently wake her up at night. Why should we do further testing on this patient?

A

This patient has alarm symptoms, specifically nocturnal diarrhea (this suggests secretory diarrhea, as it occurs during the night when fasting)

*IBS is a clinical diagnosis and involves limited work-up (CBC, serological for celiac dz, inflammatory markers) and reassurance. However, you need to do further testing on patients with alarm features: older age of onset (>50), GI bleeding, nocturnal diarrhea, worsening pain, unintended weight loss, concerning labs (Fe def anemia, elevated CRP), or family hx of IBD or colon CA.

573
Q

Lady has an adrenal mass. She has dark hair growth on her face, acne, clitoromegaly, LMP was 12 wks ago. Will levels of testosterone and LH be high, low, or normal?

A

Adrenal mass with hirsutism (androgen excess)—> HIGH testosterone, LOW LH (due to negative feedback)

574
Q

65 year old with PMH of T2DM (diagnosed 6 mo ago, diet controlled) presents with a progressive rash. It was initially painful, itchy red spots. Now it is crusty. ROS is positive for watery stools and weight loss. On exam, there are erythematous plaques with central clearing and eroded borders on the thigh. There are scattered papules with crusting over the lower abdomen. Labs significant for Hb of 10 and glucose of 175. Next step?

A

Order a glucagon level

  • She likely has glucagonoma (DM, weight loss, watery diarrhea, rash)
  • The rash is “necrolytic migratory erythema” (red blistering rash that spreads across the skin), seen in 70% of glucagonoma patients
  • normocytic anemia likely due to anemia of chronic disease
  • *glucagon >500 confirms the diagnosis + CT abdomen will localize the tumor and check for metastasis
575
Q

50 year old man who works for a new battery manufacturing plant comes in due to frequent stumbling and falls. He also has abdominal pain, constipation, recurrent headaches, and a “pins and needles sensation” in his palms and soles. His wife says he’s more forgetful lately. BP is 160/90, HR is 85. Neuro exams shows reduced pinprick sensation in the hands and feet bilaterally and a wide-based gait. Labs show Hb of 9, MCV of 65, Cr of 2, uric acid of 13 (high). Diagnosis?

A

Lead poisoning

  • Microcytic anemia with basophilic stippling (lead toxicity blocks ALAD, an enzyme needed for heme synthesis)
  • Hyperuricemia (due to impaired purine metabolism)
  • Neuropsych manifestations (memory loss, sensorimotor neuropathy, headaches, ataxia)
  • GI manifestations (abdominal pain, constipation)
  • HTN and possible nephrotoxicity (elevated creatinine)
  • remember that it causes anemia, GI and kidney disease + Kids (lead paint)-> mental deterioration. Adults (batteries, environmental exposure)-> headache, memory loss, demyelination.
576
Q

Gross, painless hematuria. What 2 differential diagnoses should come to mind w/o any further information?

A

Gross, painless hematuria= BLADDER CANCER or KIDNEY CANCER (RCC) until proven otherwise!

577
Q

Old man with PMH of non-small cell lung carcinoma s/p surgical resection comes in after having a seizure. MRI brain shows a solitary cortical mass at the right hemisphere grey-white matter junction. It is consistent with metastasis, but CT chest shows no evidence of recurrent malignancy. He is started on Phenytoin. Next step?

A

Surgical resection of brain mass

  • Metastatic brain tumors= most common brain tumors, often seen at the grey-white matter junction (grey matter= neuron cell bodies, white matter= axons)
  • Single brain metastasis-> surgery is the best choice if possible!
  • *Multiple brain metastasis-> whole brain radiation therapy (or supportive care, but then expectation is that the person will live <2 mo.)
578
Q

Patient has a pleural effusion. Pleural fluid has LDH of 300. Serum LDH is 100. Is this transudative or exudative? Note that normal serum LDH is 45-90.

A

EXUDATIVE

  • Remember Light’s criteria based on Pleural: Serum ratios. Exudative if:
    1. LDH >0.6
    2. Protein >0.5
    3. LDH > 2/3rds the upper limit of normal

P:S LDH is 300/100= 3 >0.5, so exudative (by the 2nd criteria)

Pleural LDH is 300, which is more than 2/3rds the upper limit of normal Serum LDH (upper limit of the normal serum LDH is 90…2/3rds of 90 is 60…300 >60), so exudative (by the 3rd criteria)

579
Q
70kg patient on a ventilator has the following settings. Next best step? 
RR of 18
Tidal Volume of 450 
FiO2 of 40%
PEEP of 5
pH of 7.5
pCO2 of 22 
pO2 of 120
A

Decrease the RR (18 is too high)

-His pH is 7.5 and CO2 is low= respiratory alkalosis (due to hyperventilation-> blowing off too much CO2)

**GUIDELINES:
Ventilation settings (CO2):
-Tidal Volume of 4-8 mL/kg is appropriate (may go lower for COPD or ARDS)
-RR of 10-12 breaths/ min is appropriate

Oxygenation settings (O2):

  • Start with FiO2 of 100% and titrate down to <60% (you want the lowest FiO2 to maintain PaO2 of 50-60 or O2 sat of >90%)
  • PEEP around 5 cm H2O is appropriate
580
Q

Regarding ventilator settings, what are “rules” about appropriate settings? Give specific numbers on what we generally want to keep Tidal Volume, RR, FiO2, and PEEP at.

A

Ventilation settings (CO2):

  • Tidal Volume of 4-8 mL/kg is appropriate (may go lower for COPD or ARDS)
  • RR of 10-12 breaths/ min is appropriate

Oxygenation settings (O2):

  • Start with FiO2 of 100% and titrate down to <60% (you want the lowest FiO2 to maintain PaO2 of 50-60 or O2 sat of >90%)
  • PEEP around 5 cm H2O is appropriate
581
Q

Generally, what do we want to keep Tidal Volume at for a patient on a ventilator?

A

4-8 mL/kg is appropriate (may go lower for COPD or ARDS)

582
Q

Generally, what do we want to keep RR at for a patient on a ventilator?

A

10-12 breaths/ min is appropriate

583
Q

Generally, what do we want to keep FiO2 at for a patient on a ventilator?

A

Start with FiO2 of 100% and titrate down to <60% (you want the lowest FiO2 to maintain PaO2 of 50-60 or O2 sat of >90%)

584
Q

Generally, what do we want to keep PEEP at for a patient on a ventilator?

A

Around 5 cm H2O is appropriate

*PEEP keeps alveoli propped open, but too high of PEEP increases risk of barotrauma (injury to alveoli) and can lead to pneumothorax and decreased CO (inc intrathoracic pressure-> dec venous return)

585
Q

20 year old obese woman has headaches for 6 wks that wake her from sleep. Fundoscopy shows papilledema. CT brain is normal. LP shows elevated opening pressure, otherwise CSF analysis is normal. Diagnosis and next step?

A

Idiopathic Intracranial Hypertension (IIH) aka Psuedotumor Cerebri

Give Acetazolamide

  • High ICP (papilledema is a sign), high CSF opening pressure, otherwise normal neuro exam (may have CN6 palsy), CT, and CSF analysis
  • Often seen in young, obese females
  • We don’t use Mannitol- that’s used in inpatient setting for increased ICP due to cerebral edema (given IV and requires close monitoring for renal function and electrolytes)
586
Q

What are the neoplasias/ findings in MEN 1, MEN 2a and MEN 2b?

A

MEN 1: “pans of pitted pears”

  1. Pancreatic tumors
  2. Pituitary tumors
  3. Parathyroid tumors

MEN 2a:

  1. Medullary thyroid CA
  2. Pheochromocytoma
  3. Parathyroid tumors

MEN 2b:

  1. Medullary thyroid CA
  2. Pheochromocytoma
  3. Mucosal neuromas/ marfanoid habitus
587
Q

Crazy patient insists on being seen right now for an urgent breast exam with only the male doctor in the room because it’s a “private matter.” She doesn’t have an appointment. What do you do?

A

Have the receptionist instruct the patient to schedule an appointment

(Stay professional, she has to make an appointment like everyone else, but don’t blow her off either…obviously you’ll have to tell her a chaperone is required at her appointment if a private exam is indicated)

588
Q

Cavernous hemangioma (benign vascular tumor w/ dilated blood vessels) is associated with what disease?

A

Von Hippel-Lindau Disease

589
Q

IV drug user presents with acute-onset right-sided weakness. She also has fever, HTN, tachycardia. MRI brain shows acute infarct of the left MCA. TEE shows an aortic valve vegetation with mild aortic regurg. You order blood cultures and start her on IV antibiotics for bacterial endocarditis. Should you continue current care with observation, start her on aspirin, or start her on IV heparin?

A

Continue current care with observation

IV antibiotics significantly reduce risk of septic cardioembolism (thrown vegetation-> stroke). Aspirin would only be indicated in ischemic stroke due to atherosclerotic thrombosis or embolism. Heparin is NEVER indicated in stroke due to risk of hemorrhagic stroke (and no proof it helps in the acute setting)/

590
Q

What kind of drug is Amiloride?

A

K+ sparing diuretic

“Almonds” in Sketchy

591
Q

IV drug user has bacterial endocarditis affecting the aortic valve. When would you consider aortic valve surgery?

A

If there is significant valvular dysfunction-> Heart Failure (or if infection persists/ septic embolization is recurrent)

592
Q

30 year old comes in due to epigastric abdominal discomfort and 2 episodes of vomiting a little blood. The patient had an alcohol and cocaine party last night and took aspirin tablets this morning due to headache. Most likely cause of his hematemesis?

A

Gastritis (stomach inflammation) aka Acute erosive gastropathy (or gastric mucosal erosion)

  • alcohol-> stomach mucosal injury
  • cocaine-> vasoconstriction-> dec gastric blood flow to sweep away acid-> stomach mucosal injury
  • aspirin blocks protective prostaglandins-> gastric mucosal injury
  • NOT pancreatitis (this is usually associated with chronic alcohol abuse- not one binge drinking party and doesn’t include hematemesis)
  • NOT esophageal varices (he is young, no signs of cirrhosis)
  • NOT Mallory-Weiss esophageal tear (he hasn’t had multiple vomiting/ retching episodes)
593
Q

Why are diabetic patients at increased risk for non-alcoholic fatty liver disease (NAFLD)? (May present as RUQ fullness, transaminitis.)

A

NAFLD is associated with insulin resistance!

Insulin resistance (body not responding to insulin)-> increased lipolysis (break down of fats for energy, since glucose is not getting into cells), more triglyceride synthesis, and hepatic uptake of fatty acids

594
Q

23 year old with PMH of asthma comes into the ED with difficulty breathing “feels like an asthma attack.” She is non-compliant with her ICS (inhaled corticosteroid) but takes albuterol as needed. She was recently diagnosed with sinusitis and started antibiotics this morning. Her BP is 80/50, HR 120. Exam is significant for bilateral wheezing, prolonged expiration, and urticarial (hives) rash on trunk and arms, and arms are warm to touch. Next step?

A

Give intramuscular epi- this is anaphylaxis!

  • likely caused by the antibiotic she started in the morning for sinusitis
  • epi is a beta > alpha agonist, so causes bronchodilation (beta-2) and vasoconstriction to inc BP (alpha-1). The perfect drug for anaphylactic shock!
595
Q

Explain what anaphylaxis is. Give details.

A

Severe, life-threatening allergic reaction. IgE-> mast cell degranulation-> MASSIVE histamine release-> systemic vasodilation-> hypotension-> poor organ perfusion. Also, causes edema (due to histamine + complement), which compromises the airway.

  • Give epi (beta > alpha agonist) to cause bronchodilation (beta-2) and vasoconstriction to inc BP (alpha-1)
  • Order: give IM epi (if airway compromise)-> antihistamines-> steroids
596
Q

IV drug user presents with difficultly walking and “funny” sensations in his feet. He recently had a URI and is sexually active. He is afebrile, vitals are normal. On exam, he has reduced sensation and muscle strength bilaterally in the legs. He also has mild hyperreflexia and positive Babinski. What test do you need to order?

A

MRI of the spine

  • He is presenting with signs and symptoms of spinal cord compression (bilateral in just lower extremities means you can localize to spinal cord. No UMNs below compression-> UMN signs).
  • Since he’s an IV drug user, an epidural abscess (even w/o fever) is a concern.

**if you were thinking Guillain-Barre, you’d do an LP. But that wouldn’t cause UMN signs or sensory deficit. Bad idea to do LP when abscess is in the differential bc that can introduce more bacteria.

597
Q

Patient with hx of anemia and dark brown urine (did not go to doctor for diagnosis) comes in for abdominal pain, vomiting, fatigue. He has fever, RUQ tenderness worse with deep inspiration, splenomegaly, and jaundice. Labs significant for high WBC count, low Hb, normal MCV, high MCHC, high reticulocytes, high total bili, and high LDH. Cause of anemia?

A

Hereditary spherocytosis

-Classic triad is: (1) hemolytic anemia, (2) jaundice, (3) splenomegaly

  • RBC protein (Ankrin, band 3) deficiency-> RBCs are spherical instead of donut-shaped-> spleen kills them-> hemolytic anemia, splenomegaly, and jaundice is due to too much bilirubin (UCB) for the liver to conjugate.
  • *inc MCHC (mean corpuscular Hb concentration) is due to membrane loss and RBC dehydration

*Patient is presenting with RUQ pain worse with deep inspiration (positive Murphy’s sign) wi nausea/ vomiting. This is due to bilirubin (pigmented) gallstones, which he has increased risk for due to his underlying disease (hemolysis).

598
Q

30 year old woman has worsening SOB after vacation. It started with SOB on exertion, and now she wakes up at night with a choking sensation until she sits up. On exam, she has bilateral pitting ankle edema, hepatomegaly, decreased breath sounds at the bilateral lung bases, and an S3. EKG shows nonspecific ST segment changes. CXR shows enlarged cardiac silhouette and bilateral pleural effusions. Most likely diagnosis?

A

Viral myocarditis

  • presents with CHF symptoms (usually a few weeks after a viral prodrome of fever, malaise, myalgias)
  • EKG shows nonspecific ST changes, echo shows 4 chamber dilation
  • common cause of dilated cardiomyopathy in young adults
  • can be caused by parvovirus B19, HHV 6, coxsackievirus, adenovirus, influenza, HIV
  • thought to happen due to an initially inadequate immune response to viral infection-> virus invades cardiac myocytes-> cytotoxicity (toxic to cells) and impaired contractile function
  • gold standard to diagnose is endomyocardial biopsy with PCR (but cardiac MRI is noninvasive and can help)
599
Q

30 year old lady has a few weeks of feeling sick (fever, myalgia). Now she presents with CHF symptoms. You suspect viral myocarditis. What is the gold standard diagnostic test?

A

Endomyocardial biopsy with PCR (but cardiac MRI is noninvasive and can help)

600
Q

Patient with renal transplant and recently diagnosed paroxysmal a-fib comes in for decreased urine output and fatigue for 2 weeks. Meds include Diltiazem, Apixaban, Prednisone, Tacrolimus, and Mycophenolate. She is hypertensive. Labs show Cr of 2.3 (increased from baseline of 1.4) and Tracrolimus level is elevated. Cause of worsening renal function?

A

AKI due to acute calcineurin inhibitor (CNI) renal toxicity
(*calcineurin= protein that activates T-cells and is targeted by immunosuppressant agents like Tacrolimus)

She was recently diagnosed with a-fib, so recently started Diltiazem (CCB), which blocks P450-> allows other drugs like Tacrolimus to stay in the system longer-> increased Tacrolimus levels-> vasoconstriction and AKI

601
Q

70 year old lady has leg swelling and fatigue. She has had bilaterally lower extremity edema and joint pains for years, which she attributes to old age. She has pitting edema, scattered ecchymoses, and deformed hands. Hb is 11, albumin is 3. U/A shows proteinuria. Diagnosis?

A

AA amyloidosis nephrotic syndrome (peeing out protein, pitting edema)

  • rheumatoid arthritis (deformed hands, joint pains, fatigue)-> chronic inflammation-> release of the acute phase protein serum amyloid associated protein-> AA amyloidos is (that protein misfolds and deposits in tissue)
  • diagnose with renal biopsy showing Congo-red glomerular deposits
602
Q

How do you diagnose AA amyloidosis-> nephrotic syndrome?

A

Renal biopsy showing Congo-red glomerular deposits

*suspect this is patients with nephrotic syndrome (peeing out protein, pitting edema) with history of rheumatoid arthritis (or other chronic inflammatory disease)

603
Q

Name the 6 nephrotic syndromes.

A
  1. Minimal change dz
  2. Focal segmental GN
  3. Membranous GN
  4. Membranoproliferative GN
  5. DM nephrotic syndrome
  6. Amyloidosis (AL in multiple myeloma, AA in chronic inflammation like RA)
604
Q

Name the 4 nephritic syndromes.

A
  1. Post-strep GN
  2. Rapidly progressive GN
    - Goodpastrues, post-strep GN, diffuse proliferative GN, Wegner’s, microscopic polyangiits, or Churg Strauss
  3. IgA nephropathy (Bergers)
  4. Alport syndrome
605
Q

Proliferative glomerularnephritis is associated with what condition?

A

SLE (lupus)

  • also called DIFFUSE proliferative GN (most common nephritic syndrome in lupus with “wire-looping”)
  • *renal biopsy would show glomerular immune deposits on histo
606
Q

25 year old female presents with acute pain in her left knee joint. A few days ago, she had pain in her wrist, then in her ankle. She is sexually active on OCPs. She has a fever and synovial fluid analysis shows elevated WBC count and predominance of neutrophils. Diagnosis?

A

Gonococcal septic arthritis (from Disseminated Gonococcal Infection)

  • asymmetric polyarthralgias (hers is migratory), fever, high neutrophil count in synovial fluid
  • confirm diagnosis with gram stain of synovial fluid, blood cultures, and genital NAAT (PCR) for Neisseria Gonorrhea
607
Q

30 year old man has worsening low back pain and tightness for 3 months. It improves throughout the day with activity. Exam is unremarkable (no tenderness over spine, no weakness, negative straight leg test). Most likely diagnosis?

A

Ankylosing spondylitis

Joint disorder that affects the sacroiliac joints-> fusion “bamboo spine”

608
Q

What is enthesitis? What seronegative spondyloarthropathy is this seen in?

A

Inflammation of entheses, the sites where tendons (attaching bone to muscle) or ligaments (attaching bone to bone) insert into bone

-this is seen in ankylosing spondylitis!

609
Q

What is radiculopathy?

A

Compression of a spinal nerve root/ “pinched nerve”

610
Q

What is the treatment for Rosacea?

A
Topical Metronidazole (gel form) 
(Other options: Azelaic acid or Ivermectin) 

*2nd line: oral tetracyclines

611
Q

35 year old woman has a spinning sensation associated with N/V. 2 weeks ago, she had a URI and “muffled hearing.” PMH includes mild asthma and she takes OCPs. Diagnosis?

A

Vestibular neuritis
(Inflammation of vestibular nerve/ CN8-> vertigo)

  • it is a self-limited disorder that follows a viral URI
  • vestibular neuritis + unilateral hearing loss= labyrinthitis
612
Q

50 year old man is in the ICU for septic shock from PNA. He is treated with IV fluids, antibiotics, and vasopressors. 4 days later, his lungs are doing better but he has anemia and positive occult blood in his stool. Most likely cause?

A

Stress-induced ulcer

  • common in ICU patients w/ severe physiologic stress (risk factors: shock, sepsis, coagulopathy, mechanical ventilation, traumatic spinal cord/ brain injury, burns, high-dose corticosteroids)
  • multifactorial: due to poor blood flow to abdomen (splanchnic hypoperfusion) and kidneys-> uremic toxins accumulate and impair formation of the protective mucosal layer of the stomach-> mucosal injury and bleeding
  • this is the very reason we give GI prophylaxis (PPIs) to patients at high-risk! (*but don’t just throw them at everybody either, as PPIs inc risk for C diff)
613
Q

35 year old man presents with substernal pain and palpitations. BP is 160/110, HR 110, pupils are dilated and reactive to light. EKG shows sinus tachy with ST elevations in leads V1-V4. Most likely explanation?

A

STEMI 2/2 cocaine abuse

614
Q

60 year old man has sudden-onset pain and redness in his left eye associated with severe headache and photophobia. He has been getting over a cold and taking over-the-counter decongestants. Exam shows a non reactive, dilated left pupil that is red. Next step?

A

Tonometry (measure that IOP)

-This is acute angle-closure glaucoma, possibly triggered by decongestant use (alpha-1 agonists like Phenylephrine-> pupillary dilation, which can completely close the anterior chamber angle in a person who’s predisposed bc of their anatomy/ already has a narrower angle)

don’t confuse alpha-1 and alpha-2. Alpha-1-> pupillary dilation. Alpha-2-> decreases aqueous humor production. When we treat open angle glaucoma with alpha agonists (“activate alpha, block beta”), that’s alpha-2 agonists

615
Q

29 year old man presents with weakness in the legs. He recently had trigeminal neuralgia and a URI. Exam shows increased resistance to passive flexion and extension of the legs and decreased vibration and proprioception in the left arm. What will LP likely show?

A

Oligoclonal bands (this is MS)

  • He has neurologic deficits separated by time and space (trigeminal neuralgia, lower limb motor loss, upper limb sensory loss)
  • For diagnosis, do an MRI (expect to see demyelinating lesions affecting white matter). If not classic, follow-up with LP (expect to see oligoclonal bands of Ig).
616
Q

What is albuminocytologic dissociation and what condition is it associated with?

A

Elevated CSF protein w/ normal cell count (on LP CSF analysis)

Characteristic of Guillain-Barré syndrome

617
Q

What does pleocytosis mean?

A

Increased cell count

618
Q

Presence of 14-3-3 protein in CSF analysis (from LP) indicates what?

A

Prison disease (Creutzfeldt-Jakob disease)

*these patients have rapid, profound deterioration of mental status (dementia, behavioral changes) with startle myoclonus

619
Q

50 year old lady with PMH of breast cancer s/p mastectomy presents with worsening SOB for 2 wks. She denies fever and cough. CXR shows white on the bottom right lobe. What is going on?

A

Right-sided pleural effusion

*given her history of breast cancer, this raises concern for a malignant effusion due to breast cancer recurrence

620
Q

In a pleural effusion, what would you expect on exam:

  1. Breath sounds (inc or dec)?
  2. Tactile fremitus (in or dec)?
  3. Percussion (dull, resonant, or hyperresonant)?
A
  1. Breath sounds- DECREASED
  2. Tactile fremitus- DECREASED
  3. Percussion (dull, resonant, or hyperresonant)- DULL
  • There’s fluid between you and the lungs, so lung sounds are more distant
  • There’s fluid between you and the lungs, os vibrations from the lung will be felt less
  • Fluid causes dullness to percussion (vs air creates a hollow, hyperresonant sound)
621
Q

80 year old has muscle pain and morning stiffness in the shoulders and hips. ROS is positive for fatigue and fever. On exam, aBduction of shoulders worsens pain. Labs show high ESR. In 1 week, patient responds well to glucocorticoid therapy. What additional tests, if any, do you need to confirm the diagnosis?

A

None

This is Polymyalgia rehumatica (pain and stiffness in shoulders/ hips/ neck, fatigue, weight loss, low-grade fever)
-classic symptoms, elevated inflammatory markers, and response to glucocorticoids-> no further testing needed

*a small number of cases are associated with giant cell (temporal) arteritis (headache, temporal tenderness, jaw claudication, visual disturbance)

622
Q

70 year old man with hx of chronic constipation has rectal bleeding that is bright red and lightheadedness. NG tube returns nonbilious stomach contents w/o blood. Most likely cause of the rectal bleeding?

A

Diverticulosis (diverticulum hemorrhage)

  • the most common cause of large-volume rectal bleed in adults (painless, but can be associated with lightheadedness or hemodynamic instability)
  • usually resolves on its own, but occasionally requires endoscopic or surgical evaluation

**diverticulosis is most common in the sigmoid colon, but diverticular bleeding is more common in the right colon

623
Q

What is the most common cause of large-volume rectal bleed in adults?

A

Diverticulosis (diverticular hemorrhage)

  • painless, but can be associated with lightheadedness or hemodynamic instability
  • usually goes away on its own, but may require endoscopic or surgical intervention
624
Q

Patient presents with joint pain and fat fingers that look like sausages (“sausage digits”). Diagnosis?

A

Psoriatic arthritis

  • psoriasis + arthritis
  • characteristic “sausage digits” (also called dactylitits, meaning inflammation of the fingers and toes) w/ nail pitting may be present
625
Q

High, low, or normal DLCO and FEV1/FVC ratio in interstitial lung dz?

A
Decreased DLCO (diffusion problem)
Normal FEV1/FVC ratio (restrictive lung dz= problem getting air in, so less air in= less air out, so FEV1 and FVC are both lower, but ratio is maintained)
626
Q

COPD involves what type of emphysema- panacinar or centriacinar?

A

centriacinar

627
Q

Is DLCO high, low, or normal in chronic bronchitis? Emphysema?

A

Chronic bronchitis- DLCO is normal

Emphysema- DLCO is low (alveolar destruction impairs diffusion capacity)

628
Q

Name the 4 types of pneumoconioses (interstitial lung disease due to occupational exposure) and their associated jobs.

A
  1. Coal workers pneumoconiosis (coal miners)
  2. Silicosis (sandblasters and silica miners)
  3. Asbestosis (construction workers, plumbers, and shipyard workers)
  4. Berylliosis (beryllium miners and aerospace workers like in NASA)
629
Q

Patient with PMH of hypothyroidism (on Levothyroxine) is going through menopause, having bad insomnia, hot flashes, and mood swings. If you treat her with estrogen therapy, what will change in terms of her Levothyroxine dosage?

A

Requirement for Levothyroxine would increase (you’d have to up her dose)

*if hypothyroid patient is pregnant or on estrogen therapy, you need to up the thyroid dose. Why? Pregnancy/ estrogen increases TBG (thyroid-binding globulin)-> more TH will bind it-> less free TH (will require more to be at normal levels).

630
Q

Which antiemetic drug can cause extrapyramidal symptoms?

A

Metcolopramide

Remember the D2 rings in Sketchy

631
Q

50 year old man with PMH of HTN and hypercholesterolemia feels “dizzy” like the room is spinning when he turns over in bed or looks up. On exam, he has an ejection murmur at the left sternal border. Most likely diagnosis?

A

Benign paroxysmal positional vertigo (BPPV)

  • vertigo episodes triggered by head positional changes
  • caused by calcium debris in the semicircular canal-> disruption of normal fluid flow through the vestibular system
  • diagnose with Dix-Hallpike maneuver (quickly lie patient down with head rotated 45 degrees)-> will reproduce vertigo and nystagmus
  • treat with Epley/ canalith repositioning maneuvers

*murmur is unrelated! Ejection murmur at left sternal border= still murmur aka functional murmur (an innocent murmur)

632
Q

70 year old lady has confusion, memory loss, and poor sleep. She has also been walking more slowly, has fallen, and is seeing “strangers in the backyard” that don’t exist. MRI brain shows cortical atrophy. Diagnosis?

A

Dementia with Lewy bodies

-dementia with Parkinsonism and visual hallucinations (the dementia may come first or at the same time as the other symptoms)

  • treat with carbidopa-levodopa for Parkinsonism and anticholinergics for cognitive impairment
  • on autopsy: Lewy bodies (eosinophilic inclusion bodies of alpha-synuclein protein) in brain
633
Q

50 year old man has fever/ chills, low back pain, and urinary frequency/ urgency. He is not sexually active and has no discharge. Digital rectal exam reveals anterior tenderness. Diagnosis?

A

Acute bacterial prostatitis
(Most likely due to E. Coli)

-presents as a flu-like illness w/ lower urinary tract symptoms

634
Q

Equation for anion gap?

A

AG= sodium- bicarb- chloride

635
Q

Patient undergoes a lung biopsy. After the procedure, he develops severe SOB and chest pain. BP is 70/40, HR 120, RR 30. He is diaphoretic with cold, clammy skin. CO is low and PCWP is high. Most likely diagnosis?

A

MI-> cardiogenic shock

  • He’s in shock
  • cold skin= vasoconstriction diverts blood away from extremities to vital organs + lack of perfusion
  • CO is low, so heart isn’t pumping well
  • PCWP is high, so LA is extra full since heart isn’t pumping forward well
636
Q

32 year old man comes in for palpitations. He has a family hx of sudden death at a young age due to “heart issues.” Exam reveals decrescendo early diastolic murmur at the left sternal border. Likely diagnosis?

A

Bicuspid aortic valve (causing aortic regurg)

  • bicuspid aortic valve is often associated with aortic STENOSIS, but can also be a cause of aortic regurg (from aortic root dilation)!
  • remember the 3 murmurs best heard at the left sternal border: (1) aortic regurg, (2) pulmonary regurg, (3) HOCM
637
Q

What 3 murmurs best heard at the left sternal border?

A
  1. aortic regurg
  2. pulmonary regurg
  3. HOCM
638
Q

What patients should get an annual low-dose CT scan for lung CA screening?

A

Smokers age 55-80

  • start earlier if 30+ pack year history comes before age 55
  • stop earlier if 15+ years w/o smoking comes before age 80
639
Q

Lady is on Phenytoin for a general tonic-clonic seizure she had 5 yrs ago. She is contemplating pregnancy. Next step?

A

Slowly taper her off Phenytoin and discontinue it

  • she’s been seizure free >2 yrs, so discontinuation of anti-seizure medication is reasonable (wein her off bc rapid withdrawal could cause seizure reoccurrence)
  • Phenytoin is a teratogen! Causes fetal hydantoin syndrome (group of defects due to Phenytoin): cleft palate, microcephaly, nail/ digit hypoplasia, cardiac defects, facial feature abnormalities
640
Q

Lady complains of excessive urination at night for 2 wks. She has a PMH of hypothyroidism (on Levothyroxine) and bipolar (on Lithium). Labs show high serum osmolality, lower urine osmolality, and high TSH. What’s going on?

A

Nephrogenic DI 2/2 Lithium use

  • Lithium accumulation in renal collecting ducts-> ADH resistance (kidneys not responding to ADH, so not retaining water-> pee out the water too much)
  • will have concentrated blood, dilute urine (since you’re peeing out the water)
641
Q

30 year old man is brought into the ED after being found confused in his garage. His pulse is 48 bpm. He is lethargic, diaphoretic, drooling, has constricted pupils, and is wheezing. Next best step?

A

Give Atropine

This is organophosphate poisoning (cholinergic toxicity).

  • remember DUMBBELS (Diarrhea, Urination, Miosis/ constricted pupils, Bronchospasm and Bradycardia, Emesis, Lacrimation, Salivation)
  • PNS symptoms

*Manage A,B,Cs (airway, breathing, circulation), give Atropine (anti-muscurinic), also give Pralidoxime if neuromuscular dysfunction is present

642
Q

Car crash victim is unconscious. CT/ MRI head shows numerous small hemorrhages with blurring of the gray-white matter interface. Most likely diagnosis?

A

Diffuse axonal injury

-traumatic brain injury that results in unconsciousness-> vegetative state-> death

643
Q

75 year old man with T2DM is brought in for weakness and blurry vision. He’s had a cough and sore throat over the last week. Patient appears dry and weak. Labs are significant for high K+, high BUN and Cr, and high glucose of 1,070. Diagnosis?

A

Hyperosmolar hyperglycemic state

-state of severe hyperglycemia (>600), hyperosmolarity, and dehydration in T2DM patients

  • they have such high blood sugar (body not responding to insulin to pull glc into cells)-> hyperosmolarity and glc is spilling out into urine, water follows-> dehydration.
  • hyperkalemia is an issue (lack of insulin to pull K+ into cells and hyperosmolarity also pushes K+ out of cells). Giving insulin to treat can then rapidly shift K+ back into cells-> hypokalemia, so watch it.
644
Q

Huntington disease causes atrophy in what area of the brain?

A

The caudate nucleus and putamen

645
Q

What do ACE inhibitors do to the renal arteries (mechanism)?

A

ACE inhibitors dilate the efferent arteriole
(Angiotensin II constricts the efferent and ACE i’s block that-> dilation of efferent-> dec GFR, inc RPF)

*since they decrease GFR (reduce glomerular hydrostatic pressure, while still preserving renal perfusion flow), they take pressure off the kidneys and are important in preventing nephrotic syndrome (DM nephropathy) in diabetics

646
Q

Lady has altered mental status. She has had headaches and intermittent vision changes over the last week. BP is 220/115. On exam, she has shiny thickened skin with multiple telangiectasias. Cr is high and she has 1+ proteinuria. Next step?

A

Start her on an ACE inhibitor

  • She has systemic sclerosis (diffuse scleroderma)
  • Widespread fibrosis of multiple organ systems (autoimmune vasculopathy involving collagen deposition)-> thick skin, telangiectasias, HTN emergency and acute renal failure from renal vascular injury (scleroderma renal crisis)
  • ACE inhibitors are used to prevent and treat renal HTN crisis (StepUp2Med pg 249), they reduce RAAS activation (*the high Cr is not a contraindication for ACEi’s in this population)
647
Q

Guy has worsening weakness and exertional dyspnea. He had a URI 3 wks ago, symptoms never fully went away. BP is 90/60, HR 110. JVP is 11 cm. Heart sounds are muffled. Diagnosis?

A

Cardiac tamponade (likely 2/2 viral pericarditis)

Beck’s triad: (1) hypotension, (2) distended neck veins, (3) muffled heart sounds
*pulsus paradoxus (>10 drop in BP during inspiration) is another common finding

*normal JVP= 6-8 cm (11 is elevated!)

648
Q

What’s a normal JVP?

A

6-8 cm

649
Q

WHY does cardiac tamponade result in low BP?

A

Due to decreased LV preload

Fluid filling the pericardial sac RESTRICTS FILLING-> decreased preload-> decreased stroke volume-> decreased CO

650
Q

25 year old with PMH of childhood asthma has dry cough since he contracted a URI 10 days ago. He is afebrile. On exam, he has wheezing. Next step?

A

Inhaled bronchodilator

  • This is likely ACUTE BRONCHITIS (90% viral)
  • cough can be dry or productive
  • self-limited (lasts 3-4 wks), but treat with supportive therapy (throat lozenges, over-the-counter cough suppressants, albuterol in patients with underlying asthma)—NO antibiotics
  • fever is not typical (if fever is present, think PNA)
  • no need to get a CXR here to r/o PNA—do this for patients with high fever, tachypnea, crackles/ dullness on lung exam, or underlying COPD
651
Q

68 year old woman with PMH of breast cancer and vertebral mets complains of general weakness and dizziness. Radiation + chemo are planned, but she hasn’t started yet. BP is 105/65 with orthostatic changes and HR is 102. Na+ is low and K+ is at the upper limit of normal. Diagnosis?

A

PRIMARY ADRENAL INSUFFICIENCY (lack of aldosterone and cortisol production by the adrenal glands)
-likely 2/2 metastatic breast cancer (adrenal glands are a common site of metastasis, and when they are destroyed by cancer, they stop working like they should)

  • > low BP, low Na+ (not reabsorbing it), high K+ (not peeing it out)
  • confirm the diagnosis by cosyntropin (synthetic ACTH) stimulation testing (after giving ACTH to stimulate the adrenal glands, her adrenals would still fail to produce cortisol)
652
Q

How do you confirm a diagnosis of primary adrenal insufficiency?

A

Do a Cosyntropin (synthetic ACTH) stimulations test (after giving ACTH to stimulate the adrenal glands, adrenals will still fail to produce cortisol if adrenal insufficiency is present)

  • this test is usually done in the morning when cortisol levels are normally most pronounced (body gets ready for the day of stress)
  • remember adrenal insufficiency= lack of aldosterone and cortisol by the adrenals
653
Q

Man got into a motorcycle crash, now has weakness + loss of pain and temp in both legs. Touch and vibration are intact. What spinal cord problem does this patient likely have?

A

Anterior cord syndrome

(Motor + pain/ temp are broken on both sides, since the anterior part of the spinal cord is where CST and STT fibers cross)

*from injury to anterior spinal artery in trauma

654
Q

How does Anterior cord syndrome present?

A

Motor + pain/ temp are broken on both sides

since the anterior part of the spinal cord is where CST and STT fibers cross

655
Q

How does central cord syndrome present?

A

Pain/ temp broken on both sides (affects STT crossing)

  • paralysis and burning pain in upper extremities
  • usually in old patients after forced hyperextension of the neck (whiplash)
  • can be assoc with bladder dysfunction
656
Q

35 year old obese woman complains of periodic visual disturbance. She goes blind for several seconds when standing up and has morning headaches. Visual testing reveals large blind spots. What eye condition do we call this?

A

Papilledema (swollen optic nerve due to elevated ICP)

  • causes transient vision loss with change in head position
  • headaches due to inc ICP are usually worse in the morning

*she may have idiopathic intracranial HTN (pseudotumor cerebri), given her young age, obesity, and lack of other comorbidities…next step would be neuroimaging (CT or MRI brain)

657
Q

45 year old man has headache and confusion for 2 days. Hb is 8, BUN is 30, Cr is 2.2. Peripheral blood smear shows fragmented RBCs. PT is normal. Next best step?

A

Plasmapheresis (plasma exchange)

  • This patient has TTP (Thrombotic Thrombocytopenic Purpura)
  • Presents with thrombocytopenia, microangiopathic hemolytic anemia, renal insufficiency, neuro changes (headache, confusion, coma, stroke), fever
  • autoantibody against ADAM 13, an enzyme that normally degrades vWF-> vWF stays there, clumping up platelets-> dec platelets/ thrombocytopenia (since they are getting used up forming microthrombi) + CNS abnormalities (tends to involve CNS vessels)
  • fragmented RBCs= shistocytes/ helmet cells
  • PT, PTT are normal (unlike in DIC)
658
Q

Multiple people are brought into the ED from the shopping mall after smelling a fruity odor then having symptoms. Your patient developed rapid SOB. She is diaphoretic and drooling. Pupils are pinpoint and non reactive. She is wheezing. Likely diagnosis and test to confirm?

A

Organophosphate poisoning

RBC acetylcholinesterase activity test
*you can also just give Atropine and if it works in resolving the symptoms, you know your diagnosis was correct

  • organophosphates block AChE in muscarinic and nicotinic synapses-> ACh can’t be broken down-> too much ACh-> overstimulation at the NMJ to where muscles stop responding
  • PNS symptoms: DUMBBELS (Diarrhea/ Diaphoresis, Urination, Miosis, Bronchospasm/ Bradycardia, Emesis, Lacrimation, Salivation)
  • think of this in an agricultural worker (pesticides) or in a group of people with similar symtpoms (nerve agent—these chemical weapons are used in terrorist attacks and the gas is colorless, tasteless, but has a slight fruity odor)
659
Q

55 year old man comes in due to having many falls over the past few wks. He’s also had dry skin, dry mouth, erectile dysfunction, and resting tremor. BP is 120/80 supine, 90/60 standing. Exam shows rigidity and bradykinesia (slow movement). Most likely diagnosis?

A

Multiple system atrophy (Shy-Drager syndrome)
-degenerative disease w/ Parkinson’s symptoms + autonomic insufficiency

  • this patient has Parkinson’s symptoms—resting tremor, rigidity, bradykinesia
  • and autonomic dysfunction (postural hypotension, erectile dysfunction, dry skin, dry mouth)
  • other autonomic symptoms may include: loss of bowel or bladder control, abnormal salivation or lacrimation, gastroparesis…
660
Q

80 year old man with PMH of HTN (on Ramipril and Chlorthalidone) comes in due to sudden painless vision loss in his left eye. Funduscopic exam of the left eye shows swelling of the optic disk, retinal hemorrhages, dilated and tortuous veins, and cotton wool spots. Diagnosis?

A

Central retinal vein occlusion
(Blood not draining from the eye)

*this is describing a “blood and thunder” appearance (optic disk swelling, retinal hemorrhages, dilated and tortuous veins, cotton wool spots)

661
Q
65 year old man with PMH of osteoarthritis (on naproxen for pain relief) presents with fatigue. Hb is 8.5, MCV is 72, rest of labs are normal. Would the following most likely be high, low, or normal? 
Serum iron
Serum ferritin
TIBC (total iron binding capacity) 
Transferrin saturation
A

*He has microcytic anemia= iron deficiency, anemia of chronic dz, sideroblastic anemia, or thalassemia. Most likely IRON DEFICIENCY (usually due to GI blood loss in men)- this is most common + he has no inflammatory disorders that point to anemia of chronic dz (osteoarthritis is wear and tear—it’s not autoimmune like rheumatoid)

Serum iron- LOW
Serum ferritin (iron in storage)- LOW
TIBC (total iron binding capacity) (Tf)- HIGH
Transferrin (Tf) saturation- LOW

662
Q

What type of drug is Naproxen?

A

NSAID

Trade name= Aleve

663
Q

Man has burning pain and cramping in his feet and a pins and needles sensation in his toes. He works as a construction worker and is exposed to vibrating machinery. He doesn’t smoke, drinks 6 beers/ day. Exam shows loss of ankle reflex bilaterally and impaired touch and vibration sensation on the feet. Hb A1c and MMA levels are normal. Diagnosis?

A

Alcoholic peripheral neuropathy
(a toxic neuropathy)

  • alcohol is neurotoxic-> symmetric distal polyneuropathy in a “stocking and glove pattern”
  • loss of deep tendon reflexes begins with loss of ankle reflexes
  • treat with stopping alcohol (improves symptoms), give thiamine supplements, and may give gabapentin or TCAs for refractory neuropathic pain
664
Q

What hereditary polyneuropathy is slowly progressive and results in foot drop, muscle weakness, and lower leg atrophy?

A

Charcot-Marie-Tooth disease

665
Q

Most common autoimmune neuropathy?

A

Guillain-Barré syndrome

  • rapidly progressive, ascending
  • usually follows a GI infection or URI
666
Q

If a CHF patient has hypOnatremia, do we care?

A

Yes—hypOnatremia in patients with CHF parallels the severity of HF and is a predictor of clinical outcomes

-it is caused by increased ADH (more water retention-> dilutes [Na+]) and increased renin and NE (increase Na+ and water reabsorption in the distal tubules)

667
Q

HIV patient presents with jerking movements of his arms and face. He also has been having headaches. MRI brain shows ring-enhancing lesions at the gray-white matter junction and basal ganglia. Diagnosis and treatment?

A

Toxoplasma encephalitis

Sulfadiazine + Pyrimethamine
Sulfa dyed eggs + pyramid shape on dyed eggs

668
Q

60 year old patient with PMH of HTN and hyperlipidemia. 6 hrs ago he had weakness that resolved after 30 min. On exam, patient has an S4 and mild right-sided pronator drift. EKG shows T-wave inversion in leads I and V6. CT non con of brain is normal, MRI is pending. Next step?

A

Give Aspirin

  • This is likely ischemic stroke and patient is not a TPA candidate (last normal was >4.5 hrs ago)
  • ASA is the only antiplatelet agent hat is effective in reducing the risk of early ischemic stroke recurrence- give within 24 hrs!
669
Q

What are the dietary recommendations to prevent recurrent calcium oxalate kidney stones?

A
  1. Increase fluid intake
  2. Decrease sodium intake
  3. Normal calcium intake
  • hydration always helps prevent kidney stones
  • low sodium in the diet promotes sodium and calcium reabsorption (by pushing more calcium in the blood, you pee out less= less calcium kidney stones)
  • calcium intake has no effect
670
Q

65 year old man has unsteady gait and frequent falls for 2 months. His left side feels weak. He also has morning headaches. Exam shows swaying to the left when walking and increased resistance to passive flexion of the left upper and lower extremity. Most likely cause?

A

Brain tumor

  • morning headaches + focal neuro deficits
  • diagnose with MRI brain
671
Q

Dullness to percussion and increased breath sounds. What does this indicate?

A

Consolidation of the lung

  • dullness= trapped fluid
  • increased breath sounds (bronchial, crackles)= fluid in lungs (**vs pleural effusion breath sounds are decreased bc the fluid is in the pleura b/w you and the lungs)
672
Q

40 year old woman complains of palpitations. Over the past few months, she has had anxiety and weight loss. BP is 110/80, HR is 125 and irregular. Exam shows lid lag and fine tremor of the hands. EKG shows a-fib with RVR. What initial medical therapy should you give?

A

Beta blocker (Propranolol)

This is hyperthyroidism

  • a-fib with RVR, tachycardia
  • anxiety
  • weight loss
  • tremor
  • beta-blockers are used for initial treatment of a-fib 2/2 hyperthyroidism to control HR and sympathetic symptoms (they also block peripheral conversion of T4-> to active T3)
  • continue beta-blockers until patient becomes euthyroid + treat hyperthyroidism with medication, radioactive ablation, and/or surgery
673
Q

55 year old woman with PMH of HTN (on Lisinopril) presents with severe headache that got rapidly worse over a few minutes and hasn’t responded to analgesics. It is associated with vomiting and photophobia. Exam shows mild neck stiffness, but full neuro exam is normal. Non-con CT head is normal. Next step?

A

Lumbar puncture (LP)

This sounds like subarachnoid hemorrhage (*from ruptured berry aneurysms)

  • thunderclap headache (reaches max intensity in few minutes) “worst headache of my life”
  • associated with N/V, photophobia
  • neck stiffness= meningeal irritation (from the blood)

You would expect a non-con CT to show bleeding, but if it comes back negative and you are highly suspicious of subarachnoid hemorrhage, do a LP to check!
Xanthochromia (yellow/ bloody CSF)= SAH

*NOT a migraine (migraine headaches don’t irritate the meninges)

674
Q

A hospital undergoes review of malpractice claims. They tally up the number of missed or delayed diagnoses over the last 10 yrs and find the rate is 25% higher in their ER compared to other hospitals nationwide. Most likely underlying cause of these medical errors?

A

Communication failures between providers

675
Q

What has been proven to most effectively improve communication of relevant information in patient transfers (signing off a patient from one doc to another)?

A

Implementing a signout checklist

*face-to-face handoffs w/o interruptions are also ideal, but not proven to be AS good as checklists in the signout procedure for improving medical outcomes

676
Q

32 year old man is seen to “check up on my liver.” 2 yrs ago, he shared needles with another IV drug user and was positive for hep B surface antigen, negative for all other antigens and antibodies. He stopped doing IV drugs since. His hep B serology is most likely to show what—positive or negative?

  • HBsAg
  • HBsAb
  • HBcAb
A

HBsAg- NEGATIVE
HBsAb- POSITIVE
HBcAb- POSITIVE

  • remember “SpECiES” (HBsAg, HBeAg, HBcAb, HBeAb, HBsAb)
  • serum antigen, e antigen (infectivity), core antibody, e antibody, and serum antibody

This patient had the serum antigen at the time he presented with infection, but not the e antigen and no antibodies (hadn’t been produced yet)…so now he will have the core antibody (1st one that gets made- in “window period”) and serum antibody (may or may not have the e antibody)
(No antigens anymore bc the infection is resolved)

677
Q

IV drug user gets hep B. What’s the most likely prognosis?

A

95% of hep B infections are self-limited (resolve on their own)

**vs. babies who get it from vertical transmission are likely to get the chronic form and children age 1-5 have a 30-50% chance of getting the chronic form

678
Q

65 year old obese smoker man has high fevers, confusion, and productive cough for 2 days. On exam, his cervical lymph nodes are enlarged but not painful, he has rales at the R lung bases, and he has hepatosplenomegaly. CXR shows R lower lobe infiltrate. Labs show low Hct, low platelets, and leukocytosis (low neutrophils and monocytes, high lymphocytes). Blood cultures, antibiotics, and supportive care are initiated. Next step?

A

Get a FLOW CYTOMETRY of the peripheral blood
(To diagnose CLL—will show too many naive B-cells)

  • Fever + cough + R lung infiltrate= PNA
  • Note that most bacterial infections, including PNA, cause leukocytosis with neutrophil predominance—this guy has dramatic lymphocyte predominance
  • Old patient + lymphocytosis (too many lymphocytes) + anemia + thrombocytopenia + hepatosplenomegaly= CLL (chronic lymphocytic leukemia) *the most common leukemia
  • *anemia and thrombocytopenia due to the B-cells crowding out other cell lines
  • *note that a peripheral blood smear could also aid in the diagnosis (would show smudge cells)
679
Q

Rales vs. rhonchi vs. crackles vs. wheezing?

A

“Rales in the tails, rhonchi in the bronchi”

Rales= Crackles
-from the alveoli

Rhonchi= low-pitched wheezing
-from the bronchus

680
Q

75 year old man comes in due to tremor when holding the newspaper or drinking his morning cup of coffee. He has an obvious tremor when touching his finger to the doctor’s finger. Diagnosis and treatment?

A

Essential tremor
Propranolol

  • this is an action tremor (vs. the resting tremor you get in Parkinson’s)
  • Propranolol is first-line, but other options are Primidone and Topiramate
681
Q

Patient has colon cancer. You explain his treatment options, but he says he does not want treatment. He is mentally competent. Do you ask him why he doesn’t want treatment or respect his wishes?

A

Ask him why he doesn’t want to have treatment

-before you respect his decision to not have treatment, you need to explore why he feels this way…there’s a chance he’s just in denial or afraid and needs more clarification on treatment options or depressed…

Always discuss reasons for a patient’s decision to withhold treatment before honoring it!

682
Q

45 year old man presents with daytime headaches, dizziness, and nausea. He doesn’t snore, he smokes, he works in an underground parking garage. Vitals are normal and pulse ox is 97%. Labs show Hct of 60% (normal is 40-53%). Most likely diagnosis?

A

Chronic CO poisoning
(Exposure to car exhaust in an enclosed space + has intermittent headaches, dizziness, nausea + polycythemia)

*CO has a greater affinity for Hb than oxygen-> carboxyhemoglobinemia. Less oxygen is able to bind to Hb to be delivered to tissues (left shift on Hb curve)-> hypoxia-> reactive polycythemia (kidneys release more EPO so more RBCs are made) to try to compensate.

  • *normal pulse ox bc pulse oximetry cannot differentiate between Hb bound to CO vs. oxygen
  • *diagnose with ABG (arterial blood gas)!
683
Q

60 year old man has sudden-onset weakness in the lower extremities since swimming in a pool and cannot pee. Medical hx includes DM and he had back pain the last 2 months. Exam shows motor weakness in both legs. He has no sensation in the perineal are and rectal tone is absent. Next step?

A

Emergency neurosurgical evaluation!

This patient has acute spinal cord compression, likely due to vertebral metastasis (progressive back pain and activity brought on the onset of concerning symptoms)

  • some causes of spinal cord compression are: fracture, trauma, disk herniation, malignancy, epidural abscess
  • urinary retention occurs with lesion above S2 (autonomic tract involvement)
  • regardless of the cause, when the spinal cord is compressed, you need to get a surgery consult for decompression 1st, do neuroimaging, and IV glucocorticoids may be given
684
Q

HIV patient has severe watery diarrhea and lightheadedness and lost weight. He’s non-compliment with his HIV meds and last CD4 count was 95. He works at a horse breeding farm, denies recent travel. Most likely responsible organism?

A

Cryptosporidium

  • profuse watery diarrhea in AIDS (seen w/ CD4 count <180)
  • get it from drinking contaminated water, animal contact (horse breeding), or person-to-person contact
  • stool exam w/ modified acid-fast stain shows cryptosporidium oocytes
685
Q

AIDS patient with severe watery diarrhea and low-grade fever. CD4 <180. What should you think of?

A

Cryptosporidium

686
Q

AIDS patient with watery diarrhea and crampy abdominal pain, no fever. CD4 count is <100. What should you think of?

A

Microsporidium

687
Q

AIDS patient with watery diarrhea and high fever. CD4 count is <50. What should you think of?

A

Mycobacterium avium complex (MAC)

688
Q

AIDS patient with frequent, small-volume diarrhea w/ blood, abdominal pain, and low-grade fever. CD4 count is <50. What should you think of?

A

Cytomegalovirus (CMV colitis)

689
Q

Patient who is being treated for TB complains of fatigue. His Hb is low, MCV is normal, serum iron is high, and TIBC is low. On microscopy, some RBCs are hypochromic and others are normochronic. Next step?

A

Give Vitamin B6 (Pyridoxine)
(*the bees w/ pyramid sticks in Pixorize)

  • Isoniazid depletes Vit B6 stores. Since Vit B6 is needed to make protoporphyrin (and to make heme, since heme= Fe + protoporphyrin), this causes sideroblastic anemia (type of microcytic anemia, though it starts off normocytic).
  • increased serum Fe (there is lack of protoporphyrin to combine with Fe, so excess free iron-> ringed sideroblasts)
  • decreased TIBC (Tf)
690
Q

30 year old man comes in for a check-up. He has had a lot of stress lately, otherwise no concerns. Labs show Hb of 10.5, MCV of 60, and high RBC count. Peripheral blood smear shows target cells. Diagnosis and treatment?

A

Thalassemia minor
No treatment needed

-mild microcytic anemia, target cells, high RBC count

  • microcytic anemia means choices are: iron deficiency, anemia of chronic dz, sideroblastic anemia, or thalassemia (alpha or beta, minor or major)
  • alpha thal is due to a gene mutation-> dec synthesis of beta globin chain
  • target cells due to dec Hb (RBC “blebs up” like a deflated B-ball you can indent)
  • inc RBC count (to compensate for having less functional Hb)
691
Q

Young man comes into the ED for chronic tension headaches that are “debilitating.” He only gets partial relief with NSAIDs. He has no associated symptoms. He was evaluated for the same headaches last year and extensive neuro work-up was normal. He is worried about his schooling and constantly worried he may have an undiagnosed brain tumor. What med may help?

A

SSRI (like Fluoxetine)

This is somatic symptom disorder (when a patient has a minor symptom like a headache and is convinced they got brain cancer for sure). Manage with regular appointments and avoid unnecessary referrals and tests. SSRIs and cognitive-behavioral therapy may help too.

692
Q

Lady had a TIA. She had a mitral valve replacement 5 years ago. In addition to neuroimaging, what test do you need to do?

A

Echocardiogram

-Look for evidence of Prosthetic Valve Thrombosis (PVT), a complication of artificial valve repair (patients can have a new murmur, HF due to stenosis/ valvular obstruction or regurg from valve not aligned perfectly, or a thromboembolic event like a TIA)

693
Q

70 year old man with PMH of HTN, DM, CDK, and CAD presents with cough. CXR shows a R upper lung nodule. A CTA with IV contrast confirmed this. Patient is admitted and found to have worsening acute renal failure. BP remains 140s/90s, but BUN and Cr uptrend. What would have prevented this AKI?

A

IV hydration

He has CDK and you gave him IV contrast—stupid idea! -> contrast-induced nephropathy. If you have to give contrast for imaging, dilute the contrast with hydration to prevent injury to the kidneys.

694
Q

Would you expect electrolytes (Mg, K+, Na+, P) to be high, low, or normal in anorexia nervosa?

A

All normal to low

*HypOkalemia is common particularly in the binge-purge type (vomit out K+-> low K+ in blood)

695
Q

When is Metformin contraindicated?

A

In acute renal failure, liver failure, or sepsis—as these conditions increase risk of LACTIC ACIDOSIS

696
Q

Duke criteria is a MDCalc tool/ objective way to define what disease?

A

Infective endocarditis

*need 2 major + 1 minor or 3 minor criteria

697
Q

Patient with PMH of rheumatic fever, mitral stenosis, and recent dental procedure has fatigue and SOB. Cardiac exam is significant for a holosystolic murmur at the cardiac apex with a diastolic rumble and hemorrhages under a nail bed. Labs show leukocytosis, anemia, elevated Cr. U/A shows 1+ protein. Blood cultures are most likely to isolate what organism?

A

Strep viridans (includes strep mutants, mitis, oralis, and sanguinis)

-infective endocarditis following a dental procedure (previously affected heart valve)

698
Q

Strep mutans belongs to what class of bacteria?

A

It is a strep viridans

*In the strep viridans family: strep mutants, mitis, oralis, and sanguinis

699
Q

50 year old lady has a “stabbing” chest pain when taking a deep breath. She has also had frequent pain in her hands and feet. Exam reveals fine inspiratory crackles and decreased breath sounds over the L lung base. Wrist and hand joints are swollen and tender. Labs are normal, ANA is negative. CXR shows increased interstitial markings and L pleural effusion. Thoracentesis shows it’s exudative and pleural glucose is 10. Most likely cause of the pleural effusion?

A

Rheumatoid Arthritis (RA)

Pleural effusions due to RA have a low glc, high LDH

  • if pleural glucose level is low (<60), consider RA as the cause of the pleural effusion! (StepUp2Med pg 82)
  • exudative= due to inflammation
  • fine crackles + increased interstitial lung markings= interstitial lung disease (ILD), a pulmonary manifestation of RA
700
Q

25 year old military recruit with PMH of allergic rhinitis (on chlorpheniramine) suddenly collapsed at bootcamp. He lost consciousness, now he’s awake but confused. Temp 106, BP 90/60, HR 130, RR 22. He is sweating. On lung exam, he has rales (crackles) at both lung bases. Labs show thrombocytosis, leukocytosis, and slightly elevated PT (normal Hb, BUN, PTT). Most likely cause of his symptoms?

A

Exertional heat stroke

-heavy exercise (bootcamp) presenting with hyperthermia (>104 F), confusion, and multi-organ dysfunction

  • way too hot-> vasodilation happens to shunt blood toward skin to sweat (though sweating is inadequate to cool the body at this temp)-> distributive shock (blood is not where it needs to be feeding the central organs)-> organs die and become ischemic-> tissue factor is released-> pathologic activation of coag cascade= DIC
  • may also cause CNS dysfunction (confusion, seizures), ARDS (his lung crackles), hepatic failure, renal failure…end-organ damage from the distributive shock
  • may present with anhidrosis (lack of sweating from dehydration) or sweating
  • leukocytosis is from the severe stress
  • this patient was taking an antihistamine, which has anti-cholinergic activity and can further impair heat dissipation (“hot as a hare”)
701
Q

Treatment for exertional heat stroke and non-exertional (classic) heat stroke?

A

Exertional heat stroke—> rapid cooling with ice-water immersion

Regular heat stroke—> evaporative cooling with spraying lukewarm water as fans blow on the patient’s skin

*why? Studies have shown these treatments are based in each case in reducing mortality

702
Q

40 year old lady with no PMH comes in for 2 syncope episodes over the last few days. For the past couple weeks, she’s had anorexia, nausea, and abdominal pain. BP is 85/50. On exam, her abdomen is mildly tender and she has hyperpigmentation in the palmar creases. Diagnosis?

A

Primary adrenal insufficiency (Addison’s disease)

  • the adrenal glands are the problem (usually due to autoimmune adrenalitis, but can also be due to TB or metastatic infiltration)-> deficiency of aldosterone, cortisol (hypOnatremia, hypERkalemia, hypotension)
  • high ACTH (neg feedback) (you get hyperpigmentation from this! Since it binds Melanocyte-Stimulating Hormone)

-initial symptoms are nonspecific: fatigue, weakness, weight loss + GI symptoms: nausea, abdominal pain, diarrhea

703
Q

Man complains of pain, watering, and redness in his left eye. He had similar symptoms in the same eye a few months ago. Exam of the left eye reveals vesicles and dendritic ulcers on the cornea. Diagnosis?

A

Herpes simplex keratitis
(Keratitis= inflammation of the cornea)

*common cause of corneal blindness in the US

704
Q

Who’s at risk for bacterial keratinitis?

A
  • contact lens wearers
  • patients w/ corneal trauma
  • patients w/ foreign body in eye

(Keratitis= inflammation of the cornea)

*In bacterial keratitis, the cornea appears HAZY with central ulcer and adjacent stromal abscess

705
Q

65 year old man with PMH of HTN (on Lisinopril) presents with palpitations for 2 weeks. An echo last year showed mild LA dilation and LV hypertrophy. BP is 170/90. EKG shows irregularly irregular rhythm. Next step?

A

Beta-blocker or CCB

  • This is a-fib with RVR
  • Do not cardiovert bc he is hemodynamically stable and has had this 2+ weeks (not <48 hrs)
  • Use medications to rhythm control (*other options that may be considered are anticoaugaiton for 3 wks-> CV or medications to rhythm control)
706
Q

Regardless of whether you are managing an a-fib patient by rate or rhythm control, you need to use what scoring assessment to determine if you need to give them long-term anticoagulation to prevent thromboembolism?

A

CHADS-VASc score

*C- CHF
H- HTN
A- Age >75 (2 points) 
D- DM
S- Stroke/ TIA (2 points) 
V- Vascular dz (prior MI, PAD) 
Sc- Sex category (female) 

(Max points= 9)
0 points-> no anticoagulation indicated
1 point-> moderate risk, consider anticoagulation (none, ASA, or oral anticoagulant)
2+ points-> anticoagulation for sure

707
Q

65 year old man with PMH of DM and HTN has newly diagnosed a-fib. Should you start him on long-term anticoagulation?

A

YES- CHADS-VASc score is 2 (one point for HTN + one point for DM)

*C- CHF
H- HTN
A- Age >75 (2 points) 
D- DM
S- Stroke/ TIA (2 points) 
V- Vascular dz (prior MI, PAD) 
Sc- Sex category (female) 

(Max points= 9)
0 points-> no anticoagulation indicated
1 point-> moderate risk, consider anticoagulation (none, ASA, or oral anticoagulant)
2+ points-> anticoagulation for sure

708
Q

25 year old healthcare worker comes in after a near-syncope episode. ROS is positive for chronic diarrhea with 10-12 watery BMs/ day and abdominal cramping. The diarrhea often wakes her up at night. BP is 110/70 supine, 9/50 standing. Labs show hypOkalemia and metabolic alkalosis. Colonoscopy reveals areas of dark brown mucosal pigmentation in the proximal colon. Most likely cause of her symptoms?

A

Laxative abuse

  • young (more likely to be concerned about appearance), healthcare worker (has access to laxatives)
  • watery diarrhea, abdominal cramping
  • low BP and near-syncope (from dehydration 2/2 diarrhea)
  • hypokalemia (you lose bicarb and K+ in diarrhea)
  • metabolic alkalosis (you lose bicarb in diarrhea, so you’d expect met acidosis, but met ALKALOSIS is a common finding in laxative abuse *may also have hypermagnesemia if Mg-containing laxative is used)
  • the give-away is the dark pigmentation on colonoscopy= MELANOSIS COLI (*remember the guy tanning in Sketchy: chronic use of Senna-> melanosis coli/ brown pigmentation of colon)
709
Q

Man abruptly stopped taking steroids after a year for polymyalgia rheumatica bc he didn’t like the weight gain side effect. He comes in for fatigue, weakness, and loss of appetite. Labs show slightly low Na+. Would you expect his ACTH and Cortisol to be high, low, or normal?

A

ACTH- Low
Cortisol- Low

Why? On corticosteroids-> cortisol is high (by the exogenous supply), so ACTH is low (neg feedback)
Abruptly stop steroids-> cortisol is low. You’d expect ACTH to go high (neg feedback), but it hasn’t had time to adjust so it remains low (this is why it’s important to wean patients off steroids!)

  • primary adrenal insufficiency affects all layers of the adrenal gland (bc the adrenal gland is the source of the problem), but secondary adrenal insufficiency (as in this Q) does not affect all layers- depends on the cause. Here, aldosterone levels are normal, but cortisol is low.
  • why the mild hypOnatremia? Cortisol-> vasoconstriction and inc BP. Lack of cortisol-> vasodilation and dec BP. Body responds by activating RAAS-> aldosterone + ADH. The increase in ADH > Aldo-> excess water retention-> mildly low Na+.
710
Q

65 year old man has had fatigue, retrosternal heaviness, and nonproductive cough for weeks. He comes in due to chest pain after a party at a bar. CXR shows a large anterior mediastinal mass displacing the trachea. Labs show normocytic anemia, mild leukocytosis with high eosinophils, and really high LDH. Most likely diagnosis?

A

Hodgkin lymphoma

  • old w/ fatigue and mediastinal mass
  • Hodgkin lymphoma is a common cause of mediastinal mass. Presents with cough, SOB, retrosternal chest pain (due to compression of adjacent structures), lymphadenopathy, B symptoms, worsening pain w/ alcohol.
  • labs show elevated LDH and eosinophilia
  • MILD leukocytosis bc cancer cells are confirmed to lymph nodes, not circulating (it’s lymphoma, not leukemia!)
  • Remember eosinophils mainly w/ parasites + allergies, but also in Hodgkin lymphoma (related to cytokines activating them)
  • High LDH due to increased cell turnover in cancer (not always hemolysis)
711
Q

Is high LDH always due to RBC hemolysis (hemolytic anemia)?

A

No—high LDH is a result of cell rupture. Also consider cancer (ex: Hodgkin lymphoma), where there is increased cell turnover.

712
Q

Why would you expect lymphocytes to only be MILDLY elevated (not crazy high) in lymphoma?

A

The cancer cells in lymphoma are confined to the lymph nodes, not circulating around much as in leukemia! So the PERIPHERAL blood sample won’t show a crazy number of WBCs.

713
Q

40 year old guy with PMH of DM comes in for lower abdominal and scrotal pain w/ nausea. He has high fever, low BP, tachycardia, and he looks sick. There is pain and crepitus over the lower abdomen and scrotum. He has leukocytosis, met acidosis, and high glucose. IV fluids and antibiotics are given. Diagnosis and next step?

A

Fournier gangrene (Necrotizing fasciitis of the scrotum)

Emergency surgery! (To do debridement of dead tissues. This quickly progresses to sepsis and death!)

  • metabolic acidosis is due to lactic acidosis from sepsis-> end-organ damage
  • his diabetes is a risk factor
714
Q

Patient with PMH of HTN (on Lisinopril) and low back pain (on over-the-counter pain meds) comes in for fatigue. U/A shows acidic urine, trace protein, moderate blood, and WBC casts. BUN and Cr are both elevated. Most likely diagnosis?

A
Tubulointersitial nephritis 
(*Remember interstitial nephritis is a type of intrinsic AKI) 
  • This isn’t the classic presentation: long-term drug use (NSAIDs, Penicillin, diuretic)-> fever, rash, eosinophilia
  • But, he is taking NSAIDs for back pain and the give-away is AKI (high BUN, Cr) w/ WBC casts

**glomerulonephritis (nephritic syndrome) is less likely—although there’s trace protein and blood, you would expect RBC (not WBC) casts in GN!

715
Q

22 year old has fever, sore throat, and malaise for 2 wks after returning from Honduras. Exam shows white exudates on the tonsils, enlarged lymph nodes, mild splenomegaly, and a grade 1/6 systolic murmur at the left infraclavicular region. Labs show normocytic anemia w/ high reticulocyte count, thrombocytopenia, leukocytosis, high bilirubin, ALT, and AST. Most likely diagnosis?

A

Infectious mononucleosis

-young adult, fever, tonsillitis, lymphadenopathy, splenomegaly, high WBCs (infection)

  • don’t be thrown off by the murmur—likely a flow murmur (innocent and unrelated)
  • don’t be thrown off by the recent travel—malaria, for example, wouldn’t cause tonsil exudates
  • don’t be thrown off by the elevated liver enzymes—remember that mono can cause inflammation of the liver too (Sketchy cow)
  • hemolytic anemia and thrombocytopenia are rare complications of mono due to EBV antibodies cross-reacting with RBCs and platelets, destroying them
716
Q

What is a serious side effect of Succinylcholine?

A

Cardiac arrhythmia (from high K+)

  • Succinylcholine is a depolarizing neuromuscular blocking agent
  • depolarization of neuron-> ACh release-> Succinylcholine blocks ACh esterase from breaking down ACh-> too much ACh in cleft paralyzes the muscle
  • with increased depolarization of neuron, you get more Na+ going into cells, more K+ going out-> hypERkalemia-> life-threatening cardiac arrhythmia

*patients with crush injury who are intubated with succinylcholine are at higher risk for this adverse effect! (They already have inc K+ in blood due to muscle cell lysis)

717
Q

What is a flow murmur?

A

An innocent murmur from blood moving faster across the valves

*for example, can be heard in anemia. Blood is less viscous/ thinner, so it moves faster across valves (higher velocity), so you hear the blood flow louder

718
Q

What is the biggest risk factor for both ischemic and hemorrhagic stroke?

A

HTN

*the extra pressure on the vessels speeds up plaque formation and increases bleeding risk

**HTN increases stroke risk more than any other risk factor, including high cholesterol, DM, smoking, and sedentary lifestyle.

719
Q
What are the 2 medication class options for treatment of BPH? Also state the mechanism of action and side effects. 
Which is the preferred initial treatment?
A
  1. Alpha-1 blockers (ex: Terazosin)
    - relax smooth muscle of the prostate
    - side effects: orthostatic hypotension (dec BP), reflex tachy
  2. 5-alpha reductase inhibitors (ex: Finasteride)
    - block conversion of testosterone-> DHT, which is responsible for prostate growth (so addresses the underlying cause, not just the symptoms)
    - side effects: ED, decreased libido
    * takes 6-12 mo to work

Alpha-1 blockers are 1st line. Use 5-alpha reductase inhibitors if alpha-blockers are not tolerated (hypotension) or refractory (need a 2nd med).

720
Q

40 year old man complains of erectile dysfunction. He also has fatigue which he attributes to his work. Skin exam shows brownish pigmentation. Labs show elevated liver enzymes (ALT, AST around 80) and fasting glucose of 130. Diagnosis?

A

Hemochromatosis
(Problem with HFE iron-sensing receptor on hepatocytes-> lack of hepcidin release/ inhibition on ferroportin channel, so you get excess reabsorption of iron from enterocytes)

  • excess iron deposits in organs like the liver (elevated liver enzymes) and skin (dark pigmentation)
  • notice that fasting glucose is high (>110)= “bronze diabetes”

*can progress to cirrhosis and increases risk for hepatocellular carcinoma (HCC)

721
Q

What is a normal serum calcium?

A

About 8.5-10

722
Q

Lady with PMH of celiac sprue (non-compliant with meds) has fatigue, weakness, and diffuse bone pain. Labs show high PTH, normal calcium, low phosphate, and high alk phos. Diagnosis?

A

Osteomalacia

-softening of the bones due to Vitamin D deficiency (2/2 celiac malabsorption of fat-soluble vitamins A, D, E, K in this patient)
(**you can also get osteomalacia from inadequate calcium/ phosphorus in the diet)

*normally Vit D increases calcium and P in the blood. If deficient-> dec calcium and/ or P-> impaired bone mineralization

723
Q

50 year old lady with PMH of HTN (non-compliant with meds) develops weakness while running. Over the next hour, she gets a severe headache with N/V. BP is 175/100. Neuro exam shows right hemiplegia, right hemisensory loss, and leftward deviation of the eyes. Diagnosis?

A

Putaminal hemorrhage
(on left side of brain)

  • The basal ganglia (putamen) is a common site of brain hemorrhage due to HTN (involves the same small vessels responsible for lucunar stroke). The internal capsule, which is right next to the putamen, is almost always involved-> controlateral hemiparesis (loss of movement), contralateral sensory loss, and conjugate gaze deviation toward the side of the lesion.
  • severe headache + worsening neuro deficits are more in line with hemorrhagic stroke (vs. ischemic stroke)
724
Q

45 year old man has exertional dyspnea. A harsh systolic murmur that radiates to the carotids is heard at the right 2nd intercostal space. An S4 is heard at the apex. What is the most likely underlying cause of the patients murmur and symptoms?

A

Bicuspid aortic valve

-younger guy with aortic stenosis (systolic murmur at aortic area)
(<70 so senile calcifications are not likely to be the cause)
*S4 is due to hypertrophy of the LV-> left atrial kick—the ventricle has to thicken to push blood through the stenotic aortic valve

725
Q

3 most common causes of aortic stenosis?

A
  1. Senile calcifications (>70)
  2. Bicuspid aortic valve (<70)
  3. Rheumatic heart dz
726
Q

Lady comes in after swallowing 20 Tylenol pills. She has no symptoms and CBC, coag studies (PT, INR), serum chemistries (including liver enzymes) are all normal. Next step?

A

Give activated charcoal and order acetaminophen levels

  • charcoal can be given (to absorb Tylenol in the setting of acute toxicity) if Tylenol was ingested <4 hrs ago
  • patients can be asymptomatic during the first 24 hrs, then develop severe liver injury
727
Q

How do you manage acetaminophen (Tylenol) intoxication?

A

First give activated charcoal (to absorb the Tylenol in the setting of acute toxicity) if ingestion was <4 hrs ago + order an acetaminophen level

Next, use the Rumack-Matthew nomogram on MDCalc to calculate the likelihood of hepatotoxic effects based on plasma acetaminophen level and hrs since ingestion…this is to guide you on whether or not to give N-acetylcysteine

If not a dangerous level, repeat acetaminophen level 2 hrs later. If dangerous acetaminophen level, give N-acetylcysteine now (restores hepatic glutathione stores)

**note that patients can be asymptomatic for 24 hrs, then suffer severe liver injury

728
Q

Patient has stage 1 Syphilis (painless genital ulcer, positive RPR screen). He is allergic to Penicillin. How are you going to treat him?

A

Oral doxycycline

*In early Syphilis, try alternate treatment with Doxy (or similar agents) before resorting to Penicillin desensitization

729
Q

Besides blindness, what is another serious complication of untreated giant cell (temporal) arteritis?

A

Aortic aneurysm

Remember, it is a vasculitis meaning there is inflammation affecting large branches off the aorta

730
Q

Normal albumin level?

A

3.5-5.5

731
Q

Normal BUN?

A

7-18

732
Q

55 year old heavy smoker with PMH of DM presents with nausea and vomiting. He’s also had fatigue, poor appetite, polyuria (excessive urination), polydipsia (excessive thirst), and constipation for several weeks. Labs show low PTH, high calcium, high BUN and Cr, normal Vit D levels. Most likely cause of his hypercalcemia?

A

Hypercalcemia of malignancy

Smoker-> squamous cell carcinoma of the lung-> PTHrP production (paraneoplastic syndrome)-> hypercalcemia (*actual PTH will be low by neg feedback)

  • *other causes of PTH-independent hypercalcemia:
  • other malignancy (squamous cell cancer of head/ neck, renal, bladder, breast, ovarian CA)
  • vit D toxicity or milk-alkali syndrome
  • granulomatous dz (sarcoidosis)
  • drug-induced (thiazides)
  • thyrotoxicosis
  • vit A toxicity
  • immobilization
733
Q

45 year old lady with PMH of GERD and Raynaud phenomenon presents with SOB and cough on exertion. ROS is positive for episodes of difficulty swallowing. Exam is significant for thick skin over her face, trunk, and arms. Crackles are heard over the lungs. Cause of her symptoms?

A

Interstitial lung dz (ILD) 2/2 systemic sclerosis

(*remember scleroderma is a disease of collagen deposition and fibrosis. ILD is a group of diseases characterized by fibrosis/ scarring of the lungs.)

734
Q

25 year old goes unconscious while standing in line at a supermarket. Her friend, a nurse, noted a weak pulse of 40 bpm during the episode. She woke up not confused and recalls a feeling of warmth over her body prior to passing out. Family hx is significant for heart attack in dad, seizure disorder in brother. Most likely cause of her syncope?

A

Vasovagal (neurocardiogenic)

  • often involves a prodrome before passing out (dizziness, nausea, diaphoresis, vision changes, abdominal pain, generalized feeling of warmth)
  • can be triggered by prolonged standing (or emotional distress/ pain)-> excessive vagal tone (drop in HR, BP)
735
Q

45 year old woman with PMH of breast cancer s/p mastectomy (on Tamoxifen) presents with SOB and R-sided chest pain. She also has progressive back pain. She has a fever and decreased R lung sounds. CXR shows an infiltrate at the level of the R heart border and R-sided pleural effusion (exudative by thoracentesis and Lights criteria). Diagnosis?

A

Pneumonia (parapneumonic effusion)

-fever, SOB, R middle lobe infiltrate, R pleural effusion

*pleural effusions occur in 40% of PNA patients, but usually resolve with antibiotics
(**if bacteria cross into pleural space, a complicated pleural effusion or empyema w/ pus can develop requiring drainage w/ a chest tube in addition to Abx)

736
Q

What is gynecomastia?

A

Swelling of breast tissue in males (“man boobs”)

737
Q

Man comes in for decreased libido and inability to maintain an erection for 4 months. He’s also had fatigue and a 10 lb weight loss. He drinks alcohol, doesn’t use drugs. Exam shows bilateral gynecomastia and firm, small testes. Labs show normal TSH, low T4 and T3. Diagnosis?

A

Liver cirrhosis

  • alcohol
  • decreased libido, small testes, and gynecomastia (due to increased estrogen, since liver isn’t working well to break it down)
  • decreased TOTAL T4, T3 (liver isn’t working to make thyroid binding hormones as it normally does, so total TH goes down but free TH and TSH are unchanged)
738
Q

Patient’s dad had colon cancer at age 50. When should you begin screening the patient w/ colonoscopy?

A

At age 40

Rule is start at age 50 and do colonoscopy every 10 yrs (or sigmoidoscopy every 5 yrs or fecal occult blood testing every year). Unless there’s a family history…then start colonoscopies 10 years prior to when family member got colon cancer.

739
Q

What’s a normal percent for the following (in the WBC count)?

  • neutrophils
  • eosinophils
  • lymphocytes
  • monocytes
A
  • neutrophils: around 55-60%
  • eosinophils: 1-3%
  • lymphocytes: 25-33%
  • monocytes: 3-7%
740
Q

Pregnant lady comes in for dry cough and SOB. She was recently treated for a UTI with nitrofurantoin. She has lung crackles and a ejection-murmur at the left upper sternal border. Labs show leukocytosis with eosinophilia. CXR shows bilateral mid-lower lung opacities. Diagnosis?

A

Drug-induced lung injury

This is nitrofurantoin-induced pulmonary injury. A side effect of the drug is hypersensitivity pneumonitis-> cough, SOB, crackles, bilateral opacities, eosinophilia.

*treat by stopping the drug—should improve in 1-2 days

741
Q

Fasting glucose should be between what?

A

70-110

742
Q

Alcoholic comes in for poor appetite and nausea. He has jaundice (icteric sclera) and tender hepatomegaly. There are no signs of cirrhosis, gallbladder is not palpable, and Murphy’s sign is negative. Diagnosis? Will the following be high, low, or normal?

  • AST
  • ALT
  • GGT
  • Ferritin
A

Alcoholic hepatitis
-AST, ALT, GGT, and Ferritin all HIGH

  • alcoholic with anorexia, jaundice, tender hepatomegaly
  • expect AST <500, ALT <300, AST:ALT ratio >2
  • GGT is in liver cells and ferritin is an acute phase protein (inflammation), so both would likely be high in hepatitis
743
Q

30 year old lady has worsening, bilateral lower extremity weakness and numbness + urinary incontinence today. Exam is significant for weak legs, decreased pinprick sensation below the umbilicus, absent vibratory sense in the toes, and increased lower extremity deep tendon reflexes. Most likely diagnosis?

A

Transverse myelitis
(Segmental inflammation of the spinal cord)

  • This is a rare condition that affects tracts across the horizontal aspect of the spinal cord at a given level. Cause is unknown but can occur after viral infections. (StepUp2Med pg 229)
  • Treat w/ high-dose IV steroids (often w/ plasmapheresis)

*Less likely to be cauda equina syndorme. Why? This presents with severe back pain and sensory loss of the saddle area (thighs, buttocks). This patient has sensory loss below the umbilicus (T10).

744
Q

Homeless man with hx of alcohol and IV drug abuse is brought to the ED due to severe muscle pain, stiffness, and restlessness. There are marks on his arms from IV drug use as well as abrasions and lacerations on the legs. He cannot open his mouth completely. He has painful spasms of neck muscles triggered by physical stimuli. Most likely cause of his symptoms?

A

Tetanus

  • can’t open mouth completely= trismus/ lockjaw
  • painful muscle spasms

(Clostridium tetani enters puncture wound-> toxin travels retrograde from PNS-> CNS and blocks release of the inhibitors neurotransmitters GABA-> over-contraction of muscles)

  • almost always occurs in those with poor access to health care (not vaccinated)
  • treat with debridement of infected wound, antibiotics, tetanus Ig, and tetanus toxoid
745
Q

20 year old woman has progressive weakness and loss of energy. Her BMI is 21. She has fine hair on her scalp, dental erosions, and enlarged cheeks. Labs show low Na+, low K+, high bicarb (met alkalosis), and normal urine chloride of 14. In addition to potassium supplementation, what is the best treatment fo correct the laboratory abnormalities in the patient?

A

Give normal saline
(To promote urinary excretion of bicarb)

*She has bulimia. Fine hair, dental erosions (from vomiting), and enlarged cheeks (parotid gland hypertrophy) are findings. Vomiting-> loss of HCl, K+ -> metabolic alkalosis and hypokalemia.

*Urinary chloride is helpful is distinguishing between saline-sensitive vs. saline-resistant met alkalosis (StepUp2Med pg 323).
Saline-sensitive-> low urinary Cl- (<20)
Saline-resistant-> high urinary Cl-

-Volume down-> activates RAAS-> aldosterone relaxes-> Na+ reabsorption and Na+ brings Cl- with it-> more Cl- retention, so decreased urinary Cl-
So, low urinary Cl- means the patient is salt sensitive/ volume responsive (giving fluids will help!)

746
Q

How is urinary chloride helpful in evaluating metabolic alkalosis?

A

Urinary chloride is helpful is distinguishing between saline-sensitive (giving fluids will help) vs. saline-resistant met alkalosis
(StepUp2Med pg 323).
Saline-sensitive-> low urinary Cl- (<20)
Saline-resistant-> high urinary Cl-

-Volume down-> activates RAAS-> aldosterone relaxes-> Na+ reabsorption and Na+ brings Cl- with it-> more Cl- retention, so decreased urinary Cl-
So, low urinary Cl- means the patient is salt sensitive/ volume responsive (giving fluids will help!)

747
Q

60 year old lady has episodes of sharp chest pain, worse with deep inspiration. She also has stiffness and pain in the hands and knees. Her PMH includes obesity, HTN, and HFrEF (ejection fraction 40%). She is taking carvedilol, furosemide, hydralazine, and isosorbide mononitrate. She stopped lisinopril due to angioedema. On exam, she has an inspiratory rub and moderate swelling of her hand and knee joints. Most likely cause of her symptoms?

A

Drug-induced lupus
(from hydralazine use)

*Hydralazine + nitrates can be used in place of an ACEi for treatment of HF in ACEi intolerant patients…
But unfortunately, hydralazine can cause drug-induced SLE.

*Treat by stopping the hydralazine and give NSAIDs for SLE-like symptoms.

748
Q

30 year old lady complains of weight gain over the last couple months. She’s also had irregular periods and anxiety. Exam shows proximal muscle weakness and dark terminal hair on the lower abdomen. Her fasting glucose is high (130). Next step?

A

Overnight low-dose dexamethasone suppression test
OR late-night salivary cortisol assay OR 24-hr urine free cortisol measure

  • This sounds like Cushing’s syndrome (excess cortisol-> central obesity, proximal muscle weakness, glucose intolerance, depression/ anxiety)
  • FIRST step is to figure out if it actually is Cushing’s syndrome (THEN you’d measure ACTH to find out if it’s ACTH-dependent or independent)
749
Q

Guy steps on a rusty nail, which punctures the sole of his foot. He got a 3-dose primary tetanus vaccination series when he was a kid, but no boosters since. What should you give him for tetanus prevention?

A

Single dose of Tdap vaccine

  • This is what we do:
  • KIDS: primary 3-dose vaccine series at 2, 4, and 6 months old (add’l doses recommended at 15-18 mo and 4-6 years)
  • ADULTS: single Tdap (tetanus-diphtheria-acellular pertussis) + booster for tetanus/ diphtheria (Td) every 10 yrs

*Patients with puncture wounds who haven’t gotten a booster within 5 yrs, get vaccinated. Plus, we give tetanus Ig if the patient never got the 3-dose tetanus vaccines or if their wound is large and dirty.

750
Q

Patient with history of SOB and coughing episodes, allergic rhinitis, childhood eczema, and DM has an NSTEMI. He’s treated with Aspirin, Clopidogrel, LMWH, Metoprolol, and Lisinopril. The next morning, he no longer has chest pain, but has new-onset SOB, dry cough, and wheezing. Most likely cause?

A

Adverse effect of medication

-Aspirin and beta-blockers can trigger bronchoconstriction in asthma patients!

751
Q

Man with PMH of liver cirrhosis and large-volume paracentesis of ascites several months ago presents with SOB for 2 wks. Exam shows flat neck veins, dullness and decreased breath sounds on the right, and distended abdomen with shifting dullness. Diagnosis?

A
Hepatic hydrothorax 
(Right-sided pleural effusion in a cirrhosis patient due to ascites fluid passage through a diaphragmatic defect) 
  • rule out all other cases of pleural effusion before making this diagnosis in a cirrhosis patient
  • treat with salt restriction and diuretics (to get off fluid)
  • therapeutic thoracentesis may be done if symptoms are bad enough, but avoid chest tube placement (can result in large-volume protein, fluid, and electrolyte losses)
752
Q

Goal A1c for diabetics?

A

Less than or equal to 7%

753
Q

You screen diabetics with an annual urine albumin/ Cr ratio test. What is a normal result?

A

Urine albumin/ Cr ratio <30 is normal

If >30, this means micro-albuminuria is present. Start on ACE inhibitor to prevent diabetic nephropathy.

754
Q

Man with PMH of DM is brought in due to slurred speech (dysarthria) and weakness for the past 12 hrs. Labs show low Na+, high BUN and Cr, and glucose of 900. Normal saline and insulin are given. Next step?

A

Give IV Potassium

  • This is Hyperosmolar Hyperglycemic State (HHS) (similar to DKA, but occurs in type 2 diabetics and the little insulin present prevents ketosis and metabolic acidosis)
  • These patients have a total body K+ deficit + giving insulin stimulates the Na/K pump and shifts K+ into cells-> hypokalemia, so have to replete K+.
  • In both HHS and DKA, monitor K+ closely (replete IV K+ when drops below 5.3)
755
Q

Why are elderly patients more prone to orthostatic hypotension?

A

Aging-> Impaired baroreceptors sensitivity (autonomic failure)

756
Q

What laboratory finding is the most sensitive indicator of hypovolemia?

A

Deceased urinary sodium

-hypovolemia-> activation of RAAS-> more Na+ is reabsorbed/ pushed into blood-> less sodium is peed out

757
Q

Lady complains of palpitations and anxiety. She also lost weight. She has a diffusely enlarged, non-tender thyroid gland. Labs show low TSH, high free T4. EKG shows sinus tachy. Radioactive iodine uptake is 5%. There are high levels of anti-TPO antibodies. Next step?

A

Propranolol (beta-blocker)

*High T4, low TSH seems like hyperthyroidism. But, anti-TPO antibodies is seen in Hashimoto’s thyroiditis= hypothyroidism! Remember, Hashimoto’s can start off with high TH due to lysis of thyroid cells that already produced TH.

758
Q

65 year old lady with PMH of HTN and T2DM presents with dark maroon-colored stools for 2 wks. Exam reveals a systolic ejection murmur in the right 2nd intercostal space and delayed carotid pulses. Labs show normocytic anemia and elevated BUN and Cr. Colonoscopy 6 mo ago was normal, but limited in the ascending colon due to suboptimal bowel preparation. Most likely diagnosis?

A

Angiodysplasia

  • most common vascular abnormality of the GI tract due to tortuous dilation of vessels (usually in the R colon)-> hematochezia
  • presents as episodes of painless GI bleeding in an old person >60
  • systolic murmur in R 2nd intercostal space= aortic stenosis (common in old people due to senile calcifications of the valve)
  • Angiodysplasia is more commonly seen in patients with aortic stenosis, von Willebrand dz, and renal dz
759
Q

40 year old lady complains of bilateral knee pain and joint stiffness. She has swelling of multiple joints. Labs show low Hb, low serum Fe, low TIBC, and high Ferritin. Next step?

A

Give Methotrexate
For treatment of RA (will also improve the anemia of chronic dz)

*low iron, low TIBC, high ferritin in anemia of chronic dz bc iron is trapped in storage sites (hepcidin, an acute-phase protein, blocks the ferroportin channel that reabsorbs/ pushes iron into the blood)

760
Q

Lady has pain, tingling, and numbness in her hands, worse at nights. Her wrists and fingers are stiff in the mornings. Exam shows mildly swollen wrists and finger joints. Tapping on the flexor surface of the wrist causes tingling in the first 3 digits of her hands. Next step?

A

Get a Rheumatoid factor assay

  • tapping on flexor surface of wrist-> pain is Tinnel sign
  • This is Carpal Tunnel syndrome (compression of medial nerve)
  • BUT, she also has joint stiffness in the morning, suspicious for RA and RA can cause swelling in the wrist-> compression of the median nerve

Initial evaluation of RA should include all the following: inflammatory markers (ESR, CRP), serologic studies (Rhuamatoid factor, cyclic citrullinated peptide antibodies), and x-rays of symptomatic joints

761
Q

Lady comes in for getting thirsty and peeing more often than usual. Labs show normal glucose, serum osm of 300, urine osm of 125. After 6 hrs of water deprivation, serum osm is 320 and urine osm is 130. After vasopressin is given, urine osm is 400. Diagnosis and treatment?

A

Central DI
Desmopressin (synthetic ADH)

Water deprivation-> would expect urine to concentrate more (holds back more water in response to the dehydration). It doesn’t really change here, so we’ve ruled out psychogenic polydipsia and we know it’s DI. 
Give vasopressin (ADH)-> urine osm goes up, meaning urine gets more concentrated as it should in response to dehydration. So we know the problem was lack of ADH= central DI. 

*DI (central and nephrogenic) cause hypernatremia (lack of ADH retaining water-> higher [Na+])

762
Q

Lady with PMH of HTN (on Valsartan), urinary incontinence (on Tolterodine), and recent UTI (on Bactrim) has a severe, right-sided headache around her eye for 2 hrs. She is seeing halos. Exam shows a non reactive, dilated pupil with excessive lacrimation. ESR is elevated. Diagnosis?

A

Acute angle-closure glaucoma

  • headache, eye pain, decreased visual acuity from narrowing of the anterior angle chamber-> decreased aqueous humor outflow
  • anticholinergic medication use (Tolterodine) is a risk factor
763
Q

Patient has low K+. You give IV potassium, but his K+ remains about the same. What might explain why his potassium is difficult to correct?

A

Hypomagnesemia

*remember low Mg-> low Ca, P, K

764
Q

Muscle cramps, high Na+, low K+. What do you want to check?

A

Plasma renin and aldosterone concentration

Hyperaldosteronism would cause high Na+, low K+ and this is the screening test you do

765
Q

50 year old man with PMH of HTN and hyperlipidemia comes in due to facial drooping. He had mild left ear discomfort and difficulty closing the left eye when driving to work. 4 hrs later, he couldn’t sip through a straw and his friend noticed he had left facial droop. BP is 140/85 and pulse is 80. The left face is droopy, but sensation and pain are intact and rest of neuro exam is normal. Diagnosis and treatment?

A

Bell palsy
Prednisone (steroid)

-a peripheral neuropathy involving CN 7 (the facial nerve)

  • stroke (UMN lesion)-> spares the forehead bc the upper face receives dual innervation (can’t smile on affected side, but can raise eyebrows *FA pg 520)
  • Bell palsy (LMN lesion)-> involves the forehead (can’t smile or raise eyebrows on affected side) like in this patient
766
Q

Patient with Barrett’s esophagus has dysphagia to solids, but his GERD has actually gotten better. Barium swallow shows symmetric, circumferential narrowing of the distal esophagus. What is it?

A

Esophageal (peptic) stricture
(Abnormal narrowing of esophageal lumen)

  • chronic GERD predisposes to Barrett’s esophagus and esophageal strictures (other causes of strictures include radiation, systemic sclerosis, and caustic ingestions)
  • esophageal stricture-> dysphagia to solids and the stricture can actually block acid reflex, improving heartburn symptoms
  • must do endoscopy w/ biopsy to r/o adenocarcinoma
  • if no malignancy, you do a dilation to treat
767
Q

Lady has zero neurological symptoms but is obsessed with the idea that she may have undiagnosed MS, ever since her sister was diagnosed with MS. Her fingers cramp up after typing on the computer all day, and she worries this is a sign of early MS. Diagnosis?

A

Illness anxiety disorder

(vs. somatic symptom disorder—where you actually have a symptom, but just think it’s something more serious than it is like tummy ache= must have stomach cancer)

768
Q

Side effects of Methimazole and PTU?

A

Methimazole- 1st trimester teratogen, cholestasis

PTU- hepatic failure, ANCA-associated vasculitis

BOTH- agranulocytosis (*if patient develops sore throat and fever, discontinue the drug and check WBC count)

**note: while both of these meds are treatments for hyperthyroidism, the preferred treatment is radioactive ablation

769
Q

Cervical myelopathy vs. Cervical radiculopathy vs. Cervical spondylosis?

A

Cervical myelopathy= compression of the cervical spinal cord

Cervical radiculopathy= compression of a cervical spinal nerve root/ “pinched nerve”

Cervical spondylosis= osteoarthritis of the cervical spine (marked by osteophyte formation)

770
Q

Patient has high-grade fever, headache, severe myalgias, sore throat, nonproductive cough, and mild nasal congestion for 4 days. His oropharynx is mildly erythematous w/ normal tonsils. How would you manage this patient?

A

Recommend supportive/ symptomatic treatment

This is influenza. Patients with the flu often develop systemic (fever, myalgia, headache) and respiratory (rhinorrhea, sore throat, nonproductive cough) symptoms.

*no tonsillar exudate, so unlikely to be strep pharyngitis

771
Q

Lady has heartburn, SOB on exertion w/ bilateral inspiratory crackles, and joint pain in her hands and feet. Diagnosis?

A

Systemic sclerosis

Esophageal dysmotility, fibrotic lung dz, arthralgias

772
Q

Lady with systemic sclerosis has heartburn. Endoscopy shows mild hyperemia (increased blood vessels) in the distal esophagus. Esophageal manometry shows lack of peristaltic waves in the lower 2/3rds of the esophagus + decrease in lower esophageal tone. What mechanism explains these manometry findings?

A

Smooth muscle atrophy and fibrosis in the lower esophagus

Remember that systemic sclerosis is a disease of fibrosis

773
Q

What does hyperemia mean?

A

Increased blood vessels

774
Q

35 year old lady with PMH of hypothyroidism (on Levothyroxine) and episode of acute back pain due to demineralization of vertebral bones presents with progressive muscle weakness for 6 months. It is difficult to climb stairs or comb her hair. She is hypertensive. Exam shows facial hirsutism and muscle weakness, but no pain on palpitation of the muscles. Diagnosis?

A

Cushing’s syndrome

  • painless muscle weakness
  • bone loss
  • HTN
  • hirsutism

*she needs further work-up to determine the cause: exogenous steroids, ACTH-producing pituitary adenoma (Cushing disease), ectopic ACTH production (small cell lung CA), or primary adrenal cancer

775
Q

What are 3 physical exam findings suggestive of severe aortic stenosis?

A
  1. Pulses parvus (slow-rising) and tardus (and weak carotid pulse)
    - less blood gets through the aortic valve, so carotid pulse is slow and weak
  2. Single and soft S2
    - the stenotic aortic valve takes longer to close, so it doesn’t close faster than the pulm valve, but closes at a the same time. Soft bc the calcifications on the aortic valve makes its closure softer.
  3. Mid to late systolic murmur (w/ max intensity at the 2nd right intercostal space radiating to the carotids)
    - aortic stenosis= problem opening the aortic valve during systole
776
Q

70 year old man passed out while shoveling snow. He had a similar episode last month passing out carrying heavy groceries. He denies chest pain, palpitations, cough, and lower extremity swelling. Likely diagnosis?

A

Exertional syncope due to aortic stenosis (outflow obstruction)

777
Q

Elderly man has constant back pain, thigh pain, headache. He has a deep, achy pain in the mid lumbar region that is worse with changes in position. He has mild thoracic dextroscoliosis (spine curved right) and femoral bowing. Diagnosis?

A

Paget disease of the bone

778
Q

Name two 1st gen cephalosporins (from Sketchy).

A
  1. Cephalexin (flex) aka Keflex

2. Cafazolin (Fez hat)

779
Q

Name three 2nd gen cephalosporins (from Sketchy).

A
  1. Cefuroxime (furious)
  2. Cefotetan (tea cup)
  3. Cefoxitin (fox)
780
Q

Name three 3rd gen cephalosporins (from Sketchy).

A
  1. Ceftazidime (Taz)
  2. Ceftriaxone (3 axes) aka Rocephin
  3. Cefotaximine (3 axes)
781
Q

Name a 4th gen cephalosporin (from Sketchy).

A

Cefepime (prime)

782
Q

Name a 5th gen cephalosporin (from Sketchy).

A

Ceftaroline (Tara)

783
Q

If you suspect bacterial meningitis and CT is indicated to r/o a mass (Focal neuro deficit, Altered mental status, Immunocompromised, Lesion, Seizure, papilledema), should you get the CT and LP before administering antibiotics?

A

NO

Give empiric antibiotics first

784
Q

What is the “FAILS” mnemonic to remember when you absolutely have to do a CT before an LP to rule out a brain mass?

(if brain mass and you do an LP and remove CSF, this could cause the brain to shift-> compression of the brainstem and death)

A
Focal neuro deficit
Altered mental status
Immunocompromised 
Lesion (previous CNS disease) 
Seizure 

*also papilledema (a sign of increased ICP)

785
Q

Patient with PMH of liver transplant presents with fever, headache, and confusion. His neck is stiff. WBC count is elevated with neutrophilic predominance. CT of the head is normal, LP pending. What empiric antibiotics should he be given (prior to CT)?

A

Cefepime + Vanco + Ampcillin

*This is an immunocompromised patient with suspected bacterial meningitis. We need to cover for strep pneumo, Neisseria meningitidis, Listeria, and gram neg rods. Cefepime is a 4th gen cephalosporin with broad-spectrum coverage, esp covers gram negatives. Vanco will get cover cephalosporin-resistant pneumococci. Ampcillin will cover Listeria.

786
Q

Old man presents with right-sided weakness, numbness, and slurred speech. Exam shows right hemiplegia (muscle weakness/ paralysis on that side), hemisensory loss, homonymous hemianopsia (visual field loss on the same side of both eyes) and aphasia (speech or understanding problem). CT shows MCA infarction, but he’s not giving TPA due to unclear onset. 2 days after admission, he becomes progressively obtunded (lethargic/ out of it). He’s got a fever and deviation of the eyes and head to the left. What happened and next step?

A

Transformation of ischemic stroke to hemorrhagic stroke

Repeat CT brain (non-con so blood will show up)

*transformation to hemorrhagic stroke is a complication of ischemic stroke, most often seen when the patient is treated with TPA or when the patients suffered a major large area stroke (this guy). With any change in mental status, get a repeat CT to evaluate bc urgent surgical decompression will likely be needed!

787
Q

What is an obtunded patient?

A

Means very lethargic/ “out of it”—a state where the patient is less interested in the environment, has slowed responses to stimulation, and tends to sleep more than usual w/ drowsiness during the day

788
Q

Lady had pain, stiffness, and swelling of her hands. X-rays showed periarticular osteopenia and bony erosions and labs showed mild anemia. Therapy with a disease-modifying agent was begun. What med was likely given?

A

Methotrexate
(for rheumatoid arthritis)

  • periarticular osteopenia (swelling and weakening of bones around the joint) and bony erosions (chronic inflammation of the joint-> formation of pannus-> erodes the cartilage and bone) are signs of RA
  • anemia of chronic dz
789
Q

A lady is treated with Methotrexate for RA. 2 months later, presents with painful oral ulcers. Labs are significant for elevated liver enzymes. Most likely cause of her symptoms?

A

Side effects of Methotrexate
(Medication-induced inhibition of DNA synthesis)

  • Remember, Methotrexate blocks dihydrofolate reductase in the thymidine (DNA building block) pathway-> impaired DNA synthesis, esp in rapidly dividing cells-> oral ulcers, bone marrow suppression. Also can cause hepatotoxicity (Sketchy: meat chef w/ liver spot on apron).
  • It also suppresses the immune system, which is why it’s 1st line for RA.
790
Q

45 year old man who immigrated from Southeast Asia has cough and SOB on exertion for 3 months. He can’t lay down flat to sleep due to SOB. ROS is positive for palpitations. He has crackles over his lungs. EKG shows irregularly irregular rhythm. CXR shows enlarged heart silhouette and vascular congestion. Most likely cause?

A

Rheumatic heart disease

  • can cause mitral regurg-> mitral stenosis (and aortic stenosis)
  • he likely has mitral stenosis (hard to get blood into the LV)-> dilated LA-> a-fib and backing up of blood to lungs-> pulm congestion

**pericarditis (effusion) can present with SOB, orthopnea, enlarged heart on CXR…but this guy didn’t have a recent infection suggestive of this. He is from Asia= red flag for Rheumatic heart dz.

791
Q

85 year old lady with mild dementia is brought in due to worsening confusion over 2 weeks. She no longer recognizes her caregiver but can follow instructions. She also has become weaker and had several falls. She is tired and has a mild headache. Muscle strength is 4/5 on the right, 3/5 on the left and she has positive Babinski. Diagnosis?

A

Subdural hematoma

-elderly patient with history of falls-> tearing of bridging veins due to trauma (*cerebral atrophy from aging can also stretch these veins)-> gradual onset (veins are low-pressure vessels) neuro symptoms (headache, tiredness, confusion, focal neuro deficits)

792
Q

Lady with PMH of HIV (most recent CD4 count= 30) and asthma has burning substernal chest pain and excruciating pain when swallowing. Exam shows dental carries, otherwise is normal. Most likely diagnosis?

A

Viral esophagitis (due to HSV or CMV)

*Candida esophagitis is most common in HIV patients, but viral esophagitis is more likely in her case due to: severe odynophagia (pain w/ swallowing) is main symptom, no dysphagia (difficulty swallowing), no thrush. Not eosinophilic esophagitis (despite her asthma)—this is uncommon and presents with difficulty swallowing + GERD symptoms. *Confirm diagnosis with upper GI endoscopy.

**There are 3 major causes of esophagitis in HIV patients:

  1. Candida—oral thrush, white plaques in esophagus, pseudohyphae on biopsy
  2. HSV—orolabial lesions (cold sores above lip), vesicular or ulcerative lesions, multinucleated giant cells on biopsy
  3. CMV—large linear ulcers in distal esophagus, intranuclear inclusions on biopsy
793
Q

65 year old man with PMH of DM comes to the ED for sudden loss of vision in his right eye. Visual acuity is decreased in his right eye. Ophthalmoscopy exam shows loss of fundus details, floating debris and dark red glow. Diagnosis?

A
Vitreous hemorrhage 
(Bleeding into the vitreous humor of the eye) 
  • most common cause is diabetic retinopathy (damage to blood vessels at the back of the eye)
  • presents as sudden loss of vision and onset of floaters
  • fundus is hard to visualize on exam
794
Q

After a blood transfusion, a patient immediately develops SOB and vomits. BP is 88/40, HR 140, RR 30, pulse ox 85%. Lung sounds are decreased bilaterally and pulses are weak. Most likely cause of the patient’s respiratory distress?

A

Anaphylactic reaction

  • occurs seconds to minutes after transfusion due to massive histamine release
  • more common in IgA deficient patients (their anti-IgA antibodies react w/ donor blood IgA)
  • presents with hypoxia w/ wheezing (but severe bronchospasm can prevent air entry-> decreased lung sounds), angioedema, hypotension
  • stop the transfusion and give epi and respiratory support!
795
Q

What antibodies would you expect to be positive in RA?

A

RF (rheumatoid factor) and anti-CCP

796
Q

What is the atlantoaxial joint?

A

The joint at the upper neck between the first two cervical vertebrae

*Rheumatoid arthritis can affect this joint, leading to spinal cord compression

797
Q

What joints are most often affected by RA?

A

Small peripheral joints—MCP (metacarpophalangeal) joints of the hands, PIP (proximal interphalangeal) joints of the hands, the wrists, MTP (metatarsophalangeal) joints of the toes—and the cervical spine (Atlantoaxial joint)

798
Q

60 year old man has fever, malaise, and lower back pain for 10 days. NSAIDs provide no relief. He recently had a UTI treated with Ciprofloxacin. On exam, he has intense tenderness over the L4-L5 vertebrae and local paravertebral muscle spasm. Labs show leukocytosis and elevated ESR. X-ray of lumbar spine shows mild degenerative changes. Likely diagnosis and next step?

A

Vertebral osteomyelitis
Get an MRI

  • Back pain + FOCAL spinal tenderness, plus or minus fever (*often misdiagnosed as degenerative spine dz)
  • most likely due to hematogenous spread of his recent UTI (Staph aureus is the most common cause of vertebral osteomyelitis, but gram negative bugs can cause it too)

*MRI is the best test (X-ray can be falsely negative for the first 2-3 wks of infection)

799
Q

18 year old female presents with severe headache that began suddenly. She vomited once. Neuro exam is unremarkable. CT head is normal. LP CSF analysis shows elevated opening pressure, elevated RBCs, and mildly elevated protein. There is no difference in cell counts between the first and last collection tubes. Most likely diagnosis?

A

Subarachnoid hemorrhage (SAH)

  • “worst headache of my life”/ thunderclap (max intensity reached in a couple minutes) due to rupture of berry aneurysm (may occur spontaneously or triggered by physical exertion/ rise in BP)
  • If CT is negative and suspicion is high, you do an LP expecting to see xanthochromia (yellow CSF from bilirubin)—elevated opening pressure, increased RBCs, and slightly elevated protein are consistent with SAH
800
Q

Lynch syndrome (hereditary nonpolyposis colorectal cancer) puts one at risk for what 3 cancer types?

A
  1. Colorectal cancer
  2. Endometrial cancer
  3. Ovarian cancer
  • do genetic testing for patients with a strong family hx of colon cancer (>3 relatives, multiple generations)
  • once diagnosis is established, the patient needs to undergo regular colonoscopies and annual endometrial biopsies to screen. After age 40 (or earlier if childbearing is complete), a prophylactic hysterectomy and bilateral oophorectomy is recommended
801
Q

What is delirium?

A

An acute confusion also state (fluctuating level of consciousness), often associated with infection

802
Q

70 year old man with aortic stenosis is putting on a shirt and tie and passes out. Most likely cause of his syncope?

A

Baroreceptor hypersensitivity
(In elderly the baroreceptors can become overly sensitive)

*when carotid sinus baroreceptors detect increased pressure (carotid massage or tie around neck)-> increases firing of afferent baroreceptors-> increases PNS response-> vasodilation (to lower BP) and decreased HR-> less blood flow to brain-> syncope

(In response to high pressure, baroreceptors trigger a PNS response of vasodilation/ lower BP + slowed HR)

803
Q

Lady complains of diarrhea, weight loss, and fatigue over the last year. She gets watery diarrhea 2-3x/ day, it is foul-smelling and floats. She also has diffuse bone pain. Diagnosis?

A

Vitamin D deficiency 2/2 malabsorption

  • steatorrhea (fatty stool) and bone pain
  • malabsorption problem-> lack of vitamin D (and A, K, E, the fat-soluble vitamins)-> low calcium (also low phosphate, high PTH)
804
Q

75 year old man has headaches, fatigue, ringing in his ears, blurry vision, and feels like someone is “sticking his feet with electrical pins.” Funduscopy shows dilated, tortuous retinal veins. Neuro exam shows sensory deficits in his feet. Hb is low. Serum protein electrophoresis shows a sharp IgM spike. Diagnosis?

A

Waldenstrom macroglobulinemia

  • plasma cell malignancy that results in excessive IgM production-> hyperviscosity (explains his vision problems, headache, and ringing of ears)
  • neuropathy (end-organ damage) and general infiltrative symptoms (hepatosplenomegaly, anemia, thrombocytopenia) can also occur
805
Q

Man has fever, malaise, nonproductive cough, abdominal pain, diarrhea, and SOB. He’s got a fever and BP is low. He appears confused. There are crackles over the lungs and infiltrates, revealed by CXR. Sputum sample analysis shows many neutrophils but no organisms. Sodium is 125, AST 100, ALT 112, alk phos 95. Diagnosis?

A

Legionella

  • penumonia + diarrhea + hypOnatremia
  • test with a urinary Legionella antigen
806
Q

30 year old lady has cough and blood-tinged sputum. She developed malaise, nasal congestion, sore throat, and dry cough 10 days ago. She has scattered bilateral wheezes and crackles heard on auscultation. CXR shows clear lung fields. Diagnosis and next step?

A
Acute bronchitis 
Supportive care (symptomatic treatment only) 
  • 90% of cases are viral—avoid antibiotics because they don’t provide much benefit (even in bronchitis due to Mycoplasma bacteria)
  • cough >5 days is characteristic of acute bronchitis
  • yellow sputum may occur due to epithelial sloughing and blood in sputum can occur due to inflammation and epithelial damage
807
Q

75 year old lady with PMH of T2DM and HTN presents with sudden-onset complete vision loss in her right eye. Her vision was blurry over the last 24 hrs, but acutely worsened in the last hour. ROS is positive for generalized headaches for the last 2 months and she has malaise and fatigue. Exam is significant for swollen place optic disc with blurred margins and tenderness on palpation over the lateral scalp. Diagnosis and treatment?

A
Giant cell (temporal) arteritis 
High-dose systemic glucocorticoids (steroids) 
  • large vessel vasculitis in elderly that can present with flu-like symptoms (fever, fatigue, malaise, weight loss), headaches, jaw claudication, and vision changes/ loss (amaurosis fugax temporary loss-> permanent loss)
  • tenderness on palpation of lateral scalp was a key (temporal area) + she is old with malaise and headaches, now visual changes/ loss
808
Q

Elderly man with PMH of HTN, T2DM, and CAD presents with sudden onset weakness in his right arm and leg. Over the past month, he had episodes of weakness on his right side that resolved spontaneously. Exam shows right-sided hemiplegia (paralysis) and right lower facial paresis (weakness). He has no problems with speech or sensation and non-con CT is normal. Diagnosis?

A

Lacunar stroke

Chronic HTN-> Lenticulostriate arteries off the MCA stroked out-> posterior limb of the internal capsule infarcted-> “pure motor stroke”

**can cause controlateral motor and sensory loss (but can be just a “pure motor stroke” or “pure sensory stroke” if only part is affected)

809
Q

55 year old lady has pain and stiffness in multiple joints for 1 week, worse in the morning. There is mild swelling of the joints with no redness or tenderness. ESR is within normal range. Most likely diagnosis?

A

Parvovirus B19 (virus arthritis)

810
Q

Teenage girl has a nosebleed that won’t stop. She has a history of heavy, painful messes. Her arms and legs are full of scattered ecchymoses. Platelet count is 9,000, fibrinogen is normal, PT is normal. Diagnosis?

A

Immune thrombocytopenia

*normal platelet count= 140,000-500,000

811
Q

What population of patients have higher risk for delirium?

A

Elderly patients and those with underlying dementia or past stroke (likely due to reduced cognitive reserves)

*you can help their recovery by treating underlying cause and infection, avoiding unnecessary medication, encouraging regular activity during the day, and minimizing disturbance during the night

812
Q

Young man with PMH of multiple abdominal surgeries (after being shot in the abdomen) has chronic diarrhea that wakes him at night. He lost a few lbs. Occult blood test is negative, stool osmotic gap is low. What type of diarrhea is this?

A

Secretory diarrhea

  • secretory diarrhea (toxins, hormones, etc.)-> low stool osmotic gap
  • You get large volume diarrhea that continues during the night!
  • As in this patient, can occur after bowel resection of cholecystectomy)

*osmotic diarrhea (malabsorption problem)-> high stool osmotic gap

813
Q

40 year old woman complains of generalized pruritic (itching) and fatigue for 4 mo. She has xanthelasmas (yellow plaques) on her eyelids, skin excoriations, and mild hepatomegaly. Total bili is elevated and anti-mitochondrial antibody is positive. Diagnosis?

A

Primary biliary cholangitis

-autoimmune destruction of intrahepatic bile ducts

814
Q

Lady with PMH of renal transplant (taking meds, but went off Bactrim and Valganciclovir due to leukopenia) presents with 3 days of fever, malaise, progressive SOB on exertion, dry cough, abdominal pain, and watery diarrhea that had bright red blood this morning. Exam shows bilateral lung crackles and diffuse abdominal tenderness. Labs show elevated alk phos, ALT, and AST. CXR shows bilateral interstitial infiltrates. Diagnosis?

A

Cytomegalovirus (CMV)

  • since she is immunosuppressed due to meds for organ transplant, she is on prophylaxis for PCP (Bactrim) and CMV (Valganciclovir). She had to go off these meds, so she has increased risk for both at this time.
  • pulm symptoms (SOB, dry cough), GI symptoms (abdominal pain, diarrhea, hematochezia)
815
Q

When can we NOT give a guy Viagra (Sildenafil)?

…if he’s taking what 2 types of medications already

A
  1. On a nitrate
  2. On an alpha-blocker

*Sildenafil is a PDE-5 inhibitor. Taking with nitrates (venodilate) or alpha-blockers (block constriction, so dilates) can cause severe hypotension!

816
Q

What are the 2 most accurate markers that indicate resolution of DKA?

A
  1. Serum anion gap (the gap normalizes)

2. Serum beta-hydroxybutyrate levels (disappear)

817
Q

55 year old man with PMH of HTN presents with severe, tearing abdominal pain. BP is 240/130. CT angio confirms descending aortic dissection originating at the left subclavian artery and extending to the iliac arteries. He is given IV Labetalol and Nitroprusside and admitted to the ICU. The patient improves and BP stabilizes, but now he complains of not being able to move his lower extremities. He has weakness, decreased reflexes, loss of pain, but intact vibration in both lower extremities. Most likely cause?

A

Thoracic spinal cord ischemia

  • this is a rare (2-3%) complication of aortic dissection due to interruption of blood flow (to intercostal and/or lumbar arteries) that feeds the anterior spinal cord (doesn’t affect dorsal/ posterior cord—this has a different blood supply)
  • the aorta gives off branches that supply the spinal cord
  • thoracic region makes sense-> affects lower extremities

-neuro findings (bilateral weakness in legs, loss of motor, loss of pain, intact vibration tell you there’s an anterior cord problem…everything but the DC-ML tract in the dorsal/ posterior column is affected)

818
Q

Elderly man with PMH of smoking, HTN, and MI 6 months ago comes in for palpitations, mild dizziness, and sweating. On exam, high-amplitude jugular venous pulsations are seen intermittently at irregular intervals. EKG shows regular, wide-complex tachycardia. What best explains the physical exam findings?

A

Ventricular tachycardia

  • wide-complex tachy= ventricular problem (like v tach or torsades)
  • high-amplitude JVP (Cannon A waves) are due to RA contraction against a closed tricuspid valve (there’s no valve between the superior vena cava and the RA, so JVP tells you about the right heart. There’s high pressure there bc the atria and ventricles are both contracting but not coordinated)
  • you get atrioventricular dissociation= atria and ventricles not coordinating (happens in V tach and complete AV block)
819
Q

Patient presents with colicky RUQ pain, vomiting, and melena for 1 day. History is notable for non-alcoholic fatty liver dz, diagnosed 5 days ago with liver biopsy. The biopsy also showed mild fibrosis around the portal tracts. Labs show low Hb, high platelets, high WBCs, high total bili. Diagnosis?

A

Hemobilia
(bloody bile/ bleeding into the biliary tract)

  • complication of liver biopsy (sticking a needle into the liver)
  • RUQ pain, upper GI bleed (hematemesis, melena), jaundice and anemia from bile duct obstruction
820
Q

25 year old lady has pressure-like substernal chest pain while exercising (noticed it 6 mo ago, when she changed from being a cough potato to a workout machine). She has a BP of 130/70 on the right and 105/55 on the left. She has a palpable thrill on the suprasternal notch and loud midsystolic murmur at the 1st right intercostal space. Most likely diagnosis?

A

Supravalvular aortic stenosis

  • this is the 2nd most common type of AS (congenital LV outflow obstruction due to narrowing of the ascending aorta—the aortic valve is fine, it’s the aorta above the valve that’s stenotic)
  • chest pain during exercise due to increased myocardial oxygen demand
821
Q

Osteophytes are seen in what condition?

A

Osteoarthritis

822
Q

60 year old man has right-sided neck and shoulder pain worse with movement and associated with numbness in his forearm. Exam shows limited neck rotation and lateral bending. There is decreased pinprick sensation on the posterior right forearm. Diagnosis?

A

Osteoarthritis of the neck (cervical spondylosis)-> osteophytes (bony outgrowths) that grow around/ compress the nerve roots (cervical radiculopathy)

823
Q

75 year old man with heavy smoking history has sudden-onset back pain at L1-L4 that is constant, deep, and dull. He had trouble falling asleep bc it hurt so bad. BP is 140/90. X-ray shows no vertebral abnormalities, but prevertebral calcifications are present. Likely diagnosis and next step?

A

AAA
CT abdomen

  • old smoker (has risk factors), hypertensive
  • back pain from expanding abdominal aortic aneurysm (not ruptured bc his BP is stable)
  • pre-vertebral calcifications means the structure in front of the vertebrae (the aorta) is calcified (the atherosclerosis is so extensive it shows up on x-ray)

*CT is the best test for diagnosis of abdominal aortic aneurysm

824
Q

Smoker man has intermittent epigastric pain radiating to the back, worse with eating and associated with nausea. Vitals are normal. All labs, including lipase, are normal. CT abdomen shows pancreatic atrophy with multiple calcifications in the pancreatic parenchyma. How do you manage?

A

Advise lifestyle modification and give lipase supplementation

  • this is chronic pancreatitis
  • In acute pancreatitis, you get inflammation causing premature activation of pancreatic enzyme and auto-digestion. Lipase is elevated from pancreatic cells.
  • In chronic pancreatitis the pancreas is scarred/ non-functional tissue, so not a lot of lipase to be released (usually not elevated). The pancreas isn’t working well to make pancreatic enzymes so you need to give supplementation. Plus, giving lipase will block CCK to stop pancreatic stimulation of enzymes (tells the pancreas it doesn’t have to work anymore) to relieve pain from pancreatic hyperstimuation.
825
Q

What’s the criteria for when to give long-term supplemental oxygen to a COPD patient?

A
  1. PaO2 <55 or SaO2 <88%
  2. If right HF or hematocrit >55%:
    PaO2 <59 or SaO2 <89%
826
Q

What medical innervation has a mortality benefit in patients with COPD?

A

Long-term supplemental oxygen at home (when indicated)

827
Q

Guy gets a blood transfusion (several units of packed RBCs) then develops tingling in his fingers and toes. His calcium is low. Most likely cause?

A

Calcium chelation by citrate in the transfused blood

*patients who get more than 1 blood volume of blood transfusions or packed RBCs over 24 hrs may develop elevated plasma levels of citrate (a substance added to stored blood)-> binds up calcium and magnesium-> paresthesias

828
Q

35 year old guy has pain and redness of the left eye. ROS is positive for episodic abdominal discomfort and diarrhea for several months. On exam, he has photophobia and decreased visual acuity. He also has an aphthous ulcer on the buccal mucosa and tenderness on palpation of the RLQ. What’s wrong with his eye?

A

Anterior uveitis

  • painful, red eye associated w/ photophobia, tearing, and diminished visual acuity
  • exam findings: hypopyon (layering of WBCs in the anterior chamber) + ciliary flush (hyperemia/ excess blood vessels at the junction of the sclera and cornea)
  • he has Crohn’s (uveitis can be associated with IBD)
829
Q

Anterior uveitis is most often idiopathic or traumatic. But, it can be associated with what type of conditions?

A

Inflammatory diseases (certain infections, sarcoidosis, spondyloarthritis, IBD)

830
Q

60 year old man has constant back pain, thigh pain, and headache. He uses a hearing aid for recently acquired left-sided hearing loss. He has mild thoracic dextroscoliosis, decreased lumbar lordosis, and anterolateral femoral bowing. Diagnosis?

A

Paget disease of the bone

  • osteoClast dysfunction:
    1) osteoC’s go crazy and break down bone w/o the regulation of osteoB’s
    2) bones weaken and osteoB’s realize they need to start laying down more bone so they rush to compensate
    3) osteoC’s burn out and osteoB’s are still rushing to lay down as much bone as possible, but do a bad job of it

-bone pain, headaches (enlarging cranial bones-> inc hat size), unilateral hearing loss (entrapment of CN8), and femoral bowing are typical symptoms

831
Q

42 year old HIV positive man who is sexually active with other men comes in for a check-up. He received all childhood vaccinations. Last CD4 count was 670 and anti-HBS assay is positive. You order a influenza and pneumococcus vaccine for him. What hepatitis vaccine should you also give him?

A

Hepatitis A vaccine
(*note: this became part of the childhood vaccine series in 2006, as a 42 y.o. it is unlikely he got this vaccine)

  • recommended in travelers to countries where hep A is prevalent (contaminated water, oysters), for men who have sex with men, and for adults with chronic liver dz
  • positive anti-HBS means he’s already immune to hep B (vaccinated), so a vaccine for hep B is not needed
832
Q

Most common bug that causes pneumonia in AIDS patients?

A

Strep pneumo!

Don’t be fooled…the most common is always strep penumo. Yes, AIDS patients are at risk for additional bugs like PCP, but strep pneumo is still more common.

833
Q

Live vaccines (MMR, zoster, varicella, etc.) are contraindicated in HIV patients with a CD4+ count less than what?

A

<200

834
Q

Type 2 diabetic patient is on metformin and glimepiride. Hb A1c is 8.5%. His doctor suggests switching to insulin therapy for tighter glycemic control. This switch would decrease the patient’s risk for what diabetic complications?

A

Microvascular complications (nephropathy, retinopathy)

studies have shown tight glucose control in diabetics reduces microvascular complications, but unfortunately doesn’t reduce macrovascular complications or mortality (in fact, getting the glucose too low-> hypoglycemia can increase mortality so we aim for an A1c goal of 7% in diabetics)

835
Q

What’s our goal A1c in diabetics?

A

7%

*too tight of regulation, like A1c of 6-6.5% can actually increase mortality risk due to hypoglycemia

836
Q

70 year old man has progressive urinary urgency, hesitancy, and weak urinary stream. Digital rectal exam reveals a smooth, firm, enlarged prostate. Serum creatinine is 2.1 (baseline is 1.2), PSA is normal. Next step in evaluating his AKI?

A

Renal U/S (looking for hydronephrosis)

  • severe BPH compressing the urethra-> urine backs up to kidneys-> bilateral hydronephrosis-> AKI
  • serum Cr test is not required in routine evaluation of uncomplicated BPH, but it is recommended for some patients with more significant symptoms or add’l risk factors (HTN, DM, CKD)
837
Q

Man has two purplish pruritic (itchy) flat-topped papules/ plaques on his penis. Diagnosis?

A

Lichen planus

*although this skin condition is usually seen on the extremities and trunk, lesions may also appear on the genitals or oral mucosa

838
Q

Anti-U1 ribonucleoprotein is an antibody associated with what condition?

A

Mixed connective tissue disease

Autoimmune disorder with variable features of: SLE, systemic sclerosis, and polymyositis

839
Q

Mixed connective tissue disease (remember Anti-U1 ribonucleoprotein) presents with features of what 3 disorders?

A
  1. SLE
  2. Systemic sclerosis
  3. Polymyositis
840
Q

Man who recently underwent an elective inguinal hernia repair surgery now has knee pain. There’s redness and swelling of his right knee and arthrocentesis shows elevated WBCs (neutrophils) and rhomboid-shaped crystals in the synovial fluid. Diagnosis?

A

Pseudogout
(Acute calcium pyrophosphage crystal arthritis)

  • associated with meniscal calcification (calcifications in the cartilage are released into the joint space)
  • often occurs in the setting of surgery or medical illness
841
Q

What is the precursor to squamous cell carcinoma of the skin?

A

Actinic keratosis

842
Q

70 year old man with PMH of HTN and smoking hx comes in for a physical. You notice a right carotid bruit. Follow-up duplex ultrasonography shows 60% stenosis of the right common carotid, 40% stenosis of the left common carotid. Next step?

A

Start him on antiplatelet (aspirin) and statin therapy

*carotid endarterectomy (CEA) is recommended for SYMPTOMATIC patients with high-grade carotid stenosis (>70% stenosis) (may consider in asymptomatic patients with >80% stenosis)
(**also note CEA is preferred over carotid artery stenting)

843
Q

When is carotid artery endarterectomy (CAE) recommended?

A

In symptomatic patients with carotid artery stenosis >70%

(May also do it in asymptomatic patients with stenosis >80%)

*if stenosis is less than this, medical management with aspirin + statin is appropriate

844
Q

What type of diuretics can prevent calcium kidney stones?

A

Thiazides

They pull calcium out of the pee, into the blood

845
Q

Sensation to your cornea is supplied by what nerve?

A

V1 branch of CN5 (abducens nerve)

846
Q

What is normal pressure hydrocephalus?

A

Subtype of communicating hydrocephalus, meaning decreased CSF absorption (vs. noncommunicating, which is due to structural blockage of CSF)

-dec absorption of CSF into arachnoid granulations (outpouchings) episodically-> inc CSF-> dilated ventricles-> stretching of corona radiata (nerve fibers running along edge of ventricles)-> the disruption of motor neurons causes a triad of urinary incontinence, gait instability, and cognitive dysfunction (“wet, wobbly, and wacky”_

847
Q

What bone problems are rheumatoid arthritis (RA) patients at increased risk for developing?

A

Osteopenia (weakened bones), osteoporosis, and bone fractures

*inflammatory cytokines promote bone loss *osteoporosis risk is twice as high, so encourage exercise, vitamin D, calcium, avoid glucocorticoids if possible, and bisphosphonate therapy (**Vit D bc it promotes absorption of calcium + P, which are needed for bone mineralization)

848
Q

What FINDINGS set hyperosmolar hyperglycemic state (HHS) apart from DKA?

A
  • higher glucose levels (often >1,000 vs 300-500 in DKA)
  • normal anion gap and pH
  • absent ketosis
  • some baseline insulin (type 2 diabetes) prevents ketoacid formation and anion gap
849
Q

Man is lethargic and confused. He has a fever of 100.4, BP 90/60, HR 112, RR 24. Mucous membranes are dry. Labs show high K+ (5.9), high BUN (110), high glucose (1,000). Best immediate treatment?

A

Normal saline

  • This guy has hyperosmolar hyperglycemic state (HHS)
  • Although you will give insulin (and monitor K+), aggressive IV fluids is the best immediate step! (Replenishes extracellular volume, lowers plasma osmolality, and increases tissue perfusion and responsiveness to insulin). Use normal saline initially regardless of Na+ levels.

**remember that the high glucose spillage into urine (water follows)-> severe dehydration. High BUN is due to severe dehydration (prerenal AKI). High K+ bc no response to insulin means K+ isn’t being pushed into cells, so hangs out in the blood.

850
Q

What type of fungi is Trichophyton rubrum?

A

Onychomycosis (a tinea fungi/ dermatophyte of the toenails)

851
Q

Equation for anion gap?

A

Sodium minus chloride minus bicarb

Na- Cl- HCO3

852
Q

Avid runner presents with worsening right foot pain for 5 weeks. She recalls no trauma. Her BMI is 15, LMP 8 wks ago. She has tenderness to palpation of the 2nd metatarsal bone on the dorsal (upward side) of the foot, but no erythema, bruising, or problems with range of motion. Diagnosis?

A

Stress fracture

  • often occurs in athletes (or non-athletes who suddenly increase their activity)
  • presents as pain in the forefoot over the metatarsal bones

*more common in female runners with the “female athlete triad”: (1) amenorrhea (no periods)/ oligomenorrhea (infrequent periods), (2) decreased calorie intake, (3) osteoporosis
(This lady has a BMI of 15 and period 8 wks ago, so this is likely)

853
Q

45 year old man presents with excessive wasting of his extremity muscles, most prominent in the legs. He’s also had difficulty with swallowing, chewing, speaking, and has movements in his face and tongue. Fasciculations and hyperreflexia are noted. Diagnosis?

A

ALS (amyotrophic lateral sclerosis)

-neurodegenerative disorder-> both UMN and LMN signs

(UMN signs- spasticity and exaggerated reflexes, LMN signs- weakness, atrophy, and fasciculations (movements of face and tongue in him))

854
Q

Football player with no PMH collapses at summer practice right after complaining of dizziness and headache. Temp is 105.8 with hot, dry skin. BP is 90/60, HR 140. IV hydration is started. Next step?

A

Immersion in ice water

-This is exertional heat stroke

  • Problem is the body is too hot to cool itself by sweating alone (cooling mechanism is overwhelmed). Blood gets shunted to the periphery so that the body can sweat (heat dissipation by evaporation), resulting in a distributive shock (organs now deprived of blood).
  • Body temp >104 and CNS dysfunction (confusion, irritability, seizures)…may also get hypotension, tachycardia, hyperventilation, diarrhea, cramps, ataxia
  • Mortality is 20%! Cool the patient rapidly!
855
Q

What findings distinguish heat exhaustion from heat stroke?

A

Both will involve body temp >104

Heat stroke additionally involves CNS dysfunction (confusion, irritability, seizures)

856
Q

What’s the typical patient population and the treatment for exertional heat stroke vs. nonexertional/ classic heat stroke?

A

Exertional—> athletes (due to overexertion in the heat), rapid immersion in ice water

Classic—> old people with comorbidities (due to limited ability to cope with excessive heat), evaporative cooling (fans)

857
Q

Another name for subacute thyroiditis?

A

De Quervain

(“De Quervain causes pain”- you get pain in the thyroid, thought to be due to a post-viral inflammatory process after a URI. There is a self-limited thyrotoxic phase followed by hyperthyroidism and then recovery.)

858
Q

Diabetic patient presents with periorbital edema and bilateral pitting edema around the ankles. His last A1c was 6.9% and his BP is 145/87. Labs are significant for elevated BUN, Cr, and total cholesterol. Urine protein is also elevated. What is most likely to slow the progression of his kidney disease?

A

Intensive BP control (with ACE inhibitor or ARB)

**NOT tighter glycemic control bc he’s already around our goal for diabetics of 7% (this reduces microvascular complications like nephropathy and retinopathy). If we get A1c even lower, we’d be putting him at risk for hypoglycemia and earlier mortality.

859
Q

30 year old woman presents with tingling and numbness in both hands. Since moving to Arizona, she’s had occasional headaches, dizziness, fatigue, poor sleep, and blurry vision. Exam is notable for reduced sensation to light touch and pain in the bilateral upper extremities. Her right optic disc appears hyperemic (engorged/ increased blood flow) and swollen. Next step?

A

Get an MRI of the brain/ spine

  • This is likely MS (multiple sclerosis)
  • autoimmune inflammatory demyelinating disorder of the CNS (esp the white mater where there’s axons w/ myelin sheaths) that presents with neuro deficits separated by time and space (sensory loss/ paresthesias, dizziness, optic neuritis, uhthoff phenomenon where symptoms worsen during heat exposure *like moving to Arizona!)
  • MRI (T2 weighted) will show white mater lesions
860
Q

What is the “uhthoff phenomenon” in MS?

A

MS symptoms worsen during exposure to hot temperatures/ heat

861
Q

Guy drinks a 6-pack of beer every night. His lipid panel reveals total cholesterol of 300 and triglycerides of 470. In addition to high-intensity statin therapy, what do you need to recommend to improve this patient’s lipid disorder?

A

Reduce alcohol intake!

*Statins are 1st line for mild hypertriglyceridemia (150-500, where normal TAGS= 35-160) bc they reduce cardiovascular events. In addition, do lifestyle modification (heavy alcohol consumption increases TAGs).

**Although fibrates are the best anti-lipid meds for lowering TAGs, we rarely use them bc they don’t have proof of providing cardiovascular benefit. We only use them in severe hypertriglyceridemia (500-1000).

862
Q

What’s the “rule of thumb” for how many drinks of beer are acceptable for men and women per day?

A

Two drinks/ day for men

One drink/ day for women

863
Q

Man has joint pain and swelling, poorly controlled HTN, coarse facial features that differ from his driver’s license photo, and oily skin with multiple skin tags. Next step to work-up his condition?

A

Order a IGF-1 (insulin-like growth factor 1) level
(*ordering Growth Hormone directly isn’t as accurate bc these levels fluctuate often)

  • This is acromegaly (too much GH)
  • work-up: IGF-1 level-> oral glucose suppression-> MRI brain looking for pituitary mass (somatotroph adenoma)

GH increases insulin, so acromegaly is associated with insulin resistance, which may also induce HTN

864
Q

In what geographic places do we see Yellow fever?

A

Africa (sub-Saharan) and South American countries

865
Q

Guy is traveling to Egypt. What should you vaccinate him against?

A

Hep A

Give to those traveling to developing countries. It is the most preventable disease among travelers.

866
Q

40 year old man has fatigue, weakness, low-grade fever, fleeting joint pains, dark/ cloudy urine, pain in the fingertips, and SOB on exertion. Labs show normocytic anemia, leukocytosis, elevated ESR, and rheumatoid factor. U/A is positive for 2+ blood, 1+ protein. Diagnosis?

A

Infective endocarditis

  • clinical features:
  • fever (bacteremia)
  • other non-specific/ systemic symptoms (including arthralgia)
  • murmur (vegetations disrupt blood flow across valve) and cardiac effects (including SOB)
  • septic emboli of bacteria pieces (vegetations get shot off into circulation)—> Janeway lesions (palms), Osler nodes (fingers, painful), Splinter hemorrhages (nail beds), Roth spots (eye) (*septic emboli can reach the brain, spleen, LUNG)
  • anemia of chronic disease (from inflammation)
  • immune response-> elevated ESR, positive RF (possibly due to antibody production), IC-mediated GN (hematuria)
867
Q

80 year old is brought in for a check-up. He believes his son is seeking his money. His son says he is becoming forgetful- he cannot recall recent events but remembers stories from when he was young in the navy. He is oriented to person and place only, does not know the current president’s name, and is unable to cooperate with much fo the cognitive exam. MRI brain shows mild, cortical atrophy and reduced hippocampal volume. Syphilis screening is negative. Diagnosis?

A

Alzheimer’s disease

-progressive dementia- first you loose memory of recent events and have language difficulty (followed by impaired judgment and personality change)

868
Q

What is the most common underlying cause of an S4 heart sound?

A

Long-standing HTN

(S3: blood into an already overfilled ventricle/ dilated CM/ systolic dysfunction
S4: blood into a stiff ventricle/ hypertrophic CM/ diastolic dysfunction)

869
Q

70 year old man comes in due to a syncope episode. He was resting in bed for a week after injuring his knee. His BP is normal. Most likely cause of his syncope?

A

Orthostatic hypotension

(Drop in systolic >20 when moving from lying to standing)

  • he’s an older person experiencing syncope upon standing after a period of bed rest
  • usually patient will feel lightheaded or presyncope prodrome before passing out
870
Q

What is Phencyclidine?

A

PCP (“angel dust”)

871
Q

Patient on PCP is brought into the police after being found disoriented wandering the streets. Hs gets violent, so lorazepam and haloperidol are given to calm him. Several hours later, his confusion and agitation are improved but he reports difficulty with vision. His eyes are deviated superiorly, extraocular movement is limited, and there is rigidity in his neck. Non-con CT is normal. What’s going on and what should you give him?

A

Acute dystonia
Benztropine (anti-muscarinic, improves tremor/ rigidity) OR diphenhydramine (anti-histamine)

  • He was given Haloperidol (1st gen antipsychotic) and now has a stiff neck + forced upward gaze deviation. Remember the extrapyramidal side effects: “ADAPT”
  • Acute Dystonia (sudden muscle stiffness)
  • Anesthesia (restlessness)
  • Parkinsonism (cogwheel rigidity, slow movement)
  • Tardive dyskinesia (involuntary jerky, repetitive movements like striking out the tongue)

**see FA pg 561 for extrapyramidal side effect treatments

872
Q

Patient is having a COPD exacerbation. She is given supplemental O2, IV antibiotics, Methylprednisolone, and nebulized Ipratropium with Aluterol. She is still using accessory muscles to breathe. ABG while on 4L O2/min shows pH 7.3, pCO2 60, pO2 50. Next step?

A

Noninvasive positive pressure ventilation (BiPAP or CPAP, *though it seems BiPAP is preferred in acute respiratory distress)

**why the respiratory acidosis (low pH, high CO2)? It is a obstructive disease/ problem breathing out-> air trapping and CO2 retention

873
Q

What is “noninvasive positive-pressure ventilation (NPPV)”?

A

Ventilatory support delivered by face mask rather than endotracheal tube

  • can be delivered continuous (CPAP) or bi-level (BiPAP)
  • *only resort to invasive intubation and mechanical ventilation when this fails**

*CPAP= continuous positive airway pressure
□ Administers equal inspiratory & expiratory pressure support
□ Used in sleep apnea when people just need help propping their airway open at night

*BiPAP= bi-level positive airway pressure
□ Administers inspiratory pressure > expiratory pressure
□ Often used in acute respiratory distress patients

874
Q

Gardener lady with untreated hep C presents with blistering on the backs of her hands. The lesions are not painful but cause mild itching. Her skin is fragile and minor trauma results in superficial erosions that heal with crusting and scarring. Diagnosis?

A

Porphyria Cutanea Tarda

  • due to defective uroporphyrinogen decarboxylase enzyme-> impaired heme synthesis
  • associated with hep C
  • can present with tea-colored urine and cutaneous findings (photosensitivity, hyperpigmentation, painless blistering)
875
Q

30 year old man comes in for a physical exam. His family hx is significant for sudden death in his father. His BP today is 180/100. On exam, you palpate upper abdominal masses. Suspected diagnosis and next step?

A

Autosomal dominant polycystic kidney disease (ADPKD)

Abdominal ultrasound (looking for enlarged kidneys with multiple cysts)

  • father’s sudden death is likely due to a cerebral aneurysm in the setting of ADPKD
  • HTN is an early disease manifestation (due to excess renin release)
  • renal cysts are often palpable
876
Q

Lady comes in for weakness and occasional pain in her thighs and difficulty walking. BP is 170/100, HR 90. On exam, a bruit is heard over the left carotid artery and she has hyporeflexia. EKG shows flat and broad T waves with occasional PVC’s (premature ventricular contractions). Most likely cause?

A

Hypokalemia

  • symptoms: weakness, muscle cramps, decreased deep tendon reflexes, flattening of T waves (U waves if severe) w/ PVC’s…
  • when severe: paralysis, rhabdo, arrhythmias
877
Q

Patient is found to have infective endocarditis due to strep bovis. What other seemingly unrelated test to you need to do?

A

Colonoscopy

*there is a strong association between strep bovis (aka strep gallolyticus) and colon cancer

878
Q

Another name for strep gallolyticus?

A

Strep bovis

879
Q

Single most important prognostic factor in a breast cancer patient?

A

TNM staging

Stage 4 is the worst

880
Q

New York hiker guy with past surgical hx of splenectomy after a car crash got bit by ticks. He presents with high fever/ chills, fatigue, and dark urine. On exam, he has hepatomegaly. Labs show low Hb. High retic count, WBC count, total bili, alk phos, AST, ALT, and LDH. Most likely diagnosis and what test will confirm?

A

Babesiosis
Blood smear

  • tick-borne illness (Ixodes tick from the northeastern US) *same tick that causes Borrelia/ Lyme dz
  • the splenectomy (immunocompromised) makes this patient at higher risk
  • causes hemolytic anemia (explains jaundice, dark urine, indirect hyperbilirubinemia, reticulocytosis, high liver enzymes and LDH)
  • “Maltese crosses” on blood smear is diagnostic
881
Q

Treatment for Babesiosis?

A

Atovaquone + Azithromycin

882
Q

45 year old lady comes in due to progressive weakness for months. She has difficulty lifting her kids, getting in/ out of the car, and swallowing (episodes of choking water). Exam is unremarkable. ESR and creatine kinase are elevated. Diagnosis and treatment?

A

Polymyositis (inflammatory disorder of skeletal muscle)

Prednisone (steroids)
(*most patients also get a glucocorticoid-sparing agent like Methotrexate or Azathioprine to minimize adverse effects)

  • proximal muscle weakness (otherwise normal neuro exam), elevated CK
  • involvement of upper esophageal muscles can cause dysphagia (difficulty swallowing)

*to diagnose: elevated muscle enzymes (CK), autoantibodies (ANA, anti-Jo-1), biopsy (endomysial infiltrate, patchy necrosis)

883
Q

35 year old woman complains of aching pain and stiffness over her entire body for 3 months. She has fatigue, poor sleep, frequent headaches, and difficulty with daily tasks and concentration at her job. On exam, she has tenderness to palpation over the muscles of the neck, shoulders, and back. All labs are within normal limits. Most likely diagnosis and initial therapy?

A

Fibromyalgia

Exercise program with aerobic conditioning (*although patients will complain exercise makes it worse short-term, explain it will improve symptoms in the long-term)

  • you can use a “widespread pain index” and “symptom severity scale” for diagnosis—more accurate than trigger points
  • treatment with meds (SNRI Dulexetine, TCAs) is reserved for patients who fail initial measures
884
Q

Initial and 2nd treatment plan for fibromyalgia?

A
  1. Initially, try exercise (worsens short-term, but improves long-term symptoms!)
  2. If still struggling, try meds (SNRI Duloexetine, TCAs)
885
Q

Elderly lady with ESRD (end-stage renal disease) (on hemodialysis) and anemia of chronic disease (on EPO and iron therapy) and PHM of HTN presents with headache for 2 days. BP is 210/120. Exam shows bilateral retinal hemorrhages. Most likely CAUSE?

A

EPO therapy

She is in hypertensive crisis- this is an adverse effect of EPO treatment

886
Q

What’s an example of a rare time when you may withhold telling a patient about their cancer diagnosis?

A

The patient is severely depressed/ suicidal

887
Q

African American male with PMH of heroin abuse presents with abdominal distention. He is hypertensive. Exam shows periorbital edema, ascites, and 2+ pitting edema in both legs. U/A shows protein in the urine. Most likely diagnosis on kidney biopsy?

A

Focal segmental glomerulosclerosis (FSGS)

  • nephrotic syndrome
  • more often seen in: Hispanics, African Americans, HIV, heroin, sickle cell
888
Q

30 year old man is brought in by his roommate with confusion and agitation for 2 days. He has seizures while in the ED. Temp is 104. He is confused and disoriented and has positive Babinski sign bilaterally. Has NO neck stiffness. CT head is normal LP shows CSF with high protein, RBCs, and 90% lymphocytes. Gram stain shows no bacteria. While add’l CSF studies are pending, what is your next step?

A

Give IV Acyclovir

This is Herpes (HSV) encephalitis
(inflammation of the brain due to infection *herpes viruses are most likely to cause encephalitis in immunoCOMPETENT adults)

  • confusion, seizures, exaggerated reflexes (can also see other neuro abnormalities)
  • absence of neck stiffness (not meningitis)
  • CSF: high protein, lymphocytes, no bacteria= viral
  • PCR of CSF showing viral DNA confirms diagnosis
889
Q

What bug is most likely to cause encephalitis in immunoCOMPETENT adults?

A

Herpes viruses (HSV, EBV)

890
Q

Screening test and confirmatory test for Herpes encephalitis?

A

Screening: LP with CSF showing normal-high protein, lymphocyte predominance, no bacteria (viral)

Confirmatory: PCR of the CSF showing viral DNA

891
Q

Statin therapy is recommended in patients with ASCVD (atherosclerotic cardiovascular disease) score greater than what?

A

> 7.5%

892
Q

Guy undergoes PCI via femoral artery access after having an MI of the LAD. One week later, his left lower leg is cold and mottled in appearance. Femoral pulses are intact, but left pedal pulse is absent. You consult vascular surgery. What other test should you consider doing?

A

Echocardiogram

  • this is acute limb ischemia
  • although it’s often due to thrombosis (plaque), it can be caused by embolism (thrown clot)
  • post-MI complication (especially in big heart attacks like the LAD) is LV aneurysm
  • LV aneurysm-> heart doesn’t contract well-> low ejection fraction-> stasis of remaining blood in there, which promotes clot formation-> clot gets thrown to periphery (can lead to stroke, or acute limb ischemia like in this patient)
893
Q

Lady presents with fever/ chills, N/V, and worsening RUQ pain for 2 days. Temp is 103, BP 90/48, HR 98, RR 30. She appears ill and confused. Skin and sclera are icteric (jaundiced). Labs show elevated total bili, direct bili, alk phos, ALT, AST, and WBC count. Diagnosis?

A

Acute cholangitis

  • stone is lodged in the common bile duct and infected-> sepsis
  • Charcot triad: fever, jaundice, and RUQ pain
  • Reynold’s pentad: +shock (low BP), altered mental status
  • treat with supportive care, broad-spectrum antibiotics, and biliary drainage (preferably by ERCP)
894
Q

What would a CT scan show in acute cholangitis?

A

Dilation of the intrahepatic and common bile ducts

*remember, acute cholangitis is when a stone is lodged in the common bile duct, causing infection.

895
Q

How high would you expect transaminases (ALT, AST) to be in Tylenol overdose?

A

SUPER high (like >3,000)

896
Q

What would CT scan show in acute cholecystitis?

A

Thickening of the gallbladder wall and pericholecystic fluid

*remember, acute cholecystitis is when a stone is lodged in the cystic duct

897
Q

What are the 2 treatment issues that need to be addressed in your a-fib patients?

A
  1. Rate or rhythm control?

2. Long-term anticoagulation needed to prevent stroke? (Use CHADS-VASC score)

898
Q

What are the 2 drug class options for rate-control of a-fib?

A
  1. Beta-blocker

2. Non-dihydropyridine CCB (Verapamil or Diltiazem—the “non-dairy” ones that act on the heart)

899
Q

Lady has episodes of retrosternal pain that radiates to the interscapular region of the back. These episodes last 15 min and are triggered by stress or hot/ cold food that results in food regurgitation. Nitroglycerin tablets alleviate the pain. EKG, stress test, echo, CXR, and upper GI endoscopy are all normal. Next step?

A

Esophageal motility studies (manometry recordings)

  • episodes of dysphagia (difficulty swallowing), regurgitation, and/or chest pain brought on by stress should raise suspicious for an esophageal motility disorder like DIFFUSE ESOPHAGEAL SPASM
  • Nitrates (and CCBs) relax coronary vessels, but also smooth muscle cells of the esophagus, alleviating pain
900
Q

What does odynophagia mean?

A

Painful swallowing

901
Q

Lady comes in due to excessive hair growth on her face and body. Her menstrual cycle used to be regular and she had a successful pregnancy, but now her cycles are irregular, LMP was 4 months ago. She is muscular, has a large clitoris, and temporal balding. Urine pregnancy test is negative. Next step?

A

Testosterone and DHEAS levels

  • most women with hirsutism have PCOS
  • BUT, rapid-onset (<1yr) hirsutism with virilization suggests very high androgen levels, possibly due to an androgen-secreting tumor of the ovaries or adrenal glands

If high testosterone + DHEAS—> ovarian source

If just high DHEAS—> adrenal source

902
Q

Lady has fatigue, worsening SOB, and swelling in her feet for 5 days. She had a cold 2 wks ago. Exam shows bilateral crackles, elevated JVP, and 2+ pitting edema in the lower extremities. Diagnosis?

A

Decompensated HF most likely 2/2 Viral myocarditis (from coxsackievirus, adenovirus, parvovirus B19, etc.)

  • would expect to see dilated ventricles and diffuse hypokinesis of the ventricle walls (not pumping well) on echo
  • treat supportively and with meds for HFrEF (diuretics, beta-blocker, ACEi)
903
Q

What findings on echo would you see with viral myocarditis?

A

dilated ventricles and diffuse hypokinesis of the ventricle walls (not pumping well)

*dilated cardiomyopathy picture

904
Q

Guy comes in with sudden-onset confusion after returning from an underwater spearfishing trip. He has SOB, tingling and weakness in his arm, and slurring speech. Exam shows moderate respiratory distress, bilateral faint lung crackles, reduced muscle strength in the arm, and mottling of the skin. Diagnosis?

A

Vascular air embolus from decompression sickness (“the bends”)

-results from transfer of nitrogen gas from tissues into venous bloodstream when a diver ascends too rapidly from high pressure/ deep waters
-these small air bubbles in the venous bloodstream can lodge into skin capillaries, causing mottled skin. Into pulmonary capillaries, causing respiratory distress and pulm edema. Into the RV outflow tract, causing obstructive shock.
Can also pass into arterial circulation and travel to the brain, causing confusion and stroke or MI.

  • *from PATHOMA: scuba diving/ spearfishing-> pressure increases the lower you go-> that pressure forces nitrogen to dissolve in the blood (inc pressure= inc gas that gets dissolved int he body)-> when the diver rapidly ascends back up, nitrogen can precipitate out of the blood as little gas bubbles that lodge in various tissues
  • > joint and muscle pain (“the bends”)
  • > respiratory symptoms (“the chokes”)
905
Q

1st step to determining the cause of ascites in a question stem?

A

Calculate SAAG (serum albumin ascites gradient)= serum albumin- ascites fluid albumin

If >1.1–> cause of ascites is portal HTN
(liver cirrhosis or cardiac ascites)

If <1.1–> other cause
(cancer, pancreatitis, nephrotic syndrome, TB, etc.)

906
Q

What are the 3 things that should make you suspect HIT in a patient recently treated with heparin?

A
  1. Thrombocytopenia
  2. Thrombosis (arterial or venous)
  3. > 50% drop in the platelet count from baseline

Within 5-10 days after initiation of heparin treatment

907
Q

HIV patient (non-compliant with meds) has progressive memory impairment, difficulty communicating, and left-sided weakness for 1 month. He is not oriented to time, place, or person and has aphasia. Left-sided hyperreflexia is present. CD4 count is 30. CT brain with contrast shows nonenhancing white matter lesions. Diagnosis?

A

Progressive multifocal leukoencephalopathy (PML)

  • caused by the JC virus
  • seen in AIDS patients with CD4 <200
  • virus affects the CNS-> white mater demyelination
  • no specific treatment, but antiretroviral therapy prevents it (most patients with this end up with long-term neuro damage)
908
Q

1st step in management of hypercalcemia?

A

IV fluids (Normal saline)

  • to promote urinary calcium excretion
  • calcitonin (directly blocks ostoC’s-> less bone breakdown-> less calcium into blood) and bisphosphonates (ostoC apoptosis-> less bone breakdown-> less calcium into blood *takes 2-4 days to work) usually given also, but aggressive hydration is the first step in management
909
Q

What 4 inflammatory/ depositional diseases are associated with carpal tunnel syndrome?

A
  1. RA
  2. Sarcoidosis
  3. Hypothyroidism
  4. Amyloidosis
910
Q

Polymyositis/ Dermatomyositis are associated with what 4 antibodies?

A
  1. ANA
  2. Anti-Jo-1 (anti-histidyl-tRNA-synthase)
  3. Antisignal recognition particle (anti-SRP)
    * worse prognosis
  4. Anti-Mi-2 antibodies
    * better prognosis
911
Q

Dermatomyositis patients are at risk for what other big problem?

A

Increased risk of malignancy

Lung, breast, ovary, GI, and myeloproliferative disorders

912
Q

50 year old lady with colon cancer (scheduled for resection) and PMH of HTN (non-compliant with meds) presents after collapsing while standing. She also reports left-sided chest pain. BP is 85/50, HR 120, pulse ox 80%. She is tachypneic and diaphoretic. JVP is 13. Lungs clear. Diagnosis?

A

Massive PE

  • cancer (pro-thrombotic state) is a risk factor
  • acute presentation of SOB/ hypoxia, chest pain, tachycardia, clear lungs
  • may hear loud S2, have elevated JVP
  • massive PE-> syncope (PE is impairing ability to breathe, so there’s insufficient oxygen getting to the brain) and hemodynamic collapse (RV has to pump against extra pressure-> dilation and impaired pumping)

**can be difficult to distinguish massive PE from RV MI (both cause RV dysfunction)—but, RV MI is less likely to cause SOB, tachycardia, syncope. More likely to cause bradycardia or arrhythmia

913
Q

Attorney complains of sleep problems. She goes to bed at midnight and has trouble falling asleep- usually doesn’t fall asleep until 3am and wakes up at 7am. Her job is enjoyable but stressful and she’s exhausted, so she drinks coffee all day, smokes a pack of cigarettes, takes an evening “power nap,” and eats dinner at 8pm. Lately, she wonders “what is my life all about” and feels irritable. Most likely cause of her sleeping problems?

A

Poor sleep hygiene

  • caffeine (should avoid caffeine after lunch) and nicotine (stimulants)
  • takes evening nap
  • works late hours
  • eats late dinner

*recommend for her to have a regular sleep schedule, quiet/ dark/ comfortable sleeping environment, avoid late afternoon naps, electronic devices before bedtime, nicotine/ caffeine, and heavy evening meals

**although she is questioning her purpose and irritable, this is likely a consequence of sleep deprivation, not an underlying psych disorder

914
Q

What 3 types of drugs prevent cardiac remodeling and improve mortality post-MI?

A
  1. ACE inhibitors (Enalapril, Lisinopril, etc.)
  2. Cardioselective beta-blockers (Metoprolol)
  3. Aldosterone antagonists (Spironolactone)
915
Q

25 year old female had achy joint pain for 4 weeks with moderate swelling of the MCP joints in both hands, wrists, knees, elbows. She had fevers at the onset of her joint pains, but is now afebrile. She is no longer experiencing pain in her joints and exam is unremarkable. X-rays of hands are normal. Diagnosis?

A

Parvovirus B19

  • self-limited viral arthritis (“slapped cheek” rash in kids, joint swelling and tenderness in adults)
  • can present similar to RA or SLE, but is acute and goes away (also more common than RA and SLE)
916
Q

Kindergarten teacher complains of gritty sensation in eyes (like sand in eyes) for 3 days. Eye exam shows mild injection and glandular appearance of the tarsal conjunctiva of the right eye with profuse watery discharge. Diagnosis and next step?

A
Viral conjunctivitis (most likely due to adenovirus) 
Cool, moist compresses/ symptomatic relief (can give antihistamine/ decongestant eyedrops) 
  • mild injection= redness (pink eye)
  • granular appearance of tarsal conjunctiva (lining of the eyelids when you pull them down look granular, secretory)
  • profuse watery discharge= viral (vs. purulent/ pus discharge in bacterial conjunctivitis)
917
Q

Woman is admitted to the ICU after a car accident, in which she was hypotensive from blood loss at the scene and required 7 L of fluids (crystalloids, blood, FFP). She had surgery and is receiving continuous IV fluids and vasopressors. Labs 24 hrs after the accident show Hb 10, leukocytes 15,000, BUN 30, Cr 2. Most likely finding on U/A?

A

Muddy brown casts (ATN)

-pre-renal AKI (hypovolemic shock)-> intrinsic AKI (ATN is the most common)
-BUN/Cr= 30/2= 15
(Remember in intrinsic: BUN/Cr is low <20:1, FeNa is high >2%, and urine osm is high >300 bc it is a kidney problem)
(*vs. pre-renal where BUN/Cr is high >20:1 due to slow blood flow-> extra time for BUN to get reabsorbed at PCT)
-muddy brown casts= necrosis of renal tubular epithelial cells (very sensitive for ATN)

918
Q

On U/A you see muddy brown casts. What diagnosis are you thinking of?

A

ATN (acute tubular necrosis)

919
Q

On U/A you see RBC casts. What diagnosis are you thinking of?

A

Glomerulonephritis

920
Q

On U/A you see WBC casts. What 2 diagnoses are you thinking of?

A
  1. Acute Interstitial nephritis
    (type of intrinsic AKI from drug like NSAID, Penicillin, diuretic)
  2. Pyelonephritis
921
Q

On U/A you see fatty casts. What diagnosis are you thinking of?

A

Nephrotic syndrome

922
Q

On U/A you see broad and waxy casts. What diagnosis are you thinking of?

A

Chronic renal failure

923
Q

HIV patient (treated with several different antiretroviral combinations due to resistance from non adherence) comes in for follow-up. Over the past year, he’s had a sensation of numbness and pinpricks in his feet, which now progressed up his lower legs. There is loss of sensation to touch, pain, and temp over the distal legs and feet. CD4 count is 180. Blood cell counts, liver/ renal function tests, and fasting glucose levels all normal. In addition to counseling about taking his meds, best treatment for his symptoms?

A

Gabapentin

  • this is Distal Symmetric Polyneuropathy 2/2 long-standing HIV infection (thought to occur due to accumulation of HIV antigens on distal sensory nerve fibers-> inflammation and nerve damage)
  • r/o other causes of polyneuropathy: diabetic (normal glc), uremic (normal renal function), Vit B12 def (normal RBC count/ no anemia)
924
Q

Patient who is a landscaper with a cat presents with itchy skin lesions. There is a nodular ulceration with nonpurulent drainage and surrounding erythema on the right index finger and several nontender subcutaneous nodules on the right forearm. No lymphadenopathy. Diagnosis?

A

Sporotrcihosis (from sporothrix schenckii fungus in decaying plant matter/ soil)

  • causes skin papules-> ulceration along lymphatic chain
  • lymphadenopathy is rare (only with deeper spread) *vs. cat scratch disease where tender lymphadenopathy is a hallmark feature
925
Q

What are the EKG findings in each type of AV block?

A

1st degree: prolonged PR interval
(>5 small boxes)

2nd degree:
-Mobitz type I (Wenckebach): progressive lengthening of PR until QRS beat is dropped (“longer, longer, longer, drop- this is how you Wenckebach”)

-Mobitz type II: random dropped QRS beats (not preceded by a change in length of PR)

3rd degree/ complete AV block: P and QRS’s not in sync

*what is AV block? Remember the AV node works to delay the signal from the atria to the ventricles so the heart beats at an appropriate rate. 1st degree is when it’s delaying too much (prolonged PR). 2nd degree is when it’s partially blocked and the missed QRS is when it fails to relay the impulse. 3rd degree is when there’s no communication btwn the atria and ventricles.

926
Q

Man is brought into the ED due to ascending paralysis after returning from a hiking trip in Colorado. He first noticed paresthesias in his lower legs, then progressed to weakness. Exam shows absent deep tendon reflexes on the left leg, diminished on right. He has lower extremity weakness with no motor activity on high left leg. He is afebrile with a normal WBC count and CSF exam. Next step?

A

Search for a tick on his body

  • most likely diagnosis is tick-borne paralysis (from neurotoxin release…the tick has to feed for 4-7 days to release this toxin so find the tick an remove it and the patient usually improves in an hour with full recovery after several days!!)
  • progressive ascending paralysis over hours to days
  • absent fever, no autonomic dysfunction (as in Guillain Barre), normal CSF exam
927
Q

50 year old female with PMH of asthma, T2DM, and HTN presents with SOB for 3 wks, worse today. She is also coughing up dark-colored sputum. Vitals are stable, pulse ox is 93%. She is using accessory muscles to breathe. On exam, the trachea is deviated to the right, there is dullness to percussion on the right, and breath sounds are reduced on the right. Rhonchi are present bilaterally. What explains these findings?

A

Atelectasis

-most likely due to asthma exacerbation-> mucus plugging in the airways-> blocks air from entering and the air molecules diffuse into the bloodstream and create a vacuum that leads to alveolar collapse

-decreased breath sounds (collapsed alveoli, so that part of the lung is not getting air)
(Also dullness to percussion due to lack of air and decreased fremitus due to lack of air to create vibrations)

-collapsed alveoli pull the trachea over to same side

928
Q

Patient is found to have a DVT and elevated plasma homocysteine levels. She is stated on heparin and warfarin. What other therapy should you give her?

A

Pyridoxine (Vitamin B6)

*Remember that Vitamins B6 (pyridoxine), B9 (folate), and B12 (cobalamin) are involved in the homocysteine pathway…giving B6, B9, and/or B12 can restore homocysteine levels in homocystinuria

929
Q

45 year old man with 50 pack-year smoking hx presents with bilateral hand pain most severe in his wrists. He has thickened distal fingers and convex nail beds. Chest exam shows decreased breath sounds and prolonged expiratory phase. He has missed work often due to being sick. Next step?

A

Chest x-ray

  • he is presenting with signs and symptoms of COPD (heavy smoker, clubbing, decreased breath sounds/ prolonged expiration since it’s hard to get air out, “sick” often probably due to URI’s/ COPD exacerbation
  • *convex nails (going outward like a lens)= digital clubbing (sign of poor oxygenation)

-his presentation is suspicious for HYPERTROPHIC OSTEOARTHROPATHY (abnormal proliferation of skin and bone at distal extremities-> clubbing, arthralgia/ joint pain). This is a sign of ADENOCARCINOMA OF THE LUNG, so start with a CXR to assess for cancer.

930
Q

65 year old female with recent successful cataract extraction (4 mo ago) sees a sudden burst of flashing lights and blurry vision in her left eye since this morning. Now she sees spots in her visual field and feels like a “curtain came down” over her eye. Ophthalmic exam reveals retinal tears and a grayish retina. Diagnosis?

A

Retinal detachment

  • separation of the layers of the retina (retina peels away from the underlying choroid layer)
  • myopia (nearsightedness where far objects are blurry) or trauma (like her recent cataract surgery) can cause breaks in the retina-> fluid seeps in and eventually pulls apart the retina layer
  • presents as photopsia (flashes of light in vision), floaters, dark “curtain coming down over eyes”
931
Q

Chronic kidney disease (CKD) is an independent risk factor for what disease?

A

Cardiovascular disease

*cardiovascular dz is the most common cause of death in patients with ESRD (50%). Due to risk factors they have (HTN, advanced age, etc.) and ESRD-related risks (anemia of chronic dz, oxidative stress related to dialysis)

932
Q

In a urine dipstick, what does presence of leukocyte esterase indicate? What does presents of nitrites indicate?

A

Leuk esterase-> means there are WBCs in the urine

Nitrites-> means there is gram (-) bacteria in the urine (most likely E. Coli)
*convert urinary nitrates to nitrites

933
Q

Young lady comes in for follow-up from a head injury while playing soccer. After being hit, she vomited and was confused (didn’t know what happened), but did not lose consciousness. CT head was normal and she was discharged from the ED with close observation by family members. Today she has a headache, but is not confused and full neuro exam is normal. What happened and when can she play soccer again?

A

She had a concussion

She can gradually increased activity over 1 week if she no longer has symptoms

  • recent head injury + vomiting + transient amnesia= concussion, even though she did not lose consciousness! Definition is a temporary neuro disturbance (includes disorientation, amnesia) resulting from a mild traumatic brain injury
  • further imaging usually not required, but advice to rest at least one day and gradually return to exercise (full return no sooner than 1 week) to reduce complication (chronic traumatic encephalopathy, second impact syndrome)
934
Q

Guy comes in due to increasing left flank pain, nausea, and hematuria for 5 days. The pain is dull, constant, and nonradiating. Yesterday, he also noticed swelling of the left testis. BP is 130/70. Exam shows left flank tenderness, bilateral 1+ pitting edema, and dilated tortuous veins in the left scrotum. Diagnosis?

A

Renal vein thrombosis (RVT)

  • this is a complication of nephrotic syndrome
  • membranous nephropathy (nephrotic syndrome)-> RVT
  • proteinuria (>3.5 g/day), hypoalbuminemia (<3 g/dL), edema (decreased oncotic pressure), hypercholesterolemia (liver pushed cholesterol into “thinned out” blood), hypercoagulable state (due to loss of anticoagulant proteins)
  • men can get edema in the scrotum and the RVT causes blockage preventing the left testicle from draining blood
935
Q

45 year old man presents with fatigue. He’s a vegetarian, avoids dairy, and drinks alcohol daily. Labs show Hb of 10 and normal serum iron. He is started on thiamine (Vit B1) and folic acid (Vit B9). Months later, he reports recurrent falls and increasing forgetfulness. What’s going on?

A

Vitamin B12 deficiency

-suspect this in hard-core vegetarians/ vegans with anemia and neuro complications (loss of proprioception/ vibration, memory deficit, irritability)

  • he most likely has a combined folate (from veggies, risk from having a poor diet/ alcoholic) and B12 deficiency (from meat, risk from being a vegetarian many years)
  • replenishing the folate only will partially correct hematologic abnormalities, but will not help (may worsen) neuro issues!

**if you give a vit B12 deficient patient folate, you pull their limited B12 stores to making more RBCs (not myelin)—this can worsen neuro problems!

936
Q

Another name for superior pulmonary sulcus tumor?

A

Pancoast tumor

937
Q

What’s the triad to diagnose acute liver failure (ALF)?

A
  1. Elevated aminotransferases (high ALT, AST)
  2. Signs of hepatic encephalopathy (confusion, somnolence, flapping tremor)
  3. Synthetic liver dysfunction (INR >1.5)

**acetaminophen (Tylenol) toxicity is the no. 1 cause of ALF

938
Q

Why is alcohol + Tylenol a bad mix?

A

Chronic alcohol use accelerates Tylenol’s toxicity

Why? Alcohol depletes glutathione levels (glutathione protects the liver from oxidative damage). Tylenol also does this by the NAPQI enzyme. So this combination can lead to hepatic necrosis and acute liver failure!

939
Q

Serious side effect of Trazodone?

A

Priapism (persistent, painful erections)

  • atypical antidepressant (serotonin-modulating)
  • this happens in <1% of cases, but is a medical emergency (can lead to permanent tissue damage)
940
Q

How do you confirm a diagnosis of leprosy?

A

Skin biopsy (from edge of lesion)

941
Q

Besides high fluid intake, what can you tell a patient with uric acid kidneys stone to do in order to prevent recurrence?

A

Take potassium citrate

*uric acid kidney stones are seen in low pH acidic urine, so we can prevent them through alkalization of the urine and a low-purine diet. Citrate helps to alkalize the urine.

942
Q

Lady has malodorous vaginal discharge for 1 month and sometimes feels like gas is passing through her vagina. On exam, discharge is tan-colored, there is a patch of erythema on the posterior vaginal wall, and a sinus with purulent drainage on the perianal skin. What’s going on?

A

Rectovaginal fistula
(Connection between rectum and vagina)

*seen in patients with ob/ pelvic trauma/ surgery or in patients with underlying Crohn’s disease (transmural inflammation of the GI tract)

943
Q

What lobe of the brain is atrophied in late Alzheimer’s disease?

A

Temporal lobe

944
Q

65 year old man presents with vague abdominal pain and fatigue. He smokes and drinks. On exam, the liver edge is palpated and is hard. He has trace ankle edema. Labs show microcytic anemia, mildly high AST and ALT, and high alk phos. CXR shows a small left-sided pleural effusion. Fecal occult blood is positive. Diagnosis?

A

GI cancer (likely colon cancer) that metastasized to the liver

(Abdominal pain, microcytic anemia, positive fecal occult blood, hepatomegaly with hard liver edge)

  • liver= most common site for colon CA metastasis
  • pleural effusion likely due to malignancy (NOT hepatic hydrothorax—this would be right-sided)
  • get a CT abdomen
945
Q

Lady has a sore throat for a week, so she takes amoxicillin that she finds in the medicine cabinet from 1 year ago. 30 min after, she develops an itchy rash. On exam, her tonsils are red without exudates and she has scratching on her skin due to the rash. Lungs are clear, no wheezing. Next step?

A

Treat with antihistamine

  • this is a type I hypersensitivity reaction (immediate onset, IgE mediated)
  • In patients with mono (EBV), amoxicillin can cause rash after 24 hrs. Mono is not really a concern here since her rash came on shortly after taking amoxicillin (not after a day) and she doesn’t have other mono-like symptoms (fever, fatigue, posterior cervical lymphadenopathy)
946
Q

35 year old lady wakes up with left-sided facial droop and increased sensitivity to noise. She cannot smile or raise her eyebrow on the left side. Sensation is normal. Diagnosis?

A

Bell palsy

  • upper face involvement distinguishes Bell palsy from UMN disorders (stroke), which spare the upper face
  • it is a peripheral neuropathy of the facial nerve (CN 7) characterized by facial weakness that involves the upper + lower face
  • thought to be caused by reactivation of neurotrophic viruses (like HSV)-> inflammation, edema, and degeneration of the myelin sheath
  • treat with glucocorticoids
947
Q

40 year old man presents with abdominal pain and watery diarrhea. He also has a skin rash worse with sun exposure. He has felt depressed lately and had loss of appetite. Exam is remarkable for a pigmented scaly skin rash on his face, neck, and back of his hands.

A

Pellegra
(Niacin/ Vitamin B3 deficiency)

3 D’s: “Diarrhea, Dermatitis, and Dementia”

948
Q

25 year old presents with fever and joint pains. He is being treated with cephalexin for a skin infection. His urine is dark and he has a fever. U/A shows RBCs, WBCs, WBC casts, eosinophils, and proteinuria. BUN and Cr are elevated. Diagnosis and next step?

A

Drug-induced interstitial nephritis (type of intrinsic AKI)
D/C the cephalexin

  • drug (NSAID, Penicillin, diuretic, etc.)-> fever, rash, dec urine output, eosinophils
  • WBC casts-> think interstitial nephritis or pyelonephritis
949
Q

Lady with hx of esophageal variceal band ligation presents with bloody emesis. Despite resuscitation with crystalloids, she is minimally responsive. She’s barely arousable to sternal rub. BP is 89/49, pulse 120. She continues to have episodes of bloody vomit w/ clots. Next step?

A

Endotracheal intubation

  • she needs airway protection bc she has depressed level of consciousness + ongoing hematemesis (she is unstable!)
  • THEN, promptly stop the bleeding with endoscopy ligation or sclerotherapy (solution injected into veins to cause them to constrict)
950
Q

70 year old man with chronic back pain (takes NSAIDs) presents with worsening fatigue. Exam shows an S4 heart sound and clear lungs. Labs show anemia and elevated BUN and Cr. U/A shows trace protein and no casts. U/S shows bilateral small kidneys with no hydronephrosis. What best explains these findings?

A

HTN (hypertensive nephrosclerosis)

  • renal dz with high BUN, Cr, bilateral small kidneys, trace proteinuria, S4 (blood into thick ventricle/ hypertrophic CM)
  • long-standing HTN-> hyaline arteriOlosclerosis (renal arterioles undergo medial hypertrophy and intimal fibrosis and this endothelial damage leads to protein and BM deposition into arteriolar walls)-> decreased blood flow to glomerulus (glomerulosclerosis)

**no casts is a clue it’s not ATN (muddy brown casts), interstitial nephritis (WBC casts), glomerulonephritis (RBC casts), etc.

951
Q

Patient has a DVT, was hospitalized, and was discharged on Warfarin. INR was 2.2 (goal 2-3). He has been taking his Warfarin daily, but missed his last follow-up appt. Today he presents with right leg edema, INR 1.3, U/S shows right popliteal vein thrombosis. Next step?

A

Replace Warfarin with an oral factor Xa inhibitor (RivaroXaBAN, ApiXaBAN)

*oral direct factor 10 inhibitors work similar to warfarin in treating acute venous thromboembolism and do not increase risk of bleeding complications (in fact, they are not preferred since they don’t require INR monitoring or dietary restrictions)

**he got a clot in the setting of sub-therapeutic INR (normal INR= 1, but for patients on Warfarin we aim for 2-3 bc higher INR= less clotting)

952
Q

27 year old male presents with a nose bleed. He’s had a similar nose bleed episode that brought him to the ED in the past. On exam, he has several ruby-colored papules on his lips that blanch partially with pressure. He also has clubbing. CBC is significant for elevated hematocrit (60%). Diagnosis?

A

Hereditary Hemorrhagic Telangiectasia (HHT)
aka Osler-Weber-Rendu syndrome

  • Autosomal dominant disorder
  • Diffuse telangiectasias (ruby-colored papules that blanch with pressure)
  • Recurrent epistaxis (nosebleeds)
  • AV malformations (shunts blood from arterial to venous system)-> hypoxemia (explains the clubbing and elevated Hct to compensate)
953
Q

1st line treatment for RA?

A

Methotrexate (a disease-modifying agent)

*If persistent symptoms for >6 months, add another agent (ex: TNF-alpha inhibitor, sulfasalazine, hydroxycloroquine)

954
Q

HIV patient with CD4 count of 49 presents with pain with swallowing and a substernal burning sensation. Exam shows white plaques on the buccal mucosa and palate that can be easily scraped off. Diagnosis and treatment?

A
Candida esophagitis 
Oral fluconazole (start empiric treatment) 
  • HIV patients w/ CD4 <100 and esophagitis-> most likely to be Candida, HSV, or CMV
  • oral thrush is the big hint here that it’s Candida
  • if doesn’t respond to the anti-fungal meds, then you’d proceed to upper endoscopy and biopsy
955
Q

Patient complains of depression and poor sleep. BP is 160/110. Labs are all normal except calcium is elevated at 12. Most likely cause of her HTN?

A

Hyperparathyroidism

Usually from a parathyroid adenoma-> too much PTH-> too much calcium into the blood

This is associated with HTN. Why? Thought to be due to increased renin release, sympathetic activation, and/or peripheral artery vasoconstriction

956
Q

What do you do for patients who complain in headaches that are new/ different in character from prior headaches, that present when awakening, and/or are associated with frequent N/V and vision disturbance?

A

MRI brain

*got to assess for a brain tumor

957
Q

26 year old lady with 2 day hx of fever and headache is brought in due to a seizure. She was behaving strangely this morning. Temp is 102 and hyperreflexia is noted. CSF analysis shows elevated protein, RBCs, and lymphocyte predominance. Diagnosis?

A

Herpes simplex encephalitis

  • most common fatal sporadic encephalitis (swelling of the brain) (due to HSV-1)
  • presents with acute focal neurological findings and usually fever
  • CSF shows lymphocyte pleocytosis, increased number of erythrocytes (due to hemorrhagic destruction of temporal lobe), and elevated protein
  • MRI shows temporal lobe involvement/ lesions
  • PCR showing HSV DNA in CSF is diagnostic
  • treat with IV Acyclovir
958
Q

Lady is hospitalized for fever/ chills, dysuria, and flank pain. BP is 85/40, HR is 120. She has CVA tenderness. U/A shows bacteria in the urine. You get urine and blood cultures and start IV fluids + empiric antibiotics. The patient is still hypotensive, so you give NE. Two days later, the patient improves, but fingertips are discolored and black. Why?

A

This is pyelonephritis complicated by septic shock
You gave NE (vasopressor, alpha-1 > beta agonist)-> vasoconstriction to raise MAP
-shunts blood to central organs, but away from periphery
-decreased perfusion to the digits-> NECROSIS of tissue (dry gangrene) if prolonged

959
Q

IV drug user with recent diagnosis of Candida esophagitis (received nystatin but refused further work-up) presents with SOB, productive cough, and fever. He is in mild respiratory distress. A midsystolic murmur is noted at the right upper sternal border and there are crackles over the lungs. No JVP or peripheral edema. Most likely diagnosis?

A

Pneumonia
(SOB, productive cough, fever)

  • IV drug user + candida esophagitis= he’s got AIDS
  • midsystolic murmur at R upper sternal border (aortic valve)= sounds like aortic stenosis, but it’s actually a flow murmur from fever causing hyperdynamic state, so you hear the blood flow across the valve louder (IV drug use, fever, and murmur are suspicious for endocarditis, but the patient lacks other signs of HF—JVP and edema)
  • V/Q mismatch is the mechanism for his hypoxemia
960
Q

What besides cardiac tamponade can cause pulsus paradoxus (drop in BP >10 mmHg during inspiration)? Explain.

A

Obstructive lung dz, like COPD or severe asthma attack

Hard to get air out-> air trapping (air stacking)-> when you inhale the lungs expand more, compressing the heart-> heart fills less-> drop in CO and BP

961
Q

What disease is associated with:

  1. Anti-smooth muscle antibody
  2. Anti-mitochondrial antibody
A
  1. Anti-smooth muscle antibody—> autoimmune hepatitis
  2. Anti-mitochondrial antibody—> primary biliary cholangitis/ cirrhosis (autoimmune destruction of intrahepatic bile ducts)
962
Q

60 year old woman comes into the hospital due to episodes of exertional chest pain. She has had episodes for 3 weeks whenever she goes walking. Today she had an episode in the morning while in the shower. Next step?

A

Admit the patient to the hospital to cath lab (stable angina has progressed to unstable angina)

*definitely want to cath females >50 with typical chest pain

963
Q

What is a gastric bezoar?

A

An ingested foreign body (hard mass in the stomach)

964
Q

Man with intellectual disability has episodes of vomiting after eating. He’s also had some weight loss during this period. Exam shows an area of alopecia on his scalp. Diagnosis?

A

Gastric bezoar (foreign body/ mass in stomach)

-he is swallowing his hair

965
Q

Elderly man has memory loss and is forgetting people’s names more often lately. Neuro exam is normal. MMSE (mini-mental state exam) is 27/30. Most likely cause of memory loss?

A

Mild cognitive impairment

966
Q

African American man with PMH of retinoblastoma (treated in infancy) has swelling and pain above his right knee. Alk phos is high and x-ray of the right femur shows a lytic lesion. Most likely diagnosis?

A

Osteosarcoma

*patients with familial retinoblastoma are at increased risk for osteosarcoma (Pathoma fact)

967
Q

What is the pH and glucose cut-off for COMPLICATED parapneumonic effusion?

A

PH <7.2
Glucose <60

(If pH >7.2 and glucose >60, it’s uncomplicated)

968
Q

40 year old lady has dizziness and difficulty with gait and balance for 3 days. She also is a little nauseous. Family hx is significant for MS in her sister. On exam, optic discs are normal. Tympanic membranes are dull and light reflex is distorted. Diagnosis?

A

Labyrinthitis

  • inflammation of the vestibular nerves in the inner ear, usually triggered by an infection (self-limited)
  • acute onset with vertigo, N/V, gait disturbance
  • distortion of light reflex off tympanic membrane is due to ear fullness
969
Q

Patient with PMH of ulcerative colitis presents with fever, chills, abdominal pain, and dark urine for 1 day. Exam shows jaundice, epigastric tenderness, and a mildly tender and firm liver. Labs show increased PT, total and direct bili, alk phos, and ALT. Diagnosis?

A

(Primary) Sclerosing cholangitis (PSC)

  • associated with Ulcerative Colitis
  • inflammation and fibrosis of intrahepatic and extrahepatic bile ducts (“string on pearl” appearance)
  • direct hyperbilirubinemia
970
Q

African American woman comes in due to tightness in her chest + nonproductive cough, worse at night. She has a history of needing breathing treatments. Exam shows wheezing. Diagnosis?

A

Asthma

*don’t be stupid and put sarcoidosis just cuz she’s African American!!

971
Q

Guy recently traveled to the Middle East and is now having watery diarrhea that’s brown and occasionally green. Also has abdominal bloating and gas. Most likely diagnosis?

A

ETEC (Enterotoxigenic E. Coli)

*aka traveler’s diarrhea

972
Q

Slow rising weak carotid pulse. What comes to mind?

A

Aortic stenosis

973
Q

25 year old woman comes in due to fever and right knee pain. Exam shows few raised erythematous papules on both palms. The right knee is warm, erythematous, and tender to palpation. Analysis of synovial fluid shows leukocytosis with neutrophil predominance. Diagnosis?

A

Infection of the joint from Disseminated Gonorrhea

-TRAID: polyarthralgias (pain in multiple joints) + tenosynovitis (inflammation of flexor tendon sheath in wrist) + vesiculopustular skin rash

974
Q

What pneumoconioses?

  • sandblasters
  • aerospace workers (NASA)
  • construction/ plumbers/ shipyard workers
A

Sandblasters—> Silicosis

Aerospace workers (NASA)—> Berylliosis

Construction/ plumbers/ shipyard workers—> Asbestosis

975
Q

43 year old lady complains of generalized pruritus (itching) and fatigue. Exam is notable for xanthelasmas (soft yellow plaques on the eyelids), skin excoriations, and mild hepatomegaly. Total bili is elevated and antimitochondrial antibody is positive. Diagnosis?

A

Primary biliary cholangitis/ cirrhosis

Autoimmune destruction of intrahepatic bile ducts

976
Q

What bone problem is primary biliary cholangitis (autoimmune destruction of intrahepatic bile ducts) associated with?

A

Metabolic bone disease, including osteoporosis and osteomalacia (decreased bone mineralization)

*etiology of this is unknown, but there is an association

977
Q

PR interval is prolonged if it’s more than how many big boxes?

A

> 1 big box (or >5 small boxes)

978
Q

60 year old female with PMH of breast cancer s/p mastectomy and chemo/ radiation has SOB and nonproductive cough. “Can’t breathe” when lying down. She is using accessory muscles to breathe. She has an irregularly irregular tachycardia, distended neck veins, and bilateral crackles. Next step?

A

Noninvasive ventilation (BiPAP)

  • she is having acute decompensated HF (dilated cardiomyopathy/ systolic dysfunction from chemo and heart hasn’t had time to appropriately compensate for the changes)-> CHF symptoms (distended neck veins, crackles from pulm edema)
  • the stretching of the heart-> a fib (irregularly irregular)

**although Dobutamine may be helpful to help the heart pump, her HF-> is causing respiratory failure that is severe and needs to be addressed 1st (aggressive diuresis, oxygen, assisted ventilation)

979
Q

Car crash victim undergoes surgery for femoral fracture repair. One week later, he is lethargic. Multiple contusions are on his anterior abdomen and thighs. He has a flapping tremor (asterixis). Labs show low Hb, high BUN, high AST, and crazy high creatinine kinase. Diagnosis and next step?

A

Uremic encephalopathy

Urgent hemodialysis

-car crash with contusions (suggests trauma/ crush injury)-> rhabdo-> intrinsic AKI from myoglobin clogging up tubules (explains high BUN)-> the high BUN/ inability to clear urea (uremia) is an indication for dialysis (“AEIOU”)-> uremic encephalopathy-> flapping tremor

980
Q

Man presents with gait imbalance and objects moving around in his visual field. He was treated with gentamicin and ampicillin for a diabetic foot infection 2 weeks ago. Exam shows decreased hearing in the right ear and horizontal nystagmus with rapid head movement away from a fixed target. Most likely cause of symptoms?

A

Drug toxicity
(aminoglycosides, like gentamicin, are ototoxic to both the cochlea-> sensorineural hearing loss and vestibular system-> imbalance)

*nystagmus with rapid head movement away from a fixed target= head thrust test (vs. Dix-Hallpike maneuver used to dx Benign Paroxysmal Positional Vertigo—this would also have <1 min episodes of vertigo)

981
Q

Patient with subarachnoid hemorrhage undergoes endovascular coiling. 5 days later, he complains of confusion and tingling in his right hand. There is right-sided weakness and mild facial droop. Most likely cause?

A

Cerebral vasospasm and infarction

*vasospasm (the bleed causes irritation to the surrounding vessels) is the major cause of death within 24 hrs of SAH. Prevent with Nimodipine (CCB).

982
Q

Lady has hemoptysis and CXR reveals a nodule in the upper lung periphery (no hilar or mediastinal lymph node enlargement). She never smoked a day in her life and denies recent travel. Diagnosis?

A

Adenocarcinoma of the lung

*most common lung ca in non-smokers

983
Q

30 year old man complains of right ear pain worse with chewing. Exam is unremarkable. Diagnosis?

A

TMJ (Temporomandibular joint) disorder

  • ear pain in the setting of a normal ear exam is likely to be referred pain
  • often associated with teeth grinding (brutish) at night
  • clinical diagnosis (TMJ test where palpation of TMJ with mouth opening/ closing-> pain and clicking is not super sensitive)
  • give bite guard + NSAIDs
984
Q

26 year old lady with T1DM comes in for numbness and weakness in both legs. She keeps feeling like her legs are “going to sleep.” She also reports episodes of urinary incontinence. 1 year ago, she was seen for right eye pain and visual disturbance, but these symptoms resolved. Neuro exam is significant for decreased sensation to vibration, light touch, and pain. Fingerstick glucose is 140. Diagnosis?

A

Multiple sclerosis (MS)

  • autoimmune inflammatory demyelinating disorder of the CNS (the fact that she has T1DM, another autoimmune disorder, makes this even more likely)
  • young woman with neuro symptoms separated by time and space—optic neuritis (monocular blindness, painful eye movements) and transverse myelitis (motor and sensory loss below level of lesion, incontinence)
985
Q

Lady comes in for follow-up after having her 2nd miscarriage. Exam shows an enlarged nontender thyroid gland. TSH is high, free T4 is normal. Most likely diagnosis and what antibody titers do you expect will be elevated?

A

Hashimoto thyroiditis
anti-TPO antibodies

  • thyroid disease increases risk for miscarriages
  • she has subclincial hypothyroidism (normal free T4, but high TSH as you’d expect with hypothyroidism)
  • treatment with Levothyroxine is recommended for subclinical hypothyroidism for patients with high anti-TPO levels even if they don’t have symptoms (this will reduce risk of future miscarriage)
986
Q

What 3 criteria are required to make a diagnosis of acute liver failure (ALF)?

A
  1. Super high aminotransferases (>1000)
  2. Signs of hepatic encephalopathy
  3. Impaired hepatic synthetic function (INR >1.5)

In a patient w/o cirrhosis or underlying liver dz!!

987
Q

21 year old football player presents with irritability and aggressive behavior. BP is 135/85, HR 60. Exam shows mild gynecomastia. Labs show elevated Hct. Diagnosis?

A

Anabolic (androgens) steroid abuse

  • In men-> testicular atrophy, reduced spermatogenesis, gynecomastia, mood disturbance, aggressive behavior, hepatotoxicity
  • In women-> acne, hirsutism, deepening of the voice, menstrual irregularities, hepatotoxicity

*causes erythrocytosis (inc Hct and Hb) for unknown reasons but this is a clue they’re abusing

988
Q

35 year old lady comes in due to progressive SOB and drowsiness. One week ago she was treated for a UTI with Cipro. She has had difficulty swallowing food and her speech sounds nasal. Exam shows use of accessory muscles to breathe and decreased muscle strength bilaterally. ABG shows respiratory acidosis (low pH, high CO2). Diagnosis?

A

Myasthenic crisis

  • life-threatening complication of MG characterized by severe respiratory muscle weakness-> respiratory failure
  • triggers include: infection, surgery, certain meds (fluoroquinolones)
989
Q

45 year old man with PMH of MI 6 months ago (on Metoprolol, ASA, Rosuvastatin) has SOB on exertion and dry cough worse when lying down. His BP is 150/100. Exam shows bilateral crackles, strong PMI, and pitting edema in the legs. What’s going on?

A

Decompensated HF
(HF exacerbation)

  • HF (pumping problem)-> activation of RAAS + sympathetic NS (try to compensate for poor perfusion)…eventually, these systems are maladaptive, as Na+ retention leads to increased preload and vasoconstriction leads to increased afterload. This makes the pumping problem worse!! (It’s a vicious cycle of decompensated HF)
  • His HF is in the setting of recent MI
990
Q

What does the RAAS do to the afferent and/or efferent arterioles of the kidneys?

A

Constricts the efferent arterioles

*this increases GFR (and decreases RPF)

991
Q

Man with PMH of asthma and depression is found in a suicide attempt. He overdosed on an unknown drug. Vitals are normal. He is drowsy, has dry mucous membranes, blurred vision, urinary retention, and decreased bowel sounds. Most likely drug he overdosed on?

A

A Diphenhydramine

-have anticholinergic properties: “dry as a bone, blind as a bat, hot as a hare, full as a flask”

992
Q

Man has fever, chills, SOB, and productive cough for 2 days. He also vomited in the ED. Temp is 101.8, BP is 90/60, HR is 102. Repeat BP after 2L NS bolus is 120/80. Labs show leukocytosis. ABG shows uncompensated respiratory alkalosis. Diagnosis?

A

Sepsis 2/2 PNA

  • fever/ chills, SOB, productive cough, leukocytosis
  • met alkalosis due to hyperventilation (blow off more CO2-> dec CO2)

**SIRS criteria:
1. Temp >38 (100.4) or <36 (96.8)
2. WBC >12 or <4
3. HR >90
4. RR >20
SIRS= 2/4, sepsis= SIRS (+) w/ infection source

993
Q

Lady comes in for follow-up from an ischemic stroke 1 month ago. She feels some sensation is returning, but still has burning pain in the right limbs. Neuro exam shows hemianesthesia (inability to feel sensations) on the right, athetosis (abnormal muscle contractions) on the right hand, and hyperesthesia (increase in sensitivity) on the right demonstrated by exaggerated pain w/ light touch. Motor strength is normal. Brain location of her stroke last month?

A

Lucunar stroke of the Thalamus
(Pure sensory stroke)

*Internal capsule is often a pure motor stroke

994
Q

What does hemianesthesia mean?

A

Inability to feel sensation on one side

995
Q

What does athetosis mean?

A

Abnormal muscle contractions

996
Q

What does hyperesthesia mean?

A

Increased sensitivity (ex: light touch is painful)

997
Q

What does allodynia mean?

A

Severe burning pain over an area exacerbated by light touch

998
Q

75 year old man with PMH of osteoarthritis and HTN comes in for a yearly exam. Labs show an alk phos of 420, other labs normal. Cause?

A

Paget disease of the bone

  • most patients are asymptomatic! You get an isolated increase in alk phos
  • osteoclasts go crazy breaking down bone-> osteoblasts rush to catch up and do a poor job of laying down bone-> thick bone that fractures easily
  • treatment: bisphosphonates
999
Q

What bugs cause cellulitis?

A

Gram positives (staph, strep)

1000
Q

1st line treatment for postmenopausal osteoporosis?

A

Bisphosphonates

*note: treatment w/ bisphosphonates is usually stopped after 5 yrs, as prolonged use can cause atypical fracture

1001
Q

Patient has joint pains in many joints. He now has developed fever, diarrhea, and weight loss. Small intestine biopsy shows PAS-positive macrophages. Diagnosis?

A

Whipple’s disease

  • infection with T. Whippelii (how you get it is not fully understood)
  • causes a variety of symptoms, including malabsorptive diarrhea, weight loss, arthritis, lymphadenopathy, and low-grade fever
1002
Q

Elderly guy with PMH of chronic bronchitis and BPH has worsening SOB, wheezing, and productive cough for 3 days. Pulse ox is 85% on room air. Exam shows decreased breath sounds and diffuse wheezing over the lung fields. He is admitted and started on antibiotics, systemic corticosteroids, and nebulized albuterol and ipratropium. The next day, labs show hypokalemia. Why?

A

Albuterol (beta-2 agonist)-> shift of K+ into the cells (hypokalemia)

*this is a COPD exacerbation

1003
Q

55 year old lady comes in for SOB which began suddenly 2 hrs ago while watching TV. She also has sharp left-sided chest pain worse with coughing. BP is 110/60, HR 140. WBC count is elevated. EKG shows irregular RR intervals with no definite P waves and narrow QRS complexes. Cause of her symptoms?

A

Pulmonary Embolism (PE)

  • Acute-onset pleuritic chest pain, tachycardia
  • EKG w/ irregular RR intervals, no definite P waves, and narrow QRS’s= a-fib (associated with PE)

*Acute-onset pleuritic chest pain occurs in 70% of PE patients. Hemoptysis in <20%. DVT symptoms in <30.

1004
Q

Patient is having a ulcerative colitis flare being treated with glucocorticoids. Labs show normal TSH and T4, but low T3. Why?

A

Euthyroid sick syndrome
(“Low T3 syndrome”)
-less peripheral conversion of T4-> T3 in the setting of acute illness

  • factors in acute illness that block peripheral deiodination of T4-> T3 include high endogenous cortisol levels, inflammatory cytokines (TNF-alpha), and starvation
  • also, he’s on glucocorticoids which we know blocks conversion of T4-> T3
  • no treatment needed
1005
Q

Elderly man with PMH of asthma gets screening tonometry done and is found to have elevated intraocular pressure. He has no visual symptoms or headache. Funduscopy reveals thinning of the optic disc rim and asymmetry of the cup/disc ratio between the eyes. Diagnosis and next step?

A

Open-angle glaucoma
(High intraocular pressure-> atrophy of the optic nerve head—thins out w/ increased cup-to-disc ratio/ “cupping”)

Prescribe a med to lower IOP (by decreasing aqueous humor production or increasing its outflow)—1st line is a PG eye drop (Latanoprost, Bimatoprost)

2nd line= beta-blockers (Timolol) (would have to use w/ caution in this guy with asthma!)

**Don’t give Acetazolamide—this is used for closed-angle glaucoma

1006
Q

25 year old man comes in for dysuria and increased urinary frequency. He has multiple sexual partners and doesn’t always use condoms. Exam shows discharge at the urethral meatus. U/A is positive for leukocyte esterase and WBCs, but no bacteria. Gram stain and culture of the discharge shows no bacteria. Diagnosis?

A

Chlamydial urethritis

  • not Gonorrhea, or you’d get a gram negative staining
  • confirm dx with NAAT or first-catch urine sample (w/o pre-cleaning of genital area)
  • treat with Azithromycin or Doxy
1007
Q

What Grave’s disease treatment will worsen Grave’s ophthalmopathy?

A

Radioactive iodine (RAI)

  • Graves ophthalmopathy is due to effects of activated T-cells and thyrotropin receptor antibodies on TSH receptors on retro-orbital fibroblasts and adipocytes
  • RAI can raise titers of these antibodies-> worsening ophthalmopathy
  • give glucocorticoids to prevent this complication of RAI

**Sketchy: Radioactive iodine initially worsens Graves hyperthyroidism and ophthalmopathy due to inflam-> release of TH into the blood

1008
Q

What are the 2 lab values that provide the best picture of acid-base status in any patient?

A

pH and PaCO2

*HCO3- can be calculated from these 2 values using the Henderson-Hasselbalch equation

1009
Q

38 year old female with MS (was previously on disease-modifying therapy but stopped after being symptom free for years) presents with left arm and leg numbness for 3 days. Sensation to light touch and pin prick is diminished on the left side. Neuroimaging reveals new areas of demyelination compared to previous MRIs. Next step?

A

Glucocorticoids

-this is an acute MS exacerbation (we treat exacerbations with steroids, plasmapheresis if refractory)

1010
Q

20 year old college kid is brought in due to a suicide attempt by acetaminophen overdose after his girlfriend dumped him. He is hospitalized for N-acetylcysteine therapy. During his stay, he becomes confused and incoherent. Repeat exam shows scleral icterus and asterixis. Labs show AST and ALT in the 9,000’s, T. Bili of 4, PT 120sec, and Cr of 3.5. Next step?

A

Refer to liver transplant center

  • this is acute liver failure (ALF) 2/2 acetaminophen toxicity
    • severe acute liver injury w/o underlying liver disease + LFT’s >1,000, hepatic encephalopathy, and synthetic liver dysfunction (prolonged PT)
  • only 1/2 patients with ALF will survive w/o transplant
  • there are ethical issues with transplants in suicide attempts, but it is usually pursued in patients with reactive depression (no psych hx)
1011
Q

30 year old woman with hypothyroidism (on Levothyroxine) who gave birth 6 months ago presents with lightheadedness. Yesterday she felt like she was “going to pass out.” BP is 110/60 supine, 85/45 standing, HR is 105. Skin is hyperpigmented. Labs show low Na+, high K+, low bicarb (metabolic acidosis), and high Cr. Diagnosis?

A

Autoimmune adrenalitis-> primary adrenal insufficiency (lack of aldosterone, cortisol)

  • postural hypotension, lightheadedness, low Na+, high K+ (lack of aldo)
  • primary adrenal insuff-> neg fdbk-> high ACTH-> hyperpigmentation (due to subsequent inc of melanocyte-stimulating hormone)
1012
Q

Air-fluid level in the lungs. Think what?

A

Abscess

1013
Q

What does hypercarbia mean?

A

The same thing as hypercapnia! Retained CO2 (as in COPD patients).

1014
Q

35 year old man presents with weakness, fatigue, weight loss, and intermittent diarrhea. BP is 105/65, HR 95. Exam is normal. Labs show low Hb, low Na+ (130), K+ at upper limit of normal (5.5), 8am cortisol at lower limit of normal, normal TSH. Next step?

A

ACTH stimulation test (aka cosyntropin test)

  • weakness, weight loss, low Na+, high K+ -> primary adrenal insufficiency (PAI) aka Addison’s disease (lack of aldosterone + cortisol)
  • initial presentation: nonspecific symptoms, GI symptoms, hypotension
  • start by getting an 8am cortisol level and plasma ACTH (ACTH stimulation test)
    • normal-high cortisol r/o adrenal insuff
    • low cortisol + high ACTH= primary
    • low cortisol + low ACTH= secondary (get pituitary MRI)

*this Q is tricky bc his K+ is not technically high and cortisol is not technically low. But be aware, each patients baseline differs.

1015
Q

Treatment for primary adrenal insuff (Addisons)? Secondary adrenal insuff?

A

Primary—> Prednisone + Fludrocortisone
(Cortisol + Aldo replacement)

Secondary—> Prednisone only
(Cortisol replacement)

**Remember:
Primary-> problem at the level of the adrenal glands-> lack of aldosterone + cortisol

Secondary-> problem at the level of the pituitary-> lack of cortisol (aldo is normal bc it is NOT controlled by the ACTH axis from the pituitary gland)

1016
Q

What are the things you do to manage a patient coming into the hospital with COPD exacerbation?

A
  • combination short-acting bronchodilators (beta-2 agonist + anticholinergic drugs—this is your muscurinic antagonist)
  • systemic glucocorticoids (IV methylprednisolone)
  • broad-spectrum antibiotics (Azithro, Doxy)
  • Tamiflu (Oseltamivir) if flu is suspected as the cause
  • supplemental oxygen (goal SpO2 88-92%)
  • noninvasive positive-pressure ventilation (BiPAP) (tracheal intubation if fails)
1017
Q

Patient is brought in after ingesting a sodium hydroxide drain cleaner in a suicide attempt. He is having difficulty swallowing his saliva and is drooling. Vitals are normal. Exam shows oropharyngeal erythema and mild edema and epigastric pain. IV NS is started. Next step?

A

Upper GI endoscopy
(To assess severity of esophageal damage and guide further management)

*Activated charcoal can help get poisons out of the body, but will not help the damage already done to the esophagus. Plus, you want to do the GI endoscopy 1st bc charcoal would otherwise obstruct the view during endoscopy.

**NEVER induce vomiting (re-exposes the esophagus to the toxins) or give a neutralizing agent (will cause a chemical rxn that worsens the damage)!!

1018
Q

55 year old patient has a DVT. He’s had no recent travel or surgeries. CXR is unremarkable. What should you do to evaluate the patients condition?

A

Cancer screening

  • it is either inherited (factor 5 Leiden, prothrombin gene mutation, protein C/S deficiency) OR acquired (immobilization, surgery, malignancy, medications)
  • since this is unprovoked (no recent travel or surgeries), cancer screening (colonoscopy) is appropriate. Remember cancer-> hypercoagulable state-> inc DVT risk

**you do NOT need to test for inherited causes (like factor 5 Leiden)—we do this in patients with clots 45 years old or family hx or recurrent clots

1019
Q

Who gets lung cancer screening?

A

Patients age 55-80

w/ 30+ pack-year smoking hx

And patient is a current smoker or quit less than 15 yrs ago

*do annual low-dose CT scan

1020
Q

Patient has a venous thromboembolism (VTE) like a DVT. When should you do testing for an inherited cause (factor 5 Leiden, protein C/S deficiency)?

A

Patient is less than 45 y.o. OR

Patient has a family hx of these OR

Patient has had recurrent DVTs

1021
Q

25 year old guy presents with fever, malaise, fatigue, and sore throat. He is sexually active with 3 male partners. He has a fever. Exam shows pharyngeal erythema, mild splenomegaly, and a morbilliform rash (macular rash that looks like measles). Labs show an elevated WBC count with lymphocyte predominance. ALT, AST, and alk phos are all elevated in the 100s. Peripheral blood smear shows atypical lymphocytes. Heterophile antibody testing is negative. Diagnosis?

A

Cytomegalovirus (CMV) infection
(CMV mononucleosis)

  • asymptomatic in 90% of cases, but causes mono-like illness in 10% (same symptoms, negative mono-spot test/ Heterophile antibody testing)
  • gay men at higher risk
  • confirm with CMV IgM serology
  • treatment is supportive only (though, sometimes Valganciclovir is given—may speed up recovery but data is weak)
1022
Q

How do you confirm a diagnosis of CMV infection (CMV mononucleosis)?

A

Negative mono-spot test/ heterophile antibody testing (r/o EBV)

Confirm with CMV IgM serology

1023
Q

65 year old man with COPD comes in due to worsening SOB, productive cough, and wheezing for several days. Pulse ox is 85%, HR is 114. Exam shows moderate respiratory distress, bilateral expiratory wheezing, and distant heart sounds. EKG shows irregular narrow complex tachycardia with 3 different P-wave morphologies and a variable PR interval. Next step in managing his arrhythmia?

A

Monitor while treating the COPD exacerbation

  • SOB, productive cough, wheezing in a COPDer= COPD exacerbation
  • *distant heart sounds due to air trapping in COPD making it harder to hear the heart beats
  • irregular rhythm, rapid rate, 3+ P-wave morphologies= multifocal atrial tachycardia (MAT)
    • seen in elderly pts w/ COPD
    • treat the underlying COPD
  • *most likely due to a disturbance (RA enlargement, catecholamine surge from sepsis, or electrolyte imbalance)-> triggers atrial conduction abnormalities-> you get this supraventricular tachyarrhythmia
1024
Q

Elderly COPD patient has an irregular rhythm, rapid rate, 3+ P-wave morphologies. What arrhythmia is this and how do you treat it?

A

Multifocal atrial tachycardia

Treat the underlying COPD

*most likely due to a disturbance (RA enlargement, catecholamine surge from sepsis, or electrolyte imbalance)-> triggers atrial conduction abnormalities-> you get this supraventricular tachyarrhythmia

1025
Q

65 year old woman with T2DM, HTN, and ALS presents with worsening orthopnea, daytime fatigue, and morning headache for 1 month. Exam and labs are unremarkable. What’s going on? Next step in management?

A

Respiratory insufficiency in the setting of ALS

Noninvasive positive-pressure ventilation (CPAP, BiPAP)
(Helps prop open airways for support)

  • She has ALS—a neurodegenerative disease-> UMN + LMN signs
  • Most ALS patients die of respiratory failure (inspiratory muscle/ diaphragm weakness-> poor inspiratory strength, expiratory muscle weakness-> ineffective cough, bulbar muscle weakness-> chronic aspiration)
1026
Q

How do you diagnose antiphospholipid antibody syndrome?

A

Patient meets these 2 criteria:

  1. Has antiphospholipid antibodies—lupus anticoagulant, anticardiolipin antibody, or anti-beta-2 glycoprotein antibody
  2. Has clotting problems (DVT, PE, stroke, MI) or pregnancy problems (3+ miscarriages before wk 10, 1+ miscarriage after wk 10, preeclampsia/ eclampsia/ placental insufficiency)

**antiphospholipid antibodies are seen in SLE (lupus), though not specific for it. They can cause 3 problems: (1) antiphospholipid antibody syndrome, (2) increased PTT, (3) false positive Syphilis test (RPR/ VDRL)

1027
Q

Antiphospholipid antibodies (lupus anticoagulant, anticardiolipin antibody, anti-beta-2 glycoprotein antibody) can cause what 3 problems?

A
  1. antiphospholipid antibody syndrome
    (presence of antiphospholipid antibodies + clots or pregnancy problems like miscarriages)
  2. increased PTT
  3. false positive Syphilis test (RPR/ VDRL)
1028
Q

What are the 3 anti-phospholipid antibodies?

A
  1. lupus anticoagulant
  2. anticardiolipin antibody
  3. anti-beta-2 glycoprotein antibody
1029
Q

30 year old lady who smokes and does meth has progressive weakness and SOB for several wks. BP is 88/60, HR 105, RR 22, pulse ox 91%. Exam shows diffuse crackles over the lungs and dullness to percussion at the right lung base. There is a low-pitched diastolic sound over the cardiac apex. Legs have edema, are cool, and pulses are reduced. Labs show low bicarb, high BUN and Cr. A medication is given that acts primarily by stimulating beta-1 adrenergic receptors. What med is this?

A

Dobutamine
(“Just do bugling” beta-1 > beta-2 agonist)
*increases heart contractility and CO

Meth use-> cardiomyopathy-> decompensated HF (progressive SOB, diffuse pulmonary crackles, S3, hypotension, cool extremities, diminished pulses, and high BUN and Cr= evidence of poor organ perfusion)

1030
Q

Will Dobutamine increase or decrease LV end-systolic volume? Why?

A

Decreases LV end-systolic volume

  • Dobutamine (“Just do bugling” beta-1 > beta-2 agonist) increases heart contractility and CO
  • increased contractility-> improved ejection fraction-> reduced LV end-systolic volume (more blood got pumped out, so less blood stays behind at end of systole)-> improvement in decompensated HF
1031
Q

Management of acute lower back pain (less than 4 wks)?

A
  • maintain moderate physical activity
  • NSAIDs or Tylenol
  • consider: muscle relaxants, spinal manipulation
1032
Q

Management of subacute (4-12 wks) or chronic (>12 wks) lower back pain?

A
  • intermittent use of NSAIDs or Tylenol
  • exercise therapy! (Stretching/ strengthening, aerobic)
  • consider: TCAs, Duloxetine
1033
Q

17 year old boy comes in due to worsening acne. He uses topical tretinoin and washes his face daily with benzoyl peroxide. On exam, he has moderate pustular acne with erythema but no scarring. Next step?

A

Add topical antibiotic
(topical erythromycin)

-mild-mod inflammatory acne vulgaris that failed initially therapy (topical retinoids + benzoyl peroxide)
—> go to topical antibiotics (erythromycin, clinamycin)
—> if that also fails, go to oral antibiotics (tetracyclines) (*only start here if the acne is severe, nodular, or widespread)

1034
Q

What GI disease is characterized by mucosal inflammation and crypt abscesses?

A

UC (ulcerative colitis)

*vs. Crohn’s: skip lesions, transmural inflammation, non-caseating granulomas

1035
Q

70 year old man presents with SOB, headache, and fatigue. BP is 185/125, HR 95. On exam, he has papilledema, bibasilar coarse crackles, a systolic ejection murmur along the left sternal border, and trace pedal edema. Labs are significant for Cr of 2. He is given oxygen, Furosemide (Lasix), and Nitroprusside drip. A day later, his BP is 140/80 and he is no longer short of breath, but he is confused and disoriented and has diffuse hyperreflexia. Bicarb is 14. Most likely cause?

A

Cyanide toxicity

  • he was given a Nitroprusside drip to lower his BP, which can cause CN toxicity as an adverse effect! (*CN poisoning is caused by Nitroprusside or smoke inhalation)
  • CN inhibits complex 4 of the ETC so that the oxygen delivered to tissues cannot be utilized
  • symptoms: neurological (headache, confusion, hyperreflexia), respiratory (tachypnea-> resp depression), GI (vomiting), skin (flushing)
  • low bicarb-> metabolic acidosis (lactic acidosis!)
  • treat with Thiosulfate
1036
Q

Girl comes in due to irregular periods. She has coarse hair along her chin. BMI is 28. TSH and pregnancy tests are negative. Next step to address the irregular menses?

A

Prescribe OCPs

*too much LH (related to insulin resistance)-> theca cells in ovaries use this to make androgens-> overwhelms the granulosa cells—they cant convert all the extra androgens to estrogen (estradiol)-> so excess androgens circulate the blood-> hirsutism.
Also, fat cells use the androgens to make estrone which feeds bk and inhibits GnRH-> anovulatory, infertility.

-Anovulatory cycles-> irregular menses, dec progesterone, and unopposed estrogen which causes uncontrolled endometrial proliferation (when they do get a period its really heavy and can cause anemia)

OCPs will make cycles regular and block adrenal androgen production to reduce hirsutism

1037
Q

21 year old man is brought it after having a generalized tonic-clonic seizure. He’s never had a seizure before this but had a URI the last 3 wks. CT head shows a ring-enhancing lesion in the left frontal lobe. There are air-fluid levels and mucosal edema in the paranasal sinuses. Rapid HIV testing is negative. Diagnosis?

A

Brain abscess
(*most likely due to staph a. OR strep viridans)

  • sinusitis-> brain abscess
  • may cause seizures or focal neuro deficits
  • single ring-enhancing lesion on CT
1038
Q

What are the 2 most common bugs responsible for causing brain abscesses?

A
  1. Staph a.

2. Strep viridans

1039
Q

65 year old woman with HTN presents with a bitemporal headache that has worsened over this past wk. BP is 140/90. Exam shows decreased visual acuity in both eyes, dilated segmented and tortuous retinal veins, and decreased pinprick sensation and loss of ankle reflex bilaterally in the feet (no tenderness over temporal area). Labs show low Hb. elevated total protein (normal albumin), and elevated ESR. Next best diagnostic test to get?

A

Serum protein electrophoresis (looking for monoclonal IgM spike)

  • this is likely Waldenstrom macroglobulinemia (B-cell lymphoma where tumor cells differentiate into plasma cells and produce too much IgM-> hyperviscosity symptoms since IgM is large)
  • headache, vision changes, vertigo/ dizziness/ ataxia, rarely stroke or coma, peripheral neuropathy (as in this pt)
  • treat with plasmapheresis to remove IgM from serum
1040
Q

27 year old patient complains of weight gain (no other symptoms). Says her diets are not working. 1st step?

A

Recommend a diet (and exercise) diary

Track food intake and changes in weight

1041
Q

45 year old woman with PMH of migraines relieved by NSAIDs has severe abdominal pain and N/V for 2 hrs. She’s had episodes of epigastric and RUQ abdominal pain over the past several weeks. She has a fever and diffuse abdominal tenderness and guarding. Stool guaiac is positive. Next step?

A

Upright x-ray of the chest/ abdomen

  • this is peptic ulcer disease complicated by perforation-> peritonitis (acute onset worsening of symptoms, fever and guarding)
  • *remember that rebound tenderness + guarding are signs of an acute abdomen (consult surgery!)

-look for free air under the diaphragm (*if the x-ray is negative, you may follow-up with CT w/ contrast to detect smaller amounts of free fluid, but don’t get the CT if it will delay surgery)

1042
Q

44 year old man has progressive fatigue and SOB on exertion for several weeks—doesn’t even have the energy to mow the lawn. As of yesterday, he has fever, chills, and productive cough.

Vitals: fever and tachycardia
Exam: mucosal pallor (pale skin), scattered ecchymoses, left-sided lung crackles (no lymphadenopathy, no splenomegaly)
Labs: decreased Hb (6), reticulocyte count, leukocytes (2,000), and platelets (17,000)
CXR: left lower lobe consolidation
Peripheral blood smear: decreased leukocytes and platelets (no unusual cells)

What’s going on?

A

Aplastic anemia
(Pancytopenia—dec RBCs, WBCs, and platelets)

Dec RBCs-> anemia (fatigue, pallor)
Dec WBCs-> infection (pneumonia)
Dec platelets-> bleeding (ecchymoses)

  • acquired deficiency/ absence of pluripotent stem cells from exposures (drugs, toxins, radiation), viral infections (parvovirus B19, HIV, EBV), and autoimmune conditions (SLE, eosinophilic fasciitis)
  • definitive diagnosis made w/ bone marrow biopsy showing hypocellular marrow (few normal hematopoietic stem cells, no myeloid infiltration/ fibrosis, and mainly stroma and adipocytes/ fat cells)
1043
Q

What disease has “excessive collagen deposition in the bone marrow”?

A

Myelofibrosis

1044
Q

Diagnosis based on these key words?

  1. Lymphocytes with hairy projections
  2. Smudge cells
A
  1. Lymphocytes with hairy projections—> Hairy Cell Leukemia
  2. Smudge cells—> Chronic Lymphocytic Leukemia (CLL)
1045
Q

Patient comes in for a TB skin test required for her job. She works in a cafeteria at an elementary school, has no ill contacts, and has never been in jail. There is a 12-mm induration. Next step?

A

Nothing

A PPD skin test (TB screening) is considered positive if…
>15 mm in a healthy person
>10 mm in a healthcare worker, prison worker, recent immigrant in TB area, kid less than 4 years or adult with a comorbidity
>5 mm in an immunocompromised person
(*we are more conservative with the cut-offs in high-risk individuals to avoid missing cases of TB!)

1046
Q

PPD skin test is positive. Follow up with what other TB screening test to avoid a false positive?

A

IGRA (interferon-gamma blood test)

1047
Q

50 year old male with PMH of DM and ESRD (due to diabetic nephropathy) s/p kidney transplant 5 months ago (on Prednisone, Tacrolimus, Mycophenolate sodium), presents for follow-up. Labs are remarkable for BUN of 30, Cr of 2.5. U/A shows 1+ glucose, 2+ protein. Biopsy of the transplanted kidney shows heavy lymphocyte infiltration with vascular involvement and swelling of the intima. Cause of his renal dysfunction?

A

Acute transplant rejection

  • Think of this w/ elevated Cr + proteinuria in a patient with a transplant less than 6 months ago
  • Confirm w/ renal biopsy showing lymphocytic infiltration + intimal arteritis
  • T-cell mediated immune response to antigens from the donor kidney within 6 mo of transplant
  • Patients are often asymptomatic (but may have fever, decreased urine output, graft tenderness)
1048
Q

Kidney transplant patient (5 mo ago, on Prednisone, Tacrolimus, and Mycophenolate) comes in for follow-up. He is asymptomatic but you notice a bump in Creatinine on his labs. What diagnosis if…

  1. Renal biopsy is unremarkable
  2. Renal biopsy shows intranuclear inclusions and mixed lymphocytic + neutrophilic infiltrate
  3. Renal biopsy shows heavy lymphocytic infiltration w/ vascular involvement of the intima
A
  1. Renal biopsy is unremarkable—> ACUTE TOXICITY OT CALCINEURIN INHIBITORS (TACROLIMUS) CAUSING AKI
  2. Renal biopsy shows intranuclear inclusions and mixed lymphocytic + neutrophilic infiltrate—> BK VIRUS
  3. Renal biopsy shows heavy lymphocytic infiltration w/ vascular involvement of the intima—> ACUTE TRANSPLANT REJECTION (*T-cell mediated)
1049
Q

Patient with DM is treated in the hospital for persistent N/V. Initial BP was 85/40 so she was given IV fluids and BP went up to 110/70. It’s day 3 and she has decreased bowel sounds and decreased sensation to light touch over the feet. BUN was 40 on admission, now 55. Cr was 1.8 on admission, now 2.5. U/A shows coarse granular casts. Reason for this patients decreased urine output?

A

ATN (Acute Tubular Necrosis)

  • hypotension-> pre-renal AKI-> progression to ATN
  • the fact that fluids did not improve the AKI (BUN, Cr still high—in fact, worse than on admission) means it is not pre-renal
  • “granular casts” is the give-away (remember, ATN-> brown muddy granular casts—but here they dont mention the brown muddy color)
1050
Q

Patient has a peritonsillar abscess. Without treatment (incision and drainage or needle aspiration), this patient will be at risk for what?

A

Airway compromise

  • due to swelling
  • other complications of peritonsillar abscess: aspiration pneumonitis, lung abscess, hemorrhage due to septic necrosis of the carotid sheath, extension of infection into deep neck tissue or posterior mediastinum, and post-strep sequelae like rheumatic fever or GN (does NOT lead to necrosis of the tonsils)
1051
Q

Most common causative organism of peritonsillar abscess?

A

Group A strep

1052
Q

Patient has numbness and weakness of his left hand. Exam shows atrophy of the hypothenar muscles. There is weakness on aBduction of the small finger and the patient cannot prevent paper from being pulled through his small fingers. Cause?

A

Ulnar nerve compression

*remember the hypothenar muscles are on the opposite side as the thumb and the ulnar nerve innervates this area (medial side of hand)

1053
Q

Patient is hit in the flank with a hockey stick and is having 6/10 pain. U/A shows no abnormalities. Next step?

A

No further testing needed

-blunt trauma to flank w/o hematuria on U/A means chances of renal trauma are LOW
(*would get a CT abdomen if trauma to flank + hematuria)

1054
Q

Lady is brought in due to a recurrent episode of PNA. She has vascular dementia. Her husband says she swallows food well, but has been coughing a little more than before after meals. Next step?

A

Swallowing study

-recurrent PNA in a dementia patient-> think aspiration PNA

1055
Q

What GI side effect may Verapamil cause?

A

Constipation

*remember the toilet plunger in the CCB sketchy

1056
Q

How does PNA cause a right to left shunt?

A

PNA-> consolidation (alveoli filled with mucous)-> V/Q mismatch (perfusion w/o ventilation in those alveoli)

Right to Left shunt bc the shunt is in the normal direction of right heart-> lungs-> left heart. Deoxygenated blood is not getting oxygenated-> hypoxemia (R->L shunts are cyanosic vs. L->R shunts just push already oxygenated blood to the right heart as in early ASD or VSD)

1057
Q

55 year old women recently diagnosed with systemic scleroderma presents with headache, increasing SOB, and nausea. She is anxious with a BP of 235/115. Labs show Hb 7, platelets 55,000, leukocytes 5,000, total bili 2.2, Cr 3.9. What’s going on?

A

Systemic Sclerodermia Renal Crisis

  • Malignant HTN and AKI (high BUN, Cr) in the setting of Systemic Scleroderma
  • The damage to the kidneys can cause microangiopathic hemolytic anemia (low Hb, platelets- getting used up) like an HUS-type clinical picture
1058
Q

In Systemic Scleroderma Renal Crisis, what type of RBCs may you see on peripheral blood smear?

A

Schistocytes (helmet cells)

-the renal involvement-> microangiopathic hemolytic anemia

1059
Q

80 year old male with CAD presents with progressive bilateral buttock pain radiating to his thighs and calves. It is worse with movement, better with leaning forward. Ankle-brachial index is 1.1 on the right and 1.2 on the left (normal: 0.9-1.3). What’s most likely responsible for this patients condition?

A

Osteoarthritis of the spine (spondylosis)

  • osteoarthritis-> bone spurs (osteophytes)-> lumbar spinal stenosis (narrowing of spinal canal w/ compression on lumbar nerve roots)-> pseudoclaudication
  • “shopping cart sign”-leaning forward relieves pressure on the spinal cord
  • NOT vascular claudication bc ankle-brachial index is normal
1060
Q

Healthy 25 year old guy gets into a car accident. BP is 85/50, HR 125. Exam is normal and FAST exams shows no fluid. Most likely cause of his hemodynamic instability?

A

Pelvic fracture

-blood can hide in the retroperitoneum (negative FAST exam)

  • NOT intracranial hemorrhage (Cushing’s reflex is hypertension-> bradycardia, which is opposite of his vitals)
  • most common shock in trauma= hemorrhagic (hypovolemic)
1061
Q

44 year old lady has HTN (145/90). She is on no medications. No bruits are heard on exam. CBC and chem panel are normal. Should you do testing to look for a secondary cause of HTN?

A

No—that would be premature.
She is on no BP meds yet…only do secondary HTN work-up if atypical presentation 30 y.o., resistant HTN (requiring >3 meds), or signs/ symptoms suggestive of a specific etiology (ex: high Na+ and low K+ on labs, raising a flag for hyperaldo)

1062
Q

Why might you get a lipid panel and U/A for a patient presenting with HTN?

A

Lipid panel- to evaluate their risk factors
U/A- to evaluate if they are peeing out protein or blood

*primary HTN is far more common than secondary HTN, so get routine lab work done and start on BP meds. Only do work-up for secondary causes if BP is refractory (fails >3 meds) or if specific signs/ symptoms are present.

1063
Q

30 year old woman with hypothyroidism (on Levothyroxine) presents with lightheadedness and generalized weakness 6 weeks after delivering a baby. Her BP is 110/60 supine, 85/45 standing, HR 104. Skin is hyperpigmented. Labs show low Na+, high K+. Diagnosis?

A

Autoimmune adrenalitis

  • her presentation is consistent with Addison’s disease/ low cortisol (lightheaded, hyperpigmented from high ACTH from neg fdbk) (*so primary/ at the level of the adrenal glands, NOT at the level of the pituitary so NOT Sheehans)
  • labs (low Na+, high K+) show aldosterone is also low—not just cortisol, so primary adrenal insufficiency
  • the fact she has Hashimotos (an autoimmune dz) makes another autoimmune dz more likely
1064
Q

75 year old male with PMH of HTN, DM, and recent hospitalization for PNA (2 mo ago) is found unresponsive. He has a fever, is hypotensive, and tachycardic. CXR shows a new lung infiltrate. He is treated with NS, Abx, vasopressors, and mechanical ventilation. The next day labs show ALT and AST levels in the 3,000s. Most likely cause of the abnormal liver panel?

A

Shock liver (ischemic hepatic injury)

PNA-> septic shock-> liver is deprived of blood-> shock liver (diffuse liver injury)

**liver enzymes usually return to normal within 1-2 wks

1065
Q

65 year old lady feels like the “room is spinning.” Denies hearing loss, tinnitus, fever/ chills. When patient is lying still, she has rotatory nystagmus. Walking worsens her dizziness and she falls toward the left side. Next step and why?

A

Non-contrast CT head
To evaluate for cerebellar hemorrhage/ stroke

  • she is having CENTRAL vertigo (peripheral vertical is horizontal only, central can vary and hers is rotatory= going in circles)
  • vertigo from a central cause may also affect stability (cerebellum-> coordination)
  • central vertigo may also present w/ headache

**NOT BPPV, Meniere dz, or vestibular neuritis, as these are causes of peripheral vertigo

1066
Q

What is Ludwig angina?

A

Infection in the floor of the mouth
(Rapidly progressive cellulitis of the submandibular and sublingual spaces, usually from an infected molar)

*symptoms: fever, dysphagia (difficulty swallowing), odynophagia (pain w/ swallowing), and drooling

1067
Q

Lady has anxiety, palpitations, weight loss. Thyroid gland is diffusely enlarged and nontender. Labs show high free T4, low TSH. EKG shows sinus tachy at 120 bpm. Radioactive iodine uptake is 5% and there are high titers of anti-thyroid peroxidase antibodies. Next step?

A

Propranolol (beta-blocker)

  • this sounds like hyperthyroidism (Graves)…
  • but in Graves you’d expect high radioactive uptake (“hot thyroid”) and TSI (thyroid stimulating immunoglobulin) antibodies…
  • low uptake + anti-TPO antibodies are consistent with Hashimotos (hypothyroid, but can start out with hyperthyroidism due to lysis of thyroid cells that already made TH)
  • this is a variant of chronic lymphocytic (Hashimotos) thyroiditis
  • give a beta-blocker to control the current hyperthyroid symptoms
  • Methimazole would NOT work, as this blocks TH production (the problem here is not too much TH being produced, but too much that was already made being released)
1068
Q

65 year old man is brought in by his wife due to increasing forgetfulness, unsteady gait, and fatigue. He’s a retired guy and loves his homemade whiskey. Exam is remarkable for weakness on dorsiflexion of the wrists and feet, and reduced pinprick sensation over the hands and feet. Labs show low Hb, normal MCV. Diagnosis?

A

Lead poisoning
(-> impaired heme synthesis)

  • he prob mixed/ poured his whiskey through things with lead when homemaking it
  • lead blocks ALAD in heme synthesis pathway-> anemia w/ basophilic stippling, Lead lines on long bone x-ray, Encephalopathy, Abdominal sxs, and Drops-foot and wrist drop (“LEAD”)

*this is NOT methanol poisoning (seen in ppl who make their own moonshine), as that would present with eye problems

1069
Q

Associate what disease with the term “glycosylation?”

A

Diabetes

Glycosylation= adding sugar to stuff

1070
Q

Car crash victim has absent breath sounds in the right chest on arrival. A chest tube is placed, resulting in a loud gush of air. Exam shows multiple bruises over the anterior chest wall with crepitus on palpation. Over the next hour, the patient’s oxygen sat declines. X-ray shows proper ET and chest tube placement, reaccumulation of air in the right pleural space, pneumomediastinum, and increased subcutaneous emphysema. Diagnosis?

A

Bronchial rupture

  • he had a right-sided tension pneumothorax from trauma (chest tube placed-> air gushed out)
  • now, the air came back and filled the right pleural space
  • bronchial rupture makes sense bc you intubated him-> pumping more air in to the pleural space (would successfully go into the lungs if the esophagus or something else was ruptured, not the bronchus)

“Persistent pneumothorax + large air leak despite thoracostomy in the setting of blunt trauma suggests tracheobronchial rupture”

  • confirm with bronchoscopy
  • repair with surgery
1071
Q

What is a talc pleurodesis?

A

Removal of the pleural lining

*rarely done, but can be done for things like recurrent pleural effusions

1072
Q

22 year old man comes in due to sudden-onset SOB while watching TV. It’s gradually improving, but he still gets sharp pain with deep inspiration or coughing. Vitals are normal, pulse ox is 98%. CXR shows a small right apical pneumothorax. Most appropriate management?

A

Oxygen supplementation

-This is spontaneous pneumo (rupture of bleb)
-Management depends on the size:
Small and patient is stable-> supp O2 (regardless of O2 sat, since it speeds recovery)
Large and patient is stable-> needle decompression and/ or chest tube

1073
Q

Management of a spontaneous pneumothorax…

  1. If small, patient is stable
  2. If large, patient is stable OR large, patient is unstable
A

Small-> supplemental oxygen (*regardless of O2 sat, since it speeds recovery)

Large-> needle decompression and/ or chest tube

1074
Q

60 year old man with PMH of smoking and IV drug use has right-sided neck and shoulder pain, worse with movement and associated with numbness in his forearm. He’s had several episodes, relieved with rest and NSAIDs. There is decreased pinprick sensation over the right lateral forearm, but no other abnormalities. Most likely diagnosis?

A

Cervical spondylosis/ osteoarthritis

-> cervical radiculopathy (compression on spinal nerve root)

1075
Q

Osteophytes are associated with what type of arthritis?

A

Osteoarthritis

*Remember the O and O for Osteoarthritis and Osteophytes

1076
Q

ARDS…

  1. Hypoxemia correct with oxygen? (Yes/no)
  2. Lung compliance? (High/low)
  3. A-a gradient? (High/low)
A
  1. Hypoxemia correct with oxygen? NO
  2. Lung compliance? LOW (fluid collection restricts expansion of the lungs)
  3. A-a gradient? HIGH (hyaline membranes get in the way of gas exchange)
1077
Q

27 year old male has cough, chest discomfort, SOB on exertion. He also lost weight in the past couple months. CXR shows a large anterior mediastinal mass. Blood work shows high beta-hCG and AFP (alpha fetoprotein) levels. Diagnosis?

A

Non-seminoma germ cell tumor

  • seminomas also have inc beta-hCG, but NOT AFP (you see inc AFP-> think a non-seminoma type)
  • germ cell tumors are NOT only found in the testicles! They can occur in other places too
1078
Q

“Atlantoaxial subluxation”

Whatcha thinking of?

A

Rheumatoid arthritis

Atlantoxial joint= between C1, C2
Sublaxation= one moves in relation to another-> can compress the spinal cord

*this is the risk of intubating RA patients—don’t do it

1079
Q

25 year old man is hospitalized for fever/ chills. Blood cultures came back positive for gram positive cocci in clusters. Echo showed a mobile density on the posterior mitral valve leaflet. On day 3 of hospitalization, he is SOB and can’t lie flat bc of it. He has lung crackles and a holosystolic murmur over the cardiac apex. What’s going on? Would the LA and LV size be normal or dilated? Would LV ejection fraction be decreased or normal-increased?

A

Infective endocarditis-> mitral valve affected-> mitral regurg-> acute CHF symptoms (from the mitral regurg and backing up of blood)

LA size- normal
LV size- normal
LV EF- normal-increased (heart working harder to pump in this illness/ acute condition)

*you would think LA and LV dilated and LV EF low (CHF picture). But this is so acute (3 days) that the heart hasn’t had time to dilate and fail at pumping yet.

1080
Q
70 kg man is on a ventilator with the following settings:
RR 18
TV 450 mL 
FiO2 40%
PEEP 5
ABG shows the following: 
pH 7.5
pCO2 22
pO2 69
What is the next step (what do you want to change on the settings)?
A

Decrease respiratory rate (RR)

-pCO2 is low, so he’s blowing off lots of CO2= hyperventilation (also pH shows alkalosis, as it is a respiratory alkalosis)

  • there are 2 things you can do to mess with the ventilation (CO2): change Tv or RR
    (vs. O2: FiO2 or PEEP)

-appropriate tidal volume= ~6mL/kg
=6*70 kg= 420 mL (his is 450 mL, so appropriate)

-since Tv is appropriate, lower RR!!

1081
Q

What is an appropriate tidal volume (Tv) setting on a ventilator?

A

4-8 mL/kg

*just remember ~6 mL/kg…so multiply 6 times the patient’s weight in kg to get the appropriate tidal volume in mL

1082
Q

35 year old guy who recently lost his job and just moved in with his parents is brought in due to confusion. Temp is 101, BP 165/90, HR 110, RR 22. He is restless and yelling “get these bugs off me!” Labs show macrocytic anemia, high Na+, low K+, low P. What’s going on and what should you give to this patient?

A

Alcohol withdrawal
Long-acting benzo (chlordiazepoxide)

  • alcohol is a CNS depressant (more GABA inhibitory signals, less NMDA/ glutamate excitatory signals)
  • alcoholics adapt to a new homeostasis…extra excitatory signaling develops to balance out the extra inhibitory signaling from the alcohol
  • take the alcohol away (lost job, moved in parents)-> CNS overexcitation:
    • anxiety, agitation, tremor, diaphoresis, nausea
    • hallucinations
    • delirium tremens (DT): rapid-onset delirium and autonomic stability (fever, tachy, HTN, diaphoresis)
    • seizures
1083
Q

How common is alcohol use disorder?

A

Common!

About 14% of the US population has it!
(It is often masked, but if these people abruptly stop drinking alcohol, they will develop withdrawal symptoms)

1084
Q

Old man comes in due to fatigue. Exam shows pale conjunctiva. Labs show low Hb, low MCV, low ferritin. Next step?

A

Endoscopic evaluation (colonoscopy and/or EGD/endoscopy)

  • Microcytic anemia + low ferritin (iron in storage sites) is consistent with iron def anemia
  • iron def anemia in old folks is colon cancer until proven otherwise!
1085
Q

What 2 mediation treatment options are there for BPH?

A
  1. Alpha blockers (Terazosin, Tamsulosin)

2. 5-alpha reductase inhibitors (Finasteride)

1086
Q

30 year old woman presents with palpitations and weight loss. Labs show a high free T4, low TSH. Radioactive iodine scan shows uptake of tracer only in the right lobe. Diagnosis?

A

Toxic adenoma

-hyperthyroidism with nodular uptake-> toxic adenoma (1 nodule of uptake) or multinodular goiter (multiple nodules of uptake) (*vs. hyperthyroidism with diffuse uptake-> Graves’ disease)

1087
Q

In toxic adenoma and multinodular goiter, what is responsible for increased TH levels?
In Graves dz, what is responsible for increased TH levels?

A

Toxic adenoma and multinodular goiter (inc radioactive uptake at nodule(s))-> autonomous TH production (the hyperplastic cells in the nodules produce TH w/o TSH stimulation)

Graves’ disease (inc radioactive uptake diffusely)-> extra TH production is from the TSI (thyroid stimulating immunoglobulin) antibodies

1088
Q

65 year old diabetic is having burning, stabbing pains in his feet due to diabetic neuropathy. Should you start with treating him with Gabapentin or a TCA (Amitriptyline)?

A

Gabapentin

*TCAs should be avoided in elderly due to anticholinergic side effects

1089
Q

Treatment for cryptococcal meningitis?

A

Amp B + Flucytosine (2 wks)

Followed by Fluconazole (8 wks)

1090
Q

23 year old guy complains of generalized weakness and abdominal pain a week after starting a “vegetable juice cleanse” diet challenge. The oral mucosa is dry. Labs show low Na+ (132), high K+ (5.4), normal chloride (96), low bicarb (12, metabolic acidosis), high glucose (375).

What’s going on?

A

DKA (indicating a new diagnosis of T1DM *often how it first presents)

  • acute stress (including calorie restriction)-> cortisol-> triggers worsening of hyperglycemia (no insulin to bring glucose into cells)
  • severe hyperglycemia (375) (*normal non-fasting glucose is less than 120, *glucose in DKA is 300-500)
  • anion gap metabolic acidosis: Na-bicarb-Cl= 132-12-96= 24= high anion gap (>12 +/- 2) from ketones formed from FA breakdown
  • glucose spills into urine with water and Na+ (glc-Na co-transporter)-> dehydration (dry oral mucosa) and hypoNa+
  • hyperK+ due to low pH (high H+ goes into cells-> K+ shifts out of cells into blood) and lack of insulin (to stimulate Na/K pump and bring K+ into cells, so more K+ stays in blood)
1091
Q

65 year old lady is brought in due to confusion, loss of mobility, and behavioral changes. She cries for no reason and said “a lion was sleeping in the backyard.” She has memory loss and gets lost in her neighborhood. Exam shows increased muscle tone and resting tremors in the extremities. Labs are normal (RPR negative). She is given Risperidone after becoming combative, but this only worsens her stiffness. Diagnosis?

A

Dementia with Lewy bodies
(Lewy Body Dementia)

-fluctuating cognitive impairment, recurrent visual hallucinations, Parkinsonism

**called Lewy body dementia if cognitive and motor (tremors) symptoms are less than 1 yr apart (vs. dementia 2/2 Parkinson’s disease)

1092
Q

Poor glycemic control is A1c greater than what?

A

A1c > 7%

1093
Q

35 year old man with untreated HIV and hep C is brought in by his roommate for acting “weird” the past 2 days. He’s been staying in his room, not eating much. Temp is 100.2, BP 140/85, HR 95. He has mild sclera icterus. Labs show normocytic anemia, inc reticulocyte count, indirect bili, and LDH, decreased platelets, inc BUN and Cr, and mildly inc AST and ALT.

Diagnosis and next step?

A

TTP (Thrombotic Thrombocytopenic Purpura)

Get a peripheral blood smear (looking for shistocytes)

  • dec ADAM13 level-> vWF does not get degraded-> too much sticking of platelets
  • thrombocytopenia (low platelet count), microangiopathic hemolytic anemia (RBCs get sheared when going past the microthrombi)
  • “FAT RN” Fever, Anemia (microangiopathic), Thrombocytopenia, Renal failure, Neuro manifestations
1094
Q

How to calculate GCS (Glasgow Come Scale)?

A

“my Extra Value Meal costs $4.56”

Eye response- up to 4 points
Verbal response- up to 5 points
Motor response- up to 6 points

1095
Q

70 year old man presents with a deep headache for 2 months. He’s a construction worker and his hard hat has felt more snug lately, so he thinks this is contributing to his symptoms. Labs show an elevated alk phos of 650. The skull has thickened cortices with mixed lytic and osteoBlastic lesions. Nuclear bone scan shows increased uptake in the skull and tibia. In addition to vitamin D and calcium intake, what’s the next step?

A

Prescribe a bisphosphonate
(like Alendronate)

  • this is Paget disease of the bone
  • osteoCs go crazy breaking down bone-> osteoBs lay down bone quickly and do a sucky job of it rushing-> thick brittle bone
  • clinical features:
    • bone pain (microfractures)
    • increasing hat size (skull affected)
    • hearing loss (impingement on CNs)
    • lion-like faces (craniofacial bones involved)
    • isolated high alk phos
  • bisphosphonates to treat (cause osteoC apoptosis to stop this madness)
1096
Q

Man is vomiting and “feels a little dizzy.” Labs show his chloride level is low—why?

A

GI fluid loss

Vomiting-> loss of HCl (acid) and K+ (so loss of chloride!)

**metabolic alkalosis is the consequence, not the cause, of vomiting!

1097
Q

17 year old lady presents with fatigue for 4 months. Exam shows scars on the dorsum of her hands and dental erosions. Labs show low K+ and high bicarb. Urine chloride is low. Diagnosis?

A

Surreptitious vomiting

  • she is making herself vomit (scares on hands, dental caries)
  • metabolic alkalosis-> low urinary chloride-> means diuretics or vomiting
1098
Q

30 year old woman with PMH of asthma, chronic cough, and recent uveitis presents with intermittent dizziness and unsteadiness for 2 wks. He has mild SOB. EKG shows AV block and left bundle branch block. CXR shows bilateral midfield lung opacities. Diagnosis?

A

Sarcoidosis (w/ cardiac involvement)

  • cardiac noncaseating granulomas are in 25% of sarcoidosis patients, but only 5% develop cardiac symptoms
  • cardiac sarcoidosis is a disease of non-caseating granuloma infiltration of the myocardium-> arrhythmia, cardiomyopathy, HF, and sudden cardiac death
1099
Q

60 year old man with DM and decreased visual acuity comes in due to nocturnal urinary frequency, occasional dribbling, and a weak urinary stream for 3 months. Exam shows a left-sided carotid bruit and trace bilateral ankle edema. Postvoid bladder residual volume is 40 mL. Urine dipstick shows 2+ protein. Cr is 2.1. Hb A1c is 7.3%. Cause of his chronic kidney dz?

A

Diabetic nephropathy

  • microvascular process
  • NOT BPH—this explains his urinary symptoms (urinary outflow obstruction), but a postvoid residual less than 50 mL means the obstruction is not significant (and you’d need complete obstruction to lead to a spike in Cr). Also BPH wouldn’t cause proteinuria.
1100
Q

What two type 2 diabetes drugs have an added protective effect on cardiac and weight-related comorbidities?

A
  1. GLP-1 agonists (EsenaTIDE)
  2. SGLT-2 (sodium-glucose co-transporter 2) inhibitors (CanagliFLOZIN)
    - both are good for add-on therapy in patients with cardiovascular disease, as they are associated with weight loss and reduce cardiovascular mortality and morbidity

(*insulin, sulfonylureas, and TZDs are NOT associated with weight loss)

1101
Q

30 year old man is brought in due to confusion for hours. His wife says he’s had a fever, malaise, and cough for the past 2 days. One year ago, he underwent extensive surgery for multiple gunshot wounds to the abdomen. Temp is 105, BP 80/50, HR 110, RR 30. There is dullness to percussion and crackles over the lower left chest. IV fluids and antibiotics are started. The next day, blood cultures show gram (+) cocci. What’s going on?

A

He probably had a splenectomy during his operation for multiple gunshot wounds to the abdomen (at increased risk for encapsulated bacterial infections—strep pneumo, H flu, Neisseria meningitidis)

Now has strep pneumo (PNA)-> septic shock

1102
Q

How are encapsulated organisms (strep pneumo, H. Flu, and Neisseria meningitidis) eliminated in the body? (What’s the immuno mechanism?)

A

These pathogens are eliminated by humoral response with antibody-mediated phagocytosis (opsonization) and complement activation

*much of this is dependent on splenic macrophages and splenic opsonizing antibodies (so patients without a spleen are at higher risk for encapsulated bacterial infections and need vaccination)

1103
Q

66 year old man has severe constipation and increasing back pain for 2 months. Labs show low Hb, high BUN and Cr, and high Ca2+. Best explanation for his constipation? Likely diagnosis?

A

Constipation due to hypercalcemia

Likely has multiple myeloma

  • back pain (lytic lesions), anemia (too many plasma cells, not enough of other blood cells), renal damage (from Ig light chains stuck in tubules), hypercalcemia
  • fatigue, constipation, and depression are common symptoms of mild hypercalcemia
1104
Q

30 year old man is fatigued and forgetful lately. He works as a contractor and is renovating old houses for sale. He is “clumsy” and constipated. Labs show low Hb, normal MCV, high reticulocyte count, high uric acid. Diagnosis?

A

Lead poisoning

-GI symptoms (constipation), neuropsych symptoms (sensorimotor neuropathy, memory loss), hematologic symptoms (microcytic anemia w/ basophilic stippling, hyperuricemia due to impaired purine metabolism)

1105
Q

Treatment for lead poisoning?

A

Chelation therapy with an agent like Calcium disodium EDTA

1106
Q
Based on what the renal casts look like, what’s your diagnosis? 
Granular or muddy brown
RBC casts 
WBC casts
Fatty casts 
Broad, waxy casts
A
Granular or muddy brown—> ATN 
RBC casts—> GN (glomerulonephritis) 
WBC casts—> pyelonephritis or interstitial nephritis 
Fatty casts—> nephrotic syndrome
Broad, waxy casts—> CKD
1107
Q

Pre-renal AKI. What would you expect the BUN/Cr ratio to be? How about the FeNa and urine osm?

A

BUN/Cr >20 (high, bc the slow blood flow to kidneys gives the PCT more time to reabsorb urea)

FeNa less than 1% (normal, bc kidneys are working to reabsorb Na+)

Urine osm >500 (normal, bc kidneys are working to concentrate urine)

1108
Q

Intrinsic AKI. What would you expect the BUN/Cr ratio to be? How about the FeNa and urine osm?

A

BUN/Cr less than 20 (low, bc kidneys are NOT working to reabsorb sufficient urea)

FeNa > 1% (high, bc kidneys are NOT working to reabsorb Na+)

Urine osm less than 500 (low, bc kidneys are NOT working to concentrate urine)

1109
Q

Mnemonic for causes of normal anion gap metabolic acidosis?

A
“HARD ASS” 
Hyperchloremia 
Addisons (primary adrenal insuff) 
RTA (renal tubular acidosis) 
Diarrhea 
Acetazolamide
Spironolactone 
Saline infusion 

**the big ones are: RTA, diarrhea, Addisons

1110
Q

Ischemic colitis vs. Acute mesenteric ischemia vs. Chronic mesenteric ischemia?

A

Ischemic colitis- cut off blood supply to watershed areas of colon (due to hypovolemia)
Acute mesenteric ischemia- “MI of the colon” (due to thrombotic event)
Chronic mesenteric ischemia- “angina of the colon”

1111
Q

What are the “ABC-Os” for treatment of a COPD exacerbation?

A

Antibiotics (Azithro, Doxy)
Bronchodilators w/ anticholinergics (albuterol + ipratropium)
Corticosteroids (IV methylprednisolone)
Oxygen (keep O2 sat 89-92%)

  • also BiPAP
  • do not use tiotropium in place of ipratropium, as this is long-acting/ too slow in an exacerbation
1112
Q

Treatment steps for asthma patient (based on severity, be specific)?

A
  1. SABA (if intermittent- symptoms less than 2 days/ wk)
  2. ICS (add if mild persistent- symptoms >2 days/ wk)
  3. LABA (add if moderate persistent- symptoms daily)
  4. Oral corticosteroid and Omalizumab (if allergies) (add if severe persistent- symptoms many times daily)
1113
Q

What diagnosis should come to mind for these penile ulcers?

  1. Painless ulcer (chancre) + painless lymphadenopathy
  2. Painful ulcer (chancroid) + painful lymphadenopathy
  3. Painless single ulcer + painful lymphadenopathy

*bonus points: how to treat?

A
  1. Painless ulcer (chancre) + painless lymphadenopathy—> primary Syphilis (*give Penicillin G or Doxy)
  2. Painful ulcer (chancroid) + painful lymphadenopathy—> H. Ducreyi (*give Azithro)
  3. Painless single ulcer + painful lymphadenopathy—> LMV (*due to Chlamydia so give Doxy)
1114
Q

Low magnesium (Mg) can cause low what other ions?

A

Low Ca2+, low P, and low K+

*so check those mag levels!

1115
Q

When do you do the following screening tests? For who?

  1. Low-dose CT checking for lung cancer
  2. DEXA scan checking for osteoporosis
  3. Abdominal U/S checking for AAA
  4. Mammogram checking for breast cancer
  5. Colonoscopy checking for colon cancer
A
  1. Low-dose CT checking for lung cancer—> smokers 55+ (who smoked more than 30 years or quit less than 15 years ago), annually/ every year
  2. DEXA scan checking for osteoporosis—> women 65+, once
  3. Abdominal U/S checking for AAA—> men 65+ who smoked ever, once
  4. Mammogram checking for breast cancer—> women 50+ every 2 years OR 40+ every year (conflicting recommendations are out)
  5. Colonoscopy checking for colon cancer—> everyone 50+ every 10 years (or do fecal occult blood check every year) *test more if got a genetic disease or ulcerative colitis that puts at higher risk
1116
Q

Pleural effusion. Fluid has high amylase. Differentials?

A

Esophageal rupture, pancreatitis, or malignancy

1117
Q

Pleural effusion. Fluid has low glucose (less than 60). Differentials?

A

RA, TB, SLE, esophageal rupture, or malignancy